Comprehensive

Lakukan tugas rumah & ujian kamu dengan baik sekarang menggunakan Quizwiz!

The correct response is Option B. The most appropriate method to reconstruct the wound is a local musculofascial flap and skin advancement flap. The major principle of myelomeningocele repair is to obtain a well-vascularized layer of soft-tissue coverage between the dural and skin repairs. The fascia overlying the paraspinous muscles can be turned over as flaps, followed by paraspinous muscle advancement flaps to cover the underlying dural repair. This vascularized soft-tissue layer will minimize the risk of cerebrospinal fluid contact with cutaneous bacteria and subsequent meningitis if either the dural repair or skin repair breaks down. A split-thickness skin graft over the dura would not adequately protect the spinal cord. Closing the skin directly over the dural repair using skin advancement flaps would place the child at risk for meningitis in the event of a cerebrospinal fluid leak or if wound breakdown occurred along the incision line of the widely undermined skin flaps. The use of a regional gluteal or latissimus muscle flap to cover the dural repair is unnecessary because local tissue (paraspinous muscles and fascia) is available. Harvesting the gluteal or latissimus muscles also may cause significant donor site morbidity in a child already at risk for ambulatory problems from a neurologic deficit.

2013 A 1-day-old female newborn is evaluated because of repair of a lumbar myelomeningocele. After dural repair, physical examination shows the spinal cord at the base of the wound with a 4 × 4-cm soft-tissue and skin defect. Which of the following is most appropriate to reconstruct the wound? A ) Gluteal muscle flap and skin advancement flap B ) Paraspinous musculofascial flap and skin advancement flap C ) Skin advancement flap only D ) Split-thickness skin grafting

The correct response is Option D. This patient is presenting with an uncommon but major complication of dextran administration, specifically acute renal failure, thought to be caused by either direct toxic effect on the tubules and glomeruli or intraluminal hyperviscosity. Surgeons who employ its use must be aware of this potential side effect as well as other serious side effects such as anaphylaxis, volume overload, pulmonary edema, cerebral edema, and platelet dysfunction. At-risk patients include those with a history of diabetes, renal insufficiency, or vascular disorders. It is recommended to avoid dextran in patients with chronic renal insufficiency for this reason. This head and neck cancer patient also was on aspirin for anticoagulation. It should be noted that a prospective randomized study of dextran- and aspirin-related complications in 100 patients undergoing microsurgical flap reconstruction for head and neck malignancy demonstrated that aspirin and dextran were actually equally efficacious in preventing flap failure. However, despite this, it was demonstrated that patients on dextran had a 3.9- to 7.2-fold increased relative risk of systemic complications after 48 and 120 hours of dextran infusion, respectively. Given this, aspirin should be used over dextran if anticoagulation is desired. A 500-mL bolus of Ringer's lactate would not be warranted because it contains potassium, which would already be elevated in acute-on-chronic renal failure and would exacerbate the hyperkalemia. Administration of a diuretic would not be warranted in this case because there is no evidence of fluid retention. Low-dose dopamine is commonly administered to critically ill patients in the belief that it reduces risk of renal failure by increasing renal blood flow. This has never been definitely proven, however, in multiple trials. This patient has chronic renal insufficiency exacerbated by dextran administration. Low-dose dopamine has not been demonstrated to confer any benefit in this clinical scenario. If a hematoma, arterial insufficiency, or venous congestion were suspected, returning to the operating room would be the next most appropriate step. As this is not the working diagnosis, this would be inappropriate.

2013 A 154-lb (70-kg), 54-year-old man is evaluated because of oliguria and malaise 2 days after he underwent subtotal glossectomy with reconstruction with a free radial forearm flap. History includes chronic renal insufficiency (baseline creatinine concentration was 1.8 mg/dL and is now 3.3 mg/dL). After the procedure, administration of aspirin 81 mg by mouth daily and dextran 40 at 20 mL/hr was initiated. Temperature is 99.9°F (37.7°C), heart rate is 88 bpm, respiratory rate is 20/min, and blood pressure is 110/60 mmHg. On examination, the flap is pink and soft. Urine output is 15 mL/hr. Which of the following is the most appropriate management? A ) Administer a 500-mL bolus of Ringer's lactate B ) Administer a diuretic C ) Administer dopamine D ) Discontinue dextran E ) Return to the operating room for neck exploration

The correct response is Option E. Massive injury and burns result in a systemic inflammatory response with resultant leakage of fluid into the interstitial space. Large fluid shifts can decrease perfusion to vital organs and inadequate resuscitation will result in acidosis, oliguria, and relative polycythemia. Fluid replacement is based on the observation that intravascular fluid loss into the interstitium is relatively constant during the first post-injury day. All fluid, therefore, is administered at a constant rate to avoid excessive interstitial edema. Patients are monitored for an adequate clinical response by measurement of hourly urine output (0.5-1 mL/kg/hr for adults) with adjustments as needed. Invasive monitoring may be necessary in elderly patients, patients with cardiac dysfunction, or patients with severe pulmonary injury. A variety of fluid resuscitation formulas are available to guide the initial management of fluid replacement. The Parkland formula and its variations have become the standard methods for resuscitation. Isotonic crystalloid, Ringer's lactate in particular (sodium concentration of 130 mEq/L) is the fluid of choice. The Parkland formula directs the resuscitation as follows: 4 mL of fluid × patient weight in kilograms × total body surface area (TBSA) percentage of second-and third-degree burns. Half of this total volume is delivered in the first 8 hours and the second half over the ensuing 16 hours. In a delayed presentation (3 hours in this clinical example), half of the total volume must be delivered within the 8-hour window (remaining 5 hours in this example). The TBSA is calculated using the "Rule of Nines," as follows: Head/Neck = 9% Each upper extremity = 9% Anterior thorax = 18% Posterior thorax = 18% Each lower extremity = 18% Groin = 1% In this patient example, the TBSA of burn is 54%: Anterior thorax (18%) + each lower extremity (18% + 18%). The Parkland formula considers only second- and third-degree burns for calculation of resuscitation volume; therefore, the first-degree burns in this case are ignored. In this clinical example, 4 mL × 75 kg × 54% TBSA = 16,200 mL total fluid over 24 hours. The patient will require 8100 mL in the first 8 hours and 8100 mL over the ensuing 16 hours. Because the patient presented 3 hours post burn, he will require 8100 mL over the next 5 hours which gives an hourly rate of 1620 mL/hr.

2013 A 165-lb (75-kg), 40-year-old man is brought to the emergency department 3 hours after sustaining first-degree burns to the hands and second- and third-degree burns to the entire anterior thorax and both anterior and posterior lower extremities. According to the Parkland formula, administration of which of the following is the most appropriate method of initial fluid resuscitation in this patient? A ) Hypertonic saline solution 253 mL/hr for 5 hours B ) Hypertonic saline solution 1181 mL/hr for 8 hours C ) Ringer's lactate 506 mL/hr for 24 hours D ) Ringer's lactate 1013 mL/hr for 8 hours E ) Ringer's lactate 1620 mL/hr for 5 hours

The correct response is Option E. The lip is a favorable area for early surgical intervention for infantile hemangiomas and vascular malformations. The amount of lip tissue that can safely be resected allows for primary closure in the majority of cases. The tumors often distort the local tissues and are "additive" to the amount of available tissue, so resection typically allows for safe linear closure along the vertical resting tension lines. Reconstruction of other facial areas, such as the cheek and eyelids, is usually more complicated, and nonoperative measures may be a better first line of treatment. Surgical treatment is further warranted in this case by the functional impact of the lesion. Pulsed-dye lasers are effective treatment for thin vascular lesions. Multiple treatments are needed, they can be painful, and some residual scarring is usually left after resolution. Intralesional bleomycin is another option for local control, though multiple treatments, often with anesthesia, are required. Propranolol therapy has recently come to the forefront of hemangioma treatment. Oral treatment is preferred at 1 mg/kg three times daily. Dramatic responses are seen within 24 hours of starting treatment, and significant resolution occurs over a course of weeks. Contraindications may include bronchospasm, significant cardiac abnormalities, or cerebrovascular abnormalities. Pretreatment MRI and pediatric medical management may help determine suitability for treatment. Propranolol is most effective for early lesions that are in a proliferative growth phase. In this case, by 2 years, the lesion is likely done with growth and in a stable to involuting phase. Oral prednisolone is also effective at treating hemangiomas and is a common treatment option. Hyperglycemia and cushingoid features are temporary drawbacks. Given the symptoms in this case and the relative ease of surgical resection, medical therapies such as steroids and beta blockers would be secondary choices. In asymptomatic patients with minimal disfigurement, observation alone may be the preferred first line of treatment.

2013 A 2-year-old boy is brought to the office because of an infantile hemangioma of the right lower lip. His parents say that he frequently drools and has difficulty in phonation. On examination, the lesion measures 1 cm in width and distorts the surrounding tissues. A photograph is shown. Which of the following is the most definitive treatment? A ) Bleomycin B ) Oral prednisolone C ) Propranolol D ) Pulsed-dye laser therapy E ) Surgical resection

The correct response is Option A. The most appropriate next step in management is admission to the intensive care unit (ICU) for close monitoring as this patient has significant soot in his airway, which is a sign of inhalation injury. Although endotracheal intubation may become necessary, currently the patient is stable. With access to an ICU and close monitoring, intubation is not necessary for this patient at this time. Bronchoscopy is indicated if the patient requires intubation to assess the extent of injury and manage secretions but is not currently indicated in this patient who is awake and stable. Discharge with close follow-up is not acceptable as this patient may decompensate rapidly due to the delayed onset of inhalation injury. Tracheostomy would only be indicated if the patient required prolonged intubation or if the airway could not be obtained orally.

2013 A 23-year-old man is brought to the emergency department 2 hours after a nightclub fire. Oxygen saturation is 98% on 4 L of oxygen via nasal cannula. Marked soot is noted in the pharynx. He sustained burns to less than 1% of the total body surface area. Which of the following is the most appropriate next step in management? A ) Admission to the intensive care unit for monitoring B ) Discharge with follow-up in 3 days C ) Evaluation of the upper airway with bronchoscopy D ) Securing of the airway with endotracheal intubation E ) Tracheostomy

The correct response is Option C. The patient is experiencing signs of early malignant hyperthermia, which is a life-threatening crisis and requires immediate attention. The typical symptoms of malignant hyperthermia are caused by a hypercatabolic state with increased heart rate, increased breathing rate, increased carbon dioxide production, increased oxygen consumption, acidosis, masseter muscle rigidity, and rhabdomyolysis. Very high temperature (110.0°F [43.0°C]) usually presents late. Even if treated properly, death may occur as brain damage, muscle damage, renal failure, and multiple organ failure ensue. The malignant hyperthermia crisis is a biochemical chain reaction response that is "triggered" by commonly used general anesthetics and the paralyzing agent succinylcholine within the skeletal muscles of susceptible individuals. Volatile gaseous inhalation anesthetics like sevoflurane, desflurane, isoflurane, and halothane can trigger malignant hyperthermia. The exact incidence of malignant hyperthermia is not known. Estimates vary from a frequency of one in 5000 to one in 65,000. Over 80 genetic defects have been associated with malignant hyperthermia. Malignant hyperthermia susceptibility is inherited with an autosomal dominant inheritance pattern. Children and siblings of a patient with malignant hyperthermia susceptibility usually have a 50% chance of inheriting a gene defect for malignant hyperthermia. The medical antidote is dantrolene. Additional methods include use of a hypothermia blanket (under/over the patient) and cold isotonic saline for intravenous solution.

2013 A 23-year-old woman with no history of surgery undergoes augmentation mammaplasty with administration of nitrous oxide, isoflurane, and propofol. Sixty minutes into the procedure, the nurse anesthetist notifies the surgeon that the patient has increasing end-tidal carbon dioxide concentrations, tachycardia, and severe masseter muscle rigidity. Which of the following is the most appropriate next step in management? A ) Administer intravenous saline B ) Administer a muscle relaxant C ) Stop isoflurane D ) Stop nitrous oxide E ) Stop propofol

The correct response is Option A. Any tissue transplantation from another genetically nonidentical human is termed allograft, previously referred to as homograft. As these transplanted tissues are immunologically different from the recipient, they will eventually undergo rejection from the host immune system without immunosuppressive medications. There are many distinct antigens responsible for the rejection process, the most important of which are the major histocompatability complex (MHC) antigens, known as HLA-1 and HLA-2, which reside on the surface of cells. Autograft implies that the donor tissues come from the same patient - such as the common autologous skin graft. Isograft comes from a genetically identical donor, namely an identical twin. While such donations are very rare, it is of important historical note that the first human kidney transplantation was performed by Dr. Joseph Murray between identical twin donor and recipient. In this clinical scenario, as one patient is Asian and the other Hispanic, it is clear that the transplant is not an isograft. Xenograft is a cross-species graft. Porcine skin grafts are commonly used as temporary skin substitutes to promote granulation formation in difficult wound beds.

2013 A 24-year-old Chinese-American, right-hand-dominant man undergoes cadaveric hand transplantation after traumatic amputation at the right wrist from a machinery accident. The donor is a Hispanic man. Which of the following terms best describes the antigenicity of the transplant? A ) Allograft B ) Autograft C ) Isograft D ) Xenograft

The correct response is Option C. Restoration of craniofacial contour after infection, tumor resection, or trauma can be quite challenging. Autologous bone grafts have long been considered the gold standard due to their high likelihood of osseointegration/healing and low risk of rejection or infection. Autologous bone grafts, however, have several drawbacks, including unpredictable resorption, donor site morbidity, limited availability, prolonged operative times, and difficulty in contouring. As a result, there has been an ongoing search for alternative means of reconstruction with alloplastic material. The ideal bone substitute should be chemically inert, easily contoured, retain a stable shape over time, strong, resistant to infection or foreign body reaction, inexpensive, and capable of osseointegration and tissue ingrowth. Methylmethacrylate has been used frequently for calvarial reconstruction but suffers several drawbacks, including infection requiring removal of implant, plate fracture, lack of osseointegration, difficulty shaping after polymerization, and necrosis of surrounding tissue due to the exothermic nature of the curing process. Some of the most promising and well-tolerated alloplastic materials for craniofacial skeletal reconstruction are the calcium phosphate-based compounds. Hydroxyapatite [Ca(PO4)6(OH)2] forms the principal mineral component of bone and constitutes 60% of the calcified human skeleton. Calcium phosphate compounds are bioactive and capable of osteoconduction and osseointegration. Osseointegration refers to the direct chemical bonding of an alloplast to the bony surface without an intervening fibrous tissue layer. During osteoconduction (creeping substitution), the alloplast acts as a nonviable scaffold for ingrowth of blood vessels and osteoprogenitor cells from the recipient site. Subsequently, the graft/alloplast is resorbed and replaced with new bone. This mechanism is also associated with the healing of cortical bone grafts. Hydroxyapatite cement is a mixture of tetracalcium phosphate and dicalcium phosphate anhydrous which reacts in an aqueous environment to form a paste that can be easily applied and sculpted to fit the surgical defect. Hydroxyapatite cement sets isothermically, so there is no risk of thermal damage to the surrounding tissues. Additional benefits of hydroxyapatite include "off the shelf" ease of use, maintenance of volume over time, lack of radiologic scatter, and low incidence of infection. Osteoinduction refers to the direct stimulation of mesenchymal cells at the recipient site by bone morphogenetic protein to differentiate into osteoprogenitor cells. This mechanism of action is associated with the healing of cancellous bone grafts and demineralized bone matrix. Endochondral ossification is the process by which the cartilaginous soft callus covering a fracture is transformed into bone. Osteogenesis is the process by which vascularized bone grafts heal. Viable osteocytes survive the transplantation process and produce new bone at the recipient site. Osteochondrosis refers to a family of ossification disorders in children.

2013 A 25-year-old man is scheduled to undergo reconstruction of a 5 × 5-cm, full-thickness calvarial defect. A titanium/hydroxyapatite cement cranioplasty reconstruction is planned. Which of the following mechanisms best describes the healing process involved? A ) Endochondral ossification B ) Osteochondrosis C ) Osteoconduction D ) Osteogenesis E ) Osteoinduction

The correct response is Option B. Although not commonly used, the thoracic intercostal nerves can provide graft material up to 12 cm in length. Harvest of the intercostal nerve has minimal donor site numbness and can be harvested through an open or endoscopic approach. The intrinsic chest muscles include (from superficial to deep) the external intercostal, internal intercostal, innermost intercostal, and transverse thoracis muscles. The external intercostal muscle is most active during inspiration. It functions to stiffen the chest wall during descent of the diaphragm to prevent paradoxical collapse of the chest. The other more internal intercostal muscles are weaker and are more involved in expiration. The intercostal nerve emerges from the spinal cord and immediately splits into a dorsal ramus (that innervates the back) and ventral ramus. The ventral ramus runs between the internal and innermost internal intercostal muscles before crossing over the internal thoracic vessels and penetrating through the intercostal muscles to supply the anterior chest skin.

2013 A 25-year-old man is scheduled to undergo soft-tissue coverage and nerve grafting using a seventh intercostal space nerve graft after he sustained a gunshot wound just above the left clavicle. On preoperative examination, he had difficulty abducting the shoulder, and injury to the brachial plexus was suspected. Between which of the following structures is the thoracic intercostal nerve located? A ) External intercostal and pectoralis major muscles B ) Innermost intercostal and internal intercostal muscles C ) Internal and external intercostal muscles D ) Parietal pleura and transverse thoracis muscle E ) Transverse thoracis innermost intercostal muscles

The correct response is Option A. Frostbite is a common injury to the homeless and outdoor adventurers. Direct injury is caused by extracellular freezing of tissues with significant changes of the osmotic gradient of cells which can cause significant electrolyte imbalances. Many inflammatory mediators, including thromboxanes, prostaglandins, histamine, and bradykinin are released which lead to significant edema, endothelial injury, and tissue damage. The mainstay of frostbite injuries, regardless of severity, includes rewarming, pain control, administration of tetanus prophylaxis, and frequent dressing changes and wound care. In cases of severe frostbite injuries with absent pulses, emergent angiography and infusion of tissue plasminogen activator (tPA) have been shown to significantly decrease the rate of amputation if administered within 24 hours of the onset of frostbite. Although fasciotomy might be necessary after reperfusion, it is not indicated as a first-line therapy for frostbite. Likewise, heparin and hyperbaric oxygen have not been shown to improve the outcomes in frostbite injuries. Technetium-99 triple-phase scanning can accurately estimate the level of eventual amputation required if performed in the first several days, but it is not a therapeutic modality and would delay the angiography and administration of tPA if indicated.

2013 A 27-year-old man is brought to the regional burn center 4 hours after he got lost in a snowstorm while he was hiking. Physical examination shows severe frostbite of the feet, purple coloring of the toes, heavy blistering, and marked edema. On Doppler examination, pulses are absent bilaterally. He sustained no other injuries and is otherwise healthy. In addition to warming and pain control, which of the following is the most appropriate next step in the treatment of this patient? A ) Angiography B ) Fasciotomy C ) Hyperbaric oxygen therapy D ) Intravenous administration of heparin E ) Technetium-99 triple-phase scanning

The correct response is Option E. In women, the pudendal nerve terminates in the dorsal nerve of the clitoris, which is responsible for most erogenous sensation. Neurorrhaphy to this nerve has been shown to result in superior sensory outcomes in phalloplasty. The dorsal nerve of the clitoris is analogous to the dorsal nerve of the penis, which is used as a target for neurorrhaphy during penile reconstruction for amputation or congenital microphallus. Other branches of the pudendal nerve include the inferior rectal and perineal nerves. The ilioinguinal nerve supplies sensation to the skin of the upper medial thigh and to the skin of the mons pubis and labia majora. The genitofemoral nerve also provides sensation to the labia majora and skin of the femoral triangle. The posterior femoral cutaneous nerve innervates the skin of the posterior surface of the thigh as well as part of the perineum. The iliohypogastric nerve innervates the skin above the pubis and in the gluteal region.

2013 A 27-year-old woman is scheduled to undergo female-to-male transgender surgery. A fasciocutaneous radial forearm flap is designed to construct the phallus. This procedure includes neurorrhaphy of the lateral antebrachial cutaneous nerve to a terminal branch of which of the following? A ) Genitofemoral nerve B ) Iliohypogastric nerve C ) Ilioinguinal nerve D ) Posterior femoral cutaneous nerve E ) Pudendal nerve

The correction response is Option C. Vaginal reconstruction for congenital vaginal agenesis or after tumor resection remains a challenging reconstructive endeavor. The ideal technique provides a vaginal canal with adequate dimensions, texture, appearance, and sexual function without excessive donor site morbidity. The deep inferior epigastric artery perforator flap as a local regional flap has been described recently for vaginal reconstruction. It is a non-hair-bearing flap that is performed in a single stage but is insensate. Depending on the patient's body habitus, it is less bulky than the musculocutaneous flaps (gracilis, vertical rectus) used for vaginal reconstruction, but more bulky than the Singapore flap. Two perforators are included in the flap to ensure viability. The major disadvantage is a conspicuous donor site scar. The pudendal thigh flap, commonly referred to as the Singapore flap, is a local fasciocutaneous flap that has been widely used for neovaginal reconstruction. It is a thin, sensate flap based on the posterior labial arteries, which are a continuation of the perineal artery. Modifications of the flap have been described to enhance the viability and reach of the flap. It is a reliable, single-staged operation, but can result in endovaginal hair growth. This can be improved with preoperative electric or laser depilation of the vulvar portion of the flap. Neither flap secretes mucus. Both are single-staged operations.

2013 A 32-year-old woman is scheduled to undergo subtotal reconstruction of the vagina using a pedicled deep inferior epigastric artery perforator flap after tumor ablation. Which of the following is the most likely advantage of using this type of flap coverage rather than the pudendal thigh (Singapore) flap? A ) Better maintenance of sensory innervation B ) Less vaginal bulkiness C ) Less vaginal hair growth D ) More optimal mucus secretion E ) Single-staged procedure

The correct response is Option E. The most consistent complication of costal cartilage grafting is the propensity to warp or change shape over time. This may be due to the presence of perichondrium or the nonuniform composition of the matrix that can affect the shape when it is placed. Although there is a risk of pleural violation and pneumothorax, these complications are uncommon and can be recognized intraoperatively with the Valsalva maneuver. Through the same exposure for harvesting the rib graft, a red rubber catheter is placed in the chest to evacuate intrathoracic air. As long as the visceral pleura remain intact, a thoracostomy tube is unnecessary. Fresh autologous grafts easily survive transplantation procedures and do not appear to resorb over time. Autologous cartilage from septum, concha, or rib is considered the ideal graft material. These grafts have very low risk of infection or extrusion compared with an alloplast. Cartilage grafts are tolerated well by nasal tissue.

2013 A 35-year-old Asian woman desires dorsal augmentation to achieve a more "Western" nose. Autologous rib cartilage grafting is planned. Which of the following complications is most common with this procedure? A ) Extrusion B ) Infection C ) Pneumothorax D ) Resorption E ) Warping

The correct response is Option A. The skin on the palm has specialized, encapsulated nerve endings called Meissner corpuscles in the dermal papillae and Vater-Pacini corpuscles in the deep dermis. These special dermal neural mechanoreceptors are unique to glabrous skin. Skin grafts from nonglabrous donor sites lack this feature and will have poor return of sensibility. There are a few features that are common between palmar and regular skin. Both contain intraepidermal nerve endings and sweat ducts and glands. Both have an irregular border between the basal layer of the epidermis and dermis, at which juncture are the dermal papillae and epidermal rete ridges. A network of blood vessels and sensory and autonomic nerve fibers in the dermis is shared by all skin. Palmar skin has deeper papillae and ridges, as the keratin layer is considerably thicker; however, glabrous donor sites for grafting are limited. Finally, pilosebaceous structures are absent in the palm.

2013 A 35-year-old man undergoes reconstruction of a degloving injury of the palmar surface of the hand. Full-thickness skin grafting from the groin is planned. Which of the following structures of healthy palmar skin will be missing from this graft? A ) Dermal neural mechanoreceptors B ) Dermal sweat ducts and glands C ) Irregular dermal-epidermal border D ) Pilosebaceous structures E ) Sensory and autonomic nerve fibers

The correct response is Option A. A prospective, randomized study has shown that antibiotic prophylaxis is superior to placebo in decreasing infections after human bites that are less than 24 hours old. The most common pathogens in human bite wounds are S aureus, E corrodens, H influenzae, and beta lactamase-producing anaerobic bacteria. Eikenella species are resistant to clindamycin. Meanwhile, doxycycline and trimethoprim-sulfamethoxazole are not effective against anaerobes. Of the options mentioned, only amoxicillin-clavulanate has good activity against all common oral pathogens.

2013 A 37-year-old man comes to the emergency department 4 hours after he sustained a human bite wound to the nondominant hand. Examination shows no erythema, swelling, purulent drainage, lymphangitis, or fever. Exploration of the wound shows no joint or tendon involvement. Debridement and irrigation of the wound is performed. Which of the following is the most appropriate next step in management? A ) Administration of amoxicillin-clavulanate 875/125 mg twice daily B ) Administration of clindamycin 450 mg three times daily C ) Administration of doxycycline 100 mg twice daily D ) Administration of trimethoprim-sulfamethoxazole 1 double-strength tablet twice daily E ) Observation

The correct response is Option D. Vascularized composite allotransplantation refers to the transplantation of an allograft consisting of heterogeneous cadaveric tissues. It provides a means of restoring structural, functional, and aesthetic form in severely injured patients. The potential for improvements in quality of life must be offset by the need for lifelong immunosuppression in adults with non-life-threatening injuries. The benefits and difficulties of immunosuppressive drugs have been established in solid organ transplantation. Regimens derived from renal transplantation have been successfully applied to composite tissue allografts. Overall, more than 60 hand/forearm/arm transplantations and 16 face transplantations have been performed in the past 12 years. The overall functional and aesthetic outcome is satisfactory, but side effects and complications related to immunosuppression are challenges hindering progress in this field. The high levels of immunosuppression, skin rejection, nerve regeneration, donor legislation, and the acceptance level need to be addressed to promote growth of this promising new field in transplantation and reconstructive surgery. Because composite tissue allograft transplantations are not life-saving procedures, much consideration is devoted to the issue of minimizing or withdrawing immunosuppression. When compared with solid organ transplants, composite tissue allografts are histologically heterogeneous, composed of different tissue types (e.g., skin, muscle, bone, bone marrow, lymph nodes, nerve, and tendon) and express different immunogenicity of transplanted elements. The hierarchy of antigenicity of composite tissue allografts was introduced in an experimental model of limb transplantation and showed that skin is the most antigenic tissue, and together with muscle, subcutaneous tissue, and bone (including bone marrow), may generate high immunologic response. The vascularized muscle component of limb allografts may induce a cell-mediated response greater than the skin; however, muscle as a single component is less antigenic than skin. Bone represents lower immunogenicity, and cartilage, tendon, and vessels are the least antigenic.

2013 A 37-year-old man is waiting for a facial deceased-donor vascularized composite allotransplant. Which of the following components of a total face transplant is most antigenic? A ) Bone marrow B ) Muscle C ) Nerve D ) Skin E ) Tendon

The correct response is Option D. Trichoepitheliomas are neoplasms of follicular differentiation. Trichoepithelioma usually presents as multiple, yellowish-pink, translucent papules distributed symmetrically on the cheeks, eyelids, and nasolabial area. The lesions are more frequently seen in women. Lesions are benign but can be confused with basal cell carcinomas clinically and histologically. As they are benign, no further measures should be taken. However, in cases of desmoplastic trichoepithelioma, complete excision or Mohs surgery may be needed to clearly differentiate this entity from a carcinoma. Eccrine poroma occurs as a solitary lesion usually on the sole of the foot or the palm of the hand in persons older than 40 years. It may also occur on the chest, the neck, or other locations. Eccrine poromas are seen as firm papules less than 2 cm in size. Lesions may occasionally be pedunculated and have a normal or erythematous color and a firm consistency. In rare instances, malignant eccrine poroma or porocarcinoma develops either spontaneously or from long-standing benign eccrine poroma. Treatment is surgical excision. Verrucous nevus consists of closely set verrucous papules that may coalesce to form well-demarcated plaques. They may be skin colored, brown, or gray-brown. A linear configuration is common, especially for lesions on the limb. Such lesions may appear to follow skin tension lines. On histologic evaluation, there is hyperkeratosis, acanthosis, and papillomatosis. The histologic appearance is essentially that of a benign papilloma. Excision is the most reliable treatment. This may not be practical or advisable if the lesion is extensive or at sites not amenable to simple surgery. The excision should extend to the deep dermis; otherwise, the lesion may recur. Alternative treatments have included laser cryotherapy and electrodesiccation dermabrasion. Cylindroma presents as either solitary or multiple lesions. Nodules may also be present on the face and rarely on the extremities. The lesion appears in adulthood. The surface is smooth and may be telangiectatic. Cylindromas are usually benign, but malignant changes have been reported. For solitary lesions, treatment is by excision or electrosurgery. For small cylindromas, the carbon dioxide laser may be used. Multiple cylindromas usually require extensive plastic surgery that may be obviated by progressively excising a group of nodules in multiple procedures. Nevus sebaceous is a distinctive growth most commonly found on the scalp, followed by the forehead and retroauricular region. A nevus of epithelial and nonepithelial skin components, nevus sebaceous sustains age-related modifications in morphologic appearance. The nevus occurs singly and is asymptomatic. Two thirds are present at birth; the remaining third develop in infancy or early childhood. Male and female infants are equally affected. The three-stage evolution of the nevoid condition (newborn, puberty, and adult) parallels the natural histologic differentiation of normal sebaceous glands. In approximately 20% of patients, a third phase of evolution involves the development of secondary neoplasia in the mass of the nevus. A number of benign and malignant "nevoid tumors" may occur, the most common of which is the basal cell epithelioma. The malignant degenerations are relatively low grade; only a few instances of metastasis have been reported. Surgical excision of a nevus sebaceous is recommended because of the high potential for development of basal cell carcinoma and other tumors. The lesion should preferably be excised before puberty because it may enlarge, and the risk of malignant transformation increases after puberty.

2013 A 39-year-old woman is evaluated because of a 6-month history of a growth on the face. A biopsy is planned. Which of the following findings on pathology can be safely treated with observation only in this patient? A ) Cylindroma B ) Eccrine poroma C ) Nevus sebaceous D ) Trichoepithelioma

The correct response is Option D. Tretinoin (all-trans-retinoic acid; Retin-A) is one of the best long-term topical therapies available for chronically photoaged skin. The mechanism of action of retinoids is regulated through specific nuclear receptors. Ultraviolet (UV) radiation activates a series of phosphokinases that stimulate c-Fos and c-Jun proto-oncogenes and thereby activate AP-1 transcription factor. AP-1 causes activation of metalloproteases such as collagenase, gelatinase, and stromelysin, which then break down collagen. Tretinoin results in a 70% inhibition of AP-1 transcription factor binding to DNA and a significant reduction in protease activity. Long-term use of tretinoin is associated with improved skin texture, decreased sallowness, a reduction in fine rhytides and actinic keratosis, fading of pigmented macules, and an overall improvement in skin appearance. Histologic effects of tretinoin include increased epidermal and granular layer thickness; elimination of dysplasia, atypia, and microscopic actinic keratoses; uniform dispersion of melanin granules; increased collagen and glycosaminoglycan deposition in the papillary dermis; and diminished dermal elastosis, angiogenesis, and compaction/thinning of the stratum corneum. Tretinoin side effects include erythema, photosensitivity, and desquamation. Patients are initially started on a low dose with nightly application until tolerance is achieved. Because tretinoin is a photosensitizer, sunscreen use is absolutely imperative. Topical retinoids should be used for a minimum of 3 to 4 months, with the greatest improvement after 1 year of use. Patients who use alpha-hydroxy acids concomitantly with topical retinoids will see a synergistic effect, and this combination is tolerated well in most patients. Isotretinoin (13-cis retinoic acid; Accutane) impairs sebaceous gland activity, impairs epithelialization, and thins the stratum corneum. Alpha-hydroxy acids cause desquamation as a result of diminished corneocyte cohesion just above the granular cell layer. Dermal effects of these acids include increased collagen and glycosaminoglycan production. UV radiation-stimulated oxygen free radicals are the primary mediators of UV skin damage. Vitamin C has been shown to be the primary water-soluble nonenzymatic antioxidant that helps protect skin cells from UV radiation. Other functions of Vitamin C include increased Types I and III collagen production, decreased pigment synthesis, improved epidermal barrier function, and regeneration of oxidized Vitamin E. Hydroquinone impairs the conversion of dopamine to melanin. The result is decreased formation of melanin and increased degradation of melanosomes.

2013 A 40-year-old woman comes to the office because of fine rhytides and skin discoloration caused by photoaging. Topical application of 0.025% tretinoin is planned. Which of the following best describes the mechanism of action of topical retinoid therapy? A) Decreased activation of skin appendages resulting in thinning of the stratum corneum B) Decreased corneocyte cohesion resulting in increased desquamation C) Increased free radical scavenging activity and synthesis of Types I and III collagen D) Inhibition of AP-1 transcription factor binding to DNA resulting in diminished protease activity E) Inhibition of tyrosinase resulting in diminished conversion of dopamine to melanin

The correct response is Option B. The most appropriate response is to obtain consent from the patient to access his medical record. Confirming that the x-ray studies have been performed, sharing the report, viewing the x-ray studies in person with the radiologist, and reviewing the x-ray studies with the spouse would require accessing the patient's medical record, for which the plastic surgeon has not obtained consent. Doing so would be a violation of the Health Insurance Portability and Accountability Act (HIPAA).

2013 A 42-year-old man who is a close friend of the on-call plastic surgeon is brought to the emergency department because of a metacarpal fracture. He is being treated by the on-call orthopaedic surgeon. The patient's wife sees the plastic surgeon and asks to view her husband's post-reduction x-ray studies. Which of the following is the most appropriate response? A ) Confirm only that the x-ray study was completed B ) Obtain verbal consent from the patient C ) Review the x-ray studies with the spouse D ) Share the report without showing the x-ray studies E ) View the x-ray studies with the radiologist

The correct response is Option E. Pre-reconstruction radiation is typically seen in two groups of patients: those who underwent mastectomy without reconstruction followed by radiation therapy, or those who underwent breast-conserving therapy and radiation with recurrence or new cancer. Although these patients may present with a reasonable skin envelope, complication rates associated with tissue expander/prosthesis have been reported as high as 70%, with a 40% rate of failure or conversion to flap. Additionally, aesthetic outcomes in most patients who completed expander/prosthesis reconstruction after previous radiation therapy were deemed good or very good compared with the majority of non-irradiated patients who achieved excellent results. Tissue expander/prosthesis can be considered as an option for patients with a history of previous irradiation who wish to avoid the scars and recovery of flap-type reconstructions; however, they should be counseled of the high risks of complications.

2013 A 45-year-old woman comes to the office for consultation regarding mastectomy and immediate breast reconstruction because of recurrent right-sided breast cancer. She underwent breast-conserving therapy and radiation therapy 5 years ago. BMI is 23 kg/m2. The patient is otherwise healthy, and she does not smoke cigarettes. On examination, the breast is soft with obvious fibrosis. Use of which of the following has the highest risk for reconstructive failure in this patient? A ) Deep inferior epigastric artery perforator (DIEP) flap B ) Free transverse rectus abdominis musculocutaneous (TRAM) flap C ) Latissimus flap with prosthesis D ) Pedicled TRAM flap E ) Tissue expander and prosthesis

This item was originally keyed to option C. This item was rekeyed to option D. The diagnosis of the described lesion is dermatofibrosarcoma protuberans (DFSP). It accounts for less than 0.1% of all malignant neoplasms and approximately 1% of all soft-tissue sarcomas, but is the most common type of cutaneous sarcoma. It is a malignant mesenchymal tumor that arises in the dermis and is characterized by latency in its initial detection, slow infiltrative growth, and local recurrence if not adequately treated. Distant metastasis is rare and generally occurs as a late sequela after repeated local recurrences. DFSP is most commonly found on the trunk followed by the proximal extremities, and rarely in the head and neck. These tumors have irregular shapes, frequent finger-like extensions, and an infiltrating growth pattern extending beyond clinical margins that result in incomplete removal and a propensity for local recurrence. Treatment primarily consists of wide surgical excision to include margins of 2 to 3 cm beyond the clinical tumor border if possible. Mohs micrographic surgery has been used with good outcomes in aesthetically sensitive areas such as the head and neck where tissue sparing is important. Reconstruction with tissue rearrangement or flaps should be performed after negative margins are confirmed. Most recurrences occur within 3 years of the primary excision, and close follow-up is indicated. Conventional chemotherapy is rarely used. Radiation therapy is used as an adjunct to surgery for close or positive margins in areas where adequate wide resection alone may result in major cosmetic or functional deficits. Molecular targeted therapy such as imatinib mesylate (Gleevec) is indicated for unresectable, recurrent, or metastatic DFSP. Sentinel node biopsy is not indicated in the treatment for DFSP.

2013 A 47-year-old man comes to the office because of an asymptomatic lesion of the anterior abdominal wall that has been enlarging gradually for the past 10 years. The lesion has accelerated in growth during the past several months and recently ulcerated. Examination shows a 6-cm, raised, indurated, and irregularly shaped violaceous plaque consisting of firm, irregular nodules. Examination of a specimen obtained on incisional biopsy shows a soft-tissue malignancy arising from mesenchymal cells in the dermis. Which of the following is the most appropriate treatment? A ) Wide local incision and molecular targeted therapy B ) Wide local excision and sentinel node biopsy C ) Wide local excision, molecular targeted therapy, and radiation therapy D ) Wide local excision only E ) Wide local excision, sentinel node biopsy, and chemotherapy

The correct response is Option D. The patient has pyoderma gangrenosum, which is often associated with ulcerative colitis, Crohn disease, or rheumatoid arthritis. The lesion may begin as a small erythematous plaque or have purplish discoloration, and it commonly occurs on the lower extremities after minor trauma. The lesions may rapidly become painful and appear as a necrotizing ulcer. The exact cause of pyoderma gangrenosum is not entirely understood. It is believed the disease is immune-related, such that initial treatments may be primarily medical, including systemic steroids or immunosuppression. Generally speaking, surgery is reserved as a last resort as it is believed that further surgery may exacerbate the condition. Furthermore, recurrent, ulcerative lesions after surgery are not uncommon. Marjolin ulcer is typically a squamous cell carcinoma arising from a long-standing wound, chronically inflamed tissue, or previous burn scar. The latency period may range from 5 to 30 years from the time of initial injury before developing into a Marjolin ulcer. Treatment generally involves radical resection, as Marjolin ulcers tend to be aggressive. Leishmaniasis is a disease caused by a protozoan parasite (Leishmania), which is typically transmitted by the bite of a sand fly. In the area of the bite, patients develop ulcerations of the skin. Diagnosis can be made by obtaining a history of an insect bite, a history of travel and exposure to where sand flies reside (Central America, South America, West Asia, or the Middle East), and by obtaining scrapings from the ulcers and looking for the organisms under the microscope. Scleroderma is a chronic autoimmune disease that is characterized by fibrosis of the skin. The underlying cause is not entirely understood. Patients may present with skin ulcers. Scleroderma may be categorized as systemic or limited. Patients with limited disease may present with Raynaud phenomenon as an early sign. The hands and fingers may be affected with distal tip ulceration and thin, taut skin over joints. Patients with systemic disease may have additional manifestations including gastrointestinal, pulmonary, renal, and cardiac fibrosis. Necrotizing fasciitis is a rapidly spreading soft-tissue infection involving the skin and subcutaneous tissues. It typically travels along fascial planes and can result in tissue necrosis, sepsis, and death. It can be mono- or polymicrobial, with typical offending pathogens including Group A Streptococcus, Staphylococcus aureus, Bacteroides fragilis, and Clostridium. Treatment includes high-dose intravenous antibiotic therapy and timely surgical debridement. Patients may require repeat debridement frequently in the early stages of treatment to limit progression of the disease.

2013 A 47-year-old man with hypertension and ulcerative colitis comes to the office because of a painful lesion of the right lower leg. He says that he dropped a typewriter on his leg 1 year ago and noticed a small wound that slowly began to grow larger despite treatment with bandages and topical antibacterial ointment. Examination of a specimen obtained on incisional biopsy shows neutrophilic dermatosis. Cultures grow Staphylococcus aureus. Which of the following is the most likely diagnosis? A ) Leishmaniasis B ) Marjolin ulcer C ) Necrotizing fasciitis D ) Pyoderma gangrenosum E ) Scleroderma

The correct response is Option E. Postoperative nausea and vomiting (PONV) is a distressing complication of general anesthesia and occurs in 25 to 30% of surgeries. Oftentimes, multimodality treatment is implemented for prophylaxis. Transdermal scopolamine is a potential long-acting prophylactic antiemetic initially developed to prevent motion sickness but approved in 2001 by the US Food and Drug Administration (FDA) for the prevention of PONV. Scopolamine is a centrally acting anticholinergic agent and is designed as a patch placed behind the ear that will deliver 1.5 mg of scopolamine transdermally at a constant rate over 3 days. It has been shown to be effective in decreasing PONV for up to 24 hours after surgery. Several adverse events can occur including sedation, dry mouth, and blurred vision. In addition, mydriasis, usually ipsilateral to the side of patch application, can occur if direct contamination to the eye occurs from rubbing the eyes after manipulating the patch without hand washing. The diagnosis of scopolamine contamination can be confirmed by placing 0.5 to 1.0% pilocarpine hydrochloride in the affected eye. A dilated pupil from pharmacologic mydriasis will not constrict, whereas a dilated pupil from paralytic mydriasis will constrict. Misdiagnosis can result in an unnecessary and extensive workup for an intracranial etiology. The other options including dexamethasone, droperidol, metoclopramide, and ondansetron are effective antiemetic agents but have no anticholinergic profile.

2013 A 48-year-old woman is evaluated because of a unilateral fixed, dilated, and nonresponsive pupil 1 day after she underwent prophylactic mastectomy and immediate reconstruction with a superior gluteal artery perforator free flap. History includes marked postoperative nausea. Multimodal antiemetic prophylaxis therapy was implemented during the procedure. Which of the following is the most likely causal agent of the fixed and dilated pupil? A ) Intravenous dexamethasone B ) Intravenous droperidol C ) Intravenous ondansetron D ) Oral metoclopramide E ) Scopolamine patch

The correct response is Option E. The most appropriate option for this patient would be a right TRAM flap. The defect encompasses a composite defect of the left side of the sternum, including the left internal mammary artery, four contiguous ribs, and the entire left breast. The question specifically asks for soft-tissue thoracic reconstruction. The analysis of the defect yields a very large soft-tissue requirement. Given this, the most logical choice of flaps to provide this amount of soft tissue on a reliable pedicle would be a right (contralateral) TRAM flap. Furthermore, this flap could also be shaped to provide a breast mound versus all the other choices. A left latissimus muscle flap with skin graft is not the optimal choice as it would not provide sufficient soft-tissue coverage with its associated skin paddle without undue donor site morbidity in this particular patient with a large defect that spans parasternal and lateral chest wall. As a muscle-only flap with skin graft, however, it may be considered as a backup option in case of primary flap failure. An omental flap with skin graft is an option for reconstruction of this defect; however, it would not be the most appropriate option in this case as it would necessitate an otherwise unnecessary intra-abdominal procedure and would not reshape a breast mound. It has been shown, however, to be optimal in contaminated areas due to its associated lymphatic vessels and nodes associated with its pedicle, so in different circumstances, this may be a preferred choice. A left rectus turnover flap with skin graft would not be a good option for two reasons: the left internal mammary artery has been harvested, thereby compromising the superior epigastric vessel on which this flap would be based; and it would not cover the entirety of the defect. A reverse abdominoplasty flap would not be able to reach the full superior extent of the defect.

2013 A 48-year-old woman undergoes radical resection of the left breast, left hemisternectomy, four-rib resection, and visceral pleurectomy because of invasive inflammatory breast cancer extending through the thoracic wall into the mediastinum and chest. Examination of specimens obtained from intraoperative frozen section biopsies shows no residual disease. Postoperative adjuvant chemotherapy and radiation therapy to the area are planned. An intraoperative photograph of the 17 × 17-cm defect is shown. Which of the following is the most appropriate technique for soft-tissue reconstruction? A ) Left latissimus muscle flap with skin graft B ) Left rectus muscle turnover flap and skin graft C ) Omental flap and skin graft D ) Reverse abdominoplasty flap E) Right transverse rectus abdominis musculocutaneous (TRAM) flap

The correct response is Option E. Local flaps in the gluteal region are the first choice for reconstruction of sacral pressure sores. Various designs of gluteal fasciocutaneous and musculocutaneous flaps are commonly used. The gluteal fasciocutaneous V-Y advancement flap is a method that has been preferred for management of these ulcers. Use of this type of flap conserves the gluteus maximus muscle and does not preclude ambulation in a mobile patient. Fasciocutaneous flaps, which are less sensitive to ischemia and more resistant to pressure than muscle flaps, have high mechanical resistance. These flaps are especially useful in wounds of limited depth, as are many in the sacral area. Traditional wound closure of back defects under tension, with skin (split- or full-thickness) grafts or local random transposition flaps, is a poor choice for the patient with a complicated posterior trunk wound. These treatment options fail because of the poor tissue vascularity and the presence of a significant zone of injury that produced the wound breakdown in the first place. The majority of the defects encountered in the posterior trunk can be reconstructed with local flaps. In cases of irradiation of the spine, extensive trauma, or significant debridement of surrounding tissue, the patient may be left with poor local solutions for providing vascularized tissue to the wound. In these rare cases, free tissue transfer is used to provide coverage.

2013 A 50-year-old woman is scheduled to undergo debridement and reconstruction of a Stage IV sacral pressure sore. Which of the following is the most appropriate method of reconstruction? A ) Free tissue transfer B ) Full-thickness skin grafting C ) Inferior gluteus maximus island flap D ) Split-thickness skin grafting E ) V-Y fasciocutaneous flap

The correct response is Option B. The best choice for managing an ambulatory patient with a chronic pressure sore is using a posterior thigh fasciocutaneous rotation flap. Fasciocutaneous flaps offer an axial blood supply with durable coverage and minimal potential for a functional deformity. The flap more closely reconstructs the normal anatomic arrangement over bony prominences. Disadvantages include limited bulk for large ulcers that have a significant three-dimensionality. Musculocutaneous flaps offer excellent blood supply and bulky tissue and can be re-rotated like fasciocutaneous flaps. However, in an ambulatory patient they may cause functional impairment. The tensor fasciae latae flap is occasionally used to close the ischial ulcer. Unfortunately, the distal aspect of the flap that is used to reach the ischial region is usually too thin to offer adequate padding to be effective. Therefore, this flap is not the best option. Primary closure is associated with a high recurrence rate. A large dead space is usually present due to the absence of tissue, and the induration of adjacent areas makes them difficult to mobilize for a tension-free closure.

2013 A 53-year-old ambulatory man comes to the office because of a chronic pressure ulcer of the inferior portion of the left buttock that developed during a prolonged hospitalization 1 year ago. Repeated packing and dressing changes have not been successful in healing. Which of the following is the most appropriate surgical option? A ) Excision of ulcer bursa and primary closure B ) Reconstruction with a posterior thigh fasciocutaneous rotation flap C ) Reconstruction with a superiorly based gluteal musculocutaneous rotation flap D ) Reconstruction with a tensor fasciae latae island rotation flap E ) Reconstruction with a V-Y hamstring musculocutaneous advancement flap

The correct response is Option A. The patient described has an epidermal inclusion cyst or sebaceous cyst. Cysts can occur anywhere on the body, including the face, back, and chest. These cysts are benign and occur as a result of proliferation of epidermal cells within the dermis. They are usually well circumscribed by a cyst wall made of stratified squamous epithelium. They communicate with the surface through a small opening, which may contain a keratinous plug or blackhead. Epidermal inclusion cysts grow slowly and occasionally can become inflamed and infected. Manual squeezing of the cysts may produce white keratinous material, which is often foul-smelling. Treatment is excision of the cyst in its entirety with care to leave no epithelial remnants in the wound that could develop into a recurrent cyst. Wide local resection is performed for malignant skin lesions with appropriate negative margins that can be evaluated by frozen-section pathology.

2013 A 54-year-old man comes to the office because of a lesion on his back that has enlarged gradually for the past 2 years. He says it occasionally expresses white material when he squeezes it. Physical examination shows a mobile, firm, 2 × 1.5-cm nodule with a central opening and white plug on the mid back. No tenderness is noted on palpation. Which of the following is the most appropriate initial step in management? A ) Excision B ) Incision and drainage C ) Punch biopsy D ) Wide local resection

The correct response is Option A. Fentanyl is a short-acting, synthetic opioid that is significantly more potent than morphine. Fentanyl generally produces less nausea compared to morphine, but fentanyl may produce marked respiratory depression. An occasional opioid side effect is skeletal muscle rigidity, particularly if large doses are rapidly given. Common offending agents include fentanyl, sufentanil, and alfentanil. Rigidity may occur in the chest wall, abdomen, neck, and extremity. It may be so severe that ventilation is inhibited. Treatment is administration of a neuromuscular blocking agent. Propofol is a drug used for induction and maintenance of anesthesia, and can be used as a sedative. It is not considered an analgesic so other drugs like opioids are required to manage pain. Side effects include hypotension, apnea, and dystonia. Some patients report euphoria with propofol, which has occasionally led to its illicit use, as well as death. Midazolam (Versed) is a benzodiazepine that can be used to treat seizures, insomnia, and to produce sedation and amnesia. It has a relatively fast recovery time and high effectiveness. Disadvantages include drug interactions, tolerance, and withdrawal. Succinylcholine (suxamethonium chloride) is a depolarizing neuromuscular blocker that induces muscle relaxation and acts as a paralytic agent. Side effects include malignant hyperthermia, muscle pain, rhabdomyolysis, hyperkalemia, and cardiac arrhythmias. Patients with burns should not have succinylcholine because it can lead to rapid release of potassium by skeletal muscle that can result in cardiac arrest. Nitrous oxide gas is an inhaled anesthetic agent that can be used for general anesthesia. It can cause analgesia, euphoria, derealization, and depersonalization. The major hazards of nitrous oxide are the risk of asphyxiation and the dissociative affect it has on patients. In addition, there can be short-term decreases in mental performance, audiovisual ability, and manual dexterity. Long-term exposure can cause Vitamin B12 (cobalamin) deficiency, numbness, and reproductive side effects (in pregnant women).

2013 A 54-year-old woman with Stage IIIA right-sided breast cancer is scheduled to undergo right mastectomy and immediate reconstruction with a pedicled transverse rectus abdominis musculocutaneous (TRAM) flap. After induction of anesthesia, upon positioning the patient for surgery, she becomes extremely rigid, and the plastic surgeon is not able to move her arms. Which of the following drugs is most likely responsible? A ) Fentanyl B ) Midazolam C ) Nitrous oxide D ) Propofol E ) Succinylcholine

The correct response is Option B. The patient described has an occupation that exposes him to contaminated water and raw seafood. Infections from Mycobacterium marinum and Vibrio vulnificus are both possible. Mycobacterium marinum is the most common atypical mycobacterium seen in hand infections, often seen after penetration by aquatic equipment, colonized marine life, or contact with contaminated water. The most common deep infection is flexor tenosynovitis, and may present as a chronic tenosynovitis. Sporothrix schenckii is a fungus found in plants and soil. Inoculation with the pathogen results in a papule at the entry site, followed by development of lesions along the lymphatic chain. Suppurative flexor tenosynovitis typically presents with pain over the tendon sheath, semiflexed position of the involved digit, pain on passive extension, and symmetrical swelling of the finger. This classic presentation is commonly caused by pathogens such as Staphylococcus aureus or Eikenella corrodens (often seen in association with human bite injuries). In tenosynovitis infections involving atypical mycobacterium, however, there is absence of pain. Vibrio vulnificus is a species of gram-negative, motile, curved bacterium found in the coastal waters of the United States. Infections from V vulnificus may be caused by direct exposure of an open wound to warm seawater containing the organism or from handling raw seafood or marine wildlife. Infections caused by V vulnificus result in painful cellulitis that progresses rapidly and presents with marked local tissue swelling with hemorrhagic bullae. Bacteremia with systemic symptoms is commonly seen.

2013 A 55-year-old man who is a commercial fisherman comes to the office because of a 3-week history of a swollen index finger. The patient holds the finger in a flexed position. Extension of the digit is difficult but not painful. Surgical exploration shows nonpurulent fluid in the tendon sheath. Culture of the fluid is most likely to show which of the following pathogens? A ) Eikenella corrodens B ) Mycobacterium marinum C ) Sporothrix schencki D ) Staphylococcus aureus E ) Vibrio vulnificus

The correct response is Option E. Reconstruction of the vagina after oncologic resection can be challenging for plastic surgeons, as restoration of form and function must be considered. Restoration of form is often achievable by a variety of procedures, including the vertical rectus musculocutaneous flap, gracilis musculocutaneous flap, pudendal artery fasciocutaneous flap, skin grafting, and interpositional colonic grafts. Restoration of functional outcome, however, is controversial. Return of sexual activity across multiple studies shows wide variability from 31 to 100% (53.8% pooled data). A recent survey study found that 50% of responders were sexually active and were able to achieve orgasm after reconstruction. However, a majority of responders reported wishing that they had been given more information on the procedure before surgery. The most common long-term complication from total reconstruction of the vagina is stenosis with rates reported from 18 to 22%. This requires regular use of obturators for dilatation and this should be clearly discussed with patients preoperatively. Abdominal hernia, rectovaginal fistula, pelvic abscess, and small-bowel obstruction are less common complications.

2013 A 55-year-old woman with recurrent rectal cancer comes to the office for preoperative consultation for pelvic exenteration, which will include total vaginal resection and reconstruction with a vertical rectus musculocutaneous flap. Which of the following long-term complications is most likely in this patient? A ) Abdominal hernia B ) Pelvic abscess C ) Rectovaginal fistula D ) Small-bowel obstruction E ) Vaginal stenosis

The correct response is Option E. Several biologic dressings have been FDA-approved for the treatment of diabetic foot ulcers. Apligraf® (Organogenesis, Inc., Canton, MA) and OrCel® (Ortec International, New York) are bilayered constructs of bovine collagen with human keratinocytes and fibroblasts. Although it appears like normal skin, it does not take like a skin graft, but rather the viable cells in the construct release and stimulate growth factors and matrix proteins to encourage wound healing. Neo-dermis creation is the primary function of Integra TM (Integra LifeSciences Corp, Plainsboro NJ) which is an acellular construct composed of collagen and chondroitin-6-sulfate covered with a silicone top layer to prevent evaporative losses. It does not have any active cells or release growth factors or matrix proteins. Enzymatic degradation of a wound bed is not a function of biologic grafts, but is the main function of topical wound treatments like the collagenase Santyl® (Healthpoint Biotherapeutics, Fort Worth TX).

2013 A 56-year-old man with type 1 diabetes mellitus comes to the office because of a foot ulcer over the first metatarsal head without evidence of exposed bone or osteomyelitis. A bioengineered cellular bilayered skin substitute (Apligraf®) is considered for treatment. Which of the following is the primary mechanism of action of this skin substitute to stimulate wound healing in this patient's ulcer? A ) Creation of a moist environment through creation of a barrier B ) Enzymatic degradation of the wound bed C ) Inhibition of growth factor release D ) Integration of the graft and creation of neo-dermis E ) Release of matrix proteins

The correct response is Option E. Stages I and II pressure sores usually can be managed nonsurgically. Stage I pressure sores are non-blanchable erythema of intact skin and usually resolve after 1 hour. Stage II is partial-thickness skin loss presenting clinically as a blister, abrasion, or shallow crater. Stage III pressure sores are full-thickness tissue loss down to, but not through, fascia. Stage IV pressure sores are full-thickness loss with involvement of underlying muscle, bone, tendon, ligament, or joint capsule. A tensor fasciae latae flap would usually be the flap of choice for a Stage III or IV trochanteric pressure sore. A lumbosacral flap would usually be used for a Stage III or IV sacral decubitus ulcer. With reported recurrence rates up to 82%, cessation of smoking would be recommended before operating on a Stage III or IV pressure ulcer. Unless predisposing factors can be controlled, there is no reason to provide treatment to a clean Stage III or IV decubitus ulcer. Excision and closure has a high failure rate and would not be indicated for a Stage II pressure sore.

2013 A 60-year-old man with paraplegia who lives in an assisted care facility is evaluated because of a Stage II pressure sore of the right ischium. He spends each day sitting in a wheelchair. He smokes one pack of cigarettes daily. Which of the following is the most appropriate treatment? A ) Coverage with a lumbosacral flap B ) Coverage with a posterior hamstring musculocutaneous flap C ) Coverage with a tensor fasciae latae flap D ) Excision and direct closure E ) Placement of a hydrocolloid dressing

The correct response is Option E. The most appropriate first step in management is to obtain a ventilation/perfusion scan. The patient described presents with the clinical picture of a pulmonary embolus (PE). Normal chest X-ray study rules out an obvious pulmonary process like pneumonia, atelectasis, or pneumothorax. The right-sided strain pattern on electrocardiography points to a PE as well. Vitals are abnormal, but she is hemodynamically stable. CT angiography (CTA) would have been the investigation of choice; however, serum creatinine is abnormally high, which precludes giving her the high doses of contrast dye a CTA would require. Empirically starting her on anticoagulation is another management option, though one has to weigh the risk-benefit of full-anticoagulation therapy so soon after major surgery. However, subcutaneous enoxaparin therapy in a patient with renal insufficiency is not ideal. Instead, systemic heparin therapy would be the agent of choice because it is not dependent on renal excretion, has a much shorter half-life, and its therapy can easily be measured with activated partial thromboplastin time levels. Therefore, it can be titrated more reliably, and its anticoagulation effects can be shut off quickly if the need arises. A venous Doppler examination of the lower extremities will probably need to be done, but it is not the first step in management. Additionally, it does not diagnose a PE. An emergent cardiac catheterization is not indicated because this is not an acute myocardial ischemic event. A ventilation/perfusion scan is performed in situations where an angiogram (conventional or CTA) is not indicated or available. It is especially useful in patients with renal insufficiency because the perfusion portion does not utilize contrast that could negatively affect the kidneys, but rather uses a radioactive tracer. The ventilation/perfusion scan helps stratify the probability of a patient having a PE, and thus helps the clinician to choose the appropriate way to potentially treat the patient.

2013 A 60-year-old woman comes to the emergency department because of shortness of breath and right-sided chest pain 2 days after she underwent an uncomplicated abdominoplasty. Heart rate is 100 bpm, respiratory rate is 20/min, blood pressure is 110/60 mmHg, and oxygen saturation is 92% on room air. Chest x-ray study shows no abnormalities. Serum creatinine concentration is 2.5 mg/dL. Which of the following is the most appropriate first step in management? A ) CT angiography B ) Emergency cardiac catheterization C ) Enoxaparin therapy D ) Lower extremity venous Doppler E ) Ventilation/perfusion scanning

The correct response is Option C. The most appropriate treatment in this patient is thrombectomy to restore flow within the artery and vein and administration of intra-flap tissue plasminogen activator (tPA) to dissolve clot formed within the flap. Systemic urokinase will carry a high risk of bleeding from all surgical sites and is unlikely to dissolve clot in both the arterial and venous systems of the flap. Administration of intra-flap heparin will prevent further clot formation but will not dissolve the clot in the flap pedicle. Systemic dextran following thrombectomy might prevent clot formation and has mild thrombolytic properties, but it is not nearly as effective as tPA in lysing clot within the flap. Aspirin will potentially prevent further clot formation but has no thrombolytic properties and a slow onset of action.

2013 A 64-year-old man is evaluated 2 days after undergoing soft-tissue coverage of an open distal tibia fracture with a free rectus abdominis flap. On examination, the flap appears dark and swollen. Doppler signals are not present. The patient is brought to the operating room for reexploration, and thrombus is noted within the artery and vein. In addition to thrombectomy, which of the following is the most appropriate treatment to salvage this flap? A ) Aspirin B ) Intra-flap heparin C ) Intra-flap tissue plasminogen activator D ) Systemic dextran E ) Systemic urokinase

The correct response is Option B. The most appropriate next step in management is to initiate intravenous dexrazoxane. Recent data support the use of this agent in extravasation of anthracyclines (e.g., doxorubicin) as an antidote. It has been shown to decrease the frequency and severity of tissue injury. The mechanism by which dexrazoxane diminishes tissue damage is unknown. However, two mechanisms of action are hypothesized: 1) reduction of oxidative stress due to complexes of metal ions and anthracyclines by chelating metal ions; 2) and blockade of topoisomerase II poisons by catalytic inhibition of topoisomerase II. Patients receive treatment with dexrazoxane 1000 mg/m2 administered intravenously on days 1 and 2, and 500 mg/m2 on day 3. The infusion is given in the opposite arm of the extravasation site and is administered as soon as possible and no longer than 6 hours after the extravasation accident. Flushing the port is not indicated because the port may be malfunctioning and this action could extravasate the flushing agent. Adding more fluid to the area (extravasation of saline from the port, subcutaneous saline injection, etc.) will only add to the pressure on the overlying skin, injuring the skin further. Removal of the port is not indicated until it has been ascertained it is malfunctioning. Dimethyl sulfoxide (DMSO) is a known treatment option for anthracycline extravasations. However, it is used as a topical agent with or without cortisone, not as a subcutaneous injection. Additionally, it should not be used in patients who are receiving dexrazoxane because it has been shown to decrease the efficacy of the dexrazoxane therapy.

2013 A 65-year-old woman undergoes treatment for breast cancer with intravenous doxorubicin through a subcutaneous port on the left side of the chest. The patient complains of severe pain around the port 5 minutes after the treatment is initiated, and the infusion is discontinued. Examination shows marked swelling (8 × 6 cm) and erythema of the skin. Which of the following is the most appropriate next step in management? A ) Flushing of the port with a saline solution B ) Intravenous administration of dexrazoxane C ) Removal of the port D ) Subcutaneous injection of dimethyl sulfoxide E ) Subcutaneous injection of saline solution

The correct response is Option D. The patient has cutaneous angiosarcoma of the scalp that requires wide local excision with reconstruction. Angiosarcoma is a rare (2% of all soft-tissue sarcomas) but highly aggressive tumor that is most commonly found in the face and scalp in older Caucasian men. Fifty percent of all cutaneous angiosarcomas are found in the head and neck, and they are also commonly found in the breast and extremities, particularly in patients with a history of lymphedema or radiation therapy. It appears initially as a purple plaque which is often confused with a bruise or cellulitis, which can delay diagnosis. It is frequently multifocal, and local recurrences are common, so wide local excision is recommended, frequently combined with radiation therapy. Preoperative serial punch biopsies of the surrounding tissue can help in determining appropriate resection margins. The tumor can microscopically infiltrate normal tissues for some distance away from the obvious tumor, so wide local excision is necessary. Observation or embolization is not indicated for an aggressive cancer like angiosarcoma. Although radiation is frequently used postoperatively, its role as a neoadjuvant therapy is not well established. There is no current standard of care for chemotherapy in angiosarcoma patients, and most trials of chemotherapy have shown no survival benefit. There may be a role for taxanes in the treatment in metastatic angiosarcoma.

2013 A 67-year-old Caucasian man comes to the office because of a 3-month history of a lesion on his forehead that he says has enlarged gradually and sometimes bleeds. Examination shows a nontender, soft, flat, purple lesion on the anterior hairline that is 4 cm in its largest dimension. Results of punch biopsy are consistent with angiosarcoma. Which of the following is the most appropriate treatment? A ) High-dose chemotherapy B ) Radiation therapy C ) Vascular embolization D ) Wide local excision E) Observation only

The correct response is Option C. The most appropriate next step in management is elevation of the extremity. Elevation facilitates venous return of the infiltrate, decreases swelling, and minimizes the risk of skin necrosis. The vasoconstrictive effects of epinephrine only last for 60 to 90 minutes; much longer ischemia times are necessary to cause skin necrosis. For example, amputated digits may be successfully replanted after 33 hours of warm ischemia time. The use of warm or cold compresses on an infiltration site is controversial. Heat may theoretically stimulate the evacuation of the infiltrate through vasodilation and increased blood flow. Ice may theoretically limit the inflammatory reaction as well as the diffusion of the infiltrated substance by causing vasoconstriction. However, warm or cold compresses may worsen tissue damage. Heat can cause thermal injury, and ice can cause ischemia from vasoconstriction, resulting in a "second hit" at the extravasation site. A partial-thickness skin injury may be converted to a full-thickness wound. Although phentolamine, an alpha-adrenergic antagonist, has been used to treat epinephrine infiltrations, the added volume of phentolamine can worsen the injury. Injection of an antidote adds more fluid to the subcutis and may increase the risk of pressure necrosis. In addition, because epinephrine causes vasoconstriction for only 60 to 90 minutes, its effects have usually worn off by the time the extravasation is noted, the patient is evaluated by a physician, the phentolamine is ordered, and the antidote is obtained from the pharmacy. Saline flush-out, using stab incisions through which saline is flushed, may be helpful for the infiltration of chemotherapeutic agents, but it requires several incisions and would be considered after elevation of the extremity.

2013 A 7-year-old boy with a history of anaphylactic reaction to bee stings is brought to the office 45 minutes after epinephrine was mistakenly injected to the tip of the index finger from his automatic injection device. Physical examination of the finger shows swelling, tenderness, and decreased capillary refill. Which of the following is the most appropriate next step in management? A ) Apply a cold compress B ) Apply a warm compress C ) Elevate the extremity D ) Perform a stab incision and saline irrigation E ) Subcutaneously administer an antidote

The correct response is Option A. This patient's presentation is consistent with late radiation tissue injury (LRTI), which occurs in 5 to 15% of long-term cancer survivors who have received radiation and can occur months to years (even decades) after treatment. It can vary significantly with age, dose, and site of treatment. Characteristics include progressive tissue deterioration secondary to decreased vascularity followed by replacement of normal soft-tissue architecture by dense fibrotic tissue until there is insufficient oxygen delivery to sustain normal function. When LTRI of the chest wall progresses to skin breakdown and ulceration, as has happened in this patient, a biopsy is always required to rule out recurrence of the primary tumor or a radiation-induced squamous cell carcinoma or soft-tissue sarcoma. Hyperbaric oxygen therapy has been proposed as a treatment modality that can improve tissue quality and prevent tissue breakdown in irradiated areas. While most data on this relate to treatment of osteoradionecrosis of the mandible, there have been some reports of its use on the chest wall after irradiation due to breast cancer. However, the question specifically asks for the most appropriate next step, which would be biopsy rather than hyperbaric oxygen therapy. The principles of management once malignancy has been ruled out include debridement of necrotic tissues (including ribs) and reconstruction with well-vascularized flaps. In this case, a left latissimus muscle flap and skin graft was used for reconstruction after aggressive debridement. No alloplastic material was placed or thoracic cage reconstruction performed. This is common in these types of patients as excessive fibrosis caused by the radiation to the chest wall prevents loss of respiratory efficiency through paradoxical motion. CT scan or MRI may be useful in evaluating the size, extent, and nature of the problem, but does not take priority over tissue biopsy. The redness that is visualized on the patient's skin in the case is related to post-radiation changes and telangiectasias, which are common in LTRI, and is not infection. Therefore, antibiotics would be unnecessary.

2013 A 74-year-old woman comes to the office because of a 2-month history of increasing soreness over the left lateral chest wall. History includes bilateral radical mastectomy and adjuvant radiation to the left chest 30 years ago because of left-sided breast cancer. On examination, a small, nonhealing 1 × 2-cm wound is visible in the subaxillary region within an area of dense fibrotic skin. A photograph is shown. Which of the following is the most appropriate next step? A ) Biopsy of the nonhealing wound B ) Coverage with a left latissimus muscle flap and skin grafting C ) Hyperbaric oxygen treatment D ) MRI of the chest wall E ) Oral administration of antibiotics for 7 days

The correct response is Option B. Aging skin can be identified histologically by an increase in the flattening of the dermal-epidermal junction. This results in a decreased area of contact between these two surfaces and predisposes older individuals to separation at this junction. The number of Langerhans cells, fraction of Type III collagen, and amount of glycosaminoglycan ground substance are all decreased with chronological aging. Other changes include disorganization of the major extracellular matrix components, such as collagen and other elastic fibers. The number of available keratinizing cells is also decreased with aging. It is hypothesized that this contributes to the problems associated with increased dry skin in the elderly.

2013 A 75-year-old woman undergoes rhytidectomy for facial rejuvenation. When compared with the skin of a 20-year-old woman, this patient's results are most likely to show an increase in which of the following? A ) Amount of glycosaminoglycan ground substance B ) Flattening of the dermal-epidermal junction C ) Fraction of Type III collagen D ) Number of keratinizing cells E ) Number of Langerhans cells

The correct response is Option B. The defect is a full-thickness skin defect with exposed subcutaneous fat. A local flap or skin grafting are possible reconstructive options. If skin grafting were to be performed, then a full-thickness skin graft would minimize the amount of long-term contracture (secondary contracture). A skin graft can be either split-thickness or full-thickness. Full-thickness skin grafts contain both the epidermis and the dermis and would have less long-term contracture. Defatting of the skin graft is important in the case of a full-thickness graft to optimize graft "take" in the early postoperative period. Various donor sites are available for skin grafting. Cultured epidermal autografts (CEAs) are useful when there are limited areas on the body to be used as skin graft donor sites, such as in a massive burn patient. However, CEAs are costly, and the resulting skin is often very thin and fragile. Therefore, judicious use of CEAs is warranted. Split-thickness grafts can be harvested with a mechanical dermatome or a free hand with a scalpel. Split-thickness skin grafts do not include the entire thickness of the dermis and have less initial contracture at the time of harvest (primary contracture) but are expected to have long-term contracture (secondary contracture). Harvesting with a mechanical dermatome is useful for obtaining split-thickness skin grafts. Typical thicknesses may range from 8/1000 to 14/1000 of an inch. Meshing allows for a broader surface area to be covered by a skin graft but will lead to greater contracture within the open interstices of the graft. Of note for deeper wounds extending below the superficial musculoaponeurotic system, evaluation of facial motor nerve function and status of the parotid duct may be important considerations.

2013 A 77-year-old woman undergoes excision of a basal cell carcinoma of the mid cheek. Physical examination shows a 4 × 4-cm circular skin defect with exposed subcutaneous fat. Reconstruction with skin grafting is planned. Use of which of the following is most appropriate to minimize long-term graft contracture? A ) Cultured epidermal autografting B ) Full-thickness skin grafting C ) Split-thickness skin grafting with meshing D ) Split-thickness skin grafting with no meshing

The correct response is Option D. The W-plasty was first described by Borges in 1953. The repair involves a regular pattern of interdigitating triangular advancement flaps. This repair allows for the scar to have an accordion-like effect secondary to the broken line configuration. The W-plasty and geometric broken line closure (GBLC) techniques are commonly used for scar revisions and are best indicated for scars that cross the relaxed skin tension lines (RSTL) to redirect portions of the scar. Because of the flexibility these scars have, they are also indicated over convex or concave surfaces. Compared with linear closure, the W-plasty will have decreased contracture of the wound because of the interrupted orientation of the scar. Even though a portion of the final closure will go against the RSTLs, W-plasties can be designed to orient a significant portion along the RSTLs. Because of the many triangular flaps made and repaired, it takes significantly longer to perform a W-plasty repair. The main disadvantage of the W-plasty and GBLC procedures is the need to remove a significant amount of healthy tissue on either side of the scar to be revised, which can lead to increased wound tension and the need for significant undermining.

2013 A 9-year-old girl is evaluated for scar revision after sustaining a laceration of the left cheek that was repaired in the emergency department 2 years ago. Examination shows a 4 × 0.7-cm scar that is pale, flat, and wide in appearance. It has an oblique orientation between the oral commissure and zygomatic arch. W-plasty is considered. Which of the following is the most likely outcome of the W-plasty when compared with linear closure in this patient? A ) Decreased initial wound tension B ) Decreased procedural time C ) Increased contracture of the wound D ) Increased removal of healthy tissue

The correct response is Option E. In the patient described with an intermediate thickness tumor (1-4 mm) and clinically negative neck, the most appropriate treatment is wide excision with 2-cm margins (shown) and a concurrent sentinel lymph node biopsy. The defect may be reconstructed with a local flap or skin graft depending on patient and surgeon preference. In Stage I and II melanomas (localized disease T1-4, and no evidence of regional lymphadenopathy, N0), Breslow tumor thickness is the most important predictor of local recurrence, regional/distant metastases, and overall survival. Current recommended excisional margins are 0.5 to 1.0 cm for melanoma in situ/lentigo maligna. For invasive lesions less than 1 mm thick, a 1-cm margin is adequate. Lesions with Breslow thickness of 1 to 2 mm should be resected with a 1- to 2-cm margin, using closer to 2 cm when the anatomical area is more forgiving (scalp/trunk), the thickness approaches 2 mm, or the lesion displays more aggressive histopathologic features, such as ulceration, lymphovascular invasion, tumor regression, or a mitotic index greater than one figure per high-power field. Lesions between 2 to 4 mm are adequately treated with 2-cm margins. Balch, et al., have shown that 2-cm margins are safe for lesions of intermediate thickness with equivalent survival rates, less use of skin grafting, shorter hospital stays, and lower medical costs when compared with more aggressive peripheral margins. When possible, 3-cm margins should be used for tumors greater than 4 mm thick because of their high local recurrence rate (greater than 20%). Patients with intermediate-thickness melanomas have a 20 to 25% chance of microscopic regional disease. Before the advent of sentinel lymph node biopsy, elective lymphadenectomy (ELD) was advocated for patients with intermediate-thickness melanoma because of a significant improvement in overall survival at 10 years. The primary disadvantage of routine ELD, however, was that approximately 75 to 80% of patients underwent an unnecessary procedure. The status of the sentinel lymph node is a powerful predictor of survival in melanoma because it identifies (1) those patients with a relatively favorable prognosis requiring no further therapy; and (2) high-risk patients who might benefit from additional surgery (completion lymphadenectomy) and interferon. Current indications for sentinel lymph node biopsy include all of the following: male patients with truncal melanoma less than 0.76 mm thick (9% incidence of nodal metastasis); all patients with melanoma thickness 0.76 to 1.0 mm (5% incidence of nodal metastasis); male patients with "thin" melanomas with aggressive features (Clark Level III or greater, ulcerated, evidence of regression, or axial location; these patients have a 10% risk of metastasis); and all melanomas greater than 1 mm in thickness. In the scenario described, 1-cm margins would be too narrow. A 3-cm margin is excessive for the lesion described. As noted, elective lymphadenectomy has been replaced with sentinel lymph node biopsy in a clinically negative neck. Finally, sentinel lymph node biopsy should be performed at the time of the primary tumor resection if possible due to variations in the lymphatic drainage that may occur after wide excision, skin grafting, or flap closure.

2013 An 88-year-old woman comes to the office because of the 2-cm pigmented lesion on the vertex of the scalp shown. There is no evidence of cervical or suboccipital lymphadenopathy. Examination of a specimen obtained on punch biopsy shows a Breslow thickness of 2.1 mm, Clark Level IV, two mitotic figures per high-power field, and no evidence of ulceration. Which of the following is the most appropriate management? A ) Excision with 1-cm margins and bilateral cervical lymphadenectomy B ) Excision with 2-cm margins and delayed sentinel lymph node biopsy C ) Excision with 3-cm margins and bilateral cervical lymphadenectomy D ) Preoperative lymphoscintigraphy, excision with 1-cm margins, and immediate sentinel lymph node biopsy E ) Preoperative lymphoscintigraphy, excision with 2-cm margins, and immediate sentinel lymph node biopsy

The correct response is Option D. Lentigo maligna is melanoma in situ that primarily occurs in elderly patients with a history of extensive sun exposure. It represents 4 to 15% of all melanomas and is slow-growing in a radial phase, but can progress to lentigo maligna melanoma with invasion and metastatic potential. Surgical resection remains the standard of care for treatment of lentigo maligna. In 1992, the National Institutes of Health Consensus Conference on Melanoma recommended a 5-mm margin for excision of lentigo maligna. However, the use of 5-mm margins has been associated with recurrence rates of 8 to 20%. In 2008, the National Cancer Comprehensive Network released guidelines indicating that 5-mm margins may be inadequate for treatment of lentigo maligna. The use of a staged excision technique has shown that 10-mm margins or greater were required in a majority of patients and resulted in low (1.7%) recurrence rates at 2 years. Nonsurgical modalities have been investigated in the treatment of this lesion, as they tend to occur in elderly patients who may not be surgical candidates. Nonsurgical treatments are associated with recurrence rates of 20 to 100%, with laser ablation associated with the highest recurrence rates. Topical imiquimod has shown promise, but data is limited, and long-term cure rates are unknown at this point.

2013 An 89-year-old man comes to the office because of a 2-year history of a pigmented lesion of the left cheek. The patient has an extensive history of sun exposure but no history of skin malignancy. On examination, the lesion is flat and light brown with irregular borders, but has no nodularity or ulceration. Examination of a specimen obtained on punch biopsy shows lentigo maligna. Which of the following is the most appropriate treatment? A ) Cryotherapy with liquid nitrogen B ) External beam radiation C ) Laser ablation D ) Resection with 1-cm margins E ) Topical treatment with imiquimod

The correct response is Option C. The most appropriate reconstructive option is to place a subcutaneous expander on the left side and perform a reduction of the right breast. This patient is still going through puberty and has a few years of growth left. However, her breast asymmetry is severe, symptomatic, and negatively impacting her life. Additionally, she has a good social support system with parents who are actively engaged in her well-being. Surgery that can potentially have permanent changes in a person's physical anatomy (e.g., scars), and potentially negative consequences (e.g., inability to breast-feed) must be performed with very detailed and complete informed consent, especially in minors. On the other hand, doing nothing could also lead to potentially negative social, emotional, psychological, and physical consequences. Therefore, if the patient and parents understand the risks and benefits, and are fully informed, surgical treatment is warranted in situations such as the one described. In this situation, simply placing an expander on the left side and not addressing the symptomatic right breast does not take care of all of her issues. A reduction of the right breast will not only help with her symptoms of breast hypertrophy, but will also enable a more symmetric reconstruction. Using a prosthesis to mimic an E cup breast is not ideal and will not succeed. Reducing the breast to about a C cup will allow the surgeon a realistic chance to reconstruct the left side to mimic the right breast. The expansion process on the left side would then continue until volume symmetry is attained compared with the right side. Through her growing years, the expansion process could continue. When she stabilizes in terms of her breast growth, the expander can then be substituted with a permanent prosthesis. A contralateral further reduction/lift can also be performed as needed. An ipsilateral latissimus muscle flap can also be performed simultaneously to recreate the anterior axillary fold or fill the infraclavicular space if grossly concave. However, the latissimus muscle flap in and of itself does not have the amount of tissue needed for larger breast reconstructions, as in this case, and needs a simultaneous expander placed under it for that reason.

2013 An otherwise healthy 14-year-old girl with Poland syndrome is brought to the office because of breast asymmetry and severe symptoms of right macromastia. She wears a size E cup brassiere and needs to fill the left side to attain symmetry in clothes. On examination, she has amazia and an absent sternal head of the pectoralis major muscle on the left side and macromastia of the right breast. Which of the following is the most appropriate reconstructive option for this patient? A ) Left-sided breast reconstruction with a latissimus dorsi flap and right-sided reduction mammaplasty B ) Left-sided breast reconstruction with a latissimus dorsi flap only C ) Placement of a subcutaneous left-sided tissue expander and right-sided reduction mammaplasty D ) No surgery until after puberty

The correct response is Option E. The most appropriate solution to infuse around the lesion to decrease intraoperative blood loss is 20 mL of 1:200,000 epinephrine. Although the maximum dose of subcutaneous epinephrine is unknown in a healthy child, large amounts have been shown to be safe in patients undergoing liposuction. The most conservative estimate for the amount of epinephrine (1:200,000) that can be safely injected in this child is 30 mL (3 mL/kg) every 10 minutes. Because this estimate was used in the past when halothane anesthesia was being administered (halothane lowered the arrhythmogenic threshold to epinephrine), greater volumes could likely be infused because other inhalational anesthetics are now used. Twenty mL of 1% lidocaine or 0.25% bupivacaine with epinephrine cannot be given to this 22-lb (10-kg) child because it exceeds the maximum dose. The maximum dose of lidocaine with epinephrine that can be administered is 7 mg/kg; because the concentration of 1% lidocaine is 10 mg/mL, only 7 mL of this solution could be injected. The maximum dose of bupivacaine with epinephrine that can be given is 3 mg/kg; because the concentration of 0.25% bupivacaine is 2.5 mg/mL, only 12 mL of this solution could be administered.

2013 An otherwise healthy 22-lb (10-kg), 2-year-old boy undergoes extirpation of a 5 × 5-cm arteriovenous malformation of the face. To decrease intraoperative blood loss, infusion of which of the following solutions around the lesion is most appropriate? A ) 20 mL of 0.25% bupivacaine with 1:100,000 epinephrine B ) 20 mL of 0.25% bupivacaine with 1:200,000 epinephrine C ) 20 mL of 1.0% lidocaine with 1:100,000 epinephrine D ) 20 mL of 1.0% lidocaine with 1:200,000 epinephrine E ) 20 mL of 1:200,000 epinephrine

The correct response is Option C. The most appropriate prophylactic antibiotic to initiate in this patient is ceftriaxone. Leech therapy is associated with Aeromonas species infections with incidences quoted in the literature from 2.4 to 36.2%. The most common clinical presentation of Aeromonas infection in humans is of cellulitis, often with a foul odor, complicated by subcutaneous abscess formation. In severe cases, extensive tissue loss and septicemia have been reported. Of most concern to microsurgeons is the ability of Aeromonas to invade the walls of blood vessels with resultant vasculitis, thrombosis, and hemorrhagic necrosis. Aeromonas species produce beta-lactamase, so penicillins and first-generation cephalosporins, like cefazolin, are ineffective. High levels of resistance to tetracyclines and amoxicillin-clavulanate (Augmentin) have also been observed. These organisms are usually sensitive to second- and third-generation cephalosporins, aminoglycosides, chloramphenicol, fluoroquinolones, and trimethoprim. The use of fluoroquinolones in children has been limited because of the potential of these agents to induce arthropathy in juvenile animals and to potentiate development of bacterial resistance. Fluoroquinolone use should be restricted to situations in which there is no safe and effective alternative to treat an infection caused by multidrug-resistant bacteria or to provide oral therapy when parenteral therapy is not feasible and no other effective oral agent is available. Doxycycline is contraindicated in children younger than age 8 years, as it may cause permanent discoloration and altered development of teeth.

2013 An otherwise healthy 5-year-old boy is evaluated because of cyanosis and swelling of the thumb 1 day after he underwent revascularization of the right thumb after a partial amputation injury. Preoperatively, the dorsal skin was intact, and venous anastomosis was not indicated. Which of the following prophylactic antibiotics is most appropriate to administer before initiating leech therapy? A ) Ampicillin B ) Cefazolin C ) Ceftriaxone D ) Ciprofloxacin E ) Doxycycline

The correct response is Option E. The most appropriate option for this patient is to perform a replantation with the use of vein grafts to the snuffbox for arterial inflow. Roping injuries are a combination of both an amputation as well as an avulsion, resulting in a significant zone of injury to all structures, but especially to the vessels, which sometimes have up to 4 cm of involvement with bruising of the intima and adventitia and separation of the media from the vessel walls. This zone of injury can result in anastomotic failure and digital demise if not recognized both by the mechanism of injury as well as careful examination of the structures under the operating microscope during attempted repair. Primary anastomosis in these particular injuries is most often not possible; therefore, the use of vein grafts after resection of the involved segment is an optimal strategy. The use of an end-to-side anastomosis into the radial artery at the snuffbox has been described with good success, and would be the preferred choice in this situation. A completion amputation of the thumb at this level (just distal to the metacarpophalangeal joint) would sacrifice needed thumb length and compromise ultimate function. As the thumb is reported to be responsible for up to 40% of hand function, this would represent a significant impact to the patient. It is difficult to repair the FPL tendon once avulsed from the musculotendinous junction, and many authors advocate for FPL resection rather than attempted repair. As the carpometacarpal joint of the thumb is intact, it will afford good range of motion of the thumb in all directions, even if the thumb serves merely as a "post" without ability to flex at the interphalangeal joint. Nonvascularized iliac crest bone grafting with a reverse radial forearm coverage is a surgical option but would not be the primary option in this acute amputation with an intact amputated part eligible for replantation, nor would it be preferred over staged toe-to-thumb transfer for better restoration of function and appearance. A toe-to-thumb transfer would be a good option in this patient if the thumb could not be replanted due to extensive distal injury, distal contamination, or loss of the digit during injury. In this particular case, none of those criteria are met.

2013 An otherwise healthy 50-year-old right-hand-dominant rodeo cowboy is brought to the emergency department immediately after he sustained a roping injury to the nondominant thumb. On examination, the digit is completely separated from the remaining hand, and is cool and pale. The flexor pollicis longus (FPL) tendon is attached to the amputated part, which has been avulsed from its musculotendinous junction. A photograph is shown. Which of the following is the most appropriate operative management? A ) Completion amputation and wound closure B ) Immediate toe-to-thumb transfer C ) Nonvascularized bone grafting and a reverse radial forearm flap D ) Tendon repair into the FPL muscle belly E ) Replantation with vein grafting

The correct response is Option D. This patient is at highest risk for deep venous thrombosis and she will require combination therapy of compression stockings and chemical prophylaxis. Perhaps the most well-regarded set of guidelines in this matter comes from the American College of Chest Physicians. These guidelines provide treatment recommendations based on a patient's risk classification. Caprini model offers a very user-friendly method of calculating patient risk factors and categorizing them into low, moderate, high, and highest risks. The patient in question has risk factors (age, malignancy, and major surgery) that would put her in the highest risk category. In the highest risk category, prophylaxis will require combination therapy or warfarin.

2013 An otherwise healthy 67-year-old woman with advanced breast cancer is scheduled to undergo mastectomy and immediate reconstruction with a free transverse rectus abdominis musculocutaneous (TRAM) flap. BMI is 35 kg/m2. Which of the following is most appropriate for deep venous thrombosis prophylaxis? A ) Aspirin and intermittent pneumatic compression stockings B ) Elastic compression stockings only C ) Intermittent pneumatic compression stockings only D ) Low-molecular-weight heparin and intermittent pneumatic compression stockings E ) Positioning and early ambulation

The correct response is Option E. Parry-Romberg syndrome can be associated with underlying neurologic disease. The symptoms generally first manifest in the first or second decade of life and can involve bone, cartilage, fat, or skin. Free tissue transfer performed once the disease has stabilized has been shown to be a highly satisfactory correction. However, in recent years, fat grafting has become a more utilized option. Although more procedures are required, it can begin in younger patients even if the disease is still active. It also has demonstrated high patient satisfaction. Hyaluronic acid dermal fillers are not FDA-approved in children and are unlikely to provide long-term correction. Custom prostheses or calcium phosphate ceramics would not reconstruct with like tissue; one would be less likely to have a stable, soft aesthetically pleasing result.

2013 An otherwise healthy 7-year-old boy is brought to the office because of a 4-year history of progressive soft-tissue atrophy of the left forehead with coup de sabre. Examination shows skin atrophy, discoloration, and alopecia at the hairline in the V1 distribution. Which of the following is the most appropriate method to correct the deformity? A ) Forehead contouring with a calcium phosphate ceramic B ) Free tissue transfer C ) Implantation of a custom silicone prosthesis D ) Injection of hyaluronic dermal filler E ) Serial fat grafting

The correct response is Option C. The Health Insurance Portability and Accountability Act of 1996 (HIPAA) was enacted by the US Congress in 1996. Title I of HIPAA protects health insurance coverage for workers and their families when they change or lose their jobs. Title II of HIPAA, known as the Administrative Simplification (AS) provisions, requires the establishment of national standards for electronic health care transactions and national identifiers for providers, health insurance plans, and employers. The AS provisions also address the security and privacy of health data. They allow the protection of individuals' health information, while also allowing communication between parties involved with patient care. The privacy rule outlines six permitted disclosures of individual health information including the following: per request of the patient, for treatment or payment, to individuals identified by the patient, incidental disclosure, limited data set with removal of individual identifiers, and public interest such as organ donation. Identifiable health information is treated as patient property. Portable media devices and smartphones also have the ability to store data. Digital pictures and videos taken by cell phones are commonplace, and technology exists that can directly download to the Internet. Breach in HIPAA compliance from this new technology is viewed with increasing scrutiny, especially when patient features are identifiable. In addition, personal digital assistants (PDAs), cell phones, or other devices with appropriate patient lists, properly consented pictures, or other data should be password-protected and data properly disposed, destroyed, or used. Computer hard drives must be demagnetized, overwritten, and/or physically destroyed when disposed. The Emergency Medical Treatment and Active Labor Act (EMTALA) is a US Act of Congress passed in 1986 as part of the Consolidated Omnibus Budget Reconciliation Act (COBRA). It requires hospitals to provide care to anyone needing emergency health care treatment regardless of citizenship, legal status, or ability to pay. The Consolidated Omnibus Budget Reconciliation Act of 1985 (or COBRA) is a law passed by the US Congress that mandates an insurance program giving some employees the ability to continue health insurance coverage after leaving employment. The National Health Security Strategy (NHSS) is a strategic plan developed by the US Department of Health and Human Services that is intended to minimize the health consequences associated with significant health incidents. The NHSS is a provision of the Pandemic and All Hazards Preparedness Act (PAHPA). The purpose of the NHSS is to ensure that the nation is prepared for health threats or incidents. The Patient Protection and Affordable Care Act (PPACA) is a US federal statute signed into law in 2010. PPACA reforms certain aspects of the private health insurance industry and public health insurance programs, increases insurance coverage of pre-existing conditions, and expands access to insurance to over 30 million Americans.

2013 During a procedure with general anesthesia, a 26-year-old male intern is told that the patient in the operating room next door has a very interesting tattoo on her lower abdomen. He takes a picture of the tattoo with his cell phone. This action is a violation of which of the following health care regulations? A ) Consolidated Omnibus Budget Reconciliation Act B ) Emergency Medical Treatment and Active Labor Act C ) Health Insurance Portability and Accountability Act D ) National Health Security Strategy E ) Patient Protection and Affordable Care Act

The correct response is Option E. The most appropriate first step in management is sclerotherapy. First-line intervention for a large symptomatic venous malformation is sclerotherapy. Sclerotherapy is the injection of an inflammatory substance into a lesion which causes endothelial damage, fibrosis, and shrinkage of the malformation. Sclerotherapy is more effective and less morbid than resection. Propranolol and corticosteroids are treatment options for a problematic infantile hemangioma, but have no efficacy for vascular malformations. Embolization is first-line intervention for an arteriovenous malformation, and is not a treatment option for venous malformation. Resection is second-line therapy for a large problematic venous malformation. Extirpation can cause significant morbidity (i.e., bleeding, nerve injury, infection, wound breakdown). In addition, excision leaves a cutaneous scar and recurrence is common because a venous malformation can rarely be completely removed.

2014 A 12-year-old girl has a 7 × 7-cm venous malformation of the thigh that is slowly enlarging and causing pain. The lesion involves the skin, subcutaneous tissue, and muscle. Which of the following is the most appropriate first step in management? A ) Corticosteroid therapy B ) Embolization of the lesion C ) Propranolol therapy D ) Resection of the lesion E ) Sclerotherapy

The correct response is Option E. This patient has the acquired form of nevus of Ota, also known as nevus fuscoceruleus ophthalmomaxillaris or oculodermal melanocytosis, a dermal melanocytic hamartoma that demonstrates bluish hyperpigmentation along the ophthalmic and maxillary divisions of the trigeminal nerve. The failure of complete embryonic migration of melanocytes from the neural crest to the epidermis results in dermal nesting with the resultant dermal melanin causing the Tyndall effect. This disorder primarily affects darker-pigmented individuals and is more prevalent in females. It has a bimodal age incidence, with a peak at 1 year of age and a second around puberty. The lesion tends to become increasingly prominent with age, puberty, and postmenopausal state. Most patients have no family history. Ophthalmologic examination is recommended because of a reported 10% association with ipsilateral glaucoma. Malignant degeneration to melanoma occurs in approximately 4% of reported cases and is more frequent in lighter-skinned individuals. Diagnosis is mainly clinical with confirmatory biopsy indicated when the diagnosis is uncertain or in rapidly expanding or nodular lesions suggestive of malignancy. The most effective treatment option is laser therapy, particularly with a Q-switched laser with ruby (694 nm), alexandrite (755 nm), or neodymium: yttrium-aluminum-garnet (1064 nm). The wavelength, pulse duration, and energy densities inherent in the Q-switched laser provide the desired parameters for melanin photothermolysis. Dyspigmentation is a possible complication, although it is mostly transient. Before the advent of laser therapy, treatment options were suboptimal. Makeup or camouflage therapy offered only temporary improvement. Dermabrasion followed by cryotherapy had the potential for dermal scarring and atrophy and was ineffective for those lesions with deep dermal melanocytes. Surgical excision options were also associated with scarring. Mohs micrographic excision has not been described for excision of these lesions.

2014 A 14-year-old girl with Fitzpatrick Type V skin comes to the office for evaluation of a nevus on the right side of the face. Physical examination shows a macular, bluish grey, irregular area of hyperpigmentation involving the right infrapalpebral region, nasolabial fold, and zygomatic region. Pigmentation of the right sclera is noted. Which of the following is the most appropriate treatment for this lesion? A ) Camouflage therapy B ) Cryotherapy C ) Dermabrasion D ) Mohs micrographic excision E ) Q-switched ruby laser

The correct response is Option E. Staphylococcus is still the most common organism in hand infections. The most common in felons is Staphylococcus aureus. Methicillin-resistant Staphylococcus aureus community-acquired (MRSA-CA) infections are now the most predominant strain in hand infections, comprising 60% of Staphylococcus aureus infections. Pasteurella multocida should be considered with most animal bites, although it is most common with cat bites. Eikenella corrodens is associated with human bites. There is no history of bites in this case. Listeria monocytogenes has been reported in flexor tenosynovitis in immunocompromised patients. Candida albicans is usually associated with chronic paronychia.

2014 A 15-year-old girl comes to the office because of a 1-day history of infection of the right index finger. Physical examination shows the tip of the finger is tender and swollen over the pulp. There is no history of trauma. Which of the following organisms is the most likely cause of this patient's condition? A ) Candida albicans B ) Eikenella corrodens C ) Listeria monocytogenes D ) Pasteurella multocida E ) Staphyloccus aureus

The correct response is Option C. The most appropriate next step in management is to perform the abdominoplasty without parental consent. Because the patient is married and has a child, she is an emancipated minor and is legally able to consent to medical procedures. The criteria for an emancipated minor generally include: marriage, military service, financial independence, living arrangements apart from the parents, and parenthood. Almost all states allow patients age 18 years or older to give their own consent for a medical procedure; non-emancipated patients younger than age 18 years require the consent of one parent.

2014 A 17-year-old girl comes to the office for evaluation for an abdominoplasty 1 year after the delivery of her first child. Following evaluation, the patient is deemed a good surgical candidate. The patient is married, but she came to the consultation alone. Which of the following is the most appropriate next step? A ) Obtain consent from her husband B ) Obtain consent from one parent C ) Obtain consent from the patient D ) Re-evaluate the patient at 18 years of age E ) Re-evaluate the patient at 21 years of age

The correct response is Option C. The demographic features of body dysmorphic disorder include an onset during late adolescence, and it appears to affect men and women with equal frequency. The clinical features of body dysmorphic disorder most frequently include preoccupation with the appearance of the skin, hair, and nose, although any body part can be a source of concern. Although several psychiatric comorbidities have been associated with body dysmorphic disorder, depression is the most common. In one study, over 75% of patients with body dysmorphic disorder had a lifetime history of major depression, and over half met criteria for current major depression. The remaining psychiatric disorders listed also occur with body dysmorphic disorder, but in lower frequencies. Lifetime rates of substance abuse disorders in patients with body dysmorphic disorder have been shown to be close to 30%. Similarly, the lifetime history of an anxiety disorder in patients with body dysmorphic disorder was reported to be more than 6%. The same study reported the lifetime comorbidity rate of anorexia to range from 7 to 14%. Body dysmorphic disorder and hypochondriasis both involve obsessional thinking and checking behaviors, but the focus of concern in body dysmorphic disorder is on appearance, whereas in hypochondriasis the concerns relate to health status. One study found that only 2% of their body dysmorphic disorder sample had comorbid hypochondriasis.

2014 A 22-year-old woman comes to the office for consultation because she is dissatisfied with the appearance of her nose. History includes two cosmetic procedures of the nose. During the consultation, she also expresses dissatisfaction with the appearance of her eyelids, chin, lower abdomen and flanks, and breast size. In this patient with body dysmorphic disorder, which of the following additional findings is most likely? A ) Anorexia B ) Anxiety disorder C ) Depression D ) Hypochondriasis E ) Substance abuse

The correct response is Option D. After establishing an airway, the best first step to controlling massive oronasal hemorrhage is nasal packing. This can be quickly accomplished by inflating Foley catheters in the posterior choanae, followed by anterior packing with either a nasal tampon or ribbon gauze. Nasal packing has been shown to control bleeding in 29% of such patients and decrease it in another 44% of patients. Establishment of mandibulo-maxillary fixation (MMF) may also control oronasal hemorrhage. However, achieving MMF in the emergency department is often limited by the availability of fixation devices and is complicated by the presence of an endotracheal tube. Emergent transcatheter arterial embolization, when available, is highly effective in identifying and controlling oronasal hemorrhage when packing has failed to do so. The internal maxillary and superficial temporal arteries are most often responsible for such bleeding. Operative ligation of the external carotid arteries is rarely effective to control oronasal hemorrhage due to rich collateral blood flow in the head and neck. The utility of nasendoscopy is limited in the presence of marked bleeding as visualization is poor and the bleeding vessel may not be readily visible.

2014 A 24-year-old man is brought to the emergency department after being ejected from a vehicle at high speed. Physical examination shows massive oronasal bleeding and an unstable maxilla. He is hemodynamically unstable, and other sources of marked bleeding have been excluded. Endotracheal intubation is performed. Which of the following is the most appropriate next step in management? A ) Establishment of mandibulo-maxillary fixation B ) Nasendoscopy with bipolar coagulation C ) Operative ligation of the external carotid arteries D ) Placement of anterior and posterior nasal packing E ) Transcatheter embolization

The correct response is Option E. Compression decreases blood flow to active scars, leading to decreased production of collagen fibers. This results in a balance of collagen synthesis and lysis that produces a flatter, softer, less vascularized scar. Clinically, burn scar hypertrophy is managed by use of pressure garments and inserts that must be worn almost 24 hours per day. They should be initiated as soon as all burn wounds have closed enough to tolerate wear and continued until the burn scar has matured. Initially, the pressure applied is low (15 to 17 mmHg). Then, as the scar progresses in maturation, custom-made pressure garments that provide 24 to 28 mmHg of pressure may be fabricated for the patient. The prompt institution of splinting techniques after the acute phase of burn injury can limit the development of long-term deformities. Splinting can combat edema, protect exposed structures and balance soft-tissue lengths to prevent contracture formation and compensate for functional deficits. Later, during the remodeling phase, serial casting can be a great adjunct to a therapeutic exercise program to restore normal range of motion. Surgical lengthening and scar band revision are options that are evaluated if hypertrophic scarring and contractures still develop after appropriate rehabilitation and management. Although the depth and distribution of the injury factor into the development of scars, the patient's own genetic predisposition also plays a role in scar formation and maturation. Injection of a corticosteroid can improve hypertrophic scars, but its use is limited tosmall, focused areas. Metabolic effects can be considerable. Due to the extent of scarring in this patient, corticosteroids are not an appropriate option. Although other topically applied therapies, such as creams containing vitamin E, have been widely used with the intent to improve wound healing, there is not substantial evidence to support regular use. Thirty-three percent delayed hypersensitivity reaction can be seen with topical vitamin E.

2014 A 29-year-old man comes to the office because of scarring 12 weeks after he sustained extensive chemical burns to 30% of the total body surface area. Examination shows thick hypertrophic scarring of the upper extremities and anterior torso. Which of the following is the most appropriate management? A ) Injection of a corticosteroid B ) Scar band revision C ) Serial casting D ) Topical application of vitamin E E ) Use of pressure garments

The correct response is Option E. The lesion in this patient is a desmoid tumor, also known as aggressive fibromatosis. It is a benign tumor, usually found in younger patients between 10 and 40 years old, and is locally aggressive. It is oftentimes associated with pregnancy and prior surgery, and can frequently recur. Treatment is en bloc full-thickness wide local excision (usually with frozen section confirmation of negative margins). As aggressive full-thickness abdominal wall resection is standard of care, reconstruction is more challenging. If midline fascia can be reapproximated, it should be, as primary fascial closure is associated with the lowest hernia recurrence rates. Reinforcement with mesh has been prospectively demonstrated to reduce recurrence rates even further, especially in defects over 4 cm. If midline fascial reapproximation is not possible, reduction in the size of the defect is crucial to decrease recurrence rates. This is done by component separation. However, in this case, only a right component separation is possible, given that the tumor has invaded the left rectus muscle and obliques, precluding their use for myofascial advancement. If the obliques had been spared, a component separation could still have been attempted even if there were violation of the rectus. The most durable reconstruction would be achieved if midline fascial reapproximation were possible with mesh reinforcement. Second best would be reduction in the size of the defect with a right component separation and placement of mesh as a bridging underlay. There is no role for neoadjuvant chemotherapy or radiation therapy in the treatment of these tumors.

2014 A 29-year-old woman comes to the office because of a firm, mildly tender, well-circumscribed mass of the abdomen. A photograph is shown. The mass has been slowly increasing in size for the past 6 months. CT scan shows a mass that occupies the left musculofascial abdominal wall, including the rectus, external, and internal oblique muscles, and penetrates through the anterior rectus sheath. Which of the following is the most appropriate management? A ) Neoadjuvant chemotherapy B ) Radiation therapy C ) Wide local excision, bilateral component separation, adjuvant chemotherapy D ) Wide local excision, mesh placement, radiation therapy E ) Wide local excision, right component separation, mesh reinforcement

The correct response is Option C. The most appropriate option to obtain definitive wound coverage is a fasciocutaneous free flap harvested from outside the zone of injury. Amputation is not indicated when the majority of the great toe is viable. In addition, this would be highly morbid for this young athlete. Negative pressure wound therapy alone would promote healing by secondary intention, but with an exposed joint this would likely result in an unstable wound. Local tissue rearrangement in this area results in marked donor site morbidity. Bony debridement and primary closure may lead to a healed wound, but functional morbidity would be high in this athlete.

2014 A 32-year-old male athlete sustains a contact burn to the right foot. Serial debridement results in exposure of the medial aspect of the first metatarsophalangeal joint. A photograph is shown. Which of the following is the most appropriate option for definitive wound management? A ) Amputation of the great toe B ) Bony debridement and primary closure C ) Coverage with a fasciocutaneous free flap D ) Local tissue rearrangement E ) Negative pressure wound therapy

The correct response is Option A. The patient described has a recurrent keloid after previous excision. Surgery alone has recurrence rates of over 50%, and combination therapies including injection of a corticosteroid, pressure earrings, and surgery can have marked recurrence rates. For recurrent keloids, post-excision radiation therapy, usually given in one to three fractions, has efficacy rates between 6 and 98%. The most common long-term complications of radiation therapy include hypo- or hyperpigmentation (62%) and telangiectasias (27%). Skin desquamation is an acute reaction to radiation therapy and occurs in 24% of patients. Secondary malignancies after radiation therapy for keloids are very rare. Itching from keloids is usually improved with treatment.

2014 A 33-year-old African American woman has a large recurrent keloid of the left earlobe. Reexcision with postoperative radiation therapy is planned. Which of the following is the most likely long-term complication of this therapeutic plan? A ) Altered pigmentation B ) Desquamation C ) Itching D ) Skin cancer E ) Telangiectasia

The correct response is Option E. Split-thickness skin grafts can provide wound coverage over a large area. A mechanical dermatome is often used for obtaining split-thickness skin grafts. Typical thicknesses may range from 8/1000th of an inch to 14/1000th of an inch. The graft can be meshed in various ratios such as 1:1.5, 1:2, and 1:3 to allow for a larger area of coverage per unit of harvested skin. It is important that the underlying wound bed be viable and free of necrotic tissue or infection in order to allow for healing of the skin graft ("skin graft take"). Adequate immobilization of a skin graft is important for take of the graft, and can be achieved with negative pressure wound therapy, or tie-over-bolster dressing. The thigh has an abundant amount of soft tissue and muscle, which is why skin grafts are often sufficient for wound coverage rather than flaps. The patient has a complex wound of the anterior thigh that is best described as a degloving injury in which the skin has been sheared off of the underlying tissues. Undermining of the skin is a hallmark of this type of injury. This type of injury disrupts the blood supply to the skin and can result in tissue ischemia and necrosis. In the acute period, it can be difficult to determine the extent of tissue injury as the skin viability evolves over this time such that areas of marginal blood supply may worsen and progress to full-thickness necrosis. Before definitive wound closure can be achieved, it is critical to debride all devitalized tissue such that there is a healthy viable wound bed. Hence, performing repeat debridement is often necessary. In some cases, debriding the surrounding skin as well as the underlying fat and muscle is required to remove all necrotic tissue. Debridement should continue until healthy tissue is encountered, which can be identified by visual inspection and the presence of punctate bleeding. The surgeon must consider several things when deciding between a flap and a graft. The reconstructive ladder may be used as a guide for management in this case. The defect is too large to achieve primary closure. The use of negative pressure wound therapy for such a large wound may be helpful as a temporary measure, but as a method of definitive wound closure would result in healing by secondary intention, scarring, and prolonged wound care. A full-thickness skin graft is not appropriate because of the large size of the defect and the amount of skin graft that would be required. A full-thickness graft would result in a major defect in another part of the body that would require primary closure. A local fasciocutaneous flap for such a large defect would require significant mobilization of tissue, and similarly, would result in a large donor site defect that would require grafting. A free flap is not necessary when there is viable soft tissue in the wound base. There is no exposed bone, tendon, nerves, blood vessels, or significant dead space, which would make a stronger argument for a flap-over-skin graft. Although not provided as an option in this question, the use of biosynthetic materials or dermal matrix tissues has been reported in the literature as an intermediate step to skin grafting, but it is important to consider the necessity of these materials in effecting outcomes in light of the significant cost of using them.

2014 A 33-year-old woman sustains trauma to the right thigh. She undergoes debridement of the wounds. Two days later, the right anterior thigh has a 15 × 25-cm wound with areas of exposed fat and muscle. Which of the following is the most appropriate intervention to achieve wound closure? A ) Free latissimus dorsi flap B ) Full-thickness skin graft C ) Local fasciocutaneous flap D ) Negative pressure wound therapy E ) Split-thickness skin graft

The correct response is Option A. Because of the skin's important function as a microbial barrier, prevention of infection after burn injury is still one of the most difficult challenges in caring for burn patients. The development of effective topical antimicrobial agents has markedly reduced the incidence of invasive burn wound infection and sepsis. Topical therapy should be started after the initial wound debridement. The three most common topical antimicrobial agents are silver sulfadiazine (Silvadene), silver nitrate, and mafenide acetate (Sulfamylon). Silver nitrate is typically delivered as a 0.5% solution as a wet dressing. Silver nitrate has excellent antibacterial properties and is effective for most Staphylococcus species and most gram-negative aerobes, including Pseudomonas. This agent is typically used when there is a history of sulfonamide allergy or when sensitivity to the other agents has developed. A common use of silver nitrate is in the setting of toxic epidermal necrolysis. Application is painless, but tissue penetration is poor. Concentrations above 5% are cytotoxic to healthy tissues. Because leaching of sodium, potassium, and calcium is common, this effect should be anticipated and replaced appropriately. Painful application is associated with mafenide acetate (Sulfamylon). Mafenide acetate is delivered as suspension in a water-soluble base. As a result of its solubility, it has excellent tissue penetration and is often used in heavily contaminated wounds with thick eschar. Because of excellent cartilage penetration, it is also the agent of choice with ear burns. Mafenide acetate is highly effective against gram-negative organisms. Adverse effects include hypersensitivity reactions (7% of patients) and inhibition of carbonic anhydrase with a resultant hyperchloremic metabolic acidosis. Silver sulfadiazine is the most common topical antimicrobial agent used. It has intermediate tissue penetration secondary to its limited water solubility. This agent has a good antibacterial spectrum, a low incidence of development of resistant organisms, and is applied painlessly. Transient leukopenia is a common adverse effect of silver sulfadiazine. This condition is self-limited and does not appear to increase mortality in burn patients. Switching to a different topical agent for a few days will allow the white blood cell count to return to normal. Thrombocytopenia is not associated with silver nitrate.

2014 A 35-year-old man is admitted to the burn unit after sustaining superficial partial-thickness burns involving 25% of the total body surface area. Medical history includes an allergy to sulfonamide. The burns are cleaned, and silver nitrate- soaked dressings are applied. Which of the following is most likely in this patient? A ) Hyponatremia B ) Metabolic acidosis C ) Neutropenia D ) Painful application E ) Thrombocytopenia

The correct response is Option D. In several studies of complications of abdominoplasty, the most common nerve injury was to the lateral femoral cutaneous nerve. Symptoms include anterior and lateral thigh burning, tingling, and/or numbness that increase with standing, walking, or hip extension. The genitofemoral nerve supplies the proximal portion of the thigh about the femoral triangle just lateral to the skin that is innervated by the ilioinguinal nerve. Nerve injury may result from hernia repair, but injury to this nerve is rare. The ilioinguinal nerve arises from the fusion of T12 and L1 nerve roots and pierces the transversus abdominis and internal oblique muscles. The nerve then supplies sensory branches to supply the pubic symphysis, the superior and medial aspect of the femoral triangle, and either the root of the penis and anterior scrotum in the male or the mons pubis and labia majora in the female. The nerve can be injured in abdominoplasty and other lower abdominal incisions. Symptoms include paresthesia of the skin along the inguinal ligament. The sensation may radiate to the lower abdomen. Pain may be localized to the medial groin, the labia majora or scrotum, and the inner thigh. The iliohypogastric nerve arises primarily from L1. The distribution of the cutaneous sensation of the iliohypogastric nerve most commonly is a small region just superior to the pubis. The iliohypogastric nerve is rarely injured in isolation. Symptoms include burning pain into the inguinal and suprapubic region. Saphenous nerve symptoms of entrapment may include a deep aching sensation in the thigh, knee pain, and paresthesia in the cutaneous distribution of the nerve in the leg and foot.

2014 A 35-year-old woman undergoes abdominoplasty and inner thigh liposuction. After the procedure, burning pain radiating down the right anterior thigh is noted. Pain increases when the patient stands and walks. Injury to which of the following nerves is most likely in this patient? A ) Genitofemoral B ) Iliohypogastric C ) Ilioinguinal D ) Lateral femoral cutaneous E ) Saphenous

The correct response is Option D. The most appropriate next step in management is to surgically explore the wound in the operating room. The clinical picture is of a severe, rapidly progressing infection, possibly necrotizing fasciitis. A high index of suspicion and early treatment are vital for successful outcomes. Necrotizing fasciitis is a rare and rapidly progressive infection of the deeper layers of skin and subcutaneous tissues, easily spreading across the superficial fascial plane, with subsequent death of the overlying skin and severe systemic toxicity. Liposuction is the most frequently associated cosmetic surgery with this infection. Signs and symptoms are insidious, nonspecific, or virtually unnoticeable early in the course of the disease. Later, erythema, prominent edema, and induration appear, accompanied by intense or intolerable pain. The clinical picture evolves into systemic toxicity and eventually multiple organ failure. Risk factors for necrotizing fasciitis include diabetes mellitus, immunosuppression, age older than 50 years, malnutrition, and peripheral vascular disease. There are two forms of the disease: one caused by Streptococcus pyogenes, and the other by mixed infections caused by a variety of microbes, including Escherichia coli, Proteus, Serratia, and Staphylococcus aureus. The progressive necrosis of the tissues typically involves the superficial fascia and the subcutaneous layer. The extent of the gangrene at the fascial layer is typically more severe and greater than at the skin level. This insidious infection is virtually unnoticeable and nonspecific in the first 24 to 48 hours; however, in the following days, an extensive, hardened area appears, which is often dark colored in the center. Intense pain and skin necrosis follow at the level of the infection. Metabolic changes occur, ending with respiratory distress, oliguria, acidosis, increased creatine kinase activity, increased troponin concentrations, and toxic syndrome. Diagnosis and treatment consists of surgical exploration and debridement that reveal necrotic, edematous subcutaneous fat. Bacteriologic analysis of exudate, cultures, and histologic evaluation complete the diagnosis. Early diagnosis is imperative to avoid a fatal outcome. Because necrotizing fasciitis is a progressive, rapid infection, a staged "second-look" operation and, if necessary, additional debridement should be performed. The mortality rates are high and range from 20 to 70%, but decrease to 4.2% after immediate surgical intervention. Though antibiotic therapy is an integral part of the treatment, surgical exploration is key. CT scans and ultrasonography will not change the treatment plan and are therefore not the appropriate next step in management.

2014 A 35-year-old woman with type 2 diabetes mellitus is evaluated in the emergency department because of severe pain and drainage from the right buttock 36 hours after undergoing bilateral buttock augmentation with autologous fat harvested from the thighs. Temperature is 102.0°F (38.9°C), heart rate is 105 bpm, respiratory rate is 16/min, and blood pressure is 90/60 mmHg. Physical examination of the right buttock shows brawny erythema and drainage of turbid fluid from an injection site. The patient has marked tenderness of the buttock, and the abdomen is nontender. White blood cell count is 18.5 × 109/L and serum creatinine concentration is 1.5 mg/dL. After resuscitation, which of the following is the most appropriate next step in management? A ) CT scan of the abdomen and pelvis B ) Inpatient intravenous antibiotic therapy C ) Outpatient oral antibiotic therapy D ) Surgical exploration of the wound E ) Ultrasonography of the buttock

A 38-year-old right-hand-dominant man is evaluated in the emergency department 4 hours after amputating the left thumb and index finger with a circular saw. Microvascular replantation surgery is planned. Which of the following is first in the sequence of repair? A ) Artery B ) Bone C ) Nerve D ) Tendon E ) Vein

2014 A 38-year-old right-hand-dominant man is evaluated in the emergency department 4 hours after amputating the left thumb and index finger with a circular saw. Microvascular replantation surgery is planned. Which of the following is first in the sequence of repair? A ) Artery B ) Bone C ) Nerve D ) Tendon E ) Vein

The correct response is Option B. This surgeon is at increased medico-legal risk due to the fact that there was a lack of verbal discussion of the risks of reduction mammaplasty. Three components must exist for a patient to give informed consent: disclosure, capacity, and voluntariness. In this case, the lack of verbal communication regarding the risks of reduction mammaplasty represents a lack of disclosure. Use of a medical assistant is accepted practice and does not increase the physician's risk in and of itself. One cannot rely on written documents alone, which may or may not be read or understood by the patient. Although a witness signature is required by many hospitals, the presence of a witness is not a central component of the informed consent process. A specific plastic surgery consent form can be helpful, but only if the risks pertaining to the plastic surgery procedure are discussed in person with the patient. Use of a generic hospital consent form is common and acceptable, provided a verbal discussion of the particular risks associated with the proposed procedure occurs and is documented elsewhere in the medical record.

2014 A 38-year-old woman with symptomatic macromastia comes to the office because she desires reduction mammaplasty. A medical assistant obtains the patient's history, and the plastic surgeon performs a brief history and comprehensive physical examination. The patient is considered a good candidate for surgery, and without further discussion the medical assistant is left to obtain a generic hospital consent form. The patient signs the form without a witness present. The surgeon is at increased medico-legal risk because of which of the following? A ) Lack of a specific plastic surgery consent form B ) Lack of verbal discussion of the risks of reduction mammaplasty C ) Lack of a witness signature on the consent form D ) Use of a medical assistant in the consultation E ) The physician is not at risk because the patient signed a consent form

The correct response is Option A. Freeze-dried bone allograft has been used extensively for orthopedic trauma and tumor reconstruction and has been demonstrated to be safe for nasal augmentation. The advantage of allograft is the avoidance of donor site harvesting and morbidity. Fresh autografts probably have more osteoinductive capacity and are likely to incorporate donor bone beds more thoroughly. Although this is important in bone grafting to injured bone, such as in a tibia fracture, it is less important in nasal grafting to a nasal soft-tissue bed. Freeze-dried allografts, much like acellular dermal grafts, are extensively processed to denture all cellular elements and therefore do not elicit immunologic rejection response. Irradiated costal cartilage allografts have also been used with success for nasal reconstruction. There is also a rare chance of disease transmission from the cadaveric donor. Autologous costal cartilage is one of the more commonly used graft materials for nasal reconstruction, although both donor site scarring and pain are prominent. Iliac crest is a useful graft site for cortical and cancellous bone, though the shape is not ideal for nasal contouring. Donor site pain is an issue as well. The same limitations apply to split calvarial grafts, which are most useful when a bicoronal incision has already been used for craniofacial exposure. Temporal fascia is a versatile graft source, especially when wrapped around diced cartilage. The resulting graft is pliable, soft, and has been reported to have minimal absorption. Although it is an excellent choice for this case, it does require a scalp donor site, which this patient does not want. Alloplastic materials such as silicone are also used, though they are prone to extrusion over time.

2014 A 40-year-old man desires correction of the appearance of his nose after traumatic injury 14 months ago. Examination shows collapse of the nasal bones and mid vault. The patient is concerned about additional scarring and donor site pain and requests a procedure with the least amount of donor site morbidity. Which of the following options is most appropriate for this patient? A ) Bone allograft B ) Costal cartilage graft C ) Iliac crest graft D ) Split calvarial graft E ) Temporal fascia graft

The correct response is Option A. Addition of local anesthetics during general anesthesia, whether by subcutaneous, tumescent, or regional block infiltration, can result in decreased dosage requirements of the common sedatives and analgesics that can result in nausea and emesis. Common anesthetic agents that promote nausea and emesis include opioids (fentanyl, hydromorphone, morphine) and inhalationals (halothane, isoflurane, nitrous oxide). Propofol is currently the most commonly used intravenous agent. It does not appear to directly result in nausea, but it has limited analgesic effects. Therefore, effective anesthesia with propofol requires addition of opioid narcotics (which cause nausea) and/or local anesthetics such as lidocaine and bupivacaine (which may decrease the narcotic requirement). Midazolam is a sedative-hypnotic that has anxiolytic and amnesic effects, both of which are helpful adjuncts to the surgical patient experience. Nausea is possible with midazolam, but less commonly reported than with narcotic and inhalational agents. The cause of postoperative nausea and vomiting is multifactorial and not fully understood. Strategies for prevention include: Recognition of high-risk patients (females, nonsmokers, history of motion sickness, previous postoperative nausea, general anesthesia) Pre- and postoperative treatment with multiple modalities (such as scopolamine, ondansetron, aprepitant, corticosteroids) Supplemental intraoperative oxygen and hydration

2014 A 40-year-old woman with a history of severe postoperative nausea and vomiting is scheduled for exchange of bilateral breast tissue expanders for permanent silicone implants. Use of which of the following medications is most likely to decrease the chance of postoperative nausea? A ) Bupivacaine B ) Fentanyl C ) Isoflurane D ) Midazolam E ) Nitrous oxide

The correct response is Option D. The most appropriate method for wound coverage is a local tissue flap, which could come from the intact medial upper arm and/or ulnar aspect of the forearm. A split-thickness skin graft is not appropriate coverage for vital structures. The time it takes for a dermal substitute to vascularize and form the basis of subsequent grafting is too long to leave such vital structures exposed. Free tissue transfer is an option; however, this patient is 2 weeks out from injury and the associated hypercoagulable state is a relative contraindication if local tissues are available. Above-elbow amputation is not an appropriate option as the hand is spared and there are viable coverage options for this young manual laborer.

2014 A 43-year-old electrician sustains a high-voltage electrical injury and undergoes multiple debridement procedures of the right upper extremity. The hand, ulnar aspect of the forearm, and medial upper arm are spared. Two weeks following the injury, a final debridement is performed leaving a 6-cm segment of the brachial artery and median nerve exposed in the proximal forearm. Which of the following is the most appropriate method for wound coverage? A ) Above-elbow amputation B ) Dermal substitute followed by skin graft C ) Free tissue transfer D ) Local tissue flap E ) Split-thickness skin graft

The correct response is Option B. Several reports have suggested an association between breast implants and anaplastic large cell lymphoma (ALCL), which is an extremely rare malignancy. In these cases, ALCL has usually occurred several years after implantation as swelling or a mass around the implant and is often associated with a periprosthetic seroma. Treatments have included capsulectomy with implant removal and chemotherapy and/or radiation therapy, though there is no defined consensus regimen. Despite evidence of an increased risk of ALCL in breast implant patients, the absolute risk remains extremely low. Several large epidemiologic studies have demonstrated a similar or lower incidence of breast cancer (infiltrating ductal carcinoma) among patients who have undergone prosthetic augmentation mammaplasty surgery compared with those who have not. Most cases of ALCL have been in textured implants. Angiosarcoma and malignant fibrous histiocytoma are two sarcomas that may arise in the breast. Angiosarcoma may be caused by radiation therapy for breast cancer. Neither of these sarcomas has been associated with breast implants. Acute myeloid leukemia may be associated with radiation treatment to the breast but has not been associated with breast implants.

2014 A 45-year-old woman comes to the office 10 years after undergoing subglandular implantation of textured silicone implants for augmentation mammaplasty. Physical examination shows swelling of the left breast. She is concerned about cancer. Increased incidence of which of the following malignancies is associated with breast implants? A ) Acute myeloid leukemia B ) Anaplastic large cell lymphoma C ) Angiosarcoma D ) Infiltrating ductal carcinoma E ) Malignant fibrous histiocytoma

The correct response is Option C. A systematic review of the literature was performed by Sameem et al., to determine the relative efficacy of musculocutaneous versus fasciocutaneous versus perforator flaps in the treatment of pressure ulcers. In their analysis, they concluded that all flaps had a significant recurrence and complication rate; however, there was no difference between the types of flaps used. Their paper did not specifically address the issue of how patients' conditions were subsequently managed when they did develop a recurrence. This is important, because a key surgical strategy is to consider the need for subsequent surgical procedures; in fact, perhaps more critical than the composition of the flap and the nature of its blood supply is the design of the flap and where the incisions are planned. A fundamental principle in surgical management of pressure ulcers is being able to provide healthy vascularized tissue into the wound bed and to completely obliterate dead space. Furthermore, the flap should be designed such that if a recurrence develops, the patient continues to have surgical options available, such as re-advancement or re-rotation of the prior flap(s). Preservation of sensory innervation is important for patients who still have sensation in this area. However, in patients who are completely paraplegic, there is a lack of adequate sensory innervation and a lack of the ability to ambulate, which is what ultimately leads to the development of pressure ulcers. Confining the scar to a cosmetically acceptable location is not a major priority in managing pressure ulcers. The cosmetic appearance of scars in this area is relatively unimportant in light of the presence of an open wound. Localizing individual perforators is not necessary when large rotation or advancement flaps are designed. In those situations, a broad blood supply is maintained, and multiple perforators are typically kept intact and do not require individual localization or dissection. However, it is important when a single perforator flap is being designed. Perforator flaps when used as pedicle flaps may be more prone to venous congestion and slightly higher complication rates due to the delicate dissection and isolation of the sole blood supply, which in turn is more prone to mechanical twisting and kinking. Perforator flaps ideally preserve the muscular function of the donor site, which may not be important in a patient who is paraplegic.

2014 A 47-year-old man with complete T12 paraplegia presents with a 5 × 5 × 4-cm stage IV sacral pressure ulcer. Physical examination shows the wound is clean with granulation tissue. Coverage with a local tissue flap is planned. In designing the flap, careful consideration must be made regarding which of the following? A ) Avoiding incisions near the anus to minimize bacterial contamination B ) Confining scar to a cosmetically acceptable location C ) Designing incisions allowing for future re-advancement D ) Localizing individual perforators E ) Preserving sensory innervation

The correct response is Option B. On the basis of the scenario described, fasciocutaneous free flap is the most appropriate management option. The soleus muscle flap is appropriate for defects of the middle third of the leg but lacks adequate reach for soft-tissue coverage of the distal third of the leg. Tissue expansion has been described for soft-tissue reconstruction of congenital talipes equinovarus but is usually reserved for children and in the setting of primary correction. When comparing tissue expansion in the limb versus non-limb sites, the incidence of complications associated with tissue expansion is significantly higher in the limb. Because a nonvascularized allograft is to be used, and the patient has a contracted and scarred soft-tissue envelope, tissue expansion would be associated with higher risk of expansion failure and complications when compared with free tissue transfer soft-tissue reconstruction. Cross-leg flaps are rarely used because of the availability of free tissue transfer. This flap is more appropriate in children than elderly patients, in whom stiffness is a factor. A reverse sural artery flap is not appropriate given the patient's multiple past surgeries and local scars.

2014 A 47-year-old woman is referred by orthopedic surgery for evaluation and discussion of soft-tissue reconstruction at the time of nonvascularized allograft reconstruction of recurrent Achilles tendon rupture. The patient has a history of congenital clubfoot and multiple previous Achilles tendon repairs. Physical examination shows atrophied skin and multiple longitudinal scars along both the medial and lateral distal posterior calf. Which of the following is the most appropriate management? A ) Cross-leg fasciocutaneous flap B ) Fasciocutaneous free flap C ) Reverse sural artery flap D ) Soleus muscle flap E ) Tissue expansion

The correct response is Option B. The patient described has venous insufficiency after microsurgery and the next step in management is emergent reexploration in the operating room. Multiple studies confirm that earlier reexploration improves flap salvage rates. The rate of reexploration ranges from 6 to 14%; in these cases, the flap salvage rate ranges from 36 to 94%. Time of return to the operating room is associated with flap salvage. The majority of microvascular complications occur in the first 48 hours, and the majority of these complications are due to venous thrombosis. Common presenting signs include a purple or blue skin discoloration, brisk capillary refill, edema, oozing, or hematoma. Release of sutures and pinprick of a flap and application of nitroglycerin paste can improve venous congestion in pedicled flaps, but do not obviate the need for reexploration in a microsurgical flap. Placement of leeches is a salvage option and often used when intraoperative maneuvers are unsuccessful. Streptokinase has been described for use in cases where a clot is found within the vascular system, but this should be reserved for use during reexploration, not before.

2014 A 48-year-old woman had delayed microsurgical breast reconstruction. Two hours after surgery, the patient has swelling of the breast and increased drain output. On examination at the bedside, the flap appears purple with capillary refill time of 1 second. Heart rate is 70 bpm, blood pressure is 110/60 mmHg, and most recent hematocrit is 28%. An arterial signal is identified in the skin paddle with a handheld Doppler. Which of the following is the most appropriate next step in management? A ) Application of nitroglycerin paste B ) Operative reexploration C ) Pinprick of the flap D ) Placement of leeches E ) Streptokinase therapy

The correct response is Option C. The Mathes and Nahai classification system is useful for predicting clinical applicability of various muscle and musculocutaneous flaps. With proper knowledge of the location and variation of muscle blood supply, the surgeon can safely determine the extent of muscle transposition during surgery. Five patterns of muscle circulation have been described. These patterns are based on the following relationships between the muscle and its vascular pedicle: The regional source of the vascular pedicle(s) entering the muscle Pedicle size Number of vascular pedicles Location of the pedicle in relation to muscle origin and insertion The angiographic patterns of intramuscular vessels The gluteal V-Y advancement flap used in the clinical scenario described is a Mathes/Nahai Type III flap. Type III muscle flaps demonstrate two large, independent vascular pedicles arising from separate regional arteries. Other Type III muscles include the rectus abdominis and serratus anterior. Angiographic studies have shown equal filling of the intramuscular vascular system with either pedicle injection. Type III muscle flaps can be based on either pedicle and can be split to preserve muscle function. In this particular ambulatory patient, only the superior half of the gluteal muscle (based on the superior gluteal artery) was used in order to preserve lower gluteal function. Type I muscles have a single dominant pedicle. Examples include the gastrocnemius, rectus femoris, and tensor fascia lata flaps. Type II muscle flaps demonstrate one or more large vascular pedicles near the muscle origin and several small pedicles entering the muscle belly distally. Commonly used muscle flaps in this group include the gracilis, soleus, and trapezius. The minor pedicles are typically divided to allow maximal muscle transposition. Division of the minor pedicles typically has little effect on muscle flap survival, but poorly planned musculocutaneous flaps may suffer distal skin ischemia if not planned appropriately. This vascular pattern is the most common pattern observed in anatomical studies of human cadaveric muscle. Type IV muscles demonstrate segmental vascularization along the entire length of the muscle. The sartorius and tibialis anterior muscles are the most clinically relevant muscles that display this type of pattern. The segmental nature of the blood supply severely limits the ability to transpose these muscles and therefore the utility is limited. Type V muscles display one dominant vascular pedicle near the muscle origin and multiple segmental pedicles near the muscle insertion. The latissimus and pectoralis major muscles demonstrate this vascular pattern. Angiographic studies demonstrate that the intramuscular vasculature can be supplied by either the dominant or segmental pedicles. As a result, the flaps can be elevated on either vascular system.

2014 A 50-year-old woman has wound breakdown in the lumbosacral region after spinal instrumentation, as shown in the photograph on the left. The superior aspect is closed with local paraspinal muscle advancement. The lower aspect is closed with a musculocutaneous V-Y advancement flap, as shown in the photograph on the right. Which of the following Mathes/Nahai classifications is most appropriate for this flap? A ) Type I B ) Type II C ) Type III D ) Type IV E ) Type V

The correct response is Option A. Alar rim defects present a challenging reconstructive problem. The primary reconstructive goals are to reestablish structural support, provide nasal lining if necessary, and provide external skin of similar color and texture. Complications of alar rim reconstruction include poor scars, alar notching, nasal obstruction, and narrowing of the nostril. Several choices are available, but a composite graft from the ear will often obtain an excellent cosmetic result. Skin along the alar rim, soft triangle, and columella is quite thin and firmly attached to the lower lateral cartilages. Likewise, skin along the helical rim is firmly attached to the underlying cartilage and useful for replicating the delicate topography of the columella, soft triangle, and nostril margin. Composite grafts are typically harvested from the helical root, but can be harvested from throughout the ear. Composite cartilage grafts only interface with the recipient bed around the graft's perimeter. As a result, their size should be limited to defects less than 1.0 to 1.5 cm in maximal diameter. It is recommended that no portion of the graft be greater than 1.0 cm from the wound edge. Additionally, the wound bed should be well vascularized and the patient should be a nonsmoker. Composite cartilage grafts follow a predictable healing pattern: white, then blue, and then progressively pink/red as revascularization improves. Perioperative strategies recommended by some authors to increase graft take include corticosteroids, hyperbaric oxygen, and cooling of the graft with iced compresses. Primary closure would yield a poor result and distortion of the alar rim. The other options do not provide a cartilage support, which would result in likely alar notching and potential collapse. Additionally, the skin from these donor sites would be too thick to replace the thin skin that normally inhabits this location.

2014 A 54-year-old man comes to the office for reconstruction of an 8 × 10-mm defect involving the right nasal margin after excision of basal cell carcinoma. A photograph is shown. The defect involves the skin and cartilage of the alar border. Which of the following one-stage reconstructive options is most appropriate? A ) Composite auricular graft B ) Dorsal nasal flap C ) Forehead full-thickness skin graft D ) Nasolabial flap E ) Primary closure

The correct response is Option D. The most appropriate management of acute bupivacaine toxicity is a bolus and infusion of 20% lipid emulsion. Every facility where local anesthetic is used in large doses should have a lipid rescue kit clearly labeled and available should the need arise. Although lipid rescue mechanism of action is not completely understood, it may be that the added lipid in the bloodstream acts as a "sink," allowing for the removal of lipophilic toxins from affected tissues. Major local anesthetic toxicity can have such symptoms as sudden loss of consciousness, tonic-clonic seizures, hypertension followed by progressive hypotension, tachycardia, ventricular fibrillation, bradycardia, asystole, and cardiac arrest. Arrhythmias may be refractory to treatment, and resuscitation may be prolonged, sometimes requiring more than 1 hour. In the event of a local anesthetic toxicity event, airway management, seizure suppression, and, if needed, cardiopulmonary resuscitation should be performed. Alert the nearest facility having cardiopulmonary bypass capability and administer 20% lipid emulsion (values in parentheses are for 70 kg) as follows: Bolus 1.5 mL/kg intravenously over 1 minute (~100 mL) Continuous infusion 0.25 mL/kg/min (~500 mL over 30 minutes) Repeat bolus every 5 minutes for persistent cardiovascular collapse Double infusion rate if blood pressure returns but remains decreased Continue infusion for a minimum of 30 minutes Although beta-adrenergic blockers may be useful in treating the excitatory cardiovascular phase of local anesthetic toxicity, the potential to progress to more advanced phases with myocardial depression and collapse preclude their routine use. In addition to lipid emulsion, the treatment for local anesthetic-induced cardiac toxicity is generally supportive, and may include amrinone, closed-chest cardiac massage, and cardiopulmonary bypass. Flumazenil is used to reverse the effects of benzodiazepine toxicity. Dantrolene is administered in the acute treatment of malignant hyperthermia. Atropine and dopamine are administered as part of the Advanced Cardiac Life Support protocol for bradycardia or asystole and would not be used in the scenario described

2014 A 54-year-old woman has onset of ventricular fibrillation and severe hypotension 5 minutes after 30 mL bupivacaine 0.5% is administered to the ankle for postoperative pain control during reconstruction of the foot. After initiation of cardiopulmonary resuscitation, intravenous administration of which of the following is the most appropriate management? A ) Atropine B ) Dantrolene C ) Flumazenil D ) Lipid emulsion E ) Metoprolol

The correct response is Option B. Aeromonas hydrophila is an organism present in the leech species Hirudo medicinalis gastrointestinal tract that can lead to an infection if used medicinally. In this patient with venous congestion and application of leeches, antibiotic prophylaxis is recommended with fluoroquinolones, tetracycline, or trimethoprim-sulfamethoxazole. Actinobacillus lignieresii is seen in horse bites, Pasteurella canis in dog bites, and Eikenella corrodens in human bites. In a recent review of ear reattachment methods, a variety of approaches have been used including microsurgical reattachment, burying of the part in a subcutaneous pocket, periauricular tissue flaps for coverage of the part, and direct reattachment as a composite graft. Microsurgical replantation is associated with the best aesthetic outcome even if venous anastomosis is not possible and leeching is necessary.

2014 A 55-year-old man undergoes microsurgical replantation of an amputated ear. There is venous congestion, and leeches are applied. This patient is at risk for infection by which of the following organisms? A ) Actinobacillus lignieresi B ) Aeromonas hydrophila C ) Eikenella corrodens D ) Pasteurella canis

The correct response is Option B. Successful local treatment of squamous cell carcinoma of the skin depends significantly on whether the tumors are at high or low risk for the complications of recurrence and metastasis. The external ear, lips, nose, and scalp appear to be high-risk locations for squamous cell carcinoma of the skin. Squamous cell carcinomas of the skin larger than 2 cm are twice as likely to recur locally and three times as likely to metastasize than tumors that are less than 2 cm in diameter. Frozen intraoperative examination of specimen edges can be used to judge thoroughness of excision before closure. Frozen sections of margins are recommended for high-risk squamous cell carcinoma and basal cell carcinoma in high-risk areas, lesions more than 2 cm, and any morpheaform basal cell carcinoma. Electrodessication has excellent cure rates in small, low-risk squamous cell carcinoma of the skin. Topical application of 5% fluorouracil has a role in the treatment of diffuse actinic keratoses of the face. Surgical excision is subdivided into excision with standard margins, excision with frozen-section margin evaluation, and Mohs micrographic surgery. For low-risk non-melanoma skin cancers extending into the dermis only, excision with standard margins (4 mm for basal cell carcinoma) is the usual treatment. Adequate margins of 4 mm for low-risk squamous cell carcinoma and 6 mm for high-risk squamous cell carcinoma have been demonstrated by direct tumor extension from the clinical margin but are not necessarily an estimate of cure rate. Squamous cell carcinomas are slower to invade deeper tissue than are cutaneous malignant melanomas.

2014 A 55-year-old woman is evaluated for a biopsy-proven squamous cell carcinoma of the right preauricular area measuring 2.1 cm in diameter. She is otherwise healthy. Which of the following is the most appropriate next step in management? A ) Electrodessication of the lesion B ) Excision of the lesion with frozen sections C ) Excision with a 2-mm margin D ) Excision with a 4-mm margin E ) Topical application of 5% fluorouracil

The correct response is Option E. Vitamin A is essential because it promotes epithelialization in collagen synthesis for wound healing, and supplementation is advocated in patients on chronic corticosteroid immunosuppressive medications such as prednisone. A 20,000-IU daily dosage can be useful for wound healing in immunosuppressed or irradiated patients and appears to reverse the wound healing-suppressive effects of the medication. Patients with chronic wounds frequently have some form of malnutrition that can impede the wound-healing process. In this case, the patient has a serum albumin concentration within the reference ranges, and a stable BMI, signifying adequate protein. In protein-deprived patients, supplementing amino acids that serve as the building blocks of protein synthesis is vital. L-arginine, in particular, has been shown to augment wound healing and collagen production. One study in elderly human subjects found that daily supplementation of 30 g of arginine aspartate for 14 days resulted in markedly enhanced collagen production and total protein. Ferrous gluconate is a useful supplement in iron deficiency anemia. This patient has borderline anemia, though not of a severity likely to be the central impediment to wound healing. Echinacea is a common herbal supplement used as an immunostimulant but has also been shown to have immunosuppressive effects. Lipid emulsion would be useful in a severely malnourished patient, though in this case, the patient's BMI is stable in the normal range. Of note, omega-3 fatty acids appear to inhibit the quality of collagen strength, and avoiding this common supplement during healing may be advisable.

2014 A 55-year-old woman who is wheelchair-bound has a stage IV ischial pressure ulcer. She has a history of systemic lupus erythematosus and multiple sclerosis. Medications include prednisone and gabapentin. BMI is 21 kg/m2 and has been stable for the past year. White blood cell count is 10.5 × 109/L, hematocrit is 30%, and serum albumin concentration is 3.6 mg/dL. After debridement of nonviable tissue, wound care is instituted. Supplementation with which of the following is most likely to promote wound healing? A ) Echinacea B ) Ferrous gluconate C ) Glutamine D ) Lipid emulsion E ) Vitamin A

The correct response is Option A. Percutaneous drainage of hidradenitis suppurativa pustule and fistula tracts, although a plausible short-term fix to address the immediate symptoms, does little to ensure long-term resolution of this very difficult clinical entity with marked impact on quality of life. Addition of topical antibiotic washes or oral antibiotics to percutaneous drainage had no significant effect on long-term recurrence rates. Hidradenitis suppurativa is a recurrent inflammatory disease of the apocrine glands. It initially develops from follicular occlusion with subsequent abscess, inflammation, fistulas, sinus tracts, and scarring. The sites most commonly affected are the intertrigonal regions such as the axilla, groin, and genital/anal region; although, it can also affect the breasts, hips, and thighs. Women are affected three times as often as men. Initial treatment involves local wound care and antibiotic therapy. For advanced disease, this may be followed by excision of the area of high-density apocrine glands with minimal undermining and direct closure at the site of the hidradenitis wound. At the site of inadequate resection of an area of infected glands, or if there is a recurrence, radical resection yields the best long-term result. Skin grafting and fasciocutaneous and musculocutaneous flaps have been described to cover the excisional defect. The musculocutaneous flap has been reported to be a valid option for managing infected lesions because of the abundant blood supply. Delayed secondary wound closure, with or without vacuum-assisted closure or skin substitutes, has also shown plausible outcomes. Sclerotherapy has no role in treatment of hidradenitis suppurativa.

2014 A 55-year-old woman with a BMI of 32 kg/m2 comes to the office with advanced hidradenitis suppurativa of the groin, lower abdomen, and upper thigh. Which of the following treatments is most likely to have the greatest likelihood of success in this patient? A ) Antibiotics and excision B ) Antibiotics and percutaneous drainage C ) Clindamycin irrigation D ) Intralesional injection of a corticosteroid E ) Sclerotherapy

The correct response is Option D. This patient has paclitaxel extravasation due to a malpositioned or leaking catheter with minimal symptoms; therefore, removal of the line and observation is warranted. Calcium gluconate gel is indicated after generously washing areas exposed to hydrofluoric acid as it neutralizes the fluoride ion. Topical collagenase is indicated in wounds with limited tissue necrosis and thus has no role in this patient. Changing this patient's line over a wire is contraindicated as the catheter is either malpositioned or broken. Although operative debridement is sometimes indicated in extravasation injuries, it is unusual, and expectant management is the norm. As this patient has no acute signs of compartment syndrome or tissue necrosis, line removal and observation are indicated. The incidence of extravasation is 0.01 to 6%. Chemotherapeutic agents that cause reactions are classified as irritants or vesicants. Irritants cause immediate and typically limited local reactions such as erythema, warmth, and tenderness. Common irritants are: bleomycin, carboplatin, carmustine, cisplatin, dacarbazine, etoposide, ifosfamide, and thiotepa. Vesicants can cause erythema, blistering, and skin necrosis. Itching in the absence of pain is common. In addition, vesicants can cause delayed ulceration that is self-perpetuated when the vesicant is rereleased upon lysis of affected cells. Common vesicants are: dactinomycin, daunorubicin, epirubicin, idarubicin, mechlorethamine, mitomycin, mitoxantrone, paclitaxel, vinblastine, vincristine. Paclitaxel is derived from the bark of the Pacific yew tree and induces microtubular assembly and stabilization, which leads to cell death. It is a vesicant, and if extravasation occurs, symptoms can range from localized pain, swelling, and erythema to severe skin necrosis and ulceration requiring surgical debridement. The vast majority of extravasations are managed non-operatively.

2014 A 56-year-old woman who has been undergoing treatment for breast cancer has pain around the port site 6 hours after the extravasation of paclitaxel from a subcutaneous tunneled subclavian vein catheter. The patient is hemodynamically stable and breathing comfortably. Moderate swelling and tenderness are observed between the port and clavicle. Which of the following is the most effective management? A ) Application of calcium gluconate gel B ) Application of topical collagenase C ) Line change over a wire D ) Line removal and observation E ) Operative debridement

The correct response is Option D. Infection due to retained necrotic tissue would be the most likely complication in this patient. The vacuum-assisted negative pressure wound closure device should not be used in place of good wound care principles such as debridement. Use of negative pressure wound therapy has been used for pressure ulcers, open abdomen, traumatic extremity wounds, chest wounds, burns, and skin grafts. Negative pressure wound therapy works through mechanisms that include fluid removal, drawing the wound together, microdeformation, and moist wound healing. Several randomized clinical trials support the use of negative pressure wound therapy in certain wound types. Serious complications include bleeding and infection. Negative pressure wound therapy devices should be used with caution in infected wounds. They should not be used until the wounds are adequately debrided. This wound has not been adequately debrided and negative pressure wound therapy should not be used until necrotic tissue has been removed. Bleeding is the next most common complication, but is usually seen in anticoagulated patients and after debridement. Use of a conventional gauze dressing for several hours after a debridement before placing a sponge-based negative-pressure wound therapy device may decrease the risk of excessive bleeding. Most significant bleeding has occurred secondary to disruption of major vessel grafts, cardiac bypass grafts, or the ventricle itself when sponges are placed directly on the structures. This wound is not near any major blood vessels. Use of a single sponge or a long roll of gauze within any deep wounds is recommended to avoid retained foreign bodies. Negative pressure wound therapy has been used to control wound drainage. Increased drainage would be caused by the lack of debridement and infection. Even in clean wounds, a recent report on abdominal wound closure found the most likely complication to be infection rather than recurrent hernia or enterocutaneous fistula. This wound is on the back and would not be likely to have an enterocutaneous fistula. Although initially contraindicated for use with enterocutaneous fistula, recent reports have shown its use to be safe and effective in selected cases.

2014 A 60-year-old woman is seen in the hospital for a pressure ulcer in the lumbar region. A photograph is shown. A sponge for negative pressure wound therapy is about to be applied directly to the wound. Which of the following is the most likely complication of this therapy in this patient? A ) Enterocutaneous fistula B ) Excessive bleeding C ) Excessive wound drainage D ) Infection E ) Retained sponge in wound

The correct response is Option D. The most consistent clinical stage of rejection in this case is hyperacute rejection. In hyperacute rejection, the transplanted tissue is rejected within minutes to hours because of preformed antibodies in the recipient. These antibodies are usually induced by previous blood transfusions, multiple pregnancies, or previous transplantation. The antigen-antibody complexes activate the complement system, causing massive thrombosis in the capillaries, which prevent the vascularization of the graft. If the graft is not removed, severe systemic complications such as systemic inflammatory response syndrome will result. Acute humoral rejection is also primarily mediated by antibody and complement, similar to the hyperacute form of rejection. However, these antibodies are not preexisting, but rather are rapidly induced after exposure to the graft. This usually takes a few days, and the rejection appears in about 3 to 7 days. Another important difference between the hyperacute and acute form of rejection is that there is no known treatment for the former, while the latter may be reversed by plasmapheresis and treatment with anti-B-cell reagents. Acute cellular rejection is mediated by T cells that have been activated against donor antigens, primarily in the lymphoid tissues of the recipient. This is the most common form of rejection treated by clinicians and usually occurs in the first 3 to 6 months of the transplant. Acute cellular rejection is usually treated with increased doses of standard immunosuppressive drugs or anti-lymphocytic antibodies. Chronic rejection develops months to years after acute rejection episodes have subsided. Chronic rejections are both antibody- and cell-mediated. The use of immunosuppressive drugs and tissue-typing methods has increased the survival of allografts in the first year, but chronic rejection is not prevented in most cases.

2014 A 60-year-old woman undergoes bilateral hand transplantation. Within 12 hours of the procedure, the transplanted tissues show evidence of rejection. Despite aggressive medical and surgical management, the transplants fail. Which of the following is the most likely type of tissue rejection in this patient? A ) Acute cellular B ) Acute humoral C ) Chronic D ) Hyperacute

The correct response is Option E. There has been a renewed interest in perioperative antibiotics in recent years toward more appropriate use to decrease surgical site infection (SSI) while decreasing the incidence of resistant bacteria. Current recommendations are to administer a single perioperative dose of antibiotics against common skin flora (gram positive), usually using a first-generation cephalosporin. However, the following recommendations may be underappreciated: Cefazolin intravenous: 1 g if <80 kg; 2 g if >80 kg Alternatives: clindamycin 600 to 900 mg intravenously; vancomycin 1 to 1.5 g intravenously In this case, the patient weighs 200 lb (91 kg), so 2 g of cefazolin is the recommended dosage. Additionally, an important factor is the timing of the administration of the perioperative antibiotics in order to achieve proper skin levels before incision. In one study by Classen et al., published in the New England Journal of Medicine in 1992, the optimal time was between 2 hours before the operation and skin incision, as greater than 2 hours before and any time after skin incision led to marked increases in the relative risk of SSIs. A follow-up by Weber et al. in 2008 narrowed the most appropriate window to between 30 and 59 minutes before skin incision. Finally, there is the issue of redosing. Current recommendations are to redose if there is excessive blood loss (>1500 mL) or if there are long procedures where one exceeds the half-life of the antibiotic used. In the clinical scenario described, the most appropriate choice is 2 g of cefazolin, because the patient's weight is above 80 kg, administered between 30 to 59 minutes before skin incision. Redosing on the basis of blood loss is unnecessary, although one could consider redosing at approximately 4 hours on the basis of half-life.

2014 A 60-year-old woman with breast cancer undergoes a transverse rectus abdominis musculocutaneous flap breast reconstruction after mastectomy. She has no allergies. Weight is 200 lb (91 kg). Estimated blood loss is 200 mL. Duration of the operation is 3 hours and 50 minutes. Administration of cefazolin before skin incision is planned as prophylaxis against surgical site infection. Which of the following is the most appropriate dosage and timing of this injection? Dose Timing Redosing A ) 1 g 5 minutes prior after 150 mL of blood loss B ) 1 g 15 minutes prior no C ) 1 g 40 minutes prior no D ) 2 g 15 minutes prior no E ) 2 g 40 minutes prior no

The correct response is Option A. The deep circumflex iliac artery (DCIA) arises from the lateral aspect of the external iliac artery. From its takeoff point, it travels toward the anterior superior iliac spine (ASIS) between the transversalis fascia and transversus abdominis muscle. Just medial to the ASIS, it gives off an ascending branch which supplies the internal oblique muscle. Lateral to the ascending branch, the DCIA courses through the transversalis fascia along the inner lip of the iliac crest, where it lies in the line of fusion between the iliacus and transversalis fascia, and supplies the iliac crest bone. The deep inferior epigastric vessels supply transverse rectus abdominis myocutaneous (TRAM) and deep inferior epigastric artery perforator (DIEP) flaps. The superficial circumflex iliac vessels supply the groin flap. The superficial inferior epigastric vessels supply the superficial inferior epigastric artery (SIEA) flap, which comprises the skin and subcutaneous tissue only of the lower ipsilateral hemi-abdomen. The superficial femoral vessels supply flaps such as the sartorius muscle flap.

2014 A 62-year-old man is diagnosed with osteosarcoma involving the mandible. Microsurgical reconstruction with a free osseocutaneous flap using iliac bone is planned. The vascular pedicle to this flap is which of the following? A ) Deep circumflex iliac vessels B ) Deep inferior epigastric vessels C ) Superficial circumflex iliac vessels D ) Superficial femoral vessels E ) Superficial inferior epigastric vessels

The correct response is Option B. The blood supply to the omental flap is through the right and left gastroepiploic arteries. Understanding the anatomy and blood supply to the omentum is crucial for success in omental flap transfer. The greater omentum is harvested from the transverse colon, as the short gastric vessels are ligated and the gastroepiploic vessels preserved. The omentum can be transposed to the chest through either an opening in the diaphragm or a fascial defect in the abdominal wall. The left gastric vessels arise from the celiac vessels and supply the lesser curvature of the stomach. The gastroduodenal artery arises from the celiac trunk and provides blood supply to the pylorus and proximal duodenum. One of the terminal branches of the gastroduodenal artery is the right gastroepiploic artery. The superior epigastric artery supplies the rectus abdominis muscle and is not intraperitoneal. The superior mesenteric artery arises from the aorta below the celiac trunk and supplies the lower duodenum through the transverse colon; it does not carry the blood supply necessary for design of an omental flap.

2014 A 65-year-old woman has a draining sinus tract at the lower chest 2 weeks after undergoing a cardiac bypass procedure. After extensive debridement, there is a large central defect requiring an omental flap for obliteration of the dead space. Which of the following arteries supplies the omental flap? A ) Gastroduodenal B ) Gastroepiploic C ) Left gastric D ) Superior epigastric E ) Superior mesenteric

The correct response is Option C. According to the Health Information Portability and Accountability Act (HIPAA) of 1996, it is a violation to provide personal health information about a patient without the patient's expressed consent. Consent is ideally documented in the medical record and signed by the patient. According to HIPAA, there are specific Permitted Uses and Disclosures. A physician is "permitted, but not required, to use and disclose protected health information, without an individual's authorization, for the following purposes or situations: 1) To the Individual (unless required for access or accounting of disclosures); 2) Treatment, Payment, and Health Care Operations; 3) Opportunity to Agree or Object; 4) Incident to an otherwise permitted use and disclosure; 5) Public Interest and Benefit Activities; and 6) Limited Data Set for the purposes of research, public health or health care operations. Covered entities may rely on professional ethics and best judgments in deciding which of these permissive uses and disclosures to make." Treatment is the provision, coordination, or management of health care and related services for an individual by one or more health care providers, including consultation between providers regarding a patient and referral of a patient by one provider to another.

2014 A 67-year-old man undergoes excision of a squamous cell carcinoma from the tip of his nose. The patient's daughter, who is a physician, asks for the pathology results. Which of the following is the most appropriate response? A ) Ask the patient's daughter to provide proof of her medical licensure B ) Have the patient's daughter complete a medical records release form C ) Obtain the patient's consent to release the results to his daughter D ) Refer the patient's daughter to the pathology lab E ) Release the pathology results to the patient's daughter

The correct response is Option A. Lentigo maligna is a slow-growing lesion with a substantial radial growth pattern before progressing to invasion in most cases. These lesions often occur in the head and neck region of older patients with a history of sun exposure. Clinical occurrence is variable, but many appear as irregular, sometimes extensive, pigmented patches on the face. Staging of these lesions follows the American Joint Committee on Cancer guidelines, and prognosis is based on depth of invasion. Need for sentinel node biopsy is based on staging and is independent of resection size. Wide local excision of the lesion is the current standard of care, but the surgical margin for successful excision remains controversial. Alternative techniques have been investigated to improve the 8 to 20% recurrence rates associated with standard excision with 5-mm margins. Mohs micrographic surgery shows promise in the treatment of this disease, but there remains difficulty in interpretation of melanocyte proliferation on frozen section, leading to the proposal of modifications of the procedure, including sending the final Mohs margins for rush permanent section evaluation for verification of clear margins, the so-called "slow Mohs." Clinical evaluation of margins with Wood lamp may be useful in evaluating the clinical extent of the lesion but is not adequate for determination of surgical margins. Confocal microscopy is a new technique that allows examination of melanocytes without biopsy. This modality may be useful in diagnosis of lentigo maligna, but availability is currently limited and requires training in interpretation of images.

2014 A 67-year-old man with a large lentigo maligna on the left cheek comes to the office for closure after undergoing excision. Which of the following steps is most appropriate for the surgeon prior to performing a cervical-facial rotation flap? A ) Await permanent pathology results B ) Confirm negative margins by Mohs micrographic surgery C ) Evaluate the margins clinically with a Wood lamp D ) Perform confocal microscopy E ) Refer the patient for sentinel node biopsy

The correct response is Option C. Based on the clinical scenario described, wound biopsy and culture is the most appropriate management option. Despite wound debridement and moist wound care, the wound has not improved and is in the region of a previous malignancy. Wound biopsy would allow the diagnosis of recurrent malignancy and aid in the determination of further surgical intervention. Wound culture would allow the diagnosis of soft-tissue infection contributing to the wound's persistence. Although wound debridement would be beneficial in this case, application of a skin graft in the face of possible recurrent malignancy and probable marked radiation injury would be associated with increased risk of delayed wound healing and may delay management of recurrent malignancy. If the wound was attributed only to radiation therapy, a better strategy would be to excise the irradiated soft tissues and cover the whole defect with a well-vascularized flap. Hyperbaric oxygen therapy has been shown to be beneficial for the management of radiation soft-tissue injury. This therapeutic modality should only be instituted after a complete evaluation of the patient's wound, which would include soft-tissue biopsy because the patient previously had a malignancy in the region. Complete evaluation of the wound would include pertinent history and physical examination, evaluation of the patient's nutritional status, examination of extremity vascular inflow and outflow, diagnosis and treatment of wound infection, and optimization of wound characteristics. The patient has already undergone debridement and wound care for several months; therefore, continued observation and wound care would be an inadequate management option. It is inappropriate to perform negative pressure wound therapy in an irradiated wound without diagnosis by tissue biopsy.

2014 A 73-year-old man is evaluated for a non-healing wound on the medial aspect of the calf. The wound has been present for 8 months, and he has undergone several months of serial debridements and moist wound care without improvement. A photograph is shown. Ten years ago, he was diagnosed with squamous cell carcinoma of the medial calf skin, and the condition was managed solely with radiation therapy. Which of the following is the most appropriate next step in management? A ) Hyperbaric oxygen therapy B ) Negative pressure wound therapy C ) Wound biopsy and culture D ) Wound debridement and skin graft E ) Continued observation and wound care

The correct response is Option E. The most appropriate first step in management is sclerotherapy. Lymphatic malformation is a type of vascular anomaly that results from aberrant formation of lymphatic vessels. Lymphatic malformation most commonly affects the neck and axilla. There are two major types of lymphatic malformations: macrocystic and microcystic. Macrocystic lesions have cysts large enough to be cannulated by a needle and treated with sclerotherapy. Microcystic lesions have cysts that are too small for treatment with sclerotherapy. First-line management of macrocystic lymphatic malformations is sclerotherapy, which is the injection of an inflammatory substance into the lesion that causes scarring of the cyst walls together and shrinkage of the malformation. The most commonly used sclerosants are doxycycline, sodium tetradecyl sulfate, and ethanol. Prednisolone and propranolol are drugs used to treat problematic proliferating infantile hemangioma, and have no efficacy for vascular malformations. Embolization is used to treat arteriovenous malformations and involves delivering a substance through an artery to occlude blood flow. Resection is second-line therapy for a problematic macrocystic lymphatic malformation. Extirpation can cause marked morbidity (i.e., bleeding, nerve injury, infection, wound breakdown). In addition, excision leaves a cutaneous scar and recurrence is likely because a lymphatic malformation can rarely be completely removed. Resection is considered if a lesion remains symptomatic following sclerotherapy, or for microcystic lymphatic malformations that cannot be sclerosed.

2014 A male newborn has a large macrocystic lymphatic malformation of the neck. Which of the following is the most appropriate first step in management? A ) Embolization of the lesion B ) Prednisolone therapy C ) Propranolol therapy D ) Resection of the lesion E ) Sclerotherapy

The correct response is Option B. The most likely explanation for this patient's abnormal physiology is hypovolemic shock. The patient just underwent tangential excision of a 50% total body surface area burn, and marked blood loss is to be expected. She received 2 units of packed red blood cells, but this is unlikely to be adequate for such a large burn excision. In addition, her vital signs are typical for hypovolemic shock. Sepsis and acute respiratory distress syndrome CARDS) are often seen in patients with large burns, but they are usually seen later in the hospital course. Sepsis is associated with fever and a high cardiac output. ARDS is associated with previous large-volume transfusions and lung injury, and should not cause hypotension in isolation. It is also associated with more severe hypoxia. Pneumonia and pulmonary embolism are also associated with a more profound hypoxia than this patient exhibits and are usually seen later in a burn patient's hospital course.

2014 An 18-year-old woman who sustained a flame burn involving 50% of the total body surface area is resuscitated to a stable cardiovascular and respiratory status. Four days after injury, she undergoes tangential excision and xenografting of all burned areas. Following surgery, the patient returns to the ICU intubated and ventilated. She has thick pulmonary secretions. She received 2 units of packed red blood cells during surgery. Vital signs are as follows: Temperature 99.5°F (37.5°C) Heart rate 130 bpm Respiratory rate 22/min Blood pressure 80/50 mmHg Oxygen saturation is 96% on 40% FIo2. Cardiac output is 6 L/min, and urine output is 0.1 mL/kg/h. Which of the following is the most likely explanation for these abnormal findings? A ) Acute respiratory distress syndrome B ) Hypovolemic shock C ) Pneumonia D ) Pulmonary embolism E ) Sepsis

The correct response is Option B. The majority of cleft lip and cleft palate patients undergo secondary bone grafting of the alveolar cleft between the ages of 8 and 12 years. A commonly used source of graft material, iliac crest bone, is associated with morbidity including significant pain, impaired ambulation, and prolonged recovery. Some authors have proposed the use of bone morphogenetic protein for alveolar cleft closure. Advantages to this technique include reduced operative time, quicker recovery, and a greater volume of graft material, which can be limited when harvesting iliac crest bone graft in smaller children. Osteogenesis requires both osteoconductive materials and osteoinductive factors. Iliac crest bone graft displays necessary properties, while bone morphogenetic protein provides significant osteoinductive properties, but requires an additional carrier, such as demineralized bone putty, for osteoconduction. Bone morphogenetic protein is not FDA-approved for patients younger than 12 years of age.

2014 An 18-year-old woman with a history of cleft lip and palate presents for secondary alveolar bone grafting. An iliac crest bone graft is planned. Which of the following characteristics of iliac crest bone graft is an advantage over the use of bone morphogenetic protein in this patient? A ) Greater volume of graft material B ) Osteoconductive properties C ) Osteoinductive properties D ) Reduced operative time E ) Reduced recovery time

The correct response is Option C. Vascular anomalies are a common source of pediatric morbidity, potentially resulting in cosmetic and functional abnormalities. Two main categories of lesions are hemangiomas and vascular malformations. Vascular malformations are generally named after the vessel types that are involved and are further subdivided into low-flow and high-flow lesions. Low-flow lesions include capillary, lymphatic, venous, and mixed lesions. High-flow lesions include arteriovenous malformations. Both high- and low-flow vascular malformations are almost always present at birth and either grow commensurately with the child or slowly enlarge over a period of years. Treatment is guided by the degree of functional impairment, and many require surgical therapy. Hemangiomas are rarely present at birth. They tend to appear between 2 and 8 weeks of life and grow rapidly. These lesions comprise of rapidly proliferating endothelial cells and follow a predictable clinical course. They usually undergo an aggressive proliferative phase that lasts several months before reaching a plateau phase, when they grow very little. Finally, at about 1 year of life, hemangiomas begin a process of spontaneous involution which may last for up to 4 years. Completion of involution does not mean complete involution, and many hemangiomas may leave cosmetically unacceptable residua. Hemangiomas found in inconspicuous areas can be observed. Those noted in cosmetically or functionally sensitive areas require more aggressive treatment. Treatment options have included topical, intralesional, and systemic corticosteroids, laser treatment, interferon-alfa, and surgical resection. However, each of these treatments carries marked risks and still does not provide consistent, reliable success. In June 2008, a French group reported rapid resolution of hemangiomas in children treated with propranolol for pediatric cardiomyopathy. This serendipitous finding led to a landmark paper and has resulted in a paradigm shift in the clinical care of these patients. Many groups have now confirmed their findings with very promising results. The use of propranolol for hemangiomas remains off-label, but there are centers of excellence with institutional review board protocols in place to further study its clinical effects. Recommended doses vary but frequently start at 0.5 mg/kg/day divided three times daily and slowly titrated over 1 to 2 weeks to 2 mg/kg/day. Most centers recommend obtaining pretreatment electrocardiography (ECG); if normal, then therapy can be initiated on an outpatient basis. If the ECG is abnormal, then pretreatment cardiology evaluation is warranted. Dose titration, blood pressure, and heart rate are checked regularly. The most common adverse effect is lethargy. Other adverse effects include hypoglycemia, hypotension, and bradycardia. All children drool, and this has not been reported as a complication of propranolol administration. Hypoglycemia can be a potentially life-threatening complication that can also cause seizures. Parents are counseled to give food with medication. Many centers regularly check blood glucose. An image of a child with upper lip hemangioma treated with propranolol for 14 months is shown.

2014 An 8-month-old male infant is evaluated for a lip mass. A photograph is shown. Treatment with propranolol is initiated. Which of the following adverse effects is most important to monitor? A ) Drooling B ) Hypertension C ) Hypoglycemia D ) Lethargy E ) Tachycardia

The correct response is Option C. This patient's lesions are most consistent with actinic keratoses. Actinic keratoses are most commonly seen in fair-skinned individuals in areas that have had long-term sun exposure. They are the most common skin lesions to demonstrate malignant potential and may progress to squamous cell carcinoma. Given the propensity of actinic keratoses to malignant transformation, treatment is generally recommended over observation. For multiple lesions, topical agents are generally effective and well tolerated. Imiquimod is thought to exert its effects by inducing a local immune response as well as apoptotic pathways. Other effective treatments include photodynamic therapy, cryotherapy, 5-fluorouracil, and diclofenac gel. Dexamethasone is a corticosteroid typically used for inflammatory or autoimmune skin conditions. Isotretinoin is used to treat cystic acne. Docosanol is an antiviral medication used for herpes simplex.

2014 An 87-year-old Caucasian man comes to the office with multiple 5- to 6-mm lesions on the forehead. He has a long history of sun exposure. Physical examination shows the lesions are erythematous, rough, and scaly. Which of the following is the most appropriate treatment? A ) Dexamethasone B ) Docosanol C ) Imiquimod D ) Isotretinoin E ) Observation with 1-month follow up

The correct response is Option E. The most appropriate next step in management is to consider intra-arterial thrombolytic therapy. The treatment of frostbite has remained essentially unchanged for the past 25 years. Classic management of frostbite injury includes resuscitation, rewarming, and watchful waiting. The outcome is either tissue recovery or progressive gangrene leading to eventual amputation. A variety of maneuvers aimed at advancing the care of patients with frostbite have been attempted, including hyperbaric oxygen, surgical and medical sympathectomy, pharmaceutical agents, and anticoagulation. None of these have resulted in alterations in the management of this disorder. Recent reports have described the use of thrombolytic therapy using urokinase or tissue plasminogen activator (tPA) as a potential therapy for frostbite. The rationale for this therapy is based on the understanding that tissue injury in frostbite occurs from two distinct components. Initially, tissue freezing and crystal formation occur and then are improved with tissue rewarming. The more significant cause of tissue injury occurs after thawing, and it is the robust local tissue inflammation and coagulation that stimulate microvascular thrombosis and progressive cell death. By reversing local microvascular thrombosis, tPA has been postulated to restore perfusion before irreversible ischemia and necrosis. Systemic corticosteroids or antibiotics are not indicated at this time. Systemic heparinization is usually employed as an adjunct after intra-arterial thrombolytics have been initiated. However, it is delivered intravenously. Surgical debridement is done in a delayed fashion after the area of frostbite has demarcated, sometimes weeks to months later.

2014 An otherwise healthy 35-year-old man is exposed to subzero temperatures for 24 hours. After initial management of hypothermia and rapid rewarming of the hands, bilateral upper extremity frostbite is evaluated. Physical examination shows severe frostbite of the hands and up to the wrists bilaterally. Which of the following is the most appropriate next step in management? A ) Corticosteroid therapy B ) Heparin therapy C ) Surgical debridement D ) Systemic antibiotic therapy E ) Thrombolytic therapy

The correct response is Option E. Breast cancers that are estrogen receptor positive may be responsive to adjuvant chemotherapy with selective estrogen receptor modulators such as tamoxifen, which can reduce recurrence and mortality. Tamoxifen is associated with thromboembolic events, such as deep venous thrombosis and pulmonary embolism. This prothrombotic effect has been postulated to be secondary to the effect of tamoxifen on estrogen receptors that are abundant within vascular endothelium. Tamoxifen has been shown to be associated with increased rates of total flap loss and decreased rates of flap salvage when taken within 28 days of microsurgical breast reconstruction, which represents two half-lives of the active metabolite of tamoxifen (N-desmethyl tamoxifen, t1/2=14 days). It has therefore been recommended that in patients undergoing microsurgical breast reconstruction, tamoxifen be held for at least 28 days preoperatively. Some authors have further advised holding the medication postoperatively in addition to preoperatively. Tamoxifen is not associated with impaired wound healing, increased bleeding, hemodynamic instability, or seroma formation.

2014 An otherwise healthy 37-year-old woman presents for delayed microsurgical breast reconstruction. Which of the following is associated with use of tamoxifen? A ) Hemodynamic instability B ) Impaired wound healing C ) Increased bleeding D ) Seroma formation E ) Thromboembolic events

The correct response is Option A. Venous drainage of the lower abdominal skin and subcutaneous tissue occurs primarily through the superficial venous system and secondarily through the deep venous system, with perforating veins interconnecting the two systems. These communicating veins have been identified on computed tomography angiography in approximately 90% of abdominal walls in vivo. The majority of the remaining 10% of patients likely have communicating veins that are too small to visualize or are absent. In these cases of anatomical superficial venous system dominance, venous drainage is dependent on the superficial venous system. A recently published 2012 article by Sbitany et al. demonstrated that the incidence of intraoperative venous congestion secondary to persistent superficial venous system dominance was 0.9% in 1201 muscle-sparing transverse rectus abdominis musculocutaneous and deep inferior epigastric artery perforator free flaps. A free flap that becomes congested after reperfusion in the operating room should be assessed immediately for possible etiologies including twisting, kinking, tension, or vasospasm of the vascular pedicle. If a technical problem is ruled out and the venous anastomosis remains patent, obligatory enhancement of venous drainage with the superficial venous system is necessary to salvage the free flap rather than revision of the original anastamosis. Various methods include an anastomosis of the superficial inferior epigastric vein (SIEV) to the DIEV system or any chest wall vein, including the retrograde limb of the internal mammary vessel, the branch of the internal mammary vessel, or the thoracodorsal system. This requires preemptive planning and sparing of the superficial epigastric vein or SIEV during the dissection of the flap. A vein graft can be utilized if additional length is necessary. Another option is to substitute the DIEV anastomosis with the SIEV. Tissue plasminogen activator (tPA) would not be indicated in this scenario, as it is used as a thrombolytic and there is no evidence of vascular thrombosis. Revising the DIEV anastomosis would be moot because it is patent and the deep system is being drained. Leech therapy is useful for venous congestion, but primarily as an adjunct after potential surgical etiologies have been addressed. Sacrificing the free flap without first attempting salvage is not warranted, and using a prosthetic would be possible only if prior patient consent were obtained.

2014 An otherwise healthy 38-year-old woman undergoes prophylactic bilateral mastectomy and immediate reconstruction with deep inferior epigastric artery perforator (DIEP) free flaps. Intraoperatively, the left DIEP flap appears congested before the conclusion of the case. The left deep inferior epigastric artery and vein (DIEA and DIEV) were anastomosed to the proximal internal mammary vessels. The vascular pedicle is evaluated and each anastomosis appears patent and not kinked; however, the venous congestion persists. Which of the following is the most appropriate management? A ) Anastomose the superficial inferior epigastric vein to an internal mammary vessel perforator B ) Convert to left prosthetic reconstruction C ) Infuse tissue plasminogen activator (tPA) to the DIEA D ) Initiate leech therapy E ) Revise the DIEV anastomosis to the retrograde internal mammary vessel limb

The correct response is Option C. The most appropriate treatment for this lesion is radical resection, with the goal of eradicating the lesion. Although the lesion is prone to recurrence, this approach maximizes the chances for a cure. This patient has recurrent bleeding and is at risk for death due to exsanguination. Observation is not acceptable in this otherwise healthy 68-year-old. Laser therapy is futile as it will only affect superficial dermal structures, and this lesion clearly involves large vessels and deeper tissues. Corticosteroid therapy is appropriate for management of infantile hemangioma but has no role in the treatment of arteriovenous malformations. Sclerotherapy is not indicated in this patient.

2014 An otherwise healthy 68-year-old man comes to the office because of an enlarging mass over the right ear for the past 30 years. A photograph is shown. He had no prior treatment of the mass. MRI confirms the diagnosis of arteriovenous malformation. In the past 3 months, he had bleeding from the mass requiring hospital admission and blood transfusions. Bleeding is controlled after prolonged direct pressure. Which of the following is the most appropriate management for this lesion? A ) Corticosteroid therapy B ) Laser therapy C ) Resection D ) Sclerotherapy E ) Observation

The correct response is Option B. One of the psychological contraindications to plastic surgery is when a patient gauges the success of surgery on realization of a specific goal (i.e., a job promotion). Others include the patient who is unable to contemplate an imperfect result, uncertain as to which aspect to change, under emotional stress during consultation, motivated to have surgery at the request of others, and a doctor-shopper dissatisfied with the results of multiple previous procedures. There are multiple contraindications to surgery: some anatomical and some psychological. One of the most important decisions by a surgeon is whether to perform the requested surgery. The plastic surgeon has to identify a correctable deformity or concern first. This then has to be balanced against the importance that the patient places on this deformity. According to Gorney, the patient with minor deformity but extreme concern is most likely dissatisfied with whatever the outcome. Additionally, the surgical outcome has little to do with the emotional stress. The dissatisfied patient, once discovered postoperatively, must be handled carefully. This patient must be seen frequently and offered compassion and concern. Allow the patient to see you as the ally that you are. Offer a waiting period before performing any other operations. Consider an offer to revise an operation only if you concur with the patient's complaints and you think you can improve the appearance. Also, sit with the patient and have a frank discussion of his/her complaints. All the other options in the question present patients with real identifiable, correctable problems with reasonable expectations.

2014 Which of the following individuals is at highest risk of dissatisfaction with the surgical outcome? A ) A 25-year-old man with a large amount of excess skin following a 100-lb (45-kg) weight loss who comes to the office for body contouring B ) A 35-year-old woman who comes to the office for facial rejuvenation surgery in order to advance her career as a news anchor C ) A 42-year-old woman who comes to the office with a large nasal dorsal hump and bulbous tip who is requesting an improved appearance D ) A 45-year-old mother of three with marked deflation and ptosis of the breasts who is looking for an improved appearance E ) A mother of a 3-year-old girl who brings the child in for surgery for a large congenital nevus

The correct response is Option B. Data have shown that there are predictable changes that occur to the layers of the skin and soft tissue in response to tissue expansion. Of all the layers listed, only the epidermis displays an increased thickness as a result of tissue expansion. Specifically, the epidermis becomes thicker through a process of hyperkeratosis. There is also narrowing of intercellular spaces and an increase in mitotic activity. The dermis actually thins up to 50% with fragmentation of the elastin fibers and flattening of dermal papillae. Sweat glands and hair follicles drift farther apart. Muscle decreases in both thickness and mass, although its function remains unchanged. Fat thins with some permanent loss of total fat mass. In cases of aggressive expansion, fat necrosis and fibrosis may also occur.

2014 Which of the following thickens when a tissue expander is placed and inflated? A ) Dermis B ) Epidermis C ) Fat D ) Muscle

The correct response is Option E. First-line intervention for a large, symptomatic, macrocystic, lymphatic malformation is sclerotherapy. Sclerotherapy is the injection of an anti-inflammatory substance (e.g., doxycycline) into a lesion, which causes endothelial damage, fibrosis, and shrinkage of the malformation. Sclerotherapy is more effective and less morbid than resection. Propranolol and prednisolone are treatment options for a problematic infantile hemangioma, but they have no efficacy for vascular malformations. Embolization is first-line intervention for an arteriovenous malformation, and is not a treatment option for lymphatic malformation. Resection is second-line therapy for a large macrocystic, lymphatic malformation. Extirpation can cause significant morbidity (i.e., bleeding, nerve injury, infection, wound breakdown). In addition, excision leaves a cutaneous scar, and recurrence is common because a lymphatic malformation can rarely be completely removed.

2015 A 1-year-old female infant is evaluated for a 12 × 12-cm macrocystic lymphatic malformation of the posterior trunk that has been enlarging and causing infections for the past 6 months. The lesion involves the skin, subcutaneous tissue, and muscle. Which of the following is the most appropriate next step in management? A ) Embolization B ) Prednisolone therapy C ) Propranolol therapy D ) Resection E ) Sclerotherapy

The correct response is Option A. The tensile strength of a skin incision 2 weeks following repair is approximately 10%. Classic studies by Madden and Peacock showed that a cutaneous wound achieves 5% of its ultimate strength after 1 week, 10% after 2 weeks, 20% after 3 weeks, 40% after 4 weeks, and 80% after 6 weeks. The scar has its full strength 12 weeks after repair.

2015 A 10-year-old boy underwent removal of a pigmented nevus from his scalp 2 weeks ago with suture closure. The tensile strength of the incision line today is most likely which of the following percentages of its final strength? A ) 10% B ) 20% C ) 40% D ) 60% E ) 80%

The correct response is Option A. Multiple studies have evaluated outcomes of median and ulnar nerve repair after transection injury. A meta-analysis by Ruijs et al. confirmed that younger age, specifically under 16 years old, was associated with the highest chance of satisfactory recovery of motor function in both median and ulnar nerve injuries. Patient gender was not found to be a statistically significant factor in outcome. In the same analysis, median motor nerve injuries were found to have a better chance of recovery than ulnar motor nerve injuries. Timing influenced outcome, with delay of repair adversely affecting prognosis, and although the ideal window for repair was not able to be defined by this review, there is some evidence that immediate repair may result in worse outcomes. The use of nerve grafts did not significantly predict motor recovery in these injuries.

2015 A 14-year-old boy sustains a laceration of the distal forearm. Physical examination and wound exploration suggest flexor carpi ulnaris tendon and ulnar nerve involvement. The patient undergoes immediate microsurgical nerve repair with a nerve graft. Which of the following factors is most likely to predict a satisfactory outcome in this patient? A ) Age B ) Gender C ) Immediate repair D ) Ulnar nerve involvement E ) Use of nerve graft

The correct response is Option C. Pilar cysts, also known as trichilemmal cysts, originate from the outer root sheath of the hair shaft. They present as firm, slow-growing subcutaneous nodules, and may be difficult to differentiate clinically from epidermoid cysts. They are commonly found on the scalp where they are the most common cutaneous cyst. They are lined by stratified squamous epithelium, which undergoes keratinization. In some cases, these lesions can demonstrate aggressive biologic behavior (proliferating trichilemmal tumors, malignant proliferating trichilemmal tumors), in which case they should be completely excised. Lesions that originate from adipocytes include lipomas and angiolipomas. Lesions of vascular origin include cherry angiomas and pyogenic

2015 A 24-year-old woman is evaluated because of a slow-growing subcutaneous mass of the scalp. An excisional biopsy is performed, and pathologic examination shows keratin and its breakdown products. Which of the following is the most likely origin of this lesion? A ) Adipocyte B ) Capillary C ) Hair follicle D ) Mechanoreceptor E ) Sebaceous gland

The correct response is Option E. Severity of injury is proportional to the cross-sectional area of tissue able to carry current. Thus, the most severe injuries are seen at the wrists and ankles, with decreasing severity proximally. The extremities are the most frequently injured body parts, with the upper extremity predominating. Most high-voltage injuries occur in workers on the job, so the voltage is known. Temperature increase parallels changes in amperage with tissue temperature being a critical factor in the magnitude of tissue damage. Tissue resistance from lowest to highest is nerve, blood vessels, muscle, skin, tendon, fat, and bone. Deep tissue seems to retain heat so that periosseous tissues, especially between two bones, often sustain a more severe injury than more superficial tissue. The associated macro- and microscopic vascular injury seems to occur nearly immediately and is not reversible. Alternating current (AC) causes tetanic muscle contraction, which may either throw victims away from contact or draw them into continued contact with the electrical source, the latter being more common, given our propensity to grasp at objects and the greater strength in our forearm flexors relative to extensors. This effect is often described as the "no-let-go" phenomenon. Altered levels of consciousness are reported in about half of high-voltage injuries. The tissue injury in electrical burns seems to be a combination of thermal and nonthermal mechanisms. If enough heat is generated, the tissue heats to supraphysiologic temperatures causing denaturation of macromolecules, which is usually irreversible. Electroporation refers to the formation of aqueous pores in lipid bilayers exposed to a supraphysiologic electric field. The applied electric field alters the transmembrane potential, with muscle fibers and nerves being the most susceptible. Subsequent pore formation likely allows calcium influx into the cytoplasm, thereby triggering apoptosis and cell death. Electroporation can therefore induce cell necrosis in the absence of heating. Transmembrane protein molecules contain polar amino acid residues that can change orientation in an electric field. This effect, known as electroconformational protein degradation, may be irreversible and form yet another mechanism of nonthermal injury.

2015 A 26-year-old lineman is brought to emergency department after accidentally grabbing a high-voltage power line. The most severe injury is seen at which of the following anatomical locations? A ) Chest wall B ) Neck C ) Shoulder D ) Upper arm E ) Wrist

The correct response is Option B. The amount of Ringer's lactate (mL) needed for initial resuscitation is formulated by the Parkland formula of 4 mL/kg/% the total body surface area (TBSA) burned, with half this volume infused over the first 8 hours from the time of injury. In other words, for the first 8 hours, the patient should receive 2 mL/% TBSA/kg. All other options are not accurate based on the Parkland formula and this patient's TBSA. One easy way to determine the hourly rate for the first 8 hours is to multiply the % TBSA and kg, and divide by 4. Because the Parkland formula is 4 mL/% TBSA/kg, with half given over the first 8 hours, the 4 in the Parkland formula can be divided by 16 (2 × 8) to give a denominator of 4. It is absolutely critical to perform accurate estimation of the burn size and weight, because overzealous fluid administration can lead to fluid creep, compartment syndromes, and pulmonary complications.

2015 A 27-year-old man is brought to the emergency department after sustaining second- and third-degree burns to most of the anterior torso and the upper extremities. Which of the following describes the appropriate amount of Ringer's lactate, according to the Parkland formula, for fluid management of this patient's condition? A ) 1 mL/% TBSA/kg given over the first 6 hours B ) 2 mL/% TBSA/kg given over the first 8 hours C ) 2 mL/% TBSA/kg, half given over the first 8 hours D ) 4 mL/% TBSA/kg given over the first 6 hours E ) 4 mL/% TBSA/kg given over the first 8 hours

The correct response is Option B. The lesion in this patient is a recurrent desmoid tumor, also known as aggressive fibromatosis. It is a benign tumor, usually found in younger patients between 10 and 40 years of age, and is locally aggressive. It is often associated with pregnancy and previous surgery and can frequently recur. Management is en bloc, full-thickness, wide local excision (usually with frozen section confirmation of negative margins). An aggressive full-thickness abdominal wall resection is standard of care, making reconstruction more challenging. The more durable and functional reconstruction entails complete restoration of the abdominal wall, especially musculofascial components, in a primary reapproximation. However, depending on the size of the tumor and resultant defect, this may not be possible. Basic principles, after obtaining proper margins after resection of the tumor, would then be reduction in defect size to the maximal extent possible and wide bridging underlay of mesh with at least 4- to 5-cm margins in all directions. Bridging inlay, whereby a mesh is simply sewn to the margins of the defect, has clearly been shown to be inferior in terms of recurrence rates. Mesh choice can be either synthetic or biologic, although if significant contamination exists, if soft-tissue coverage is tenuous, or if one desires to decrease the amount of adhesion formation when placing mesh directly against the bowel, one should consider placement of biologic mesh, accepting the fact that there is a higher incidence of postoperative bulges using these materials, by and large. In this case, only a left component separation is possible, given that the tumor has invaded the right rectus muscle and obliques, precluding their use for myofascial advancement. There is no role for chemotherapy or radiation therapy in the treatment of these tumors. Coverage with a right anterolateral thigh flap, with or without rectus femoris, can reconstruct the soft-tissue defect, but avoiding the use of mesh in a defect over 4 cm has a significantly higher chance of a recurrent hernia and would not be standard of care. Primary skin grafting on top of a nonvascularized thick piece of acellular dermal matrix will not "take," even with use of negative pressure wound therapy.

2015 A 27-year-old woman is evaluated for a recurrent abdominal desmoid tumor. CT scan shows a mass that occupies the full-thickness right musculofascial abdominal wall, involving the rectus abdominis muscle and oblique muscles, including lateral to the semilunar line. Resection is performed. Photographs of the defect are shown. Which of the following is the most appropriate management? A ) Bilateral component separation, primary skin closure with incisional topical negative pressure wound therapy, adjuvant chemotherapy B ) Left component separation, bridging wide intraperitoneal underlay biologic mesh, primary closure skin C ) Pedicled right anterolateral thigh flap with rectus femoris, no mesh D ) Placement of a bridging inlay of biologic mesh with primary split-thickness skin grafting and negative pressure wound therapy E ) Placement of a bridging inlay of uncoated heavyweight polypropylene mesh with adjuvant radiation therapy

The correct response is Option A. The only reliable information gleaned from available data is that the use of a coupler for venous anastomoses does decrease the operative time in performing the vascular technique. Most studies point to an improved patency rate in venous anastomoses as well, but this remains open to interpretation. All the available published data point to one factor with the greatest influence on patency rates: adherence to sound and well-established microvascular technique principles. There is no proof that end-to end, end-to-side, running or interrupted, eversion or mattress, etc., has any superiority over other techniques in patency rates. At the present time there are no adequate reliable data regarding use of sutureless techniques and their long-term outcomes in a clinical setting. Other options have not been proven as benefits of a coupler device over other techniques.

2015 A 28-year-old woman with a traumatic lower extremity wound undergoes free tissue transfer reconstruction. Venous anastomosis is completed with a 3-mm coupler device. Which of the following is the proven benefit of using a coupler device? A ) Decreased anastomosis time B ) Decreased thrombosis C ) Decreased twisting of vessels D ) Ease of use E ) Improved kinking

The correct response is Option E. Warping is one drawback of auricular cartilage grafts. They can curl over time. Warping is more common when the perichondrial layer is left on the graft. Pure cartilage grafts tend to warp less often. Auricular cartilage and pure cartilage warping would occur gradually over a period of weeks or months. Cartilage is a versatile graft material that is useful for providing structural support or treating contour problems. The rib, nasal septum, and ear are the most common donor sites. Grafts are primarily composed of hyaline and elastic cartilage, with only a 1 to 10% volume of cells, so they can easily survive transplantation. Graft nutrition relies on diffusion of nutrients through the matrix of proteoglycans, interstitial fluid, and chondrocytes. Early contour deformities could result from malposition or shifting of the graft. In this clinical case, the deformity occurs at 6 months postoperatively, when the graft should be stable in the soft-tissue envelope. Rejection is not likely with autologous cartilage and would likely occur with extrusion or fluid drainage. Rejection or failure to incorporate would typically be associated with an inadequately vascularized soft-tissue envelope or infection, and these complications would be expected within the first few weeks. Resorption can occur gradually but is uncommon. A contour indentation would be more likely to result than a protruding bump. Ossification of the graft is not a known complication.

2015 A 29-year-old woman is evaluated because of a bump that developed on the nasal dorsum 6 months after she underwent reconstruction with an onlay auricular cartilage graft to treat a post-traumatic nasal deformity. Which of the following is the most likely cause of this patient's complication? A ) Infection B ) Ossification C ) Rejection D ) Resorption E ) Warping

The correct response is Option E. Cigarette smoking is a leading cause of preventable death and disability in the United States. Over the past 20 years, several studies have demonstrated an increased risk of postoperative complications following plastic surgical procedures, including rhytidectomy, breast reconstruction, digital replantation, muscle flaps, and body-contouring procedures. Tobacco smoke is a complex mix of particulate matter, volatile acids, and gases. There are over 4000 different compounds in cigarette smoke, many of which are toxic, mutagenic, and carcinogenic. Tobacco-induced vasoconstriction is mediated directly and indirectly by nicotine, a colorless, odorless, and poisonous alkaloid. Increased cellular levels of nicotine cause direct microvascular vasoconstriction. Indirect pathways of vasoconstriction include the enhancement of thromboxane A2 and stimulation of catecholamine release. Random skin flaps such as abdominoplasty, rhytidectomy, and mastectomy flaps are predominantly supplied by the subdermal plexus, which is very sensitive to sympathomimetic agonists such as catecholamines. Smoking also increases carboxyhemoglobin levels, which shifts the oxygen-hemoglobin saturation curve to the left. The net result is decreased oxygen-carrying capacity by direct competitive inhibition from carbon monoxide. Other effects caused by smoking include decreased prostaglandin I2 (prostacyclin) production, increased platelet aggregation and blood viscosity, decreased collagen production, decreased red blood cell deformability, increased fibrinogen production, and decreased leukocyte function (mediated by hydrogen cyanide). The net effect is a prothrombogenic state with impaired inflammation that also contributes to slow wound healing. Although fibrinogen production is increased and leukocyte function is decreased, the primary mechanism by which wound healing is impaired is related to the nicotine-induced vasoconstriction of the subdermal plexus. Rhytidectomy patients who smoke are 12.5 times more likely to develop skin necrosis compared with patients who do not smoke. One study showed a 47.9% rate of wound-healing problems in abdominoplasty patients who smoked compared with 14.8% in those who did not smoke. Another large study of patients undergoing breast reconstruction using a free transverse rectus abdominis musculocutaneous (TRAM) flap showed no difference in free flap survival in those patients who smoked, but the smoking population had a significantly higher rate of mastectomy skin flap loss, abdominal donor-site complications, and hernias. Current recommendations for smokers who desire elective cosmetic surgery are to avoid smoking and all nicotine products for 4 weeks before and after surgery.

2015 A 30-year-old woman who underwent uneventful abdominoplasty is evaluated 2 weeks postoperatively because of midline wound dehiscence with tissue necrosis. She reports that she did not stop smoking before surgery as instructed. A photograph is shown. Which of the following mechanisms is the most likely cause of the delayed wound healing? A ) Decreased catecholamine production B ) Decreased hemoglobin concentration C ) Decreased leukocyte function D ) Increased fibrinogen production E ) Increased microvascular vasoconstriction

The correct response is Option B. The most likely diagnosis is acute rejection, because this patient is still in the early postoperative period when acute rejection is most likely to occur (0 to 3 months). ABO incompatibility and antibody incompatibility would result in hyperacute rejection, which is mediated by the humoral immune system and occurs within minutes of transplantation. Chronic rejection occurs after years and is characterized by vasculopathy and fibrosis.

2015 A 32-year-old man with a history of self-inflicted gunshot wound is evaluated because of significant facial deformity despite multiple complex reconstructive procedures. Composite tissue allotransplantation is performed. One episode of rejection is successfully treated 4 weeks postoperatively. Three months postoperatively, the patient develops recurrent swelling and hyperemia of the facial skin. Which of the following is the most likely cause of this condition? A ) ABO incompatibility B ) Acute rejection C ) Antibody incompatibility D ) Chronic rejection E ) Hyperacute rejection

The correct response is Option A. The main rationale for performing early motion exercises is to decrease adhesion formation. During the early phases of tendon healing, large amounts of collagen are deposited and form early scarring. Although this scarring is crucial for the healing of the repaired tendon, scarring will also occur in the tendon sheath and, if allowed to progress, can lead to stiffness in the involved digit. Early passive- and active-motion protocols assist in breaking apart early scarring of the tendon to the surrounding sheath. Early motion has not shown to increase the final strength of the repair and is more likely to cause rupture. Although synovial fluid flow might be increased with tendon excursion, it does not improve outcomes. Postoperative pain is likely increased with therapy.

2015 A 34-year-old machinist undergoes repair of the flexor tendon of the index finger as the result of a work-related injury. Which of the following is the main rationale for performing early motion exercises after surgical repair? A ) Decrease adhesions B ) Decrease postoperative pain C ) Improve strength of repair D ) Increase synovial fluid flow E ) Prevent rupture

The correct response is Option E. Necrotizing fasciitis is a rapidly progressive soft-tissue infection characterized by necrosis of the fascia and subcutaneous fat with subsequent necrosis of the overlying skin. Although complication rates associated with liposuction are not unduly increased, infection is a major concern, and cases of prolonged inflammation, septic shock, and infections have been documented. Likewise, cases of necrotizing fasciitis following liposuction have been reported on several occasions and, according to data reported in the literature, the overall incidence of necrotizing fasciitis is equal to 0.4 per 100,000 patients. There are two common forms that are reported: infections caused by Streptococcus pyogenes and mixed infections caused by a variety of microbes, including Escherichia coli, Proteus, Serratia, and Staphylococcus aureus. A detailed case of necrotizing fasciitis sustained by Mycobacterium chelonae after a combined procedure of liposuction and lipofilling has also been described. The progressive necrosis of the tissues typically involves the superficial fascia and the subcutaneous layer, but is limited in extension to the skin; the extent of the gangrene at the fascial layer is usually more severe and greater than at the skin level. Necrotizing fasciitis is virtually unnoticeable in the first 48 hours with nonspecific symptoms. In the days that follow, an extensive, hardened region forms, which is often dark in the center. Severe pain and necrosis follow at the level of the infection. Drainage of "dishwater fluid" is often pathognomonic. Metabolic changes occur, ending with respiratory distress, oliguria, acidosis, increased troponin concentrations, and sepsis. Diagnosis and treatment consist of surgical exploration and debridement that reveal necrotic, edematous, subcutaneous fat. Bacteriologic analysis of exudate, cultures, and histologic evaluation complete the diagnosis. Early diagnosis is imperative to avoid a fatal outcome. Treatment is based on immediate and aggressive surgical debridement with combined antibiotic therapy. Because necrotizing fasciitis is a progressive, rapid infection, the wound typically is left open for a planned "second-look" operation and additional debridement if necessary. The mortality rates are increased and range up to 70% but decrease to 4.2% after immediate surgical intervention. Delay in debridement increases mortality.

2015 A 34-year-old woman undergoes laser-assisted liposuction of the abdomen, hips, and inner and outer thighs using a tumescent technique. A total of 2500 mL of aspirate is removed. Postoperative recovery is uneventful, and the patient is discharged home the same day. She comes to the emergency department 4 days later with intense pain over the lower abdomen and flanks. Temperature is 101°F (38.3°C). Physical examination shows the skin has well-demarcated erythema, induration, and bullae forming at multiple sites. Her incisions are seeping clear, grey fluid. Which of the following is the most appropriate management? A ) Exploratory laparotomy for presumed bowel perforation B ) Intravenous administration of antibiotics and local burn care C ) Intravenous administration of antibiotics only D ) Oral administration of antibiotics E ) Surgical debridement of the involved tissue

The correct response is Option D. Risk factors for postoperative nausea and vomiting include: female gender, nonsmoking status, prior history of postoperative nausea/vomiting/motion sickness, use of volatile anesthetics/general anesthesia, opioid/narcotic use, facial rejuvenation procedures, and long duration of surgery. Several measures can be taken to decrease postoperative nausea and vomiting. A thorough history with identification of risk factors can aid stratification of patients preoperatively. Use of long-acting local anesthetic agents, nonsteroidal anti-inflammatory drugs (NSAIDs), and cyclooxygenase-2 selective inhibitors can decrease the need for postoperative opioid use. Avoidance of nitrous oxide, especially in combination with fentanyl and volatile inhalational gases, and multimodal use of serotonin antagonists combined with other antiemetic agents can also decrease postoperative nausea and vomiting.

2015 A 35-year-old man undergoes a 90-minute rhytidectomy procedure with intravenous sedation. The patient smokes 10 cigarettes daily, but is otherwise healthy. Which of the following factors most likely places this patient at increased risk for postoperative nausea and vomiting? A ) Duration of procedure B ) Gender C ) History of cigarette smoking D ) History of postoperative nausea and vomiting E ) Type of anesthesia

The correct response is Option D. Propofol can be safely used in patients with a suspected diagnosis of malignant hyperthermia. Malignant hyperthermia is a rare, life-threatening inherited skeletal muscle disorder that shows symptoms of hypermetabolic reaction to volatile anesthetic gases and depolarizing muscle relaxants. The incidence is between 1 in 5000 to 1 in 100,000 anesthetic encounters. Mortality rates have decreased from 70% to less than 5% as awareness of this condition has led to accurate diagnosis and treatment. Malignant hyperthermia is genetically transmitted through an autosomal dominant inheritance pattern with variable penetrance. In obtaining a medical history, it is important to document family history of adverse outcomes to general anesthesia. If it is reported that a first-degree relative has had a malignant hyperthermia crisis or susceptibility, then the patient should not be exposed to triggering agents. Anesthetic agents that trigger malignant hyperthermia include: halothane, enflurane, isoflurane, desflurane, sevoflurane, and succinylcholine. Nitric oxide can be used if the anesthesia machine is "vapor-free" and contains no traces of volatile gas. Other safe agents include nondepolarizing muscle relaxants (such as vecuronium, rocuronium, and pancuronium), all ester and amide local anesthetics, ketamine, propofol, etomidate, barbiturates, opiates, and benzodiazepines. Although it is safe to undergo minor procedures with administration of a topical or local anesthetic agent, patients undergoing complex procedures with intravenous sedation, general anesthesia, or major conduction blockade should be referred to an accredited ambulatory surgical center or hospital. If symptoms are recognized in the operating room (high temperature, increased end-tidal CO2, muscle rigidity), rapid treatment with dantrolene sodium is the highest priority. Acute episodes may require stopping the procedure and transfer to an intensive care unit.

2015 A 35-year-old woman comes to the office for consultation on augmentation mammaplasty. During preoperative workup, she reports that her mother has a history of malignant hyperthermia. The patient has never undergone surgery. Which of the following anesthetic agents is most appropriate for this surgery? A ) Desflurane B ) Halothane C ) Isoflurane D ) Propofol E ) Succinylcholine

The correct response is Option C. To ensure patient safety during office-based procedures, a system of quality care needs to be established in each facility, emphasizing maintenance of the appropriate facilities, equipment, personnel, protocols, and procedures. When administering anesthesia of any kind in an office setting, the surgeon should follow the American Society of Anesthesiologists' "Guidelines for Office-Based Anesthesia," and "Practice Guidelines for Sedation and Analgesia by Non-Anesthesiologists." The facility should be outfitted with the appropriate medical equipment, materials, and drugs necessary to provide anesthesia, recovery ministration, cardiopulmonary resuscitation, and provisions for potential emergencies. Anesthesia equipment should include suctioning apparatus, appropriately sized airway equipment, including laryngoscope blades, means of positive-pressure ventilation, intravenous equipment, pharmacologic antagonists, basic resuscitative medications, and, in the event of deep sedation, defibrillator equipment. Additional equipment, such as a central venous catheter kit, electrocautery unit, warming blanket, and tracheostomy set may be desirable based on the type of patients and/or cases being performed, but are not considered essential in the case of the patient described.

2015 A 35-year-old woman is scheduled to undergo low-volume liposuction of the lower abdomen in an office setting. Intravenous sedation, in addition to local and tumescent anesthesia, is planned. Which of the following is essential to have in the operative suite according to the Guidelines for Office-Based Anesthesia? A ) Central venous catheter kit B ) Electrocautery unit C ) Intubation equipment D ) Tracheostomy set E ) Warming blanket

The correct response is Option A. The cumulative effect of multiple procedures performed during a single operation increases the potential that complications may develop. Large-volume liposuction, combined with other procedures such as abdominoplasty, can cause serious complications. Death associated with isolated lipoplasty is rare (0.0021%, or one per 47,415), but mortality increases significantly when lipoplasty is combined with other procedures. When combined with non-abdominoplasty procedures, lipoplasty mortality increases to one per 7314; when combined with abdominoplasty, with or without other procedures, the lipoplasty mortality increases to one per 3281. The presumed benefits of combined procedures must thus be weighed against potential untoward events. Studies indicate that epidural anesthesia combined with the infusion of anesthetic infiltrate provides patients with a consistent intraoperative comfort level. Data from the few anesthesia studies that have specifically assessed patients undergoing liposuction confirm the safety of general anesthesia, epidural anesthesia, spinal anesthesia, moderate sedation, and local anesthesia for this procedure. It should be noted, however, that epidural anesthesia and spinal anesthesia can cause vasodilation and hypotension, thereby necessitating the administration of excess fluid and increasing the risk of fluid overload. Based on the patient's history, physical examination, review of systems, laboratory testing, and/or a medical specialist's evaluation, the physician should select the patient's American Society of Anesthesiologists (ASA) physical classification rating: Type 1: A normal healthy patient; Type 2: A patient with mild systemic disease; Type 3: A patient with severe systemic disease; Type 4: A patient with severe systemic disease that is a constant threat to life. ASA Type 1 and Type 2 patients are candidates for ambulatory and office-based surgery. The patient described is a Type 2 patient, a classification rating that represents patients who have any of the following conditions that are under control without systemic compromise: diabetes mellitus, hypertension, asthma, gastroesophageal reflux disease, peptic ulcer disease, hematologic disorders, arthritis, and neuropathy. Large-volume liposuction is defined as the removal of 5000 mL or greater of total aspirate during a single procedure. A review of the scientific literature shows that there are no scientific data available to support a specific volume maximum at which point liposuction is no longer safe. The super-wet technique, introduced in the mid-1980s, uses larger volumes of subcutaneous infiltrate, whereby 1 to 2 mL of solution is infused for each 1 mL of fat to be removed. The infiltrate solution consists of saline or Ringer's lactate with epinephrine and, in some cases, lidocaine. Using this method, blood loss generally decreases to less than 1 to 2% of the aspirate volume.

2015 A 35-year-old woman with a history of hypertension is evaluated for body contouring. Physical examination shows generalized abdominal adiposity and moderate infraumbilical pannus. The patient undergoes abdominoplasty and large-volume liposuction after induction of epidural anesthesia. A super-wet technique is used and a total volume of 5500 mL is removed. Which of the following factors poses the greatest risk of death for this patient? A ) Abdominoplasty B ) Epidural anesthesia C ) Hypertension D ) Liposuction volume E ) Super-wet technique

The correct response is Option B. Hyaluronidase is an enzyme that breaks down hyaluronic acid, a mucopolysaccharide that is a normal component of the interstitial fluid barrier. It has been shown to increase the rate of absorption of an injected substance by facilitating diffusion of the substance over a large area. When injected locally within 1 hour of extravasation, it breaks down hyaluronic acid and decreases the viscosity of the extracellular matrix, and facilitates absorption and dispersal of the extravasated chemical. The ischemic effects of extravasated vasoconstrictive agents such as norepinephrine and dopamine may be reversed with local infiltration of phentolamine, which is an alpha-blocking agent. Topical heat application has been recommended in vinca alkaloid extravasation to promote local circulation and speed up clearance of the extravasated agent. Topical cooling in animal models has been demonstrated to increase ulcer formation. Dexrazoxane has been shown to antagonize the effects of several topoisomerase II poisons such as anthracycline agents, including doxorubicin. Recent clinical trials in Europe have demonstrated its efficacy in minimizing tissue damage from anthracycline extravasation if administered intravenously within 6 hours of extravasation. It is now the recommended initial treatment of anthracycline extravasation, especially in light of its FDA approval in 2007. Dimethyl sulfoxide (DMSO) is a free radical scavenger and an effective solvent. It may also have antibacterial, anti-inflammatory, and vasodilatory properties. Its topical application is effective in preventing ulcerations caused by doxorubicin extravasation.

2015 A 36-year-old man with traumatic injuries, who is intubated and sedated in the intensive care unit, is noted to have extravasation of concentrated calcium solution from a peripheral access intravenous line. The consult is made immediately after extravasation. Which of the following is the most appropriate management of this injury? A ) Intravenous administration of dexrazoxane B ) Local injection of hyaluronidase C ) Phentolamine infiltration D ) Topical application of dimethyl sulfoxide E ) Topical application of heat

The correct response is Option B. Higher American Society of Anesthesiologists (ASA) class has been identified as a risk factor for postoperative complications in HIV-positive patients in multiple studies. A patient who is ASA Class 2 has only mild systemic disease. Increasing class number indicates increasing severity of disease (Class 3 - severe systemic disease, Class 4 - severe systemic disease that is a constant threat to life). Acquired immunodeficiency syndrome (AIDS) is diagnosed when the CD4 count is <200 cells/mm3 or with acquisition of an AIDS-defining illness. An absolute CD4 count of <200 cells/mm3 has been associated with increased risk of complications including wound infections. HIV seropositivity alone has been found not to be an independent risk factor for postoperative complications. Percutaneous surgery, such as fat grafting, has not been associated with increased risk of infection in HIV patients, nor has skin incisional surgery. Transoral mucosal incisional surgery has been found to be associated with a significantly greater risk of wound infection in HIV patients. A viral load greater than 10,000 copies/mL suggests that antiretroviral therapy is no longer effective and has been identified as an independent risk factor for complications.

2015 A 37-year-old man, who is American Society of Anesthesiologists (ASA) Class 2, comes to the office for evaluation and treatment of human immunodeficiency virus (HIV)-associated lipodystrophy. The patient has a CD4 count of 100 cells/mm3. Autologous fat grafting is planned. Which of the following factors is most likely to increase this patient's risk of postoperative complications? A ) ASA Class B ) CD4 cell count C ) HIV seropositivity D ) Percutaneous surgery

The correct response is Option A. Free flaps can be monitored by several different modalities in the postoperative period. The main reason for monitoring free flaps postoperatively is to detect vascular complications in a timely fashion, before permanent injury to the flap occurs, and to maximize the possibility of flap salvage. Reviews of large consecutive series of free flaps indicate that the most likely time period for a vascular compromise is early on, usually within the first 24 hours after successful transfer from the operating room. Therefore, postoperative monitoring protocols should be designed to closely follow flap perfusion during this period of time. Vascular events leading to issues with flap perfusion do occur at later times, but such events are generally infrequent and more difficult to salvage.

2015 A 39-year-old woman successfully undergoes immediate bilateral breast reconstruction with coverage with free deep inferior epigastric artery perforator free flaps. Postoperative flap monitoring is planned. Vascular compromise is most likely to occur during which of the following time periods postoperatively? A ) 0-1 days B ) 2-3 days C ) 4-5 days D ) 6-7 days E ) 8-9 days

The correct response is Option D. First-line treatment of a small problematic infantile hemangioma is intralesional corticosteroid (triamcinolone 3 mg/kg). If the tumor is too large to inject, then oral corticosteroid (prednisolone 3 mg/kg daily) or propranolol is initiated. Interferon is no longer used to treat infantile hemangioma because it may cause spastic diplegia when administered to infants. The common side effects of propranolol include gastrointestinal effects (vomiting, diarrhea, and constipation), rash, fatigue, and hypersomnia. Severe side effects may include bradycardia, hypotension, chest pain, shortness of breath, bleeding, bronchospasm, and glaucoma. In pediatric patients, propranolol has been associated with hypoglycemia that may occur without the characteristic jitteriness. Proliferating hemangiomas should not be treated with pulsed-dye laser therapy because accelerated involution does not occur, and patients are at risk for ulceration, pain, bleeding, hypopigmentation, and scarring. The pulsed-dye laser is indicated, however, to treat residual telangiectasias after the tumor has involuted. Corticosteroid complications (e.g., adverse neurodevelopment, aseptic necrosis of the femoral head, diabetes mellitus, osteoporosis, adrenal insufficiency, cataracts, glaucoma, infection, gastric irritation) have not been observed in patients treated with corticosteroid for infantile hemangioma. Although increased blood pressure has been observed, the clinical significance of this is unclear, and no adverse effects have been reported. Twenty percent of infants develop a temporary cushingoid appearance that disappears during tapering of the medication. Approximately one third of infants exhibit decreased gain in height but return to their growth curve by 24 months of age.

2015 A 4-month-old male infant is brought to the office because of a rapidly enlarging mass in the eyebrow region. The patient's mother says she first noticed the lesion at 1 month of age and that it was not present at birth. A photograph is shown. Treatment with administration of propranolol is planned. Which of the following adverse effects is most likely in this patient? A ) Adrenal failure B ) Decrease in height C ) Hypertension D ) Hypoglycemia E ) Spastic diplegia

The correct response is Option C. The component separation technique can be used to achieve medial transposition of the rectus muscle and overlying anterior fascia. The surgery involves division of the external oblique fascia and muscle lateral and parallel to the linea semilunaris. The plane deep to the external oblique muscle, which is relatively avascular, is then dissected laterally. The rectus muscle is also separated off of the posterior rectus sheath (using access from the medial laparotomy or hernia incision). This allows for medial advancement of the rectus muscle, overlying anterior rectus sheath, internal oblique muscle, and transversus muscle as a unit. The segmental neurovascular bundles course deep to the internal oblique muscle and penetrate into the rectus muscle 10 to 25 mm medial to its lateral margin. The component separation technique, when performed in the scenario described, can give unilateral advancement toward the midline approximately 10 cm at the level of the umbilicus, which equates to a bilateral advancement of 20 cm. The least amount of advancement is in the subxiphoid and subcostal regions, often making more cranially located defects more difficult to close. Since it was originally reported in 1990, several modifications and variations of this technique have been described in the literature. These include perforator-preserving and/or endoscopic techniques to methods that describe additional maneuvers to increase mobilization or improve durability with the addition of biologic or prosthetic meshes.

2015 A 40-year-old man undergoes ventral hernia repair with biologic mesh and fascial closure at the midline. A bilateral component separation technique with incision of the external oblique fascia and muscle lateral to the linea semilunaris and dissection in the plane between the external and internal oblique muscles, and separation of the rectus muscle off of the posterior rectus fascia is performed. At which of the following levels can the least amount of advancement of the medial fascial edges be expected? A ) Midway between the umbilicus and pubis B ) Midway between the umbilicus and subcostal margin C ) Subcostal margin D ) Suprapubic E ) Umbilicus

The correct response is Option E. Hidradenitis is a chronic and often disabling skin condition of the apocrine glands. It results in abscesses, inflammatory nodules, and draining sinuses in the axillae, groin, perineum, and occasionally the submammary area. It affects approximately 1 in 300 people, and it is more common in women, people who smoke, and people who are overweight. The axillae are more commonly involved in women, and the perineum more commonly in men. Early treatment centers around local wound care, oral antibiotics, corticosteroid injections, hygiene, weight loss, and smoking cessation. Decreased estrogen levels may also contribute, and disease often subsides with oral contraceptives and during pregnancy. Unresponsive or extensive disease requires surgical intervention for long-term resolution. Relapse rates are high, and healing is often slow. Although limited excision and closure of solitary lesions is an effective short-term treatment, new lesions are likely to develop. Fistulotomy alone has an unacceptably high recurrence rate. A long-term resolution in this severely affected patient requires complete excision of the hair-bearing skin of the affected area. The large defect created needs broad skin coverage. The best options include split-thickness skin grafting or grafting with negative pressure wound therapy immobilization. Negative pressure wound therapy followed by healing by secondary intention or coverage with local flaps are additional options, though scar contractures and shoulder stiffness are risks. Laser treatments are too superficial to treat the deeper apocrine structures. Lymphadenectomy is not necessary for a cure.

2015 A 40-year-old woman with chronic hidradenitis suppurativa of the bilateral axillae is referred for surgical management after 4 years of local wound care, multiple corticosteroid injections, and oral antibiotics failed to improve her symptoms. Physical examination shows multiple inflamed nodules and draining sinuses. Photographs are shown. Which of the following treatment options is most likely to provide long-term resolution of this patient's symptoms? A ) Axillary lymphadenectomy and coverage with a trapezius flap B ) Carbon dioxide laser and silver sulfadiazine C ) Fistulotomy and negative pressure wound therapy D ) Lesion excision and primary closure E) Total axillary excision and skin grafting

The correct response is Option D. Similar to skin grafting, fat grafting involves a donor site as well as graft and recipient preparation. Fat is harvested, and donor site is closed and dressed. Fat graft is processed and injected into the recipient site. Fat grafting is reported with code 20926 (tissue grafts, other [e.g., paratenon, fat, dermis]) and includes the following: · Harvest of the fat graft material by any method (e.g., syringe, suction-assisted lipectomy, incision) · Closure of the donor site, if indicated, with appropriate dressing · Processing of fat graft material · Injection of fat graft into recipient site · Dressing of recipient site · 90 days of routine postoperative care Code 20926 is not anatomical site-specific, nor is it volume dependent. Thus, both the injection of 50 mL of fat into the cheek concavity and injection of 500 mL of fat into the thigh for correction of contour irregularities are coded as 20926. This code is used for each anatomical area injected; thus, if both breasts had fat grafting, the Current Procedural Terminology (CPT) codes would be 20926 and 20926-59, because the second breast is recognized as a separate and distinct procedure. Code 15877 (suction-assisted lipectomy; trunk) is not coded separately because harvest of the graft by any method is included in 20926. Subcutaneous injection codes (11950-11954) describe the injection of "off the shelf" products such as collagen. These do not involve the harvest of tissue, and hence neither donor site nor postoperative care is involved. Code 19830 (revision of reconstructed breast) may be appropriate if a large area or multiple areas of a reconstructed breast are grafted, because it may be better described as a "revision of the reconstructed breast." Currently, however, there is no uniform established minimum volume to be considered sufficient to warrant use of code 19830 instead of 20926. Preauthorization for insurance reimbursement should be done for all fat grafting procedures, because not all payers will cover them, since there is no "functional improvement."

2015 A 41-year-old woman comes to the office for follow-up examination after undergoing reconstruction of the left breast with an implant. She has an upper pole contour depression and desires correction. She has been preauthorized for fat grafting from the abdomen to the upper pole; 25 mL is injected into the upper pole. Which of the following is the most appropriate Current Procedural Terminology (CPT) code? CPT code Description 11954 subcutaneous injection of filling material (e.g., collagen) over 10.0 mL 15877 suction-assisted lipectomy; trunk 19380 revision of reconstructed breast 20926 tissue grafts, other (e.g., paratenon, fat, dermis) A ) 15877 and 11954 B ) 15877 and 19380 C ) 19380 D ) 20926 E ) 20926 and 19380

The correct response is Option E. True allergic reactions to local anesthetics are rare. Type I hypersensitivity reactions may include anaphylaxis, and are modulated by immunoglobulin E. Both ester-based local anesthetics, such as procaine, and amide-based local anesthetics may induce an allergic response. The most likely allergen is para-aminobenzoic acid (PABA). This compound consists of a benzene ring substituted with an amino group at the 4-position (leading to the analogous label of 4-aminobenzoic acid) as well as a carboxyl group. Preservatives such as methylparaben have also been shown to cause allergic reactions to local anesthetics, but are less likely than allergic reactions caused by PABA. Epinephrine can induce vasoconstriction and increase the safe dosage of administered local anesthetic, though it would not be expected to cause an allergic response itself. Iodine and acetaldehyde should not be found in local anesthetic mixtures.

2015 A 42-year-old woman, with a history of anaphylactic reaction to procaine, comes to the office for consultation regarding augmentation mammaplasty. Anesthetics that contain which of the following should be avoided in this patient? A ) Acetaldehyde B ) Epinephrine C ) Iodine D ) Methylparaben E ) Para-aminobenzoic acid

The correct response is Option D. The most appropriate management is exploration of the left flap to assess anastomotic patency and pedicle orientation. This flap is hyperemic with brisk capillary refill and present arterial signals. These are all signs of venous insufficiency, and emergent exploration is indicated to assess the vascular pedicle for kinking or thrombosis. Application of leeches will drain excess blood from the flap but will not address the underlying problem. Observation is unacceptable because there are signs of venous insufficiency, and this requires urgent intervention. Systemic heparin will prevent further clot formation but will not dissolve an acute clot or resolve pedicle kinking. Systemic tissue plasminogen activator would greatly increase this patient's risk of bleeding. This agent should only be used within a flap.

2015 A 43-year-old woman is evaluated 6 hours after undergoing delayed breast reconstruction with deep inferior epigastric artery perforator flaps. On Doppler examination, arterial signals are present. Capillary refill time is 3 seconds on the right and 1 second on the left. A photograph is shown. Which of the following is the most appropriate management of the left breast? A ) Administration of systemic heparin B ) Administration of systemic tissue plasminogen activator C ) Application of leeches D ) Return to the operating room E ) Observation

The correct response is Option B. Hypertrophic scars generally arise during the first few weeks following the initial scar, grow rapidly, and then regress. On the other hand, keloid scars appear later following the initial scar, and then gradually proliferate, often indefinitely. Both keloid and hypertrophic scars demonstrate increased fibroblast density. Keloid scars demonstrate increased fibroblast proliferation rates compared with hypertrophic scars. Keloid scars demonstrate a decreased ratio of type III to type I collagen. This is not observed in hypertrophic scars. Keloid scars demonstrate thicker, larger, and more randomly oriented collagen fibers compared with hypertrophic scars.

2015 A 45-year-old man sustains a facial laceration and develops a keloid scar. Compared with a hypertrophic scar, this patient's scar is most likely to have which of the following characteristics? A ) Decreased fibroblast density B ) Increased fibroblast proliferation rates C ) Increased ratio of type III to type I collagen D ) Regression of the scar over time E ) Smaller and thinner collagen fibers

The correct response is Option E. The defect is a full-thickness skin defect with exposed subcutaneous fat. Coverage with a local flap and skin graft are possible reconstructive options. If skin grafting were to be performed, then a full-thickness skin graft would minimize the amount of long-term contracture (secondary contracture). A skin graft can be either split- or full-thickness. Split-thickness grafts can be harvested with a mechanical dermatome, or free hand with a scalpel. Split-thickness skin grafts do not include the entire thickness of the dermis and have less initial contracture at the time of harvest (primary contracture). Full-thickness skin grafts contain both the epidermis and the dermis and would have less long-term contracture. Harvesting with a mechanical dermatome is useful for obtaining split-thickness skin grafts. Typical thicknesses may range from 8/1000th of an inch to 14/1000th of an inch. Adequate immobilization of a skin graft is critical for "take" of the graft. Negative pressure wound therapy can be employed to promote adherence of the graft to the wound bed. Defatting of the skin graft in the case of a full-thickness graft is important to optimize graft "take" in the early postoperative period. Obtaining skin from a donor site with a close color match to the recipient site is an important aesthetic consideration, but does not affect long-term contracture.

2015 A 45-year-old man undergoes excision of a basal cell carcinoma of the mid cheek. A 3.6 × 4.7-cm oval skin defect with exposed subcutaneous fat is present. Skin grafting is planned. Which of the following is most likely to minimize long-term scar contracture? A ) Choosing a donor site with appropriate color match B ) Defatting of the skin graft C ) Harvesting with a mechanical dermatome D ) Immobilizing with negative pressure wound therapy E ) Including the entire dermis

The correct response is Option B. The area below the umbilicus and above the incision is most likely to have decreased sensation in the long term (after more than 3 years). This is true for superficial touch, superficial pain, temperature (hot and cold), and vibration. The area above the umbilicus recovers sensation to touch, pain, and vibration within 3 years; sensation to hot and cold temperatures recovers beyond 3 years. The suprapubic area has decreased sensation to temperature; however, touch, pain, and vibration recover in the short term. The areas below the xiphoid, at the lateral abdomen, as well as all other areas on the abdomen, have decreased sensation to pressure, although the infraumbilical region had the greatest change. The anterior nerve branches of the 6th to 12th intercostal nerves travel in a plane between the transversus abdominis and internal oblique muscles. They enter the internal oblique fascia, divide into two branches, and enter the posterior rectus sheath. One branch ascends 3 cm from the lateral edge of the rectus muscle to supply sensation to the skin over the lateral half of the rectus muscle. The other branch travels between the rectus and the posterior sheath before penetrating the linea alba and supplying sensation to the medial half of the rectus muscle. These nerves are divided during undermining from the abdominoplasty.

2015 A 45-year-old woman comes to the office seeking consultation for an abdominoplasty. Physical examination shows diastasis recti and excess skin and fat in the infraumbilical region. Abdominoplasty with undermining of the upper abdominal flap and plication of the diastasis is planned. Decrease in overall sensation in which of the following areas is most likely after more than 3 years postoperatively? A ) Epigastric B ) Infraumbilical C ) Lateral abdominal D ) Subxiphoid E ) Suprapubic

The correct response is Option D. The scleroderma spectrum of disorders includes localized scleroderma and systemic sclerosis, both of which are characterized by thickened sclerotic skin. Localized scleroderma involves only the skin, whereas systemic sclerosis is associated with extracutaneous involvement and is subcategorized into limited and diffuse forms. Limited cutaneous systemic sclerosis is restricted to the hands, distal forearm, face, and neck. Limited scleroderma is sometimes referred to as CREST syndrome, which is an acronym for the following features: calcinosis, Raynaud syndrome, esophageal dysmotility, sclerodactyly, and telangiectasia. Diffuse cutaneous systemic sclerosis includes truncal involvement (chest, abdomen, upper arms, shoulders). In most patients with systemic sclerosis, there is gastrointestinal involvement. Symptoms are present in more than half of patients and most commonly are related to gastroesophageal reflux disease, resulting in symptoms such as heartburn and dysphagia. Other findings in systemic sclerosis include: diarrhea, mucocutaneous telangiectasia of the face/lips/oral cavity/hands, dyspnea on exertion/interstitial pulmonary disease, and digital infarctions/pitting. Amyloid infiltration of the skin may produce thickening and stiffness. Telangiectasias are not a feature of amyloidosis. Gastrointestinal involvement with amyloid typically does not cause esophageal reflux or heartburn. Dermatomyositis is an idiopathic inflammatory myopathy characterized by skin and muscle abnormalities. Cutaneous involvement manifests in the form of distinct rashes, such as of the upper eyelids (heliotrope rash), hands (Gottron sign), and chest and shoulders (shawl sign). However, thickened skin of the trunk and limbs, and telangiectasias are not features of dermatomyositis. Hypothyroidism can result in cutaneous and dermal edema (myxedema). Other findings include fatigue, cold intolerance, weight gain, constipation, dry skin, myalgia, and menstrual irregularities. It is not associated with telangiectasias. Systemic lupus erythematosus (SLE) is a chronic inflammatory disease that can affect the skin, joints, kidneys, lungs, nervous system, serous membranes, and/or other organs of the body. Mucocutaneous changes include butterfly rash, erythematous plaques (discoid lesions), and ulcers. SLE is not associated with telangiectasias.

2015 A 45-year-old woman is evaluated because of a 1-year history of skin abnormalities. Physical examination shows areas of thickened skin involving the forearms and hands. Telangiectasias are seen on the face and oral cavity. A review of systems discloses symptoms of heartburn and dysphagia. Which of the following is the most likely diagnosis? A ) Amyloidosis B ) Dermatomyositis C ) Hypothyroidism D ) Scleroderma E ) Systemic lupus erythematosus

The correct response is Option D. In some cases, patients with systemic lupus erythematosus (SLE) can develop antibodies against platelet membrane phospholipids, increasing adhesion and aggregation of platelets, and producing a state of hypercoagulability. Although not all patients with SLE will develop these antibodies, a history of SLE should raise concern when evaluating a patient for a free flap reconstruction. All of the other options are autoimmune diseases, but none are typically associated with coagulopathies.

2015 A 48-year-old woman is evaluated for immediate bilateral breast reconstruction using coverage with deep inferior epigastric artery perforator free flaps. Which of the following conditions is most likely to be associated with hypercoagulable state? A ) Celiac disease B ) Graves disease C ) Rheumatoid arthritis D ) Systemic lupus erythematosus E ) Type 1 diabetes mellitus

The correct response is Option E. Accidental self-injection of epinephrine with an EpiPen occurs in 1 in 50,000 syringes. There have been no documented cases of digital necrosis following injection, and observation is indicated in this case. The effect of epinephrine's vasoconstriction lasts for approximately 90 minutes and will likely resolve on its own. Phentolamine has been described as a reversal agent for epinephrine and has shown clinical efficacy, but there has been no study to show that there are better outcomes with phentolamine injection compared with observation. Also, the added volume of injection with phentolamine could cause pressure necrosis and the timing between EpiPen injection and definitive treatment would usually be outside of the 90 minutes it would take for the epinephrine to wear off on its own. Topical nitroglycerin paste and subcutaneous injection of calcium channel blockers such as nifedipine have not shown to be viable treatment modalities. Application of a hot pack can lead to increased tissue damage and burns and is not indicated. Emergent exploration is indicated for compartment syndrome and is not indicated in this case where the finger is soft.

2015 A 5-year-old boy is brought to the emergency department 45 minutes after accidentally injecting his palm with epinephrine from an auto-injector (EpiPen). On physical examination, the ring finger is soft and pale, and capillary refill time is poor. A small puncture mark is noted on the flexor surface of the palm just proximal to the metacarpophalangeal (MCP) joint. Which of the following is the most appropriate next step in management? A ) Application of a hot pack B ) Emergent operative exploration C ) Subcutaneous injection of nifedipine D ) Topical nitroglycerin paste E ) Observation

The correct response is Option D. The risk factor in this patient associated with the greatest risk of late recurrence is ischial ulcer location. The treatment of pressure ulcers remains a difficult problem for medical practitioners and plastic surgeons. In a recent study, risk factors were identified as predictors of flap failure after resection and reconstruction with a flap. Factors associated with dehiscence include age less than 45 years, history of previous same-site surgery failure, albumin concentration of less than 3.5 mg/dL, and poor diabetic control (hemoglobin A1c of more than 6%). Factors associated with recurrence include ischial wound location, previous same-site surgery failure, and poor diabetic control. Osteomyelitis confirmed by bone biopsy and size of defect were not associated with dehiscence or recurrence. Patients with multiple risk factors showed a dramatic trend toward dehiscence and flap failure. Other studies corroborate the finding that ischial ulcer location is difficult for achieving coverage. This is most likely secondary to the high pressure over the ischial tuberosities in the seated position. Pressure maps have shown that, when seated, the pressure exceeds 80 to 100 mmHg compared with supine sacral pressures of 60 mmHg.

2015 A 50-year-old man with paraplegia and diabetes mellitus is evaluated because of an ischial stage 4 pressure ulcer. Optimizing the patient's nutrition and diabetic control is attempted, and aggressive resection of the ulcer and underlying bone with a locoregional flap closure is planned. Which of the following characteristics places this patient at increased risk for late recurrence of the ulcer? A ) Age greater than 45 years B ) Defect size greater than 20 cm2 C ) Hemoglobin A1c of less than 6% D ) Location of ulcer E ) Osteomyelitis confirmed by bone biopsy

The correct response is Option D. The lesion is a seborrheic keratosis, also known as verruca senilis or pigmented papilloma. Seborrheic keratoses are common benign lesions that may begin to appear during the fifth decade of life. They arise from the basal layer of the epidermis and are composed of well-differentiated basal cells. They often contain cystic "inclusions" of keratinous material called "horn cysts." Lesions exhibit hyperkeratosis (thickening of the stratum corneum), acanthosis (diffuse epidermal hyperplasia and thickening of the skin), and papillomatosis (skin surface elevation). The growth and depth of pigmentation vary directly with exposure to sunlight. If left untreated, they will enlarge gradually and increase in thickness. Seborrheic keratoses typically do not involute spontaneously. They may appear on the head, neck, and trunk after age 50 years and are often distinctly marked and have a waxy, stuck-on appearance. The surface is soft and oily to the touch. Sizes can range from 1 mm to 5 cm. No treatment is necessary, but these lesions are cosmetically unappealing, and for that reason alone, patients may request to have them removed. It is extremely rare for cutaneous malignancies to develop within seborrheic keratoses. A verrucous carcinoma is a variant of squamous cell carcinoma. As such, it requires wide local excision with negative margins for treatment. Nevus sebaceous (Jadassohn nevus) may be described as cerebriform, nodular, or verrucous. It is hairless and can appear on the scalp, face, or neck. When present at birth, it persists throughout life and tends to become more verrucous and nodular during the growth phase associated with puberty. Over time, these lesions are associated with a risk of basal cell carcinoma, which occurs in approximately 15 to 20%. Keratoacanthoma and squamous cell carcinoma may also develop, although with much less frequency than basal cell carcinoma. Because of the risk of malignant transformation, complete excision is generally recommended. Melanoma results from malignant transformation of the melanocyte; the pigment-producing cell of the body can occur anywhere melanocytes are present, including skin, eye, and the mucous membranes of the upper digestive tract, sinuses, anus, and vagina. The incidence of cutaneous melanoma in the United States has increased steadily over the past 50 years and is now 15 per 100,000. Worldwide, the incidence of melanoma is generally reflective of variation in genetic, phenotypic, and ultraviolet (UV) exposure risk factors. Major risk factors include exposure to UV radiation and genetic predisposition. The exposure risk primarily involves intermittent, damaging exposure to the sun such that history of a severe sunburn (blistering or pain for more than 2 days), even in youth, confers an approximately twofold increase in risk. Patients who have fair skin, blue eyes, red hair, and are prone to freckling are at increased risk for melanoma. Mutations in two genes are associated with hereditary melanoma predisposition.

2015 A 55-year-old woman comes to the office because of a 3-year history of a raised, dark brown lesion on her back. The lesion is not painful and does not bleed. A photograph is shown. An excisional biopsy is performed. Pathologic examination shows proliferation of cells from the basal layer of the epidermis with cystic inclusions. The lesion exhibits hyperkeratosis, acanthosis, and papillomatosis. Which of the following is the most likely diagnosis? A ) Basal cell carcinoma B ) Malignant melanoma C ) Nevus sebaceous D ) Seborrheic keratosis E ) Verrucous carcinoma

The correct response is Option D. The most reasonable approach in this patient is to offer reassurance and reset her expectations. A patient with a BMI of 35 kg/m2 who undergoes mastectomy and implant reconstruction will never look like a model with augmented breasts. This patient clearly has misguided expectations. Any surgical intervention is unlikely to produce the result she is looking for, when in fact she has a very acceptable result as is. Referral of this patient to a psychiatrist will likely upset the patient and undermine her trust.

2015 A 55-year-old woman comes to the office for a second opinion because she is displeased with the results of a recent bilateral mastectomy and breast reconstruction with 800-mL high-profile silicone implants. A photograph is shown. BMI is 35 kg/m2. She repeatedly shows pictures of models with augmented breasts and says that she wants her breasts to be "perkier." She requests augmentation/mastopexy. Which of the following is the most appropriate next step in management? A ) Augmentation/mastopexy B ) Implant exchange C ) Mastopexy D ) Reassurance E ) Referral to a psychiatrist

The correct response is Option D. Although recurrence rates are generally very high for large ventral hernias, placement of mesh in the retrorectal position appears to have the most decreased rate of recurrence compared with other methods. Placement above the plane of the rectus muscle requires division of vascular perforators that traverse the rectus muscle and perfuse the overlying skin flaps. These perforating branches of the epigastric circulation are most dense in the periumbilical zone. Preservation of the perforators has been shown to be beneficial in a number of case series reports. Placement of mesh between the oblique muscles in this case would not provide support for the midline hernia because these muscles are more lateral. Hernia recurrence rates are generally more increased in the presence of infection, with large defects when the rectus muscles cannot be advanced back together in the midline, in obese patients, and in patients with multiple medical comorbidities. Patients with several risk factors can expect recurrence rates in the range of 20 to 40%, whereas patients with few risk factors have recurrence about 5% of the time.

2015 A 56-year-old man is evaluated for a ventral hernia after undergoing midline laparotomy for diverticulitis. BMI is 38 kg/m2. Physical examination shows midline fascial defect measuring 20 × 15 cm; there is no evidence of infection and skin coverage is stable. Repair with rectus advancement and polypropylene mesh is planned. Placement of mesh between which of the following planes is most likely to decrease this patient's risk of hernia recurrence? A ) Anterior rectus sheath and rectus muscle B ) Internal and external oblique muscles C ) Medial edges of rectus muscle D ) Rectus muscle and posterior sheath E ) Skin and anterior rectus sheath

The correct response is Option E. The most appropriate management of this wound is serial debridement and delayed closure. With such a high level of contamination of both the soft and hard tissues, layered closure after the initial debridement will very likely lead to infection, especially in a patient with a history of smoking. It would be a mistake to perform a free tissue transfer in a highly contaminated wound. Furthermore, there is no missing or ischemic tissue. Negative pressure wound therapy followed by skin grafting would not be appropriate for a deep wound with bone exposure when local tissues are available for closure; this would be more appropriate for a superficial wound with missing skin. Healing by secondary intention is an option; however, serial debridement and delayed closure will take less time, is less painful, and avoids scar formation in the heel.

2015 A 56-year-old man who is an active smoker sustains a degloving injury of the left foot from a motorcycle collision. The heel was avulsed from the calcaneus by a deep posterior laceration but has normal capillary refill. No tissue is missing, but the wound is heavily contaminated and the calcaneus has an abrasion that is imbedded with grit. After irrigation and debridement in the operating room, which of the following is the most appropriate next step in management of this wound? A ) Coverage with a free gracilis muscle flap B ) Healing by secondary intention C ) Layered closure over a drain D ) Negative pressure wound therapy and skin grafting E ) Serial debridement and delayed closure

The correct response is Option E. In the clinical scenario described, the vertical rectus abdominis musculocutaneous (VRAM) flap is the most appropriate choice. This approach can provide enough tissue to resurface the vaginal vault and fill dead space. For posterior defects of the vaginal vault, abdominal-based flaps are usually preferable. They can provide a large amount of vascularized tissue that rotates easily into the defect. Bilateral gracilis muscle flaps alone would offer less soft tissue, and mucosalization in a radiated field is unpredictable. A deep inferior epigastric artery perforator flap is far more complicated than a VRAM flap, and the donor site is problematic in patients with a history of smoking. Primary closure of a large vaginal defect would likely cause significant stenosis and poor healing because of previous radiation. A skin graft with obturator is not likely to be successful given her radiation and tobacco history and violation of the rectum.

2015 A 57-year-old woman undergoes resection of a squamous cell carcinoma of the vagina, resulting in a defect of the posterior two thirds of the vaginal vault from the introitus to the dome. A small portion of the rectum is involved in the resection and primarily repaired. The patient has a history of pelvic radiation therapy and tobacco use. Which of the following is the most appropriate approach for closure of this defect? A ) Bilateral gracilis muscle flaps B ) Deep inferior epigastric artery perforator flap C ) Primary repair of the vaginectomy defect D ) Split-thickness skin grafting with obturator E ) Vertical rectus abdominis musculocutaneous flap

The correct response is Option A. Acellular dermal matrices (ADM) have been advocated for the past decade as an important adjunct in the complex field of abdominal wall reconstruction. Many studies have verified the use and general safety of ADM in abdominal wall reconstruction, but conclusive evidence of its advantages over other techniques is still lacking. What can be inferred is its advantage over prosthetic mesh in contaminated fields. Polypropylene mesh would be contraindicated in the infected wound in this example. After adequate debridement and appropriate antibiotics in an optimized patient, ADM can be used for hernia repair or reconstruction along with component separation. Postoperative infections can be as common as 40%, but conservative management measures often suffice, rather than reoperation and graft explantation, which are required with prosthetic mesh. One recognized drawback in the stretchable nature of dermal matrix grafts is that they can often stretch under tension to 50% or more of their initial dimensions. Postoperative bulging without true herniation is common. Strategies for prevention include suturing the graft under maximal stretch and use of porcine dermal grafts rather than human grafts. Hernia recurrence in the complex abdominal wall reconstruction remains a common event, regardless of technique. Although some studies assert a strong advantage with ADM, others report a similar or increased recurrence rate. As successful operative techniques become more standardized, perhaps more uniform success will be demonstrated in future studies. Skin necrosis and hematoma are common surgical complications that should not vary between choice of graft material. Fistula rates are decreased with ADM versus prosthetic mesh reconstruction.

2015 A 60-year-old man undergoes sigmoid resection and colostomy for management of ruptured diverticulitis. The patient has smoked one pack of cigarettes daily for the past 35 years. BMI is 36 kg/m2. After colostomy reversal, he has an abdominal wound infection and fascial dehiscence. Reconstruction with a bridging human acellular dermal matrix is planned. Compared with traditional polypropylene mesh repair, which of the following complications is more likely with the planned approach? A ) Abdominal bulge B ) Fistula C ) Hematoma D ) Infection E ) Skin necrosis

The correct response is Option B. The most appropriate option for this patient is a left latissimus dorsi muscle flap with skin graft over acellular dermal matrix, given the alternatives listed. Basic principles of thoracic reconstruction include: debridement of devitalized tissue, removal of foreign bodies, establishment of healthy wound bed, restoration of stability/structure (generally reconstruction of skeleton if more than four ribs or a greater than 5-cm-diameter defect is involved), restoration of normal respiratory mechanics, protection of vital structures/organs, obliteration of dead space, provision of durable coverage, and delivery of an aesthetic result. However, if a patient has been previously irradiated, and therefore loses compliance of the thoracic cage because of radiation-induced fibrosis, skeletal reconstruction may not bemandatory if there is no paradoxical motion of the thoracic cage upon respirations and there is preservation of respiratory efficiency. Such is the case with this patient. A left rectus turnover flap would not be a good option for two reasons: 1) as can be seen in the image, the left internal mammary artery has been harvested, thereby compromising the superior epigastric vessel on which this flap would be based, and 2) it is insufficient to provide enough soft-tissue coverage of a defect this size. Furthermore, as indicated above, methyl methacrylate would not be mandatory in this patient. A right pectoralis turnover flap is insufficient to cover a defect this size. A free omental flap can be used to reconstruct this defect (as can a pedicled omental flap), but again, thoracic skeletal reconstruction would not be mandatory in this previously irradiated patient; furthermore, even if it were, titanium mesh and reconstruction plates would not be utilized. A reverse abdominoplasty flap (Ryan procedure) would not be able to cover a defect this size.

2015 A 62-year-old woman presents with a new-onset draining sinus of the left thoracic cage with associated indurated skin. Medical history includes bilateral breast cancer that was managed with bilateral radical mastectomy with radiation therapy 27 years ago. On CT scan, the image (shown) is consistent with osteoradionecrosis. Resection of affected skin, soft tissue, and thoracic cage produces a 35 × 20-cm soft-tissue defect and a skeletal defect spanning five ribs. A photograph of the defect is shown. The thoracic cage is fibrotic and noncompliant because of previous radiation. Which of the following approaches is most appropriate for reconstruction? A ) Free omental flap with skin graft over titanium mesh and reconstruction plates B ) Left latissimus dorsi muscle flap with skin graft over acellular dermal matrix C ) Left rectus abdominis turnover flap with skin graft over methyl methacrylate sandwich D ) Reverse abdominoplasty advancement over ePTFE patch E ) Right pectoralis muscle turnover flap over polypropylene mesh

The correct response is Option C. Venous thromboembolism (VTE) is a disorder with short-term mortality and long-term morbidity. Plastic and reconstructive surgery patients are known to be at high risk for VTE after surgery. Symptomatic VTE occurs with high frequency after post-bariatric body contouring (7.7%), abdominoplasty (5%), and breast or upper body contouring (2.9%). To fully identify VTE risk in surgical patients, individualized patient assessment is advocated. The Caprini risk assessment model (RAM) is a useful and effective tool to stratify surgical patients for VTE risk. For patients with high Caprini scores, a significantly greater likelihood of VTE events is observed. Approximately 11% of patients with Caprini score >8 will have a VTE within 60 days after surgery. Based upon recommendations from the ASPS VTE Task Force, patients undergoing elective plastic and reconstructive surgical procedures who have Caprini RAM score of 7 or more should have VTE risk reduction strategies employed, such as limiting operating room times, weight reduction, discontinuation of hormone replacement therapy, and early postoperative mobilization. Patients undergoing major plastic and reconstructive operative procedures performed during general anesthesia that last longer than 60 minutes should receive VTE prevention. For patients with Caprini score of 3 to 6, the use of postoperative low-molecular-weight heparin (LMWH) or unfractionated heparin (UH) should be considered. For patients with Caprini score of 3 or more, use of mechanical prophylaxis throughout the duration of chemical prophylaxis for non-ambulatory patients should be considered. For patients with Caprini score of 7 or more, the use of extended LMWH postoperative prophylaxis should be strongly considered. Aspirin does not decrease the risk of VTE and may increase the risk of perioperative complications.

2015 A 62-year-old woman with a history of Stage III breast cancer is scheduled for delayed autologous breast reconstruction from the abdominal donor site. She has no other medical problems. BMI is 30 kg/m2. Her mother had a lower extremity deep venous thrombosis in the past. Caprini risk assessment score is 9. Which of the following is the most appropriate method of postoperative VTE risk reduction? A ) Aspirin therapy B ) Early ambulation after surgery C ) Low-molecular-weight heparin therapy D ) Sequential compression device use E ) No VTE prevention is indicated

The correct response is Option C. The most likely factor to indicate rigid chest wall reconstruction in this very large anticipated chest wall defect is the total area of the bony chest wall that is resected. Generally, reconstruction of the lateral bony chest wall seems necessary if four or more consecutive ribs are resected or if the diameter of the total defect is larger than 5 cm. Historically, protection of a flail segment of chest wall was based on the pendelluft principle, a phenomenon in which there is airflow back and forth between the lungs, resulting in underventilated segments of lung. This out-of-phase movement of the airway gas between the intact and flail-chest-side lungs was long believed to be the major contributor to respiratory dysfunction in patients with flail chest but has failed to be proven and appears to be a flawed hypothesis. In patients who have had radiation therapy, larger defects may be tolerated without rigid chest wall stabilization owing to fibrosis. As many as five ribs may be resected in patients who have undergone radiation therapy before reconstruction is considered because increased fibrosis produces chest wall stiffness. Anterior and posterior defects are typically better tolerated than lateral defects.

2015 A 65-year-old man is evaluated for a large mass in a previously irradiated area of the posterolateral chest. Preoperatively obtained ventilation-perfusion scans and pulmonary function testing are within normal limits. A full-thickness chest wall resection to include three ribs and at least 10 cm of each rib is planned. The soft-tissue defect is anticipated to be 20 cm in diameter. Which of the following factors in this patient is most likely to necessitate a rigid chest wall reconstruction? A ) Posterolateral position of the defect B ) Previous chest wall radiation therapy C ) Total area of the chest wall defect D ) Total number of ribs resected

The correct response is Option A. The most likely diagnosis is involutional or senile ptosis. In evaluating a patient requesting a blepharoplasty, one must also evaluate for blepharoptosis. If a blepharoplasty is performed without correction of ptosis, the patient will have continuing ptosis and potentially be dissatisfied. The most common type of blepharoptosis is involutional or senile ptosis. A thorough history and physical examination should assess ophthalmologic and neurologic causes. On physical examination, there is a characteristic high skin crease (greater than 7 mm), thinned upper eyelid, and lid drop on downward gaze. The levator function (amount of lid excursion with the brow and frontalis muscle held in neutral position) should be assessed. Normal function is greater than 10 mm, moderate function is 5 to 10 mm, and poor function is less than 5 mm. The etiology of ptosis can be classified into neurogenic (oculomotor nerve palsy, Horner syndrome, Marcus Gunn jaw-winking syndrome), myogenic (myasthenia gravis, myotonic dystrophy, mitochondriopathy), mechanical (edema or tumors), traumatic (birth trauma, muscle or nerve damage), congenital, or neurotoxic (such as in envenomation, snake bites, or botulism). None of these are likely, considering this patient's history and physical examination.

2015 A 65-year-old woman comes to the office for consultation on blepharoplasty and rhytidectomy. She notes that her eyelid creases are asymmetric, with the right upper crease higher than the left. She also has difficulty seeing out of her right eye when she is tired. She notes that these symptoms have worsened progressively over the past 5 years. She has levator function of 11 mm with the brow and frontalis muscle in neutral position. This patient most likely has which of the following types of ptosis? A ) Involutional B ) Mechanical C ) Myogenic D ) Neurogenic E ) Traumatic

The correct response is Option E. Tissue expansion effectively expands hair-bearing scalp that can then be rotated, advanced, and/or transposed over the area of alopecia to correct the deformity. A latissimus dorsi musculocutaneous free flap does not contain hair, and thus, would cause a similar area of alopecia. Follicular hair transplantation would not be efficacious for a large area of alopecia, and the split-thickness graft over periosteum would be insufficient tissue to accept the grafts. Serial excision of the skin graft is not possible because of the large defect and absence of normal scalp tissue on either side of the defect. A full-thickness graft from the hair-bearing scalp would cause a similar area of donor site alopecia.

2015 A 9-year-old girl is evaluated because of an 8-month history of alopecia of the scalp caused by a skin graft over the periosteum. A photograph is shown. Which of the following is the most appropriate method to correct this patient's alopecia? A ) Coverage with a latissimus dorsi musculocutaneous free flap B ) Follicular hair transplantation C ) Full-thickness skin grafting from hair-bearing scalp D ) Serial excision of the skin graft E ) Tissue expansion

The correct response is Option B. According to the U.S. Department of Health and Human Services, "the Health Insurance Portability and Accountability Act (HIPAA) Security Rule establishes national standards to protect individuals' electronic personal health information (ePHI) that is created, received, used, or maintained by a covered entity. The Security Rule requires appropriate administrative, physical and technical safeguards to ensure the confidentiality, integrity, and security of electronic protected health information." To this end, HIPAA requires that ePHI be protected using encryption processes that have been tested by the National Institute of Standards and Technology and judged to meet this standard. Password protection can prevent access to electronic devices but does not allow for data on a device to be deleted in the event of a breach. De-identification of file names does not change the electronic information that is stored within a file and, therefore, is inadequate in protecting ePHI. Institutional review boards are primarily responsible for reviewing and monitoring research that involves human subjects and are generally separate from HIPAA-related regulations.

2015 According to the Health Insurance Portability and Accountability Act's Security Rule, which of the following is the basic requirement for storing patient photographs on a personal computer? A ) De-identification of patient names in files B ) Encryption using an approved process C ) Institutional review board approval D ) Password protection of the personal computer E ) Storage of patient photographs is not permitted on personal devices

The correct response is Option E. The most appropriate next step in management is to proceed with urgent fasciotomies of the upper extremity. Indications for emergency extremity escharotomy are the presence of a circumferential eschar with impending or established vascular compromise of the extremities or digits. Progressive flow reduction by Doppler ultrasound is the primary indication for escharotomy. When evaluating upper extremities, it is advised to Doppler ultrasound the palmar arch, not the wrist, so as to evaluate distal blood flow. Other indications include decreased capillary refill time (more than 2 seconds), cyanosis, and/or relentless deep pain progressing to numbness (in awake patients). Neurovascular integrity should be monitored frequently and in a scheduled manner. Capillary refill time, Doppler signals, pulse oximetry, and sensation distal to the burned area should be checked hourly. After the escharotomy, any continued increase in capillary refill time, decrease in Doppler signal, or change in sensation should lead to immediate further decompression via fasciotomy. A carpal tunnel release is vital during the fasciotomy. Escharotomies typically are performed at bedside under sterile conditions with intravenous sedation using electrocautery. The aim is to make surgical incisions through burned eschar to allow expansion of underlying tissues. In extremity escharotomies, full-thickness incisions along medial and lateral mid-axial lines should be made. Escharotomies should be carried to just beyond the area of the full-thickness burn. Digital escharotomies are performed along the mid-axial line between neurovascular bundle and extensor apparatus. The ideal side to perform escharotomy allows for preservation of pinch: thumb requires radial incision only, and the index finger, long finger, ring finger, and little finger require ulnar incisions only. In the scenario described, further fluid boluses, escharotomies beyond the standard releases, and management would not be appropriate next steps, and would increase the risk of local complications like further ischemia, tissue necrosis/gangrene, or systemic complications like hyperkalemia, metabolic acidosis, and renal failure.

2015 An otherwise healthy 35-year-old woman is brought to the burn unit because of severe scalding injury to 50% of the total body surface area. Physical examination shows circumferential full-thickness burns to the right upper extremity, and absent distal pulses. Capillary refill time in the fingers is more than 3 seconds. Appropriate airway control and fluid resuscitation are initiated. Emergent right upper extremity escharotomies are performed. Despite surgical intervention, the blood flow to the distal extremity remains poor based on capillary refill time and Doppler flows. Which of the following is the most appropriate next step in management? A ) Additional escharotomies B ) Administration of an intravenous fluid bolus C ) Angiography D ) Elevation of the extremity and reexamination in 45 minutes E ) Fasciotomy of the upper extremity

The correct response is Option C. The most appropriate method for wound closure is a radial forearm free flap, as it matches the thin skin and subcutaneous tissue of the temporal area and has a long pedicle that can reach recipient vessels in the neck. With a patient history of radiation therapy and superficial parotidectomy, the superficial temporal vessels are unlikely to be suitable recipient vessels. With a patient history of upper neck dissection, it is possible that suitable recipient vessels will only be found inferiorly, and thus, it is best to use a flap with a long, reliable pedicle. A split-thickness skin graft will likely fail in a radiated wound bed. A scalp rotation flap will likely include tissue in the radiation field, bring hair-bearing tissue into a non-hair-bearing area, and require a split-thickness skin graft for closure of the donor site. It could be considered in a patient who is not a candidate for free tissue transfer. The lateral arm flap and parascapular flap are slightly thicker than the radial forearm flap, but the principal reason to avoid these flaps in this patient is that their pedicle lengths are relatively short and may not reach the recipient vessels in the neck.

2015 An otherwise healthy 59-year-old woman is scheduled to undergo resection of recurrent squamous cell cancer of the right temporal area 5 years after initial resection, superficial parotidectomy, limited upper cervical lymphadenectomy, skin grafting, and adjuvant radiation therapy. The anticipated defect will be 6 × 8 cm, involving the skin and subcutaneous tissues. The superficial temporal vessels cannot be identified. Which of the following is the most appropriate method of wound closure? A ) Lateral arm flap B ) Parascapular flap C ) Radial forearm free flap D ) Scalp rotation flap E ) Split-thickness skin grafting

The correct response is Option D. The most appropriate surgical margin recommended for the clinical scenario described is 6 to 10 mm. Cutaneous squamous cell carcinoma (cSCC) is the second most common skin cancer after basal cell carcinomas. They are broadly categorized into low- and high-risk lesions depending on size, location, depth of invasion, recurrence, and patient factors such as immunosuppression. A 4-mm margin of healthy tissue is recommended for lower-risk lesions. This category includes well-differentiated tumors smaller than 2 cm in diameter that do not occur on the scalp, ears, eyelids, lips, or nose, and do not involve subcutaneous fat. Therefore, simple excision is most valuable in the treatment of small primary squamous cell carcinomas on the trunk, extremities, or neck, where tissue sparing is less essential. The recurrence rate after the excision of low-risk lesions ranges from 5 to 8%. A 6-mm margin of healthy tissue is recommended for lesions that are larger than 2 cm, invasive to fat, or in high-risk locations (i.e., central face, ears, scalp, genitalia, hands, feet). Given the cosmetic and functional impact of these wider margins, tumors in this latter category are often removed via Mohs micrographic surgery to achieve high cure rates while sparing healthy tissue. The depth of an excision should always include a portion of the subcutaneous fat. Cure rates following simple excision of well-defined T1 lesions may be as high as 95 to 99%. The generally accepted 5-year cure rate for primary tumors treated with standard excision is 92%; this rate drops to 77% for recurrent cSCC. No large randomized studies have addressed the issue of appropriate margin size in cSCC, as has been done for melanoma. The recommendations for margin size should be taken only as rough guidelines, with the understanding that large, aggressive lesions frequently have substantial extension beyond the apparent superficial boundary. Therefore, a surgeon's experience and judgment when planning surgical margins is paramount to the successful treatment of cSCC.

2015 An otherwise healthy 65-year-old man comes to the clinic because of a 3-cm ulcerated lesion of the scalp. There are no palpable regional lymph nodes. Chest x-ray study shows no abnormalities. A punch biopsy is performed and a diagnosis of well-differentiated squamous cell carcinoma is made. Wide local excision of the lesion is planned. Which of the following is the recommended minimum surgical margin in this patient? A ) 1 mm B ) 2 mm C ) 4 mm D ) 6 mm E ) 12 mm

The correct response is Option E. The surgical step required as part of the "delay" procedure in a distally based sural flap is division of the proximal lesser saphenous vein. The distally based sural flap is a neurofasciocutaneous flap used to reconstruct ankle, heel, and foot defects. The classically described and possibly most important arterial supply to the distally based sural flap is provided by septocutaneous perforators arising from the peroneal artery. The most distal of these is located 4 to 7 cm proximal to the lateral malleolus. However, there are at least three other sources described: fasciocutaneous perforators from the posterior tibial artery, venocutaneous perforators from the lesser saphenous vein, and neurocutaneous perforators from the sural nerve. The skin and fascia of the flap are drained primarily by the lesser saphenous vein. The lesser saphenous vein contains numerous valves that prevent retrograde blood flow. There are, however, one or more smaller collateral veins that run parallel to the lesser saphenous vein. These veins have anastomotic connections to the lesser saphenous vein, which can allow blood to bypass the valves of the lesser saphenous vein and flow in a retrograde fashion. In attempts to redirect blood flow and decrease the risk of flap necrosis and other complications, several authors have described sural flap delay procedures. Two distinct delay procedures have been described. In one, the flap is first elevated without completely incising the proximal edge of the skin island. A powder-free glove is then placed between the elevated fascia and the gastrocnemius muscle, and the skin is closed. Two weeks later, the flap is completely elevated and transferred into the defect site. This procedure has the goal of redirecting blood flow in a longitudinal direction before complete elevation of the flap. In the other technique, the flap is raised in its entirety and then sutured back into its donor site. The flap is then transferred into its recipient site as a second procedure. This technique allows the flap to become viable on its distal vascular pedicle before causing the additional trauma of transferring the flap, which can potentially compromise that pedicle. Division of the greater saphenous vein is not indicated because it is not in the vicinity of the flap. Similarly, division of the perforator 5 cm proximal to the lateral malleolus is not appropriate because this is the major pedicle supplying the flap.

2015 An otherwise healthy 65-year-old man is evaluated because of a 2-month history of a nonhealing wound to the back of the left heel. He has a history of smoking 50 packs of cigarettes yearly but quit 1 year ago. Physical examination shows a clean wound with exposed bone and palpable distal pulses in the lower extremities. Coverage with a distally based fasciocutaneous sural flap is planned. Because of the patient's history of smoking, a "delay" procedure is performed first. Division of which of the following is required for this procedure? A ) Distal greater saphenous vein B ) Distal lesser saphenous vein C ) Perforator 5 cm proximal to the lateral malleolus D ) Proximal greater saphenous vein E ) Proximal lesser saphenous vein

The correct response is Option E. Disciplinary actions can be taken for charging exorbitant fees, particularly of a non-contractual nature (e.g., emergency care). Fees are exorbitant when they are wholly disproportionate to the services rendered. The reasonableness of fees depends upon the novelty and difficulty of the procedures involved, the skill required to provide proper care, the time and labor required, the fee charged for similar services by similarly situated peers, and whether or not the patient had agreed in advance to the fee. Except in instances of emergencies or urgent and life-threatening disease or injury, nothing in this principle shall be construed to prohibit a member from requiring prepayment of professional fees for all elective surgical operations. Sanctions can be taken if the member participates in a charity raffle, fund-raising event, contest, or other promotion in which the prize is any procedure defined as a medical service that requires an incision. Examples of services that require an incision include, but are not limited to, rhytidectomy, augmentation mammaplasty, blepharoplasty, and liposuction. Examples of medical services that would not be considered procedures include, but are not limited to, injections (botulinum toxin, hyaluronic acid), microdermabrasion, and other skin surface treatments. Nonsurgical means do not fall under this category. Sanctions can be taken if the Web site contains a testimonial pertaining to the quality and efficacy of medical care if the experience of the endorser does not represent the typical experience of other patients or if, because of the infrequency and/or complexity of such care, results in other cases cannot be predicted with any degree of accuracy. Also, sanctions may be taken if it contains a testimonial or endorsement pertaining to the quality of the member's medical care or the member's qualifications if the endorser has been compensated by the member or a third party retained by the member for making such testimonial or endorsement. Honest, well-represented, unbiased, uncoerced testimonials are allowed. Disciplinary actions can be taken if the Web site contains photographs, images, or facsimiles of persons who falsely or deceptively portray a physical or medical condition, injury, or disease, including obesity, or recovery of relief therefrom; or if it contains photographs, images, or facsimiles of persons who have received the services advertised, but who have experienced results that are not typical of the results obtained by the average patient, without clearly and noticeably disclosing that fact; also, if it contains photographs, images, or facsimiles of persons before and after receiving services, which use different light, poses, or photographic techniques to misrepresent the results achieved by the individual.

2015 Based on the code of ethics of the American Society of Plastic Surgeons, which of the following actions, if committed by a member, can be cause for disciplinary action? A ) The board-eligible surgeon chooses to advertise in an airline magazine B ) The member chooses to place patient testimonials on his or her Web site C ) The member participates in a charity raffle, fund-raising event, contest, or other promotion in which the prize is nonsurgical facial rejuvenation D ) The member uses his or her own before-and-after photos on the Web site E) The out-of-network member charges wholly disproportionate fees for emergency care

The correct response is Option C. Implant rupture is one of the most common reasons for implant removal. While implant rupture can be associated with symptoms such as capsular contracture, it is often completely asymptomatic or "silent." For this reason, screening imaging to detect such ruptures has been recommended. According to the 2011 Update on the Safety of Silicone Gel-Filled Breast Implants published by the U.S. Food and Drug Administration, it is currently recommended that women with silicone implants get their first MRI 3 years after they receive the implants and every 2 years thereafter to detect silent ruptures. Since the rate of rupture increases the longer an implant is in place, not screening is unacceptable. Mammograms can detect extracapsular silicone when an implant ruptures, but they do not detect intracapsular ruptures. The accuracy of ultrasound largely depends on the skill of the ultrasound technologist, the type of equipment used, and the experience of the interpreting physician. Furthermore, ultrasound is limited in its ability to detect ruptures in the back wall of the implant and in the breast tissue behind it. CT scans can detect intracapsular silicone gel-filled breast implant rupture, but they are limited in their ability to detect extracapsular ruptures.

2016 36-year-old woman with genetic susceptibility to breast cancer is scheduled to undergo bilateral prophylactic mastectomy. She has elected to proceed with immediate single-stage reconstruction using permanent silicone implants and acellular dermal matrix. Which of the following is the most sensitive method to detect implant rupture in this patient? A ) CT scan B ) Mammography C ) MRI D ) Physical examination E ) Ultrasonography

The correct response is Option B. This young patient has limited, focal disease that has failed medical management. Local excision is useful for isolated, scattered individual, or linear lesions. More extensive disease that occupies a significant portion of the skin area may be better treated by complete axillary excision. In this younger patient with limited disease, direct excision of the affected areas is a less morbid approach. Negative pressure wound therapy and skin grafts may be optimal coverage treatments for extensive disease, and some local flaps have been described as well. This stubborn disease originates from the apocrine glands and can often be chronic and disabling. The axillae, groin, perineum, and submammary areas can all be affected. Initial treatments include local care, antibiotics, hygiene, and weight loss. Steroid injections may help in early, small lesions, but have limited effectiveness and are painful. After a 6-week course of antibiotics, this patient is not likely to benefit from more treatment. Only excision of the diseased apocrine glands is likely to be effective. Incision and drainage is a suitable treatment for a closed abscess associated with hidradenitis, but incising this patient's lesions would not improve her situation.

2016 A 15-year-old girl has two draining nodules of axillary hidradenitis suppurativa. Each lesion is approximately 10 mm in diameter. The remaining axillary skin shows no abnormalities. After a 6-week course of oral doxycycline and topical mupirocin, the lesions drain less but are still present. Which of the following is the most appropriate treatment for this patient? A ) Complete axillary excision and skin grafting B ) Excision and closure C ) Incision and drainage D ) Intravenous antibiotics E ) Kenalog injection

The correct response is Option E. No other tissue or donor site in the body will approximate the hair-bearing qualities of scalp tissue. Simply, the best replacement for scalp tissue is scalp tissue. In addition to calvarial coverage and wound closure, the reconstructive surgeon should strive for a cosmetically pleasing outcome. In this patient with nearly 40% scalp loss, tissue expansion will provide the optimal functional and aesthetic outcome. Tissue expansion should be considered when local tissue rearrangements are inadequate because of defect size, trauma, or unacceptable rearrangement or distortion of the hairline. Tissue expansion increases the amount of locally available tissue, preserves sensation, and maintains hair follicles and adnexal structures. Approximately 50% of the scalp can be reconstructed with expanded tissue with minimal change in hair density. Serial expansions can be performed if needed for ultimate closure. When performing expansion, the largest tolerated expander(s) should be placed in the subgaleal plane. Rectangular and crescent-shaped implants provide more expansion than circular devices. Expansion should continue until the expanded flap is approximately 20% larger than the expected defect to account for tissue recoil. Disadvantages of tissue expansion include prolonged time to complete the expansion process (up to 3 months not uncommon), need for at least two operations, and a high rate of complications (6 to 48%). Common problems include infection, implant exposure or extrusion, hematoma, flap necrosis, skull erosion, alopecia, and wide scars. Serial excisions are useful for smaller defects. The amount of scar and deformity is much less when using tissue expanders versus serial reductions and complex rotation flaps. Free tissue transfer is best utilized for reconstruction of total or near-total scalp defects, particularly in the setting of irradiation or oncologic reconstruction. The latissimus dorsi muscle free flap is one of the best choices because of its large area and long pedicle. With time, the muscle flap will atrophy and conform well to the underlying bone. Other good options include the radial forearm, anterolateral thigh, or parascapular flaps. The Orticochea and Juri flaps are ill suited for a defect this large (>30% of scalp) or in this location. Juri flaps (temporoparietal-occipital) are useful for frontal or frontoparietal defects. The flap is based on the parietal branch of the superficial temporal artery and is best used after surgical delay of the distal aspect. The Orticochea procedure (3-flap technique) was classically described for reconstruction of acquired defects of the occiput. Each flap in the design must include a vascular pedicle and the technique can be used to cover up to 30% of the scalp. The cosmetic outcome using Orticochea flaps is often inferior to what can be obtained using tissue expansion because of unnatural resultant hair orientation. For this reason, tissue expansion should be considered in lieu of Orticochea flaps if possible. Hair transplantation is not appropriate for a defect of this size and a scarred recipient site.

2016 A 17-year-old boy is evaluated for scalp reconstruction because of a burn sustained 10 years ago. On examination, there is patchy alopecia and hypertrophic scarring of the left side of the temporoparietal scalp that occupies approximately 35% of the total scalp. Which of the following is the most appropriate method for reconstruction in this patient? A ) Free latissimus dorsi musculocutaneous flap and skin grafting B ) Hair transplantation C ) Orticochea flap D ) Serial excision and closure E ) Tissue expansion

The correct response is Option A. Facial contour deformities can result from a variety of causes, can be unilateral or bilateral, and may range in severity. The most common cause is Romberg disease, progressive hemifacial atrophy. The cause is unknown but may be a localized form of scleroderma. Systemic lupus erythematosus and scleroderma are the most common connective tissue disorders resulting in soft-tissue atrophy with facial involvement. Systemic lupus erythematosus is an inflammatory connective tissue disorder of unknown cause that predominantly affects women (90%). The clinical course can be variable and involve almost all organ systems. Cutaneous manifestations include soft tissue atrophy, telangiectasia, and extremity ulcerations. Scleroderma refers to a spectrum of disorders characterized by diffuse fibrosis of the skin, blood vessels, synovia, and vital organs, such as the kidneys. Women are affected four times more frequently than men and the skin is involved in 95% of cases. The pathogenesis is unclear but may include vascular abnormalities (vasculitis), excess collagen deposition, and excess proteoglycan production in the involved tissues. Surgical options for correction of hemifacial atrophy include alloplastic implants, free dermis/fat grafts, fat injections, and fasciocutaneous free tissue flaps. For mild defects, a custom alloplastic implant composed of silicone or porous polyethylene may be acceptable, but risks such as skin breakdown with implant exposure, scar contracture, and poor long-term patient satisfaction make an alloplastic option less desirable. In the patient described, the large nature of the defect and thin nature of her skin make an alloplastic option a poor choice. Fat grafting using the techniques described by Coleman is a viable option for smaller defects. The large defect and need for multiple cycles of fat grafting make this a less desirable option in this patient. Various free flaps including omentum, parascapular, groin, and superficial inferior epigastric artery flaps have been described for correction of hemifacial atrophy. In this case, the anterolateral thigh flap is the most appropriate choice because it allows for harvest of a large amount of tissue, including skin, fat, fascia, and muscle. A gracilis free flap is a great option for facial reanimation, but it would not have enough size and bulk to correct this patient's deformity. Similarly, a pectoralis major flap would not be acceptable due to lack of bulk and inability to position the flap without tethering because of the pedicle. Additionally, it would be difficult to transpose the muscle high enough for full correction and an unaesthetic bulge would be notable in the neck.

2016 A 20-year-old woman comes to the office to discuss reconstruction for hemifacial atrophy. A photograph is shown. The atrophy began when she was 13 years old and stabilized by 16 years of age. Workup suggests mixed connective tissue etiology that falls within the lupus spectrum. Which of the following is the most appropriate intervention to correct this patient's deformity? A ) Buried anterolateral thigh free flap B ) Custom silicone prosthesis C ) Gracilis free flap D ) Injectable poly-L-lactic acid E ) Pedicled pectoralis major flap

The correct response is Option B. Although a bilaminate neodermis (Integra) artificial skin substitute has been traditionally used in the acute management of burns, there have been many successful reports incorporating its use in general reconstructive plastic surgery. Integra is a bioengineered dermal substitute consisting of a bilayer membrane system. The dermal replacement layer is composed of a cross-linked bovine tendon collagenglycosaminoglycan (chondroitin-6-sulfate) matrix coated on one side with a synthetic polysiloxane polymer (silicone) layer. This layer functions as an epidermis to control moisture loss from the wound. The collagen matrix serves as a scaffold for ingrowth of fibroblasts, macrophages, lymphocytes, and capillaries and is ultimately replacedby host tissue. Integra "take" mirrors skin graft "take" and follows the sequence of 1) imbibition, 2) fibroblast migration, 3) neovascularization, and 4) maturation. As the host tissue infiltrates the dermal layer, the collagen layer is biointegrated with the wound to form a vascular neodermis capable of accepting a split-thickness skin graft after a period of approximately 4 weeks. Proper patient selection and careful monitoring for infection or hematoma accumulation are crucial in this time period. After approximately 4 weeks, the silicone layer is removed and a thin (0.008- to 0.010-inch) skin graft can be applied. Advantages of Integra use include improved cosmesis, diminished scar contracture or development of hypertrophic scar, off-the-shelf availability in large quantities, increased elasticity, and the ability to use thinner skin grafts with subsequent improvement in donor-site morbidity, scar, and time to heal. Other cited advantages include commensurate growth of the grafted tissue in children, potential avoidance of a microsurgical procedure, and placement over tendons with no significant reduction in tendon mobility. The disadvantages of Integra include its high cost, steep learning curve, need for at least two stages with subsequent increased time to final closure, potential for infection under the silicone layer, and lack of adnexal structures with patient concerns of skin dryness. In this patient, use of Integra with delayed thin split-thickness skin grafts at 4 weeks will give the best outcome in terms of cosmesis and function. A photograph is shown. Split-thickness skin grafts are not a good option in this location because of the risk of graft loss with tendon exposure due to lack of viable paratenon and the risk of secondary contracture at the wrist and elbow. Full-thickness skin grafts would diminish the risk of secondary contracture but harvesting enough tissue to cover the entire arm would be impossible using the inguinal region as a donor site. Fullthickness grafts also require a healthier wound bed for optimal graft "take" and are not likely to heal over exposed tendons. Free tissue transfer is an excellent technique for resurfacing the forearm, but in this patient, the thigh donor site would be difficult secondary to tissue thickness and the need for an extremely large flap. A better choice could be the scapular or parascapular donor site, perhaps in combination with a preliminary tissue expansion of that location. Obtaining enough tissue would be difficult. Banking the forearm in the abdomen would be less desirable as well, given her obesity and the need for a large surface area.

2016 A 22-year-old woman is evaluated in the emergency department because of necrotizing fasciitis of the right upper extremity. BMI is 35 kg/m2. After adequate debridement, the entire extensor and flexor compartments are open with exposed tendon. A photograph is shown. Which of the following is the most appropriate initial method of reconstruction? A ) Anterolateral thigh free flap B ) Bilaminate neodermis C ) Full-thickness skin grafts D ) Pedicled abdominal flap E ) Split-thickness skin grafts

The correct response is Option C. On the basis of the scenario described, a fasciocutaneous free tissue transfer is the most appropriate management option to reconstruct the soft-tissue coverage of the tibia and foot extensor mechanism. There are several donor site options for the harvest of fasciocutaneous free tissue, and each flap has distinct characteristics. The scapular/parascapular flaps are frequently employed regionally for axilla, shoulder, posterior thorax, and upper extremity reconstruction. This donor site also provides a large surface area of fasciocutaneous tissue for distant use and is commonly used in cutaneous, head and neck, and upper and lower extremity reconstruction. The tissue transfer can measure 12 × 25 cm with primary closure of the donor site, pedicle length is 3 to 5 cm, arterial diameter is 2.5 to 4 mm, and venous diameter ranges from 1.5 to 4 mm. Due to the extent of the zone of injury, this pedicle length would be too short to minimize the need for vein grafts. The anterolateral thigh (ALT) flap has proven to be one of the most versatile free tissue transfers in reconstructive surgery. Based on perforators from the descending branch of the lateral circumflex femoral artery that traverse the vastus lateralis (VL) (80%) or the septum between the rectus femoris and VL (18 to 20%), this flap can be fashioned as large as 10 cm wide by 25 cm long. Its pedicle length can extend 10 to 12 cm, arterial diameter is 2 mm, and venous diameter is 2 to 3 mm. Due to the extent of the zone of injury, this pedicle length would be too short to minimize the need for vein grafts. The volar forearm donor site is utilized both regionally and in distant locations such as the head and neck, and upper and lower extremities. It is also employed in specialized areas such as esophageal and penile reconstruction. The skin paddle can extend 10 × 40 cm and pedicle length can extend up to 20 cm with similar arterial and venous vessel diameters when compared with the scapula and the ALT. In the scenario described, the pedicle characteristics of the radial forearm flap make this the best option for reconstruction and to decrease the need for vein grafting in this patient.

2016 A 25-year-old man has a 5 × 5-cm soft-tissue foot defect with exposed bone after trauma. Free tissue transfer reconstruction of the defect is planned. Intraoperatively, 15-cm proximal dissection of the pedicle is required to perform microvascular anastomoses outside of the zone of injury. Which of the following flaps will minimize the need for vein grafting in this patient? A ) Anterolateral thigh B ) Gracilis C ) Radial forearm D ) Scapular

The correct response is Option E. The most consistent complication of cartilage grafting is the propensity to change shape or warp over time. This may be due to the presence of perichondrium or the nonuniform composition of the matrix that can affect the shape when it is placed. Pure cartilage grafts tend to maintain shape, but grafts with an intact perichondrial layer can curl significantly and lead to unpredictable results. During septal graft harvest, care must be taken to elevate mucoperichondrial flaps in the proper plane. Likewise, auricular or costal cartilage grafts must be harvested in a subperichondrial plane. Removal of the perichondrium and softer outer cartilage layer leaves the more rigid cartilage core, which maintains shape more predictably. Extrusion, necrosis, ossification, and resorption are not known to be affected by the presence or absence of the perichondrial layer. Fresh autologous grafts easily survive transplantation procedures and do not appear to resorb over time. Autologous cartilage from septum, concha, or rib is considered the ideal graft material. These grafts have very low risk for infection or extrusion compared with an allograft. Cartilage grafts are tolerated well by nasal tissue.

2016 A 25-year-old woman is scheduled for correction of a deformity of the nasal dorsum caused by previous nasal trauma as a child. Autologous rib cartilage grafting of the dorsum is planned to correct the deformity. Which of the following complications is most likely in this patient? A ) Extrusion B ) Necrosis C ) Ossification D ) Resorption E ) Warping

The correct response is Option C. Currently, there are several techniques available for penile reconstruction in patients undergoing female-to-male sex reassignment. Multiple goals are sought, including good appearance, standing micturition, sexual sensation, coital ability, and sufficient bulk to accommodate a stiffener. In metoidioplasty, the clitoris is partially released and stretched, and the urethra is lengthened to reach the tip of the phallus using an anterior musculomucosal vaginal flap, which is done as a single-stage procedure. The advantages of this technique are a shorter procedure and lower risk of complications. Its disadvantages are a shorter phallus with decreased possibility of standing micturition. Phalloplasty, in which a phallus is created de novo, can be performed using a variety of possible flaps, including anterolateral thigh, fibula, groin, latissimus dorsi, and radial forearm. Phalloplasty can seldom be performed in one stage because of the need to create a competent neourethra and is associated with a 20 to 40% rate of urinary complications (fistulas, stenoses). Soft-tissue-only flaps typically involve placement of a stiffener, which can be prosthetic or autologous, while the fibula flap results in a permanent erection. Free flaps (radial forearm, latissimus, fibula) involve lengthier microsurgical procedures, while some pedicled flaps (groin) necessitate a minimum of two stages. Tissue expansion is not a method used for penile reconstruction in female-to-male sex reassignment surgery.

2016 A 25-year-old woman who is considering female-to-male gender reassignment surgery inquires about penile reconstruction. The patient is interested in a technique that would involve the fewest number of total procedures. Which of the following techniques would be most appropriate? A ) Free fibula flap B ) Free radial forearm flap C ) Metoidioplasty D ) Pedicled groin flap E ) Tissue expansion

The correct response is Option A. Form-stable silicone gel implants are fifth-generation, shaped, and textured implants that have additional cross-linking between molecules. They are purported to have several advantages over other round saline and silicone gel implants because they retain their shape and decrease the incidence of folding and rippling. This has translated into significantly lower capsular contracture rates. However, they do have some disadvantages. Because they are shaped and maintaining orientation is critical, they have a higher incidence of malposition. They are also more prone to seroma formation, which may be associated with their textured surface. Infection and resorption rates remain similar.

2016 A 28-year-old woman comes to discuss primary augmentation mammoplasty options and is deciding between form-stable shaped implants and less cohesive round silicone gel implants. She inquires about the benefits of each type of implant. Compared with smooth round silicone gel implants, highly cohesive formstable gel implants have a decreased incidence of which of the following? A ) Capsular contracture B ) Implant malposition C ) Infection D ) Seroma

The correct response is Option C. Increasing the cross-linking in a highly cohesive gel shaped silicone implant improves form stability. This allows for the creation of shaped implant designs that persist despite position or external forces on the implant. The current, fifth-generation silicone breast implants derive their cohesiveness from the cross-linking of the silicone. Increasing the amount of cross-linking leads to an increase in cohesiveness and a firmer implant. This may lead to less rippling and folding because of resistance to collapse; however, recent MRI studies have shown folds and distortions are still possible. Increasing cohesiveness, however, does have some disadvantages with potential risks for gel fracture and delamination of the implant shell.

2016 A 28-year-old woman comes to the office to discuss augmentation mammaplasty. She is interested in silicone implants, specifically highly cohesive gel shaped implants. Which of the following is the most likely result of increasing the crosslinking in these implants? A ) Decreased risk of gel fracture B ) Decreased risk of shell delamination C ) Improved form stability D ) Increased risk of folds E ) Softer implants

The correct response is Option E. The most likely diagnosis is kaposiform hemangioendothelioma, which is a vascular tumor that is usually present at birth. The lesion can cause Kasabach-Merritt phenomenon: extreme thrombocytopenia (<25,000/mm3) from platelet trapping within the tumor which leads to spontaneous bleeding and bruising. Treatment is either vincristine or rapamycin. Infantile hemangioma, congenital hemangioma, capillary malformation, and arteriovenous malformation do not cause Kasabach-Merritt phenomenon or any other type of coagulopathy.

2016 A 3-month-old male infant is brought to the office for evaluation of a 10 × 10-cm purple lesion of the right thigh that was present at birth. Medical history includes nosebleeds. On physical examination, bruising over the trunk is noted. Complete blood cell count shows thrombocytopenia. Which of the following is the most likely diagnosis? A ) Arteriovenous malformation B ) Capillary malformation C ) Congenital hemangioma D ) Infantile hemangioma E ) Kaposiform hemangioendothelioma

The correct response is Option D. Ribs five through seven are commonly described as the ideal sites for harvesting because of ease of access and ample bone material compared with the more cephalic or caudal ribs. The seventh rib has the added advantage of being situated over the abdominal cavity, reducing the risk for pneumothorax during harvest. Ribs five and six are also suitable, and may hide better in the inframammary crease of women. Ribs may be harvested either whole or split depending on the amount of material needed. If a whole-rib graft is required, as long as multiple adjacent ribs are not removed, morbidity is limited. Split-rib harvest offers the advantage of eventual bone regrowth, and the area can be harvested for bone grafting again in the future if needed. The eleventh rib provides a wide expanse of medial cartilage for harvest, although it is not an ideal bone donor site. Rib grafts offer both cortical and cancellous bone material. The cortical bone is rigid and provides immediate structural support, though final incorporation and strengthening occur over many months. A well-vascularized wound bed is required for bone healing and incorporation. A split-rib inset provides the cancellous bone surface, which is more rapidly revasculuarized. If an entire bone is needed for grafting, then a vascularized bone graft or flap with microvascular anastomosis would be better suited.

2016 A 30-year-old man requires bone grafting for repair of a severe nasal deformity after a bicycle collision. Which of the following is the optimal rib donor site for ease of exposure and limited morbidity? A ) First B ) Third C ) Fourth D ) Seventh E ) Eleventh

The correct response is Option C. The most appropriate option for this patient is to proceed to the operating room for decompression/drainage of the flexor tendon sheath. This patient has all four of Kanavel's signs, specifically pain on passive extension, fusiform swelling, flexor tendon sheath tenderness, and flexion of the affected digit. These point towards a diagnosis of flexor tenosynovitis, with the cause being the cat bite she sustained several days prior. Although cat bites only reflect 5% of all animal bites, they represent 76% of all infected bites, thought to be because of the morphology of their long teeth, which simulate a deep puncture wound. Outcomes studies demonstrate that because of this patient's diabetes, she is at risk for a poor outcome, specifically the need for amputation and/or decreased total active motion. Other risk factors that can lead to these outcomes include age greater than 43 years, presence of subcutaneous purulence, digital ischemia, and polymicrobial infection. Given that she is at risk and that she has all four Kanavel's signs, any intervention that is more conservative than operative decompression and drainage may lead to suboptimal outcomes. Administration of antibiotics (orally or itravenously) is not a substitute for drainage of the flexor sheath, especially in a case that presents more than 48 hours out from initiation of symptoms. Drainage in the emergency department is not as optimal as in the operating room given that the infection may limit efficacy of local anesthesia as well as the need for possible conversion to an open drainage procedure if closed catheter irrigation is insufficient.

2016 A 34-year-old woman comes to the emergency department 5 days after sustaining a cat bite to the left index finger. A photograph is shown. Medical history includes diabetes mellitus type 1. Physical examination shows punctures to the dorsum and volar surfaces of the proximal phalanx, mild fusiform swelling of the digit with tenderness over the flexor tendon sheath, pain with passive extension, and partially flexed posture of the digit. The symptoms have worsened over the past 3 days. Temperature is 99°F (37.2°C). Which of the following is the most appropriate next step in management? A ) Incision and drainage of the puncture sites B ) Inpatient intravenous antibiotics C ) Irrigation of the flexor sheath D ) Outpatient oral antibiotics E ) Splinting immobilization and elevation

The correct response is Option A. Ischial pressure ulcers can be one of the most difficult wounds for which to achieve long-term coverage and success. The best outcomes arise from multidisciplinary care teams that focus on pressure alleviation (both perioperative and chronic), nutrition, smoking cessation, muscle spasm management, and vigilant observation. despite best efforts, ischial pressure ulcers still have up to a 70% recurrence rate. Because of high recurrence rates, surgical planning needs take potential future wounds into consideration. Large rotation or advancement flaps have the benefit of being able to be re-rotated or re-advanced, whereas transposition flaps do not, and can make future surgery more difficult. With good surgical planning, rotation and transposition flaps would not have a difference in vascularity, scar placement, pressure relief, or recurrence rates

2016 A 35-year-old paraplegic man presents with a 5 × 5-cm pressure ulcer over his left ischium involving the skin, subcutaneous tissue, and bone. After debridement, a gluteal rotation flap is planned for closure. Compared with a transposition flap, which of the following is the most significant benefit of using a rotation flap for coverage of this patient's wound? A ) Ability to reuse flap for future surgery B ) Decreased recurrence rate C ) Improved pressure relief D ) Improved scar placement E ) Improved vascularity

The correct response is Option B. In a patient with a grossly dirty wound, it is appropriate to administer a tetanus shot. Tetanus (also known as lockjaw) is characterized by a prolonged contraction of skeletal muscle fibers. The primary symptoms are caused by tetanospasmin, a neurotoxin produced by Clostridium tetani, a gram-positive, rod-shaped, obligate anaerobe. Infection generally occurs through wound contamination and often involves a cut or deep puncture wound. In the acute period before definitive wound closure can be achieved, it is critical to debride all devitalized tissue such that there is a healthy, viable wound bed. The administration of broad-spectrum intravenous antibiotics has not been shown to decrease the risk of wound infection, and may, in fact, lead to the development of selecting out for resistant pathogens. The mainstay of treatment is performing repeat surgical debridement as often as necessary until the wound is clean. Debridement and cleansing of the wound are ideally performed in the operating room under controlled conditions; however, depending upon the condition of the patient, concomitant injuries, and the ability of the patient to tolerate the procedure, it may be necessary to perform a limited, conservative wound washout at the bedside or in the emergency department. Over the course of the initial hospital stay, debridement should continue until healthy tissue is encountered, which can be identified by visual inspection and the presence of punctate bleeding. The surgeon must consider several things when deciding between closure with a flap or a graft. The defect in this patient is too large to achieve primary closure. The use of negative pressure wound therapy for such a large wound may be helpful as a temporary measure, but, as a method of definitive wound closure, would result in healing by secondary intention, scarring, and prolonged wound care. If there were exposed bone, tendon, nerves, blood vessels, or significant dead space, this would make a stronger argument for a flap-over-skin graft. Although not provided as an option in this question, the use of biosynthetic materials or dermal matrix tissues has been reported in the literature as an intermediate step to skin grafting, but it is important to consider the necessity of these materials in effecting outcomes in light of the significant cost of using them. Split-thickness skin grafts can provide wound coverage over a large area. A mechanical dermatome is often used. Typical thicknesses may range from 8/1000th of an inch to 14/1000th of an inch. The graft can be meshed in various ratios such as 1:1.5, 1:2, and 1:3 to allow for a larger area of coverage per unit of harvested skin. It is important that the underlying wound bed be viable and free of necrotic tissue or infection to allow for healing of the skin graft ("skin graft take"). Adequate immobilization of a skin graft is important for "take" of the graft, and can be achieved with negative pressure wound therapy, or tie-over bolster dressing. The thigh has an abundant amount of soft tissue and muscle, which is why skin grafts are often sufficient for wound coverage rather than flaps.

2016 A 38-year-old unconscious and intoxicated woman is brought to the emergency department after being struck by a motor vehicle. She sustained multiple injuries, including a wound on the right thigh, which measures 12 × 18 cm with areas of exposed fat and muscle. There is dirt and gravel in the wound. Which of the following is the most appropriate next step in management? A ) Broad-spectrum antibiotic therapy B ) Injection of tetanus toxoid C ) Negative pressure wound therapy D ) Split-thickness skin grafting E ) Wound irrigation and debridement

The correct response is Option B. Calcium hydroxylapatite is a semipermanent material that can be injected as a soft tissue filler and lasts 1 to 2 years, which is longer than the 4 to 12 months that hyaluronic acid-based fillers last. Safe injection of this material includes prevention of overcorrection, prevention of clumping of filler due to bolus injections, injection in a subdermal plane, and postinjection massage. If nodules form (which usually occur in areas of thin soft-tissue coverage such as the eyelids, lips, and nasolabial region), there are multiple described effective treatments, which include direct excision, observation to allow for the product to resorb, and needle disruption and unroofing. However, whereas lumps caused by poly-L-lactic acid or polymethyl methacrylate respond well to intralesional steroids, these are not as effective in treating lumps caused by calcium hydroxylapatite.

2016 A 38-year-old woman seeks cosmetic enhancement of the nasolabial area to decrease deep folds. She has had good results from injection of hyaluronic acid-based soft-tissue fillers in the past but now desires a longer lasting result. Calcium hydroxylapatite is chosen based on this request. On follow-up examination 2 weeks later, white nodules are noted along the nasolabial folds. Which of the following is the LEAST effective treatment option to address the white nodules? A ) Direct excision of the filler B ) Injection of a corticosteroid C ) Massage of the folds D ) Needle disruption and unroofing of the lumps

The correct response is Option D. Heparin may be used locally or systemically during flap salvage attempts in an effort to encourage further propagation of clot and irrigate existing thrombus. Papaverine and thrombolytic agents are used locally on or within the flap vessels, but not systemically due to concern for systemic complications.

2016 A 38-year-old woman undergoes bilateral breast reconstruction using microvascular free tissue transfer from the abdomen. The patient is evaluated 8 hours later because the right breast flap appears mottled and engorged. Administration of which of the following is CONTRAINDICATED in this patient? A ) Heparin irrigation to the flap vessels B ) Papaverine to the flap vessels C ) Systemic heparin D ) Systemic thrombolytics E ) Thrombolytics to the flap vessels

The correct response is Option C. In a patient with an intravenous extravasation, if the symptoms are mild (pain, swelling, or erythema), elevation and cold compresses will usually lead to complete resolution. Patients who have more severe symptoms, such as neurovascular compromise, may need additional evaluation of compartment pressures and potentially surgical exploration for decompression; however, the first step in treatment is still elevation of the extremity. Extravasation injury was once a difficult problem. The extravasation of ionic contrast at high osmolality had increased risk for soft-tissue complications, and plastic surgery consultation was often necessary. The switch to nonionic, low-osmolality contrast over the past decade has resulted in a significant decrease in complications. In a recent study of 69,657 patients undergoing a CT scan, the rate of extravasation was 0.7% (476 patients), and only one patient required operative intervention and decompressive fasciotomy. Corticosteroids do not have a role in extravasation injuries.

2016 A 40-year-old woman has an extravasation injury following a CT scan. An automated power injector was used to inject 100 mL of nonionic contrast medium into an intravenous cannula on the dorsum of the hand approximately 60 minutes ago. On examination, the hand appears edematous with mild erythema and is moderately tender. The skin is intact with no blistering. Capillary refill time is normal, and there is no neurologic deficit. Which of the following is the most appropriate next step in management? A ) Administration of a corticosteroid B ) Bedside exploration of the intravenous cannula site C ) Elevation of the extremity D ) Surgical exploration with dorsal fasciotomy and carpal tunnel release E ) Surgical exploration with dorsal fasciotomy only

The correct response is Option B. The most significant advantage of the fibula flap over the forearm flap in penile reconstruction is the avoidance of a prosthesis to provide erectile and sexual function. Another advantage is the less conspicuous donor site. Both flaps allow for the creation of an innervated skin island. The fibula skin island can be innervated via the superficial peroneal nerve, and the forearm skin island can be innervated via the medial and lateral antebrachial cutaneous nerves. The forearm flap utilizes vascularized forearm skin for the neourethra, whereas the fibula flap employs a fullthickness skin graft. Thus, the incidence of urethral stricture is greater with the fibula flap. Both flaps are based on reliable and relatively constant anatomy, although the septocutaneous branch of the peroneal artery to the skin of the fibula flap can be variable or even absent. One disadvantage of the fibula flap is that sensation is not as good as with the forearm flap.

2016 A 42-year-old woman is scheduled to undergo penile reconstruction with a fibula flap as part of gender reassignment. She has been evaluated by all appropriate specialists in the context of a multidisciplinary transgender clinic. Which of the following is the most significant advantage of the fibula flap over the forearm flap in this patient? A ) Ability to create an innervated skin island B ) Avoidance of prosthesis to provide sexual function C ) Decreased incidence of urethral stricture D ) Improved sensory recovery E ) More reliable and constant anatomy

The correct response is Option E. Immediate breast reconstruction with tissue expanders followed by reconstruction of choice preserves the patient's skin envelope and keeps open options for definitive final reconstruction of choice whether with an implant or autologous tissue. Tissue expander-based reconstruction is associated with a higher complication rate in the setting of radiation therapy but preserves abdominal and back tissue as options for autologous reconstruction. Implant-based reconstruction does not provide the advantages of improved symmetry, sensation, or breast ptosis

2016 A 43-year-old woman would like to discuss plans for breast reconstruction after her upcoming unilateral mastectomy. Postoperative radiation therapy is planned. Which of the following is the most likely benefit of tissue expander-based breast reconstruction compared with immediate autologous breast reconstruction using this patient's abdominal tissue? A ) Better symmetry B ) Improved postoperative sensation C ) A larger, more ptotic breast reconstruction D ) Lower risk of complications from radiation E ) Preservation of the patient's options for final reconstruction

The correct response is Option C. The best reconstructive option for an ambulatory patient with a chronic ischial pressure sore is to spare muscle and use a fasciocutaneous flap. Fasciocutaneous flaps offer an axial blood supply with durable coverage and minimal potential for a functional deformity. The flaps closely reconstruct the normal anatomical arrangement over bony prominences. Disadvantages include limited bulk to fill large defects. Musculocutaneous flaps offer excellent blood supply and bulky tissue and can be rerotated like fasciocutaneous flaps. However, in an ambulatory patient, they may cause functional impairment. The tensor fasciae latae flap is occasionally used to close the ischial ulcer. Unfortunately, the distal aspect of the flap that is used to reach the ischial region is usually too thin to offer adequate padding to be effective. Therefore, this is not the most appropriate option. Primary closure is associated with an increased rate of wound recurrence. A large dead space is usually present due to the absence of tissue, and the induration of adjacent areas makes them difficult to mobilize for a tension-free closure.

2016 A 45-year-old man is hospitalized in the intensive care unit for 3 weeks after sustaining blunt force trauma to the head during an assault and develops a pressure ulcer over the right ischium. Six weeks later, the wound has not healed despite multiple bedside debridements and daily wound care. The patient is ambulatory. Physical examination shows an 8 × 4 × 4-cm stage III pressure ulcer at the right inferior gluteal fold with an indurated bursa. After nutritional optimization, which of the following surgical techniques is most appropriate for closure after excising the bursa? A ) Gracilis musculocutaneous rotation flap B ) Negative pressure wound therapy then primary closure C ) Posterior thigh fasciocutaneous rotation flap D ) Superiorly based gluteal musculocutaneous rotation flap E ) Tensor fasciae latae island rotation flap

The correct response is Option E. The most appropriate management of this injury is to repair the ulnar nerve primarily and perform a distal nerve transfer as well. The ulnar nerve is one of the two upper extremity nerves that supply motor input to the intrinsic muscles of the hand (the other being the median nerve). It supplies the interossei, hypothenar muscles, ulnar lumbricals, and the adductor pollicis muscles. These small muscles of the hand are vital for proper thumb and finger function. Because of their small size and delicate nature, they are very sensitive to denervation, and over a period of 9 to 12 months atrophy beyond repair. The aim of nerve repair is to reestablish nerve signals to the end-organ (i.e., muscle) prior to irreversible denervated muscle. The injury described is a good 18 inches or so away from the hand, and regeneration of the motor fibers to the intrinsic muscles would only occur after at a rate of an inch a month, by which time permanent atrophy would have already occurred. Primary repair alone can lead to suboptimal intrinsic muscle function despite excellent technical repairs. Distal nerve transfers help prevent the denervational atrophy by "babysitting" the muscles during the time it takes for the ulnar nerve to regenerate its motor fibers to the end-organ/muscles. The terminal branch of the anterior interosseous nerve (AIN) is most commonly used as the donor nerve. An end-side neurorrhaphy is performed to the ulnar motor fascicles in the distal forearm, a distance which results in reinnervation of the intrinsic muscles well before the 9- to 12-month mark. The known topography of the ulnar nerve allows the surgeon to coapt the donor nerves to the appropriate motor recipient site of the ulnar nerve. An end-end coaptation of the terminal AIN to the motor fascicle of the ulnar nerve is also an option. Nerve grafts or conduits are not required to repair the nerve injury in this clinical scenario. A 1-cm gap can usually be primarily repaired after dissecting the nerve and freeing it up proximally and distally. If further length is needed for tension-free repair, the ulnar nerve can be transposed anteriorly out of its natural position, giving another few centimeters of length.

2016 A 45-year-old man sustains a laceration of the ulnar nerve proximal to the elbow. He has loss of intrinsic hand function. At the time of surgical repair, there is a 1-cm gap in the nerve. Which of the following is the most appropriate management of this injury? A ) Interposition nerve conduit B ) Interposition nerve grafting C ) Primary repair only D ) Primary repair with anterior transposition E ) Primary repair with distal nerve transfer

The correct response is Option A. Aspirin, or acetylsalicylic acid, inhibits the enzyme cyclooxygenase. Cyclooxygenase ordinarily functions to form multiple compounds from arachidonic acid, including thromboxane and prostacyclin. Thromboxane is a platelet aggregator and vasoconstrictor. Prostacyclin is also a platelet aggregator. By decreasing formation of these compounds, aspirin acts as an anticoagulant. Dextran is a polysaccharide whose mechanism of action is thought to involve decreasing platelet aggregation by altering the electric charge of platelets, as well as by decreasing blood viscosity. Dextran also acts as a volume expander. Heparin is a glycosaminoglycan that binds to antithrombin III and enhances its ability to inactivate thrombin (which ordinarily converts fibrinogen to fibrin), as well as clotting factors IX, X, XI, and XII. Heparin may also additionally decrease thrombosis by causing nitric oxide-mediated vasodilation. Hirudin is derived from the medicinal leech Hirudo medicinalis and functions by directly inhibiting thrombin, in contrast to heparin, which requires the antithrombin III cofactor. Streptokinase is a thrombolytic agent that functions by activating plasminogen and its conversion to plasmin. In turn, plasmin breaks down fibrin into fibrin degradation products.

2016 A 45-year-old woman undergoes reconstruction of the right breast with a deep inferior epigastric artery perforator flap. Postoperatively, the flap shows signs of ischemia and is reexplored. During flap salvage, which of the following agents should be administered to inactivate thromboxane? A ) Aspirin B ) Dextran C ) Heparin D ) Hirudin E ) Streptokinase

The correct response is Option C. In the patient described, the most significant contraindication to replantation is the mechanism (crush injury) and multiple-level nature of the injury. It is highly unlikely that replantation will be successful with a crush mechanism due to the zone of injury. In addition, the multiple-level injury including the proximal phalanx and distal interphalangeal (DIP) joint precludes a functional result. Replantation in children does not adversely affect epiphyseal growth. Children adapt quite well to functional deficits of the hand. Microvascular surgery in children, while challenging, has been shown to have a very high success rate when performed by skilled microsurgeons. The index finger can be considered expendable; however, children tend to have more favorable results than adults when it comes to replantation and, therefore, whenever feasible, replantation should be attempted, even if it is an isolated index finger injury. The mechanism of injury plays a greater role than the type of digit in determining the feasibility of replantation in the pediatric population.

2016 A 5-year-old boy is brought to the emergency department after sustaining a crush injury to the index finger of his right dominant hand, resulting in amputation through the distal interphalangeal (DIP) joint. X-ray study shows a comminuted fracture of the proximal phalanx. Which of the following is the most significant CONTRAINDICATION to replantation in this patient? A ) Children have a difficult time adapting to functional deficits B ) The index finger is expendable C ) The mechanism and multiple-level nature of the injury preclude a functional result D ) Microvascular anastomosis is unlikely to be successful in a child of this age E ) Replantation will adversely affect epiphyseal growth

The correct response is Option E. In severely comminuted fractures of the mandible, the viability of the bony fragments depends on the blood supply from the periosteum and periosteal vessels. Significant periosteal stripping during an open repair would place these patients at risk for necrosis and bony resorption. This is the premise behind closed reduction of these injuries when indicated. There are two major types of bone: tubular (long bone) and flat bones (primarily of the facial skeleton). Tubular bones include the long bones of the extremities, clavicles, hands, and feet, and are composed of a diaphysis, paired metaphyses, and epiphyses. Tubular bones have a dual blood supply. The predominant supply is the nutrient diaphyseal arteries that often enter the middle third of the diaphysis and bifurcate upon entering the medullary canal. At the distal aspect, the smaller metaphyseal and epiphyseal arteries generally arise from arteries that supply the joint and anastomose with the diaphyseal arteries. This less robust blood supply at the joints explains why fractures in this location can lead to growth retardation. The other major blood supply is the periosteal vascular plexus; this plexus relies on connections with overlying skeletal muscle. Flat bones of the facial skeleton, as well as the scapulae, sternum, and ribs, do not contain a diaphysis, metaphyses, or epiphyses. They contain a dual blood supply with nutrient arteries (generally based off of the maxillary artery or middle meningeal artery for the calvaria) and the periosteal vessels and vascular plexus. Although the vascular connections with the periosteum are poorly developed, vascularized calvarial grafts could be based on this system if the galea and vessels were adherent to the graft. There is no galea on the mandible and, therefore, no galeal vessels.

2016 A 50-year-old man is brought to the emergency department after a high-speed motor vehicle collision. CT scan is obtained and shows a highly comminuted fracture of the mandible involving the symphysis, body, and angle on the left. Which of the following types of vessels supplies perfusion to these mandibular segments? A ) Diaphyseal B ) Epiphyseal C ) Galeal D ) Metaphyseal E ) Periosteal

The correct response is Option B. The patient is undergoing a transversus abdominis plane (TAP) block. TAP blocks have become popular adjunctive pain control measures for a variety of abdominal surgeries. They can either be performed transcutaneously with ultrasound guidance or under direct visualization in the operating room prior to closure of the abdominal donor site in deep inferior epigastric artery perforator (DIEP) or transverse rectus abdominis musculocutaneous (TRAM) flap surgeries. The segmental innervation to be blocked in the TAP block is located between the internal oblique and transversus abdominis muscles. Either infiltration of liposomal bupivacaine or placement of epidural catheters in the TAP space has been described.

2016 A 50-year-old woman is scheduled to undergo bilateral deep inferior epigastric artery perforator (DIEP) flap breast reconstruction. A donor site regional block with liposomal bupivacaine is planned for analgesia. Which of the following injection locations is most likely to improve pain control and decrease narcotic use in this patient? A ) Between external oblique and internal oblique fascias B ) Between internal oblique and transversus abdominis fascias C ) Deep to rectus abdominis fascia D ) Deep to transversus abdominis fascia E ) Subcutaneous plane

The correct response is Option C. A component separation procedure allows for the primary closure of central abdominal wall musculofascial defects without excessive tension. In the classically described procedure, the external oblique aponeurosis is released lateral to the linea semilunaris, allowing for medial advancement of the rectus abdominis along with the internal oblique and transversus muscles. The innervation and vasculature to these structures are preserved in order to maintain functional integrity of these abdominal wall muscles. The vertical and transverse rectus abdominus musculocutaneous flaps along with the anterolateral thigh flaps are primarily used to reconstruct soft-tissue defects but are unlikely to result in a functional abdominal wall.

2016 A 52-year-old man who has a ventral hernia with fascial defect that is 18 cm long and 14 cm wide is scheduled for abdominal wall reconstruction. A photograph is shown. Which of the following techniques is most likely to result in a functional tension-free closure? A ) Advancement of the vertical rectus abdominis musculocutaneous flaps B ) Release and advancement of the external oblique muscles C ) Release and advancement of the rectus, internal oblique, and transversus abdominis muscles D ) Rotation of the anteriolateral thigh flaps E ) Rotation of the transverse rectus abdominis myocutaneous flaps

The correct response is Option D. While all of the listed treatments are effective and employed commonly for facial aging, only tretinoin (Retin-A) has been found to have effectiveness in long-term collagen production. Retinoids are vitamin A derivatives that have profound effects on the skin. Both increases in dermal collagen production and decreases in degradation are seen over 6 to 12 months of treatment. The active form is tretinoin, a metabolite of vitamin A. Within 3 months of starting treatment, improvements in skin softness, texture, and reductions in fine rhytides and dyschromia are apparent. Treatment is typically a 0.05 to 0.01% topical cream applied nightly. Peeling and redness are common, but treatment tolerance improves with time. Early treatment reactions can be treated with decreased product concentration, longer treatment intervals, and topical hydrocortisones. Fat grafting is a very effective volume replacement and deeper rhytides treatment. While there are some suggestions of increased vascularity and health of overlying skin, effects are secondary and not as profound as tretinoin. Rhytidectomy (facelifting) results in physical skin tightening by removal, along with deeper tissue (SMAS) repositioning. Skin texture and collagen effects are relatively minor. Hyaluronic acid fillers are also effective rhytide and volume treatments, but have little or no effect on collagen. TCA peels effectively treat the epidermis and superficial dermis and have minor collagen stimulation effects through the natural wound-healing process. While surface appearance effects can be dramatic, the amount of collagen stimulation is far less than tretinoin. Laser resurfacing can result in more dermal injury and resultant collagen production than chemical peels, although hypopigmentation and prolonged recovery are disadvantages. Pre-treatment with tretinoin prior to peels and laser resurfacing can increase the depth of treatment and, some hypothesize, improvements in healing and recovery time.

2016 A 52-year-old woman seeks treatment for signs of facial aging. She reports diffuse fine rhytides caused by a long history of sun exposure and smoking. Which of the following is most likely to result in the greatest long-term increase in dermal collagen content? A ) Deep-plane rhytidectomy B ) Fat grafting C ) Hyaluronic acid filler injection D ) Tretinoin therapy E ) Trichloroacetic acid peel

The correct response is Option C. Preservation of the superficial inferior epigastric veins (SIEV) during flap harvest is a useful preventive measure in microsurgical free tissue transfer operations. These veins can serve as important lifeboats to augment venous outflow in the setting of venous congestion. Typically, if a free flap demonstrates venous congestion, the inset should be taken down and the pedicle, recipient vessels, and anastomoses interrogated. Simple issues, such as mechanical compression or twisting of the vein should be ruled out. Next, the SIEV should be inspected. If it is engorged, it is likely that the flap is reliant on superficial outflow, and this vein should be connected to a recipient vessel to augment the venous outflow of the flap. Options for recipient veins include an anterograde branch on the pedicle vena comitans, or in a retrograde fashion to the vena comitans that was not used in the initial set of anastomoses. Near-infrared fluorescence imaging technology can assist with flap design and may be useful for assessing the arterial inflow of the flap, but it has not been shown to correlate with flap loss or venous complications. The use of one or two veins in microsurgical free tissue transfer is a topic that has been debated for several years. While some studies indicate that the use of two venous connections may decrease the velocity of blood flow across the anatomosis, there are no data to support differences in flap outcomes or thrombotic events. Therefore, the routine use of a second vein is largely up to surgeon preference. Heparin may be a useful adjunct when thrombosis of the arterial or venous anastomosis is excised and a revision of the anastomosis is performed. Without evidence of thrombosis, it is unlikely to have any added benefit. For venous anastomoses, the use of venous coupling devices has not been associated with patency issues or increased thrombosis rates. Therefore, a hand-sewn anastomosis is not likely to prevent the issue presented in this question.

2016 A 52-year-old woman undergoes immediate unilateral breast reconstruction with a free deep inferior epigastric flap. Near completion of the procedure, the flap skin paddle is noted to have venous congestion. On exploration of the pedicle, the anastomosis between the vena comitans and the internal mammary vessel appears patent. Which of the following preventive measures would most likely have averted this issue? A ) Administration of heparin immediately before release of vessel clamps and flap revascularization B ) Anastomosis of the veins using sutures instead of a venous coupling device C ) Preservation and anastomosis of the superficial inferior epigastric vein D ) Routine anastomosis of two venae comitantes per flap E ) Use of near-infrared fluorescence imaging to assess flap blood flow

The correct response is Option A. Multiple studies have supported improved patient satisfaction with autologous breast reconstruction in the setting of a unilateral reconstruction. The clinical scenario involves an obese patient with a large, ptotic breast who does not desire a surgical procedure for symmetry and is the ideal candidate for an autologous reconstruction. Microvascular free tissue transfer of abdominal tissue for breast reconstruction demonstrates improved reliability and decreased fat necrosis compared with pedicled flap reconstruction. Implant-based reconstruction is less likely to provide adequate symmetry in this patient. An extended latissimus dorsi musculocutaneous flap-based reconstruction alone is unlikely to provide enough volume for symmetry and a traditional latissimus flap with implant is unlikely to provide adequate ptosis for symmetry.

2016 A 53-year-old woman comes to discuss breast reconstruction after undergoing left mastectomy for ductal carcinoma in situ. She does not require chemotherapy or radiation therapy and does not want surgery on the unaffected breast. She is obese but otherwise healthy with a large, ptotic right breast. Which of the following breast reconstruction techniques is most likely to result in the greatest long-term patient satisfaction? A ) Autologous breast reconstruction with microvascular free tissue transfer from the abdomen B ) Extended latissimus dorsi musculocutaneous flap reconstruction without an implant C ) Immediate reconstruction with cohesive, anatomically shaped silicone gel implant D ) Immediate tissue expander followed by implant-based reconstruction E ) Latissimus dorsi musculocutaneous flap reconstruction with immediate placement of a permanent implant

The correct response is Option D. The sartorius muscle demonstrates a Mathes-Nahai Type IV vascular pattern with multiple segmental vascular pedicles. For this reason, it is important to preserve as many segmental pedicles as possible when transferring this flap. The sartorius flap is usually transferred as a distally based flap, and not a proximally based flap as presented in the clinical scenario. When based proximally, it is necessary to ligate multiple segmental pedicles to achieve an adequate arc of rotation, and this can lead to flap necrosis. When based distally, it is often necessary to divide 1 to 2 segmental pedicles superiorly to allow an adequate arc of rotation, but this usually does not compromise the flap. If there is any concern regarding flap perfusion, the pedicles can be temporarily clamped before division to assess the effects of pedicle ligation. Examples of muscle flaps with the other Mathes-Nahai types of vascular pattern include the following: Type I - tensor fascia lata Type II - gracilis Type III - gluteus maximus Type IV - latissimus dorsi

2016 A 53-year-old woman comes to the office with an exposed vascular graft in the left groin caused by postoperative wound infection. After operative debridement, a proximally based sartorius muscle flap is transferred to cover the graft. Several days after surgery, it is evident that the distal portion of the muscle flap is necrotic. This most likely occurred because the sartorius muscle demonstrates which of the following Mathes-Nahai types of vascular pattern? Type Description (A) I single vascular pedicle (B) II dominant and minor vascular pedicle(s) (C) III two dominant vascular pedicles (D) IV segmental vascular pedicles (E) V single dominant and secondary segmental vascular pedicles

The correct response is Option D. According to National Comprehensive Cancer Network guidelines, melanomas with Breslow thickness greater than 1 mm should be treated with wide local excision and sentinel lymph node biopsy. Wide local excision alone, with no additional lymph node sampling, may be appropriate for some melanomas less than 1 mm, but additional factors such as mitotic rate and ulceration should be assessed. Random lymph node sampling would not necessarily assess the first node in the draining basin, therefore, it is important that the sentinel lymph node be identified with techniques such as tracking of an intradermally injected radionucleotide using a handheld gamma probe and visual identification with blue dye. In the past, melanomas with thickness between 1 and 4 mm were treated with elective lymph node dissection. Since several randomized, prospective studies failed to clearly demonstrate a survival advantage and the significant associated morbidity, sentinel lymph node biopsy has replaced this procedure. While there are growing indications for the use of radiation therapy for skin cancers, intraoperative radiation therapy is not routinely used.

2016 A 55-year-old man undergoes biopsy of a pigmented lesion on the neck. Examination of a biopsy specimen shows superficial spreading of a malignant melanoma with no ulceration, 1.2-mm thickness, and less than 1 mitosis per mm2. In addition to wide local excision, which of the following is most appropriate treatment? A ) Interferon therapy B ) Radiation therapy C ) Regional lymphadenectomy D ) Sentinel lymph node biopsy E ) Observation

The correct response is Option B. Given the size of the defect, the patient's comorbidities, and bowel violation, the best method to optimize results is a component separation with a bioprosthetic mesh underlay. The use of bridging mesh without approximation of the fascia is not recommended due to a high recurrence rate. Instead the Ventral Hernia Working Group (VHWG) recommends reapproximation of the rectus muscle at the midline whenever possible without undue tension. This can be done by using the component separation technique originally described by Ramirez. The procedure calls for a release of the external oblique aponeurosis 1 cm lateral to the linea semilunaris, which allows for medialization of the rectus abdominis and underlying lateral musculature for primary approximation. Given the enterotomy, a bioprosthetic mesh would be recommended as opposed to a synthetic mesh, because it is likely more resistant to infection and does not necessarily need removal in a contaminated wound. It is the VHWG's preferred method to place mesh in an underlay manner, because intra-abdominal pressure pushes the mesh against the native abdominal wall instead of away from it. It also adds another layer of tissue over the prostatic material and would be preferred with a bioprosthetic because of decreased risk for bowel adhesions. Additionally, lower rates of hernia recurrence have been shown in patients who underwent component separation with mesh as opposed to those without.

2016 A 55-year-old man who underwent abdominal surgery 10 years ago undergoes lysis of adhesions for treatment of ongoing intermittent bowel obstruction symptoms. He does not smoke cigarettes and has a history of hypertension and diabetes mellitus. A single enterotomy is made and repaired primarily. During abdominal wall closure after the intra-abdominal procedure, the fascial edges cannot be approximated without marked tension with a relaxed defect diameter maximum of 7 cm. Which of the following is the best method of repair? A ) Bioprosthetic mesh bridging B ) Component separation and bioprosthetic mesh underlay C ) Component separation and synthetic mesh underlay D ) Component separation with no mesh E ) Synthetic mesh bridging

The correct response is Option D. In the clinical setting of the patient described, the method for perineal reconstruction that has the most evidence to support it is the vertical rectus abdominis myocutaneous (VRAM) flap. Omental flaps are effective for smaller defects that primarily involve internal structures. Primary skin closure, local advancement flaps, and propeller flaps are not ideal in the setting of prior radiation because of the high risk of wound healing complications. Gracilis flaps are low in volume and may be useful for smaller defects, although bilateral flaps can be adequate for moderate-sized defects. This patient has a large composite defect that will benefit from the large volume and cutaneous component of a VRAM flap.

2016 A 56-year-old man comes to the office because of recurrent and locally advanced rectal cancer 2 years after undergoing low anterior resection and adjuvant chemotherapy and radiation therapy. Surgical extirpation involving the external perianal skin, anus, rectum, and musculature of the pelvic floor is performed. Which of the following is the most appropriate method of perineal reconstruction in this patient? A ) Gracilis flap B ) Local propeller flap C ) Omental flap D ) Rectus abdominis musculocutaneous flap E ) Rectus femoris flap

The correct response is Option E. As the treatment of anorectal cancers has shifted to more sphincter-preserving surgeries and radiochemotherapy, the need for abdominoperineal resection (APR) has decreased. However, in cases of locally persistent disease, involvement of nearby structures (vagina), or recurrent disease, there is an increased rate of survival and improved quality of life with pelvic extirpation. These cases typically result in problematic wounds secondary to previous radiation therapy, with complication rates as increased as 60%. Complications include delayed wound healing, infection, internal hernia, chronic sinus formation, and the psychological and physical concerns associated with vaginal resection in the female patient. In an effort to minimize these complications and restore sexual function in the female patient, musculocutaneous flap reconstruction is recommended. In the patient described, the most appropriate option for reconstruction is the vertical rectus abdominis musculocutaneous (VRAM) flap. The advantages of the rectus flap include an excellent and safe pedicle (Mathes/Nahai Type III flap), rapid technique, abundant well-vascularized tissue, wide arc of rotation, acceptable donor-site morbidity, and ease of access in relation to the APR procedure. The well-vascularized bulky tissue is effective in obliterating the pelvic dead space, minimizes the risk of internal herniation of the abdominal contents, and provides healthy tissue in the event of suture line dehiscence allowing more rapid healing by secondary intention if necessary. The disadvantages of the VRAM include lack of sensation in the vagina/perineum, loss of secretory function that may result in dyspareunia, abdominal weakness/hernia, and unsightly bulk in the perineum that may require a secondary debulking procedure for aesthetic improvement. In the patient described, the decision was made to close the perineal skin over the deepithelialized flap. Primary closure is inappropriate secondary to the increased rate of complications and poor sexual function without reconstruction as noted above. Bilateral pudendal thigh (Singapore) flaps are a great option for vaginal or labial reconstruction in the congenital or ablative setting when there is not a need for a large volume of tissue. In this case, the flaps are not an option because of the need for bulky tissue, previous radiation therapy administered to the base of the flaps, and transection of the flap's blood supply (posterior labial artery). The extended groin flap is inappropriate for the same reason. A skin graft is not likely to be successful, given the patient's radiation history and violation of the rectum.

2016 A 56-year-old woman undergoes abdominoperineal resection secondary to aggressive rectal cancer. Neoadjuvant chemotherapy and radiation therapy have been performed. Surgical examination shows defect of the anus, rectum, and posterior vaginal wall. An intraoperative photograph is shown. Which of the following is the most appropriate reconstructive option in this patient? A ) Bilateral Singapore (pudendal thigh) flaps B ) Gracilis muscle flap C ) Omental flap D ) Rectus femoris flap E ) Vertical rectus abdominis musculocutaneous flap

The correct response is Option D. Integra is a bilaminate neodermal material that consists of an outer silicone sheet and an inner layer of bovine collagen and glycosaminoglycan (derived from shark cartilage), which acts as a scaffold for regeneration. Specifically, there is migration of host fibroblasts into the matrix, which subsequently proliferate and form collagen. Additionally, there is migration of endothelial cells that form a vascular network within the neodermis, which can subsequently act as a foundation to support staged split-thickness skin grafting, at which time the silicone outer layer is removed and a thin (usually 6-8/1000") split-thickness autograft is applied. The success, or "take," of the skin graft depends on whether the neodermis is vascularized sufficiently to maintain graft viability. A 2001 study by Moiemen et al. demonstrated clinically and histologically that this process takes 4 weeks on average, without the use of topical negative pressure. A follow-up article from 2004 suggests this process can be accelerated from 4 weeks to an average of 7.25 days if topical negative pressure is used. Further follow-up studies based on the clinical appearance of the bilaminate neodermal matrix have suggested reduction in complication rates, improved patient tolerance, and enhanced and accelerated neovascularization with the use of topical negative pressure wound therapy. However, a study published in 2010 looking histologically at various time points could not demonstrate that topical negative pressure wound therapy accelerates neovascularization as verified by the presence of histologically patent vascular channels. The authors found that the median percentage of the template depth that demonstrated canalized channels was 0%, 20%, 61%, and 80% for days 7, 14, 21, and 28, respectively. Because this question pertains to the use of bilaminate neodermal matrix with the use of a standard bolster dressing and not negative pressure wound therapy, most existing evidence indicates that the most appropriate timing is 4 weeks after placement of the bilaminate neodermis.

2016 A 58-year-old man is referred by an orthopedic surgeon after undergoing open reduction and internal fixation of a fracture of the ankle after falling down stairs. The repair is complicated by wound dehiscence with 2 × 2 cm of exposed tibialis anterior tendon without paratenon. A photograph is shown. After debridement of healthy tissue, reconstruction is performed using bilaminate neodermis with placement of a standard bolster dressing. At which of the following times relative to placement of the bilaminate neodermis should split-thickness skin grafting be performed? A ) At the same time B ) 1 week after C ) 2 weeks after D ) 4 weeks after E ) 6 weeks after

The correct response is Option C. The currently accepted decolonization protocol from the Infectious Diseases Society of America is a combination of topical nasal mupirocin ointment and a chlorhexidine body wash for 5 days. The most robust data for decolonization are in the cardiac surgery literature; the Society of Thoracic Surgeons guidelines recommend routine prophylaxis for all patients undergoing cardiac surgery. These guidelines are based on studies that show a decreased rate of sternal wound infections. The carriage rate of methicillin-resistant Staphylococcus aureus for health care workers is approximately 5% based on large review studies. As a substantial portion of cases originate from nasal colonization, routine screening and decolonization are recommended in areas where the carriage rate exceeds 10% (Centers for Disease Control and Prevention recommendation). The other antibiotics listed are reserved for the treatment of infection rather than as part of a decontamination protocol.

2016 A 60-year-old woman who works as a nurse in the intensive care unit (ICU) is scheduled for cardiac bypass surgery. Because she has taken care of many patients with methicillin-resistant Staphylococcus aureus infection, she is interested in topical agents for decolonization. Which of the following regimens is most appropriate for this patient? A ) Clindamycin B ) Linezolid C ) Mupirocin and chlorhexidine D ) Trimethoprim-sulfamethoxazole E ) Vancomycin

The correct response is Option C. In the case presented above, the strongest indication for Mohs micrographic surgery comes from the anatomic finding of indistinct borders. Other findings are not necessarily indications for Mohs. Other proven indications are recurrent cancer, high risk zones of the face, morpheaform basal cell cancer, or evidence of neurovascular involvement. By some studies, larger sized basal cell cancers (>2 cm) may be indications. By definition, Mohs micrographic surgery is a technique for treatment of complex or ill-defined skin cancers with examination of 100% of tissue. One physician performs the procedure, acting in two distinct capacities: surgeon (excising the cancer) and pathologist (reading the slides). Usually, the final pathologic clearance is given on the same day as the resection. This is in comparison with a wide local excision technique, which is performed by two different physicians: a surgeon who removes the cancer and a pathologist who reads it separately. In the latter scenario, although a "wet" read can be done on the same day, the final pathologic evaluation has to await the permanent preparation of slides and a final read. Also, using the routine pathologic evaluation, only 2% of the margins are looked at (using the common "bread loafing" technique), compared with the 100% of tissues evaluated using the Mohs technique. The vast majority of Mohs resections is done with one or two excisions. Although at times there is a need for multiple excisions, the need for excisions is driven by the positive margins detected pathologically. In Mohs technique, the amount of healthy tissue taken to obtain "clear" margins is the minimum needed to do so, thus preserving the healthy tissue, which can be critically useful in the ensuing reconstruction. These margins are definitively assessed on that same day of surgery, so once the patient leaves the Mohs surgeon's office, he or she knows the cancer has been completely removed. This is why the margin control is superior in Mohs technique compared with other surgical techniques.

2016 A 62-year-old woman with biopsy-proven basal cell carcinoma of left mid cheek presents for consultation for surgical treatment. On physical examination, the lesion is 0.6 cm in diameter and has indistinct borders. Which of the following criteria is the most likely indication for Mohs micrographic surgery in this patient? A ) Anatomical location of the cancer B ) Diagnosis of basal cell carcinoma C ) Indistinct borders D ) Patient age E ) Size of the carcinoma

The correct response is Option C. Radiation dermatitis is one of the most common side effects of radiotherapy for cancer and can occur any time, from hours to weeks after radiation exposure. Acute radiation-induced skin changes depend on the radiation dose and include erythema, edema, pigment changes, epilation, and dry or moist desquamation. They can also be accompanied by pain and pruritis. For the lower grade changes described for this patient, topical corticosteroids with low to medium potency, such as hydrocortisone 1% cream, are recommended. This is in addition to a skin-washing protocol to keep skin clean and dry before treatments. All of the other topical agents listed - antihistamines, salicylate analgesics, vitamin E, and hyaluronic acid - have all been previously studied and shown to have no added benefit. More severe skin changes such as moist desquamation, skin necrosis, or intractable pain may require radiation to be stopped, resulting in inadequate disease treatment.

2016 A 63-year-old man underwent resection of a chest wall sarcoma that was covered with an anterolateral thigh flap. He now undergoes external beam radiation therapy, and there is erythema, edema, and dry desquamation of the surgical sites. Which of the following is the most appropriate treatment? A ) Diphenhydramine B ) Hyaluronic acid C ) Hydrocortisone D ) Salicylate E ) Vitamin E

The correct response is Option A. The results of fat grafting are dependent upon the surgeon's experience and technique. Strategies to increase the take of the fat graft and decrease fat necrosis include atraumatic harvest techniques. Placement of the graft in multiple small aliquots increases the availability for vascularity, and creating a lattice-like framework when depositing the grafts avoids large-volume deposits. Placement of the grafts in a few large-volume deposits is avoided to minimize fat necrosis and creation of fatty oil cysts. Centrifugation has mixed results in fat grafting.

2016 A 64-year-old woman is evaluated because of the aged appearance of the dorsal surfaces of the hands. Physical examination of the hands shows lipodystrophy. Autologous fat grafting is planned to improve the cosmetic appearance. Which of the following maneuvers is most likely to decrease the risk of fat necrosis and improve fat graft viability? A ) Placement of fat grafts in small aliquots B ) Postoperative prevention of pressure on grafted regions C ) Preparation of fat grafts with centrifugation D ) Use of ultrasound-assisted liposuction for fat graft harvest

The correct response is Option A. Split-thickness skin grafts require less metabolic demand from the wound bed compared with full-thickness skin grafts. In situations where the wound bed may not supply adequate nutrient diffusion through the graft, a full-thickness graft may develop superficial epidermolysis. Skin graft viability is initially based on plasmatic imbibition from the serous exudate of the wound, followed by inosculation and angiogenesis. After graft take, remodeling and scar maturation occur. Primary ontracture of a graft is from elastin fibers in the dermis, whereas secondary contracture is from myofibroblast activity. Split-thickness grafts contain epidermis and a variable amount of dermis compared with full-thickness grafts, which, by definition, contain both epidermis and the entire dermal layer. Full-thickness grafts contain intact skin appendages, whereas split thickness grafts do not; this allows the donor site for split-thickness grafts to reepithelialize. Split-thickness grafts therefore undergo less primary and more secondary contraction.

2016 A 65-year-old man comes to the office because of a 6 × 8-cm open wound on the forearm. The wound currently has healthy granulation tissue and does not appear infected. Which of the following is an advantage of split-thickness skin grafting over full-thickness skin grafting in this patient? A ) Less metabolic demand required from wound bed B ) Less secondary contraction from elastin fibers C ) More primary contraction from myofibroblasts D ) Occurrence of inosculation before plasmatic imbibition E ) Presence of intact skin appendages within graft

The correct response is Option D. Composite tissue reconstruction after tumor ablation is a significant challenge to the plastic surgeon. Generally, bony defects of the anterior mandible require bony replacement and defects greater than 6 cm require vascularized bone for reconstruction. For large defects, the optimum choice of bone is the vascularized fibula. Reconstruction with free flaps can be successfully performed in a patient with a smoking history and, in fact, is often the optimal method of reconstruction in smokers. The use of allograft cadaveric bone is not indicated, especially in a patient with a history of smoking. For defects of the size described, the fixation plate alone would not give adequate long-term support and would likely result in plate exposure or extrusion in the face of previous radiotherapy. Nonvascularized bone grafts are indicated in bony defects less than 6 cm in length.

2016 A 65-year-old man with a history of squamous cell carcinoma of the perioral skin managed with resection, reconstruction, and postoperative radiation therapy is evaluated because examination of a specimen obtained on biopsy confirmed carcinoma recurrence. History includes tobacco use. CT scan of the head and neck shows involvement of a 7-cm segment of the right mandibular parasymphyseal and body bone. Preoperative metastatic workup shows no other abnormalities. Operative perioral skin and mandibular resection is planned. Which of the following is the most appropriate method of mandibular reconstruction in this patient? A ) Autologous corticocancellous bone graft B ) Cadaveric bone graft and anterolateral thigh flap C ) Reconstruction plate and rectus abdominis musculocutaneous flap D ) Vascularized bone flap

The correct response is Option D. Tissue injury from radiation results in irreversible damage that limits the ability of wounds to heal with skin grafts or by secondary intention, such as using negative pressure wound therapy. Tissues within the field of radiation are also generally affected, which would limit use of an intercostal muscle flap. Although a latissimus dorsi musculocutaneous flap would provide an adequate amount of tissue from outside of the field of radiation, it would be unreliable in the setting of prior posterolateral thoracotomy, unless there is documentation that the latissimus was spared. A vertical rectus abdominis musculocutaneous flap would provide sufficient tissue from outside of the field of radiation and would not have been affected by the patient's prior surgery.

2016 A 66-year-old man with a history of adenocarcinoma of the lung undergoes a left pneumonectomy using a posterolateral thoracotomy incision. Postoperatively, he receives radiation therapy. The patient subsequently develops a nonhealing ulcer of the chest wall measuring 5 × 6 cm in the region of the nipple-areola complex. Which of the following is the most appropriate option for reconstruction? A ) Negative pressure wound therapy B ) Pedicled intercostal muscle flap and a split-thickness skin graft C ) Pedicled latissimus dorsi musculocutaneous flap D ) Pedicled vertical rectus abdominis musculocutaneous flap E ) Split-thickness skin graft only

The correct response is Option D. This patient described has extramammary Paget disease and the treatment is wide excision. Paget disease of the breast also presents with eczematous skin changes and is associated with breast cancer. Extramammary Paget disease, however, is an intraepithelial carcinoma that commonly involves the vulvar, perianal, perineal, scrotal, and penile regions. It presents as well-defined, moist, erythematous plaques associated with pruritis. Histopathologic examination shows epidermal acanthosis and elongated rete ridges. Paget cells are large intraepidermal cells with a large nucleus and abundant pale cytoplasm. There is a 7 to 40% rate of associated malignancy. Wide excision is the standard of care, and recent reports have shown that Mohs micrographic surgery can improve evaluation of resection margins. Observation is not appropriate for extramammary Paget disease given the potential risk for malignancy. Topical treatment with steroids or antifungals is also not appropriate. Oral miltefosine is a treatment for leishmaniasis, an infectious disease that can involve the skin, mucous membranes, and internal organs. Although cutaneous leishmaniasis can present with ulcerating lesions or a dense dermal infiltrate, the histology is predominantly histiocytes, lymphocytes, and plasma cells.

2016 A 75-year-old man comes to the office because of a 5-year history of a pruritic lesion on the right groin that has been enlarging gradually in size. A photograph is shown. Examination of a specimen obtained on biopsy shows Paget disease. Which of the following is the most appropriate next step in management? A ) Oral miltefosine B ) Topical hydrocortisone C ) Topical miconazole D ) Wide excision E ) Observation

The correct response is Option E. Osteoinduction refers to the direct stimulation of mesenchymal cells at the recipient site by bone morphogenetic protein to differentiate into osteoprogenitor cells. This mechanism of action is associated with the healing of cancellous bone grafts and demineralized bone matrix. Osteoconduction (creeping substitution) is the primary method by which cortical bone grafts heal. During osteoconduction, cells and blood vessels from the recipient bed grow into the graft. The bone graft becomes a template for the deposition of new bone and the graft resorbs. Neovascularization is complete by 6 to 8 weeks, but ultimate strength of cortical grafts is not seen until 6 to 12 months, at which time the graft is comparable to a vascularized bone graft. Osteogenesis is the formation of new bone by cells in a flap/graft that survive the transfer. This is the primary mechanism by which a vascularized bone graft heals. The pedicle keeps the bone alive so that primary bone healing can occur between the graft and recipient site. Osteoblasts from both locations participate in the formation of new bone at the interface between graft and native bone. Advantages of a vascularized bone graft include the ability to place the graft into a hostile environment such as an irradiated wound bed and immediate structural support with shortened time to bony union compared with nonvascularized cortical grafts. Osteochondrosis refers to a family of ossification disorders in children. Endochondral ossification is the process by which the cartilaginous soft callus covering a fracture is transformed into bone.

2016 A 9-year-old boy with unilateral cleft lip and palate undergoes alveolar bone grafting with a cancellous iliac bone graft. Which of the following best describes the mechanism of bone healing? A ) Endochondral ossification B ) Osteochondrosis C ) Osteoconduction D ) Osteogenesis E ) Osteoinduction

The correct response is Option E. The most appropriate prophylactic regimen to prescribe in this clinical scenario is trimethoprim-sulfamethoxazole and clindamycin. Dog bites to the hand are potentially dangerous bites that could lead to serious hand infections. The common microorganisms that cause infections in such bites are Pasteurella species, anaerobes, Staphylococcus aureus, and Streptococcus. The ideal antibiotic would have been amoxicillin-clavulanic acid, which covers most of these microorganisms. However, the child is allergic to penicillin, and, therefore, this drug is contraindicated. Another good option would have been amoxicillin and clindamycin (for the anaerobic coverage). However, for the same reason described above, it too cannot be used. Tetracyclines are contraindicated in children under 8 years of age owing to the ill effects on growing teeth and bones. Quinolones are also contraindicated in children under 18 years of age owing to their harmful effects on cartilage and joints. Although this is debatable, currently the use of quinolones in children is restricted by the Food and Drug Administration to certain specific conditions (cystic fibrosis, multidrug-resistant urinary tract infection, and inhalational anthrax). Clindamycin alone does not adequately cover most of the organisms involved, including Pasteurella, which is gram-negative.

2016 A healthy 5-year-old boy is evaluated after he is bitten on the hand by a dog. He has an allergy to penicillin. Which of the following prophylactic antibiotic regimens is most appropriate to prescribe? A ) Amoxicillin and clindamycin B ) Ciprofloxacin and metronidazole C ) Clindamycin only D ) Doxycycline and metronidazole E ) Trimethoprim-sulfamethoxazole and clindamycin

The correct response is Option C. In any reconstruction, many factors (local, regional, and systemic) have to be considered before deciding on a proper treatment option. Indeed, there may very well be multiple options. The patient described is an elderly man who is an extremely poor candidate for anesthesia (example of systemic consideration). Additionally, he will need radiation therapy to the scalp as soon as possible. Therefore, the reconstruction option needs to have excellent blood supply to heal in the first place, heal quickly, and withstand the effects of radiation. Additionally, coverage of the exposed calvarium is necessary, as periosteum has been removed by the Mohs surgeon. Out of the options given, local scalp flap coverage best accomplishes this goal. Delay of treatment is not recommended because it only creates a greater problem after radiation therapy, because all local options as well as the calvarium will be irradiated, which severely hampers the surgeon's ability to provide a low-morbidity procedure and avoid a substantial operation. Dermal matrices (any form) are not appropriate options here for many reasons. They are not the definitive treatment option in a patient who is about to undergo radiation. Once the matrices become incorporated, they will usually need a skin graft to complete reconstruction or they will need a prolonged period of dressing changes, neither of which is optimal in this patient (a second surgery or a prolonged healing phase). Also, they need to be placed on a well-vascularized bed in order for them to "take" and heal more effectively. An exposed calvarium (without additional burring of bone) is not an optimal bed for a dermal matrix. Skin grafts (of any variety) lack blood supply after harvest. They also need to be placed on a well-vascularized bed in order for them to "take" and heal more effectively. An exposed calvarium (without additional burring of bone) is not an optimal bed for a skin graft. Also, a thin skin graft may not be the best form of reconstruction in a patient who is about to undergo radiation therapy, if other options exist. Radial forearm free flap is too complex an operation for this patient with many comorbidities and a relatively small defect.

2016 An 82-year-old man is referred for reconstruction of the scalp after Mohs micrographic surgery for an aggressive squamous cell carcinoma. He is scheduled to undergo radiation therapy as soon as possible after reconstruction. The patient has pulmonary fibrosis and is receiving oxygen via nasal cannula. Physical examination shows a vertex scalp defect of 4 × 4 cm with calvarium exposed throughout. Which of the following is the most appropriate management? A ) Bilaminate neodermis B ) Delayed reconstruction C ) Local tissue rearrangement D ) Radial forearm free flap E ) Split-thickness skin graft

The correct response is Option C. The most appropriate surgical treatment option would be to re-rotate the posterior thigh flap. One of the main principles in surgical flap closure of pressure ulcers (PUs) is to utilize large rotation flaps so that they may be re-rotated in the case of recurrences. Although it is debatable whether to re-flap a patient who has a PU recurrence, some clinical situations require a redo flap closure. In those circumstances, instead of using a new flap or donor site, re-elevation of the original flap and re-advancing it is usually the first-line flap option. The posterior thigh flap is a fasciocutaneous flap based on the descending branch of the inferior gluteal artery (if elevated as a purely V-Y flap, or if raised as a superiorly based "tongue" flap). However, if elevated as a rotation-advancement flap, it also has blood flow from medial or lateral thigh skin perforators depending on where the flap incisions are made on the posterior thigh. In all forms of the flap, the entire posterior thigh skin and fascia is elevated off the hamstring muscles (semitendinosus, semimembranosus, biceps femoris), thus severing all the musculocutaneous perforators from those muscles to the overlying skin and fascia. Therefore, the V-Y hamstring flap (a musculocutaneous flap) is not available after a posterior thigh flap has been raised. Though the inferior gluteal artery flap, gracilis, and possibly the tensor fascia lata flaps may be used, they are not the most appropriate surgical option when a re-rotation thigh flap is still available.

2016 An otherwise healthy 55-year-old man with paraplegia is evaluated because of a recurrent ischial pressure ulcer. History includes previous coverage of the ulcer with a posterior thigh rotation-advancement flap. After appropriate optimization of both patient and wound, which of the following is the most appropriate surgical intervention for reclosure of the wound? A ) Gracilis musculocutaneous flap B ) Inferior gluteal flap C ) Re-rotation of posterior thigh flap D ) Tensor fascia lata flap E ) V-Y hamstring flap

The correct response is Option C. The use of negative pressure therapy (NPT) is contraindicated in wounds with active infection including osteomyelitis. Negative pressure dressings in these wounds convert an open, draining wound into a closed wound, which could potentially lead to abscess formation and/or sepsis. NPT has become an integral part of wound management over the past decade and a half. It is a commonly used wound dressing and/or chronic wound management tool. It is instrumental in acute wounds as well (e.g., lower extremity trauma, abdominal wall trauma), and as a skin graft bolster dressing. The major contraindications for its use include wounds with active infection such as untreated osteomyelitis, malignant wounds, wounds with exposed major vessels and/or organs, and wounds with unexplored and/or nonenteric fistulas. Apart from the infected wound, all the wounds mentioned in the option set may benefit from the use of NPT. It may not be the definitive management for those wounds, but it could be used as a temporary measure prior to the definitive treatment. Recently, studies have shown that the use of NPT over closed incisions may reduce the risk for dehiscence and infections.

2016 Negative pressure wound therapy with a sponge dressing is CONTRAINDICATED in which of the following clinical scenarios? A ) An abdominal wound with an enteric fistula B ) A dorsal hand wound with an exposed tendon C ) A lower extremity wound with acute osteomyelitis D ) Over a closed surgical incision E ) A radiated scalp wound with exposed bone

The correct response is Option D. The prophylactic use of trimethoprim/sufamethoxazole (TMP/SMZ) in irradiated wounds, primarily in the setting of implant-based breast reconstruction, has been shown to reduce the rate of skin-derived infections. Mirzabeigi et al1 reported a 28% reduction in infection rates with the use of TMP/SMZ when compared with cephalosporins, in an irradiated chest wall wound. Based on their data, the authors advocate a monthlong prophylactic course of TMP/SMZ to reduce infection rates in this high-risk population. This is likely due to the ability of TMP/SMZ to treat both methicillin-resistant Staphylococcus aureus (MRSA) and Staphylococcus epidermidis. This has been supported by other published reports.2,3 Cefazolin and nafcillin do not effectively treat MRSA, reducing their clinical ability to prevent skin-related infections in this high-risk population. Doxycyline has some efficacy in treating most skin flora, including MRSA, but there is no clinical evidence to suggest it outperforms TMP/SMZ.

2016 Which of the following antibiotics is most effective in decreasing Staphylococcus aureus infections in irradiated wounds? A ) Cefazolin B ) Doxycycline C ) Nafcillin D ) Trimethoprim-sulfamethoxazole

The correct response is Option E. First-line surgical intervention for penile and/or scrotal lymphedema is resection of the overgrown skin and subcutaneous tissue. Liposuction is generally considered first-line operative treatment for extremity lymphedema but does not have efficacy for penile/scrotal disease. Lymphatic venous anastomosis and vascularized lymph node transfer are microsurgical procedures reserved for early extremity lymphedema and do not have efficacy for penile/scrotal lymphedema. The Charles procedure involves the removal of the entire skin, subcutaneous tissue, and muscle fascia with grafting of the underlying muscle. The Charles procedure is rarely performed and is used only for extremity lymphedema and not penile/scrotal disease.

2017 A 12-year-old boy is brought to the office because of penile and scrotal lymphedema. He has had several infections and is dissatisfied with the appearance of his genitalia. Which of the following is the most appropriate next step in management? A) Charles procedure B) Liposuction C) Lymph node transfer D) Lymphatic venous anastomosis E) Tissue excision and skin grafting

The correct response is Option C. Pilomatricoma (also known as pilomatrixoma or calcifying epithelioma of Malherbe) is a common, benign calcifying tumor of the hair appendages that mostly occur under the age of 20. Most occur in the head and neck, but the extremities and trunk are also affected. Surgical excision is the treatment of choice. Malignancy is very rare. Intraoperative findings show a calcific, friable mass adherent to the undersurface of the skin. Unlike a keratinous or sebaceous cyst, there is no discrete capsule or punctum (plugged pore). Recurrence is reported in the 1 to 2% range. This benign growth is related to a somatic (non-inherited) gene mutation CTNNB-1, that is involved in cell replication of the hair matrix. Sebaceous nevus presents as a waxy textured skin patch, often present at birth. The scalp is a common site, although it may present elsewhere. Lesions are slow-growing and benign, but over the course of one's lifetime, they have up to a 50% transformation rate to basal cell carcinoma, with squamous cell carcinoma less likely. Hemangiomas are cutaneous blood vessel proliferations that are bright red or purple in color and are typically present at birth. While they are also common in childhood, they are more superficial in location and have a very different appearance versus pilomatricomas, which are subepithelial. Spitz nevi are melanocytic lesions that can occur in children and adults. They can mimic melanoma though they are benign spindle cell lesions. Malignant transformation is not common, though atypical variants exist, therefore excision is recommended. They appear as dark brown or black macules on the skin.

2017 A 12-year-old girl develops a 12-mm nodule on her right cheek that grows slowly over 2 months. It is firm to the touch, mildly tender, and slightly bluish. There is no redness, ulceration, or visible punctum. Which of the following is the most likely diagnosis? A) Hemangioma B) Keratinous cyst C) Pilomatricoma D) Sebaceous nevus E) Spitz nevus

The correct response is Option A. Infantile hemangioma is the most common tumor of infancy. These lesions typically present between 2 weeks and 2 months of life and have a predictable clinical course, including rapid proliferation during the first 9 months of life (proliferative phase), followed by gradual involution until 3.5 years of age. Infantile hemangiomas most commonly occur in the skin, but can also occur in other sites, with the liver being the most common extracutaneous site. For cutaneous lesions, most are located in the head and neck, followed by the trunk, and then by the extremities. >95% of infantile hemangiomas are diagnosed by history and physical examination; <5% require imaging or biopsy for diagnosis. The onset of a red lesion at 2 weeks of age followed by rapid growth is pathognomonic for infantile hemangioma. The tumor affects both sexes, but is 3 to 4 times more common in females. Infantile hemangioma is also more frequent in premature infants and twins who are low birth weight. Some infantile hemangiomas can ulcerate during the proliferating phase.

2017 A 2-week-old premature male twin develops a red, rapidly enlarging lesion of the posterior trunk. Which of the following is the most important factor suggesting a diagnosis of infantile hemangioma? A) Growth of the lesion B) Location of the lesion C) Patient gender D) Premature birth E) Twin gestation

The correct response is Option A. For most patients with omphaloceles less than 5 cm in diameter, a single operation involving a traditional anterior component separation from costal margin to iliac crest is sufficient to reduce the omphalocele and reapproximate the rectus diastasis. Extended component separations are typically only needed when the omphalocele is large and accompanied by a diaphragmatic hernia. Once the abdominal wall defect extends past 5 cm in diameter, a staged procedure involving the placement of tissue expanders and subsequent flap advancement must be considered. Autologous tissue options, such as the tensor fascia lata flap, may be needed for larger defects that can not be managed with component separation. While prosthetic or biologic mesh placement is used to correct large abdominal wall defects, it is typically used only after autologous options have failed or are not available.

2017 A 2-year-old boy with a history of omphalocele presents for correction of a 5-cm abdominal bulge with a 3-cm widened scar over the bulge. Which of the following is the most appropriate next step in management of the bulge? A) Anterior component separation B) Interposition acellular dermal matrix placement C) Interposition prosthetic mesh placement D) Tensor fascia lata flap E) Tissue expander placement

The correct response is Option C. In this patient, a posterior thigh flap consisting of the biceps femoris, semitendinosus, and semimembranosus muscles and overlying skin is the most appropriate treatment option. Advantages of this flap include a reliable vascular supply (first perforating branch of profunda femoris artery), a generous amount of skin, fat, and muscle to obliterate the dead space after ulcer excision, and the ability to re-advance the flap in the event of ulcer recurrence. Additionally, a posterior thigh flap preserves other donor sites such as the medial thigh, lateral thigh, and gluteal muscles for secondary problems or the inevitable pressure sore in another location. The chief disadvantage of the posterior thigh flap is its upward mobility, as maximal movement is 10 to 12 cm. For extensive ischial defects, a second flap or another flap option may be necessary. Generally, this flap is limited to non-ambulatory patients due to the harvest of multiple knee flexors. Transfer of the biceps femoris alone has been described for small defects in ambulatory patients. The posterior thigh flap is a Mathes/Nahai Type II muscle, with a primary dominant pedicle (first perforating branch of the profunda femoris artery) with smaller segmental distal pedicles. These segmental secondary pedicles can typically be divided with little effect on flap survival. It is recommended that the origin and insertion of the muscles be divided to maximize flap mobility and minimize tension at the inset site. These flaps can be designed as a V-Y configuration or designed in a rotational configuration as shown in the perioperative photograph. The temptation to perform a primary closure should be resisted, as pressure sores by definition indicate a soft tissue deficiency. Simply pulling tissues together over a bony prominence will very likely fail because of wound tension and dehiscence. In this patient, this was already attempted previously with a predictable outcome. Skin grafts have very limited success in this location because of a lack of bulk and resistance to pressure and shearing forces. The gluteal fasciocutaneous flap is based on the inferior gluteal artery and is a good option in the ambulatory patient with an ischial pressure sore. In this case, however, the pedicle is not available owing to the previous gluteal V-Y flap used to repair the sacral pressure sore. In cases when a gluteal fasciocutaneous flap has been utilized previously, the posterior thigh musculature may still be used in the face of a recurrent ischial pressure sore; however, a skin graft would be required. The tensor fascia lata (TFL) flap is the first-line choice for trochanteric pressure sores and has already been used in this patient. The TFL has been described for ischial pressure sores, but only after more reliable flaps have been exhausted. Because the distal 6 to 8 cm of the TFL are unreliable, flap delay or pre-transfer expansion are recommended prior to transfer for an ischial reconstruction.

2017 A 22-year-old man who is a wheelchair-bound paraplegic is evaluated because of a recurrent pressure sore. The patient has had previous surgeries to repair a left trochanteric pressure sore and a large midline sacral pressure sore. A photograph is shown. The most appropriate treatment is debridement followed by coverage with which of the following? A) Adjacent skin advancement B) Gluteal fasciocutaneous flap C) Posterior thigh musculocutaneous flap D) Split-thickness skin graft E) Tensor fascia lata flap

The correct response is Option C. Restoration of craniofacial contour after infection, tumor resection, or trauma can be quite challenging. Autologous bone grafts have long been considered the gold standard because of their high likelihood of osseointegration/healing, and low risk of rejection or infection. Autologous bone grafts, however, have several drawbacks including unpredictable resorption, donor site morbidity, limited availability, prolonged operative times, and difficulty to contour. As a result, there has been an ongoing search for alternative means of reconstruction with alloplastic material. The ideal bone substitute should be chemically inert, easily contoured, able to retain a stable shape over time, strong, resistant to infection or foreign body reaction, inexpensive, and capable of osseointegration and tissue ingrowth. Methylmethacrylate has been used frequently for calvarial reconstruction but suffers several drawbacks, including infection requiring removal of implant, plate fracture, lack of osseointegration, difficulty shaping after polymerization, and necrosis of surrounding tissue due to the exothermic nature of the curing process. Among the most promising and well-tolerated alloplastic materials for craniofacial skeletal reconstruction are the calcium phosphate-based compounds. Hydroxyapatite [Ca(PO4)6(OH)2] forms the principal mineral component of bone and constitutes 60% of the calcified human skeleton. Calcium phosphate compounds are bioactive and capable of osteoconduction and osseointegration. Osseointegration refers to the direct chemical bonding of an alloplast to the bony surface without an intervening fibrous tissue layer. During osteoconduction (creeping substitution), the alloplast acts as a nonviable scaffold for ingrowth of blood vessels and osteoprogenitor cells from the recipient site. Subsequently, the graft/alloplast is resorbed and replaced with new bone. This mechanism is also associated with the healing of cortical bone grafts. Hydroxyapatite (HA) cement is a mixture of tetracalcium phosphate and dicalcium phosphate anhydrous, which react in an aqueous environment to form a paste that can be easily applied and sculpted to fit the surgical defect. HA cement sets isothermically, so there is no risk of thermal damage to the surrounding tissues. Additional benefits of HA include "off the shelf" ease of use, maintenance of volume over time, lack of radiologic scatter, and low incidence of infection. Osteoinduction refers to the direct stimulation of mesenchymal cells at the recipient site by bone morphogenetic protein to differentiate into osteoprogenitor cells. This mechanism of action is associated with the healing of cancellous bone grafts and demineralized bone matrix. Endochondral ossification is the process by which the cartilaginous soft callus covering a fracture is transformed into bone. Osteogenesis is the process by which vascularized bone grafts heal. Viable osteocytes survive the transplantation process and produce new bone at the recipient site. Osteochondrosis refers to a family of ossification disorders in children.

2017 A 23-year-old man comes to the office for post-traumatic cranial reconstruction 6 months after a motor vehicle collision. Physical examination shows a 5 × 4-cm full-thickness calvarial defect in the left parietal region. A titanium/hydroxyapatite cement cranioplasty reconstruction is planned. Which of the following mechanisms best describes the healing process associated with hydroxyapatite? A) Endochondral ossification B) Osteochondrosis C) Osteoconduction D) Osteogenesis E) Osteoinduction

The correct response is Option E. A 60-degree z-plasty lengthens a scar by 75%. If each z-plasty covers 4 cm of scar, each will lengthen the scar by 3 cm, for a total increase of 9 cm. In contrast, a 30-degree z-plasty lengthens an incision by 25%, and a 45-degree z-plasty lengthens an incision by 50%. To prevent undue tension, angles greater than 60 degrees should be avoided.

2017 A 23-year-old man presents 2 years after sustaining full-thickness burns on the anterior neck. He has undergone tissue expansion and local flap reconstruction of the burn defect. He notes webbing and contracture at the margin of one of the prior flap reconstructions. Three identical 60-degree Z-plasties are planned over a total length of 12 cm. The expected gain in scar length is which of the following? A) 3 cm B) 4 cm C) 6 cm D) 8 cm E) 9 cm

The correct response is Option D. The vascular pedicle for the medial sural artery perforator flap arises from the popliteal vessels. The medial sural artery flap is a thin, pliable perforator flap that can provide well vascularized soft-tissue coverage, especially for relatively small defects. It is commonly used for head/neck, hand, and lower-extremity defects. The first perforator is frequently found along a line connecting the mid-popliteal area to the medial malleolus at the 8-cm mark from the proximal end. Preoperative planning is facilitated with ultrasound identification of the perforators. Sub-fascial dissection is frequently performed to protect the perforator and blood supply and to allow for a gliding surface for tendon repairs. Donor sites that are narrower than 5 cm can frequently be closed primarily. The main benefit of the medial sural artery perforator flap over an anterolateral thigh flap is the relative thinness of the flap, which can be significant in overweight or obese patients.

2017 A 24-year-old man comes to the emergency department because of a dorsal hand injury. Physical examination shows a 6 × 4-cm full-thickness defect with exposed metacarpal bones. A medial sural artery perforator flap for soft-tissue coverage is planned. From which of the following vessels does the vascular pedicle for this flap originate? A) Anterior tibial B) Descending genicular C) Peroneal D) Popliteal E) Posterior tibial

The correct response is Option A. Elective aesthetic procedures may be considered in patients with cutis laxa, a genetic disorder with variable inheritance and expressive patterns. The underlying defect is poor elastic tissues due to degeneration of elastic fibers, or a nonfunctioning elastase inhibitor. As a result, patients present with coarse, loose, excess skin throughout the body. In the autosomal dominant form of cutis laxa, the symptoms are confined only to the skin. In the recessive and X-linked forms, there may be other associated conditions such as congenital heart disease, hernias, aneurysms, emphysema, and pneumothorax. Although the effects of cutis laxa worsen with time, there is no underlying issue with wound healing. As a result, surgery may be considered to correct the facial appearance and any functional issues such as ectropion or ptosis. In the other diseases listed, surgery is contraindicated due to poor/unknown wound healing mechanisms. Ehlers-Danlos syndrome (cutis hyperelastica) includes a group of more than 10 different inherited disorders that all involve a genetic mutation affecting collagen and connective tissue synthesis and structure. The clinical presentation includes skin laxity, hyperextensibility and excessive thinness of the skin, joint hypermobility, and aortic aneurysms. Wound healing is poor and elective procedures should not be performed. Elastoderma is a disorder of unknown etiology. Clinical manifestations include pendulous skin laxity initially involving the trunk and extremities that progresses to involve the entire body. Because the effects on wound healing are unknown/unpredictable, elective surgery is not recommended. Werner syndrome is an autosomal recessive disorder characterized by pigmented, indurated, plaque-containing skin, osteoporosis, muscle atrophy, growth retardation, cardiovascular disease, and diabetes. Small vessel angiopathy and poor wound healing are associated. Progeria (Hutchinson-Gilford syndrome) is an autosomal recessive disorder of unknown cause. Findings are similar to premature aging and include lax, excess skin, growth retardation, craniofacial abnormalities, and cardiac disease. Wound healing is poor and the disease is associated with premature death.

2017 A 24-year-old woman comes to the office requesting facial rejuvenation because of premature aging and extensive cervicofacial skin laxity and skin excess. A congenital cause for this patient's condition is suspected. This patient is a candidate for elective surgery if the cause of her condition is found to be which of the following disorders? A) Cutis laxa B) Ehlers-Danlos syndrome C) Elastoderma D) Progeria E) Werner syndrome

The correct response is Option B. Principles of microsurgical nerve repair include the use of meticulous and atraumatic technique with adequate magnification, microsurgical instruments, and sutures. A primary repair is performed whenever possible, provided that the repair is tension-free in order to maximize perfusion to the repair site. In this patient, a 1-cm nerve gap in the ulnar nerve was present even after mobilizing the proximal and distal nerve ends. In this situation, the ulnar nerve may be transposed anteriorly, which would shorten the distance between the nerve ends and allow for primary repair. Nerve transfers are indicated in very proximal nerve injuries where a proximal stump is unavailable for primary repair or grafting, or when a very long nerve gap is present where there would be a concern that target muscle denervation might occur prior to nerve regeneration. Polyglycolic acid nerve conduits are bioabsorbable tubes through which nerve regeneration occurs. They represent an option for nerve reconstruction without any associated donor site morbidity when a nerve gap is present in order to achieve a tension-free repair. Although primary nerve repair is preferable to the use of a graft/conduit, doing so in the setting of this patient's 1-cm nerve gap would not result in a tension-free repair. Autologous nerve grafting, such as with the sural nerve, is an option for nerve reconstruction when a nerve gap is present in order to achieve a tension-free repair.

2017 A 25-year-old man comes to the office after sustaining a deep laceration to the elbow. Physical examination shows decreased function of the ulnar nerve, and the patient is taken for operative exploration and repair. Following proximal and distal dissection, a 1-cm gap between the proximal and distal nerve ends persists. Which of the following is the most appropriate next step in management? A) Nerve transfer B) Nerve transposition C) Polyglycolic acid nerve conduit D) Primary repair E) Sural nerve grafting

The correct response is Option B. Hidradenitis suppurativa is a chronic inflammatory skin disease. Also known as acne inversa, it is characterized by recurrent nodules and abscesses, typically of apocrine gland-bearing skin. This patient has mild hidradenitis, with an initial presentation of a few abscesses without sinus tracts or cicatrization/scarring. Clindamycin applied topically is often used as a first-line therapy for mild hidradenitis. In a randomized, placebo-controlled trial, patients treated with twice-daily topical application of 1% clindamycin solution were found to have significantly less disease burden, and the treatment was well tolerated with few side effects. Although there have been reports of the use of botulinum toxin in the treatment of hidradenitis, its role and efficacy in this setting are currently unclear. Etanercept is a TNF-alpha inhibitor. Although some TNF-alpha inhibitors, particularly infliximab, have shown efficacy in patients with moderate-to-severe hidradenitis, data are conflicting with regard to Etanercept. Prednisone is occasionally used to calm the inflammatory process in severe hidradenitis. However, it does not prevent formation of new lesions and is rarely used for long-term therapy in patients with hidradenitis because of possible adverse effects. A few case reports have described improvement with cyclosporine in refractory cases of hidradenitis. However, it is typically not used for initial medical treatment of hidradenitis, and duration of use is often limited by adverse effects.

2017 A 25-year-old obese man is evaluated because of new onset of inflamed nodules involving the bilateral axillae. A diagnosis of hidradenitis suppurativa is made. Which of the following is the most appropriate initial medication for treatment of this patient's condition? A) Botulinum toxin type A B) Clindamycin C) Cyclosporine D) Etanercept E) Prednisone

The correct response is Option B. Emergent incision and drainage is mandatory for high-pressure paint gun injuries. Although clinically these may appear benign and/or superficial, there is often significant underlying injury. Even small amounts of material can lead to compartment syndrome, poor perfusion, and closed space infections resulting in tissue necrosis and ultimately, amputation. History is critical, but plain films may be used to confirm the diagnosis, as both latex and the less common oil-based paints are easily seen. Grease may be radiolucent or radiopaque, depending on lead content. The most commonly injected materials are paint and grease but can also include paint solvents and fuel oil. Nearly all reported cases involved male occupational injuries and injury to the non-dominant second or third digit, as in this case. These machines can generate pressures of 2,000 to 12,000 pounds per square inch (psi), which far exceeds the 100 psi needed to break the skin. These extreme pressures can propel injected material through the skin and subcutaneous tissues down to the bone or along fascial planes, tendon sheaths, and neurovascular bundles. The overall rate of amputation was 30% and particularly related to the location of injury and type of material injected. Optimal time for wide surgical debridement was within 6 hours of injury. Other studies have documented an amputation rate of approximately 40% when surgery is performed within 6 hours, and an amputation rate of 57% when surgery is delayed beyond 6 hours. The amputation risk is as high as 87% without treatment or if treatment is further delayed. None of the other interventions listed are appropriate for this type of emergent injury.

2017 A 27-year-old man who is right-hand-dominant and works as a manual laborer comes to the emergency department for evaluation 6 hours after inadvertently incurring a high-pressure latex paint injection to the volar aspect of his left index finger. Which of the following is the most appropriate management? A) Admission to the hospital and intravenous administration of antibiotics B) Operative exploration C) Radial gutter splint with follow-up in 3 days D) Topical application of acetone E) Warm compresses, elevation, and observation

The correct response is Option B. The most likely change to the area undergoing expansion is increased blood flow. Expansion causes increased angiogenesis and vascularity to the tissues, which improves survival when flaps are rotated or transposed. Tissue expansion also causes 1) thickening of the epidermis and hyperkeratosis (this resolves after removal of the expander); 2) thinning of the dermis (which normalizes after approximately 2 years); 3) thinning and reduced muscle mass (without diminished function); and 4) permanent loss of up to 50% of adipose tissue.

2017 A 3-year-old girl is undergoing tissue expansion of the scalp and forehead for resection of a giant congenital nevus. Which of the following changes is most likely to be observed in the area undergoing expansion? A) Increased adipose tissue B) Increased blood flow C) Increased muscle mass D) Thickening of dermis E) Thinning of epidermis

The correct response is Option B. A large number of animal, as well as clinical, studies are now pointing to the suboptimal viability of diced cartilage when wrapped with oxidized cellulose polymer (Surgicel). Good viability has been seen when the cartilage is not wrapped, wrapped with fascia, or wrapped with acellular dermal matrix (AlloDerm). Clinically, the data seem to be more nuanced. The use of diced or crushed cartilage is predicated on reducing the noted long-term complications of larger cartilage grafts (warping and prominence or visibility). The clinical data may be favoring use of fascia for wrapping, if using anything at all. AlloDerm may be another promising option.

2017 A 30-year-old woman comes to the office for rhinoplasty. Diced cartilage grafting is planned. Which of the following techniques will result in the lowest viability of the cartilage graft? A) Leaving unwrapped B) Wrapping in oxidized cellulose polymer (Surgicel) C) Wrapping with acellular dermal matrix D) Wrapping with fascia

The correct response is Option E. The most appropriate treatment for this patient is a free nipple graft and mastectomy due to breast size and nipple ptosis. Gender dysphoria is a commonly acknowledged disorder, affecting up to 0.3% of the population. Hundreds of patients have undergone subcutaneous mastectomy surgery with a high reported patient satisfaction rate. Smaller-sized patients can achieve excellent results with periareolar mastectomy or donut excision to reduce large areola size. Large patients with ptosis, similar to gynecomastia surgery, require longer scars for skin removal and have good results, but longer scars. One study of outcomes found that patients and surgeons preferred the outcome appearance with free nipple graft and a single inframammary crease scar versus an extended areola incision with scars across the mid chest. Neither liposuction nor periareolar reduction would adequately reduce the skin envelope and breast tissue. Circumvertical and Wise pattern breast reduction would create a more feminized shape.

2017 A 32-year-old patient, who was born as a female, identifies as male and requests breast reduction surgery for a masculine appearance. The patient has C-cup breasts with grade 2 ptosis. Which of the following is the most appropriate surgical option? A) Liposuction B) Periareolar breast reduction C) Circumvertical breast reduction D) Wise pattern breast reduction E) Mastectomy with free nipple graft

The correct response is Option C. Several recent studies have confirmed that a surgical margin of 15 to 20 mm is associated with high rates of recurrence-free survival and primary closure when wide local excision is performed. Marginal excision is associated with higher rates of recurrence, and larger wide local excisions (>20 mm) are associated with similar recurrence-free survival but a much higher need for reconstructive surgery. Mohs micrographic surgery has shown promise, with higher initial clearance rates using smaller margins, but the question specifically addressed surgical margins when planning wide local excision.

2017 A 34-year-old woman comes to the office because of a 6 × 7-cm subcutaneous mass below the left scapula. Biopsy confirms dermatofibrosarcoma protuberans. To minimize recurrence yet maximize the chances of primary closure, which of the following is the most appropriate margin when planning wide local excision? A) 5 mm B) 10 mm C) 20 mm D) 40 mm E) 50 mm

The correct response is Option C. The most important vessels to anastomose in this clinical scenario are the deep dorsal arteries and vein. There are two deep arteries and one deep vein. There is a superficial dorsal vein as well, but no superficial dorsal artery runs with it. The vascular supply of the penis is from the internal iliac system, not the external iliac system. The internal pudendal artery comes off the anterior division of the internal iliac artery and is the main blood supply to the penis. It branches into the bulbar, cavernosal, and dorsal penile arterial branches. The other vessels listed are all involved in the blood supply to the penile structure, but are too small, less accessible, or too proximal for replantation purposes. The deep dorsal vessels alone can adequately revascularize the whole penile structure. The steps in replantation include: formation of a suprapubic urinary diversion, urethral anastomosis over a Foley catheter, corporal body coaptation by approximating the tunica albuginea, microsurgical anastomosis of the dorsal vessels and coaptation of the dorsal nerves, and skin closure.

2017 A 35-year-old man is brought to the emergency department for penile amputation at the base of the shaft. The amputated part has been appropriately transported with the patient. Revascularization of which of the following vessels will result in the highest probability of successful replantation? A) Bulbar artery and vein B) Cavernosal artery and vein C) Deep dorsal arteries and vein D) Internal pudendal artery and vein

The correct response is Option B. The transversus abdominis plane (TAP) block is a regional anesthetic that blocks sensory afferent nerve fibers that supply the anterior/lateral abdominal wall dermatomes of T6-L1. These sensory nerves travel below the internal oblique muscle in the plane above the transversus abdominis muscle. Traditionally, the technique is performed blindly by placing a needle through the triangle of Petit posteriorly until the needle reaches the TAP. Once the needle is in the appropriate plane, 20 mL of a long-acting local anesthetic, such as bupivacaine, is injected. More recent modifications include the use of ultrasound guidance to optimize precise placement and the use of diluted long-acting multivesicular liposomal bupivacaine (Exparel). Several studies have demonstrated the benefits of a TAP block during abdominal surgery. These benefits include decreased pain, opioid use, and nausea/vomiting, as well as faster return of bowel function. Complications include potential systemic toxicity due to dose of anesthetic delivered/inadvertent intravascular injection and intraperitoneal injection with possible injury to intraabdominal organs such as, the liver or spleen.

2017 A 35-year-old woman is scheduled for abdominoplasty with flank liposuction. Regional anesthetic block is planned. The most appropriate location for placement of the anesthetic is between which of the following? A) External oblique muscle and internal oblique muscle B) Internal oblique muscle and transversus abdominis muscle C) Skin and external oblique muscle D) Transversalis fascia and peritoneum E) Transversus abdominis muscle and transversalis fascia

The correct response is Option A. The descending genicular artery supplies the medial femoral condyle flap. The descending genicular artery is a branch of the superficial femoral artery. This corticoperiosteal flap has been used with increasing frequency for small bony defects and the treatment of nonunion. A cutaneous component can also be harvested based on a saphenous artery branch. The anterolateral thigh flap is based off the lateral femoral circumflex artery perforators. The gracilis muscle flap is based off vessels from the medial circumflex femoral artery; perforator flaps can also be designed based off this vascular system. The profunda femoris supplies the posterior thigh flap. The peroneal artery provides the vascular supply to the fibula flap.

2017 A 40-year-old man comes to the office for evaluation of a nonhealing scaphoid fracture. History includes bone grafting for avascular necrosis of the proximal pole, but there is evidence of nonunion. Carpal collapse and humpback deformity are also noted. Reconstruction with a vascularized medial femoral condyle flap is scheduled. Which of the following arteries provides the blood supply to this flap? A) Descending genicular artery B) Lateral circumflex femoral artery C) Medial circumflex femoral artery D) Peroneal artery E) Profunda femoris artery

The correct response is Option B. This patient meets the diagnostic criteria for anaphylaxis, manifested by acute onset of generalized hives and swelling along with either respiratory compromise or hypotension. Intramuscular injection of epinephrine, 0.3 to 0.5 mg, preferably in the mid-outer thigh, is the initial treatment in adults. Other treatments include administration of supplemental oxygen and rapid normal saline infusion. Absorption of epinephrine after subcutaneous injection may be too slow to reverse anaphylaxis. Bolus injections of epinephrine should be avoided because of the risks of dosing errors and overdose. Epinephrine infusion can be considered for anaphylaxis that is refractory to intramuscular injections, but it is not a first-line choice. Subcutaneous or intradermal injection would result in delayed absorption.

2017 A 42-year-old woman presents for fat grafting from the abdomen to the upper breast poles under intravenous sedation. Patient history includes implant-based breast reconstruction. She receives 1 g of intravenous cefazolin in the preoperative holding area. Ten minutes later, she has onset of generalized hives, flushing, and swelling of the lips. She reports dyspnea and has audible wheezing. The most appropriate next step is administration of epinephrine by which of the following routes? A) Intradermal injection B) Intramuscular injection C) Intravenous bolus D) Intravenous infusion E) Subcutaneous injection

The correct response is Option B. The innervation to the anterior abdominal wall comes from the anterior cutaneous branches of the 6th to 12th intercostal nerves. The anterior cutaneous nerves perforate the anterior rectus sheath and are severed during the undermining of the abdominal flap during abdominoplasty. The hypogastric or infraumbilical region becomes the distal-most point for sensory innervation from the surrounding intact lateral cutaneous branches of the intercostal nerves that innervate the lateral anterior abdominal wall. The transverse incision from the abdominoplasty limits innervation from the pubic and thigh regions. The mons pubis should not demonstrate much change in sensation as it lies inferior to the surgical scar. Patients undergoing abdominoplasty should be made aware of the likely decrease in sensation of this area, including superficial touch, superficial pain, pressure, vibration, and temperature.

2017 A 43-year-old woman comes to the office for consultation regarding abdominoplasty. She says she is concerned about postoperative hypoesthesia of the abdominal wall. Which of the following areas is most likely to have the greatest decrease in sensation after a traditional abdominoplasty in this patient? A) Epigastric B) Infraumbilical C) Lateral abdominal D) Mons pubis E) Subxiphoid

The correct response is Option B. Preoperative optimization is essential in patients undergoing pressure sore reconstruction. This includes adequate nutrition (albumin greater than 3.5 g/dL, 1.5 to 3.0 g/kg/day of protein intake, 25 to 35 cal/kg of nonprotein calories daily), eradication of infection, treatment of spasms/contractures, smoking cessation, urinary/stool diversion in select circumstances, and an appropriate postoperative plan of care including pressure off-loading. A multivariate analysis of risk factors for failure of pressure sore flap reconstruction included hemoglobin A1c greater than 6%, younger age, albumin concentration less than 3.5 g/dL, and ischial location. In this patient, the primary risk factor for failure of reconstruction would be a high hemoglobin A1c.

2017 A 45-year-old paraplegic man with a history of type 1 diabetes mellitus comes to the physician because of a stage 3 trochanteric pressure sore. The wound has been adequately debrided, and myocutaneous flap reconstruction is planned. Albumin concentration is 3.8 mg/dL, and hemoglobin A1c concentration is 6.3%. Diet includes protein intake of 2.5 g/kg daily. Which of the following factors is most likely to be associated with failure of the reconstruction? A) Albumin concentration B) Hemoglobin A1c C) Patient age D) Pressure sore location E) Protein intake

The correct response is Option C. Chromic gut is an absorbable natural monofilament suture whose strength decreases to 50% in approximately 14 days, with near complete loss of strength at approximately 3 weeks. Polyglactin (Vicryl) is an absorbable synthetic polyfilament suture whose strength decreases to 50% in approximately 2 to 3 weeks, with near complete loss of strength at approximately 1 month. Poliglecaprone (Monocryl) is an absorbable synthetic monofilament suture whose strength decreases to 50% in approximately 7 to 10 days, with near complete loss of strength at approximately 3 weeks. Polydioxanone (PDS) is an absorbable synthetic monofilament suture whose strength decreases to 50% in approximately 4 weeks, with near complete loss of strength at approximately 6 weeks. Polypropylene is a a nonabsorbable synthetic monofilament suture.

2017 A 45-year-old woman undergoes breast reconstruction with a transverse rectus abdominis musculocutaneous (TRAM) flap. During donor site closure, a size-0 absorbable suture that maintains the greatest strength over time is desired for closure of Scarpa fascia. Which of the following sutures is most appropriate? A) Chromic gut B) Poliglecaprone (Monocryl) C) Polydioxanone (PDS) D) Polyglactin (Vicryl) E) Polypropylene (Prolene)

The correct response is Option E. Injection of botulinum toxin around the digital vessels in the palm has been shown to decrease pain associated with vasospastic disorders like Raynaud phenomenon. This is a relatively quick, easy, and low-risk method of treating a patient with incapacitating ischemic pain of the hand. The exact mechanism by which botulinum toxin works in this clinical scenario is still under investigation, but some theories suggest an effect on the vessels and/or nerves of the hand through inhibition of sympathetic nerves, sensory nerves (c-fibers), substance P, and/or other signal transduction pathways. Studies show a 75 to 100% reduction in pain and up to 50% healing of chronic ulcers. Approximately 10 units of botulinum toxin is bathed around each of the digital neurovascular bundles in the palm. The most common side effect reported is temporary minor intrinsic hand weakness. Injecting botulinum toxin in the skin or too proximally in the wrist has not shown the same response as around the digital neurovascular bundles in the palm. Surgical sympathectomies by stripping the adventitia of the digital and wrist vessels have also shown some success in symptom control. Stellate ganglion blocks have also been used for this purpose among others (complex regional pain syndrome); however, local anesthetics, not botulinum toxin, are used to block the ganglion.

2017 A 45-year-old woman with scleroderma is evaluated because of a 2-year history of severe resting pain in both hands. She does not smoke cigarettes. Despite appropriate medication therapy, she has had no relief of her symptoms. Injection of botulinum toxin type A into which of the following locations is the most appropriate treatment for this patient's Raynaud phenomenon? A) Around the stellate ganglion B) Intradermal at the wrist C) Intradermal in the palm D) Perivascular at the wrist E) Perivascular in the palm

The correct response is Option D. Collagen bilayer matrices have become an important option in the reconstructive ladder for lower extremity wounds. Studies have demonstrated the ability of these dermal regeneration templates to neovascularize and heal into pliable, durable coverage in an attempt to achieve stable wound healing and maintain limb length. Many of these studies were performed in the setting of diabetic wounds with exposed bone or tendon, thus each of these settings does not represent a contraindication. Adequate debridement, including clearance of any polymicrobial infection, is one of the keys to successful reconstruction.

2017 A 46-year-old man with type 1 diabetes mellitus is evaluated for an infected foot ulcer. After adequate surgical debridement, a collagen bilayer matrix is used for coverage. Which of the following clinical factors represents the greatest risk for failure of reconstruction? A) Anatomic location B) Exposed bone C) Exposed tendon D) Polymicrobial infection E) Type 1 diabetes mellitus

The correct response is Option A. To optimize wound healing, a moist wound-healing environment has been shown to be superior to a dry wound-healing environment. Studies on split-thickness skin graft donor sites have not been very well designed, but many studies suggest that a moist dressing is better than a dry dressing, and several review papers support this concept. Although leaving petrolatum gauze open to air is very common and may be the most practical option in certain circumstances, it does not optimize wound healing compared with a moist dressing. The only options listed that provide a moist environment for the duration required for early reepithelialization are gauze covered with occlusive dressing and alginate dressings. Gauze covered with occlusive dressing would not work well, because conventional gauze would stick to the wound and be very difficult to remove without causing significant tissue injury. Alginate dressings are emerging as an excellent option for split-thickness skin graft donor site wounds. They are adaptable, absorptive, nonadhesive, antibacterial, and provide a moist environment for wound healing.

2017 A 47-year-old man undergoes split-thickness autografting for the treatment of a forearm burn. Which of the following donor site dressings is most appropriate to optimize wound healing? A) Alginate covered with occlusive dressing for 7 days B) Moist gauze covered with occlusive dressing for 7 days C) Petrolatum gauze covered with occlusive dressing for 2 days, then left open to air D) Petrolatum gauze left open to air E) Xenograft left open to air

The correct response is Option D. Mohs micrographic surgery is a surgical technique in which tumor excision and microscopic examination of tissue margins are performed by the same surgeon. Use of a beveled excision and careful mapping of the peripheral and deep margins of horizontal frozen sections allow for comprehensive examination of all the borders of the excised tissue, resulting in excellent cure rates. In addition to the high cure rate, Mohs surgery is a tissue-sparing procedure that is an important advantage in cosmetically and functionally sensitive areas and contrasts with traditional approaches in which a set margin of excision is performed. Indications for Mohs surgery include recurrent basal cell carcinomas (BCC) and squamous cell carcinomas (SCC), locations prone to recurrence ("H-zone" of the face: inner canthus, nasolabial fold, nose, periorbital, temple, upper lip and periauricular regions, retroauricular, and chin), at/near critical structures (e.g., eye, lip), large tumors (>2 cm), ill-defined tumor margins, aggressive histology (BCC - morpheaform infiltrative, basosquamous, perineural; SCC - poorly differentiated, invasive, perineural), and special hosts (immunosuppressed, basal cell nevus syndrome, xeroderma pigmentosum). Therefore, in this patient, the primary indication for Mohs surgery would be the recurrent nature of her BCC.

2017 A 50-year-old woman with a history of scleroderma is evaluated because of a 1.5-cm lesion on her right cheek. Patient history includes basal cell carcinoma excision at the same site 3 years ago. A punch biopsy shows basal cell carcinoma (micronodular subtype). Which of the following is the most appropriate indication for Mohs micrographic surgery in this patient? A) Histologic subtype B) History of scleroderma C) Location of lesion D) Recurrence of lesion E) Size of lesion

The correct response is Option E. The most appropriate next therapy option for this patient is systemic corticosteroids. These ulcerative lesions are most likely pyoderma gangrenosum (PG), an ulcerative cutaneous condition of unknown etiology. This condition is most likely associated with other systemic diseases like inflammatory bowel disease, or immunologic diseases. This diagnosis is usually one of exclusion, and one must have a high index of suspicion for ulcerative wounds that are persistent despite adequate workup and treatment. One must be especially aware of PG's association with a condition known as pathergy. This is a phenomenon in which surgical manipulation of the area or distant sites may trigger worsening of the ulcerative condition and/or development of the condition in an area of skin trauma. First-line therapy for PG involves the use of prednisone. Other anti-inflammatory agents, including immunosuppressive agents, and biologic agents have also been used. The prognosis is generally good; however, the disease can recur and residual scarring is common. Because of these factors, the other options are not the most appropriate next steps in the treatment of this patient.

2017 A 50-year-old woman with systemic lupus erythematosus is evaluated because of a nonhealing ulcer of the right lower extremity. It started as a small pustule 3 months ago and steadily worsened to an ulcerative lesion. Examination of a biopsy specimen ruled out malignancy. Cultures have been negative for more than 4 weeks. Debridement of the wound and skin grafting are attempted but result in loss of the graft and development of similar ulcerative areas at the donor site. Which of the following is the most appropriate next step in management? A) Bilayer skin substitute B) Fasciocutaneous flap C) Hyperbaric oxygen therapy D) Long-term antibiotic therapy E) Systemic corticosteroid therapy

he correct response is Option E. There are many choices for bone grafts, both vascularized and nonvascularized. In this clinical scenario, there is no question that a vascularized bone graft is indicated, given the irradiated field, anterior location, and >6-cm defect. The choice of vascularized bone grafts include the free fibula, free iliac crest, free scapula, and free radius bone grafts. All have their pros and cons individually, although collectively, vascularized bone free flaps provide 40% more strength, 56% more stiffness, higher complete arthrodesis rate, and superior functional outcomes. The free fibula flap would be the most common choice for this situation given its long pedicle (6 to 10 cm), the large amount of usable bone (22 to 24 cm), minimal donor site morbidity, ability to accept dental implants, and reliable skin flap (skin island survival rates approaching 100% due to increased anatomical understanding and improvements in harvest techniques). However, its disadvantage is that it doesn't have the vertical height of the free iliac crest.

2017 A 53-year-old man is undergoing revision of mandibular reconstruction. An iliac crest osteocutaneous free flap is planned. Which of the following is the main advantage of this flap compared with a free fibula osteocutaneous flap? A) Bone length B) Minimal donor site morbidity C) Pedicle length D) Reliability with multiple osteotomies E) Vertical height

The correct response is Option B. Polymethylmethacrylate polymerization is an exergonic or exothermic reaction. The heat generated can cause injury or necrosis of the surrounding soft tissues and bone. Saline irrigation is an effective method of heat dissipation during this process. The toxic byproducts of polymerization include cyanide gas in small amounts that are dissipated by normal operating room ventilation or suction. Water is not a necessary cofactor for the polymerization process. Reduction of bacterial burden is a rationale for irrigation in general but does not play a specific role in this clinical setting. The dielectric constant of the substrate is elevated or unchanged by the irritant and this is unlikely to have an effect on the polymerization process.

2017 A 53-year-old man presents with a calvarial defect following neurosurgical extirpation of a tumor. Polymethylmethacrylate reinforced by wire mesh is used for the cranioplasty. The use of saline irrigation during the curing of the biopolymer serves which of the following purposes? A) Dilutes toxic byproducts of polymerization B) Dissipates thermal energy C) Irrigates subclinical bacterial colonization D) Provides an aqueous catalyst for polymerization E) Reduces the dielectric constant of the substrate

The correct response is Option A. The omental flap is supplied by the gastroepiploic vessels. Common options for sternal wound reconstruction include the pectoralis major, rectus abdominis, latissimus dorsi, and omental flaps. The use of an omental flap for a mediastinal defect was described in the 1970s; however, muscle flaps became a popular choice for reconstruction in the 1980s. Based on the size of the defect, the omental flap can be used with or without a skin graft. The omentum has angiogenic and immunogenic properties that make it ideal for reconstruction of sternal wound infections. The omentum is based on the left and right gastroepiploic vessels. In order to increase length, the flap can be based on one set of vessels, usually the right gastroepiploic vessels. The left gastroepiploic vessels are a branch of the splenic vessels; the right gastroepiploic vessels are a branch of the gastroduodenal vessels. Harvest can be performed through either an upper abdominal incision, transdiaphragmatic, or laparoscopically. There is a risk of donor site morbidity such as abdominal wound infections or symptomatic hernias. The superior mesenteric vessels supply the lower part of the duodenum extending to the middle third of the transverse colon, as well as the pancreas. The left and right gastric vessels supply the lesser curvature of the stomach. The short gastric vessels supply a portion of the greater curvature of the stomach and are branches of the splenic vessels. The left and right gastroepiploic vessels supply the greater curvature of the stomach along with the omentum.

2017 A 55-year-old man who recently underwent a cardiac bypass procedure has a sternal infection that requires debridement. The defect is evaluated, and reconstruction using an omental flap is planned. Which of the following vessels provides the blood supply for this flap? A) Gastroepiploic B) Left gastric C) Right gastric D) Short gastric E) Superior mesenteric

The correct response is Option E. In patients undergoing free flap reconstruction, the use of vasopressors is typically avoided when possible because of concerns that vasoconstriction of the anastomoses will result in microvascular thrombosis. When feasible, intravenous fluid administration should be attempted first to address hypotension. However, numerous studies have examined the effect of intraoperative vasopressors on free flap reconstructions and have generally not found an increased risk of postoperative complications.

2017 A 55-year-old man with a history of squamous cell carcinoma undergoes glossectomy and reconstruction with a free radial forearm flap. Intraoperatively, the patient experiences hypotension, and norepinephrine is administered. Which of the following is the most likely effect of this treatment on the outcome of the free flap? A) Delayed wound healing B) Microvascular thrombosis C) Partial flap loss D) Total flap loss E) No effect

The correct response is Option B. The patient has actinic keratoses (AKs). AKs are common in in people with significant sun exposure and are a response to ultraviolet radiation. The likelihood of malignant transformation to squamous cell carcinoma (SCC) is approximately 10%. There are various treatment modalities for AKs, including cryotherapy, 5-flurouracil (5-FU), photodynamic therapy, superficial glycolic peels, and imiquimod. Cryotherapy with liquid nitrogen is commonly used for isolated lesions, while the other therapies are more commonly used for diffuse disease.

2017 A 58-year-old Caucasian farmer comes to the office because of several pink scaly macules on his cheeks and nose. Biopsy of one of the lesions shows pleomorphic keratinocytes within the basal layer of the epidermis and hyperkeratosis, consistent with actinic keratosis. If left untreated, which of the following is the likelihood that these lesions will become malignant? A) 0% B) 10% C) 30% D) 50% E) 70%

The correct response is Option D. The pudendal thigh flap is an axial patterned flap based on terminal branches of the superficial perineal artery. The superficial perineal nerve follows the course of this artery and becomes the posterior labial nerve to provide sensation to the proximal skin paddle. The superficial perineal nerve arises from the pudendal nerve. In a series of 19 patients who underwent pudendal thigh flap reconstruction, all reported sensation. The gracilis myocutaneous flap may provide some pressure sensation from cutaneous branches of the obturator nerve, although this is inconsistent. The anterolateral thigh perforator flap and vertical or oblique rectus abdominis myocutaneous flap may be neurotized to provide sensation; however, this would not result in immediate sensation.

2017 A 58-year-old woman is evaluated for squamous cell carcinoma of the posterior vagina. Wide local excision of the vagina with flap reconstruction is planned. Which of the following flap options is most likely to provide a reconstruction that is sensate immediately after surgery? A) Gracilis myocutaneous B) Oblique rectus abdominis myocutaneous C) Pedicled anterolateral thigh perforator D) Pudendal thigh (Singapore) E) Vertical rectus abdominis myocutaneous

The correct response is Option D. Breast implant technology has evolved greatly since implants were introduced in the 1960s. Increased cross-linking of silicone polymers (polydimethylsiloxanes) results in a more stable, cohesive form and closer shell-gel interactions. Advantages of these more "form-stable" implants include lower rates of rippling and rupture. They allow for the creation of shaped implants that offer clear advantages for certain patients, such as those seeking a natural upper pole shape transition, and those with wider or taller breast shapes. The biggest drawback of shaped implants is the need to place them in a precise surgical pocket lest they rotate, causing deformity and potentially requiring reoperation. As long as surgeons follow sound surgical principles of dissecting an appropriate pocket limited to the approximate width of the implant, malrotation rates are low, typically in the 1.5% range. In one study, half of patients with implant rotation improved with manual repositioning and taping for 3 to 6 weeks, while the other half required reoperation. Infection rates do not vary among implant types. Shaped implants have textured shells, which have been shown to have lower rates of capsule contracture, particularly in the subglandular position. Implant rupture rates are also lower in new generation implants, in the 0.7% per year range. Visible rippling rates are more common in thinner consistency implants, such as saline and older silicone devices.

2017 A 58-year-old woman undergoes removal of round 280-cc silicone gel implants she has had for over 30 years. New silicone gel implants measuring 10 cm in width by 12 cm in height with a 5-cm projection are placed. Compared with her original gel implants, the new implants are more likely to have a higher rate of which of the following complications? A) Contracture B) Infection C) Rippling D) Rotation E) Rupture

The correct response is Option A. The American Joint Committee on Cancer (AJCC) revised TNM staging of melanoma in 2010. Three subgroups of distant metastases are distinguished: skin and soft-tissue metastases (best prognosis), lung metastases (intermediate prognosis), and other visceral metastases such as liver and brain (worst prognosis). Elevated lactate dehydrogenase in either of the first subgroups up-stages to the last subgroup.

2017 A 63-year-old man is evaluated because of a 1.6-cm pigmented lesion on his right shoulder. Excisional biopsy shows superficial spreading malignant melanoma with a Breslow depth of 3 mm. Evidence of distant metastases to which of the following tissues on further staging carries the poorest prognosis for this disease? A) Liver B) Lung C) Lymph nodes D) Subcutaneous tissue

The correct response is Option A. The best method for initial excision in this critically ill patient is excision down to fascia (fascial excision). Excision down to viable tissue (tangential excision) with or without a tourniquet would result in a large amount of blood loss, which would be a significant physiologic insult for this patient; additionally, it may not eradicate the suspected fungal infection. Tangential excision using tumescence may not result in significant blood loss, but the ability to judge viable from nonviable tissue is compromised, and this approach would have a high likelihood of requiring further debridement to achieve a healthy tissue bed unless performed by extremely experienced burn surgeons. Fascial excision is also much faster than tangential excision, which is an important consideration in this critically ill patient. Hydrosurgical debridement is adequate for superficial burns but has no role in a large flame burn and would result in excessive blood loss and operative time.

2017 A 63-year-old man who was in a house fire has burns on 55% of his body including the upper limbs, chest, abdomen, and left leg. He underwent escharotomies and has been resuscitated, but he requires mechanical ventilation because of an inhalation injury. He has an evolving acute kidney injury. A photograph is shown. Immediate excision is planned for management of a suspected fungal infection of the burn wounds. Which of the following is the most appropriate method for initial excision in this patient? A) Excision down to fascia without a tourniquet B) Excision down to viable tissue using tumescence C) Excision down to viable tissue with a tourniquet D) Excision down to viable tissue without a tourniquet E) Hydrosurgical debridement without a tourniquet

The correct response is Option A. The patient has basal cell nevus (Gorlin) syndrome. It is an autosomal dominant genetic condition affecting 1 in 56,000. Males and females are equally affected. It is caused by a mutation in the PTCH1 gene. Clinical characteristics include multiple basal cell carcinomas, odontogenic cysts of the mandible, facial dysmorphism, and skeletal abnormalities of the vertebrae, skull, and ribs. 5 to 10% of patients will develop medulloblastoma. The other malignancies are not associated with Gorlin syndrome.

2017 A 7-year-old boy is evaluated because of new nodular lesions on his skin. Patient history includes a jaw cyst, pits in the hands and feet, and a treated meduloblastoma. Biopsy is planned. Which of the following is the most likely diagnosis? A) Basal cell carcinoma B) Melanoma C) Merkel cell carcinoma D) Sebaceous adenocarcinoma E) Squamous cell carcinoma

The correct response is Option A. This patient has gout and presents with a gouty tophus of the elbow. The red nodule over a joint with milky white fluid is diagnostic. The treatment of choice is an anti-inflammatory agent, and colchicine is the most common. Gout results from an imbalance in purine metabolism, resulting in uric acid crystal deposition in the joints. The great toe is most commonly affected, classically known as podagra. When these nodules occur in the upper extremity, it is easy to misdiagnose them as bacterial in origin. For unknown reasons, gout is more common in men and occurs more frequently after surgery of any type. Vancomycin would be appropriate for a severe, systemic bacterial infection such as methicillin-resistant Staphylococcus aureus (MRSA). In this case, purulent drainage would be expected, rather than the milky fluid with granules that was encountered. Fluconazole is an antifungal. Methotrexate is used to treat rheumatoid arthritis, not gout. Doxorubicin is an antineoplastic chemotherapy agent and would be used to treat a biopsy-confirmed cancer.

2017 A 72-year-old man is referred for surgical treatment of a 3-cm, tender red mass on the left elbow. It developed spontaneously over the past 3 weeks, and has not improved with 10 days of oral cephalexin therapy. During incision and drainage, the mass is found to be filled with copious milky white fluid with white solid granules. Pathology shows crystal deposits in the fluid. In addition to wound packing, which of the following is the most appropriate therapy? A) Colchicine B) Doxorubicin C) Fluconazole D) Methotrexate E) Vancomycin

The correct response is Option A. Arterial thrombosis is associated with lower flap salvage rates than venous thrombosis or mechanical causes. Bui et al. demonstrated salvage rates of 40%, 71%, and 90% for arterial thrombosis, venous thrombosis, or hematoma, respectively. Selber et al. demonstrated a similar tendency toward flap failure after arterial thrombosis. They also documented a 92% flap salvage rate for mechanical causes of ischemia, compared with 64.9% for vessel thrombosis. Mechanical or extrinsic causes of flap ischemia are generally easy to correct and are less likely to be associated with vessel injury. It is postulated that arterial thrombosis is more likely to be associated with endothelial injury than venous thrombosis.

2017 A 72-year-old man undergoes composite mandibular resection followed by fibula osteocutaneous free flap reconstruction. Patient history includes squamous cell carcinoma of the oropharynx. The morning after surgery, ischemic compromise of the flap is noted. Urgent exploration of the microvascular anastomosis is performed. Which of the following causes of flap compromise is most likely to result in failure of salvage attempts? A) Arterial thrombosis B) Hematoma C) Pedicle kinking D) Venous thrombosis

The correct response is Option C. The key insight into the proper technique for this patient is the prior use of radiation on his nose. This should prompt the reconstructive surgeon to bring in healthy, well perfused, non-irradiated tissue to the area to be reconstructed whenever possible. Out of all the options presented, nasolabial flap fits this option the best. Any local nasal flap will leave the surgeon to deal with unpredictable previously irradiated nasal skin. The outcome can be less reliable because of perfusion and possibly unfavorable tissue pliability and mobility. As was mentioned, this patient's wound bed was previously irradiated. Therefore, any type of skin graft, split- or full-thickness, may result in poor graft survival. Radial forearm free flap is not indicated in a small defect where regional tissue can be used.

2017 A 73-year-old man has recently undergone Mohs micrographic surgery for a basal cell carcinoma of the nasal sidewall with a resultant 1.5-cm skin-only defect. History includes prior irradiation to the nose for squamous cell carcinoma. The nasal skin has significant radiation skin changes. Which of the following methods of reconstruction is most appropriate for this patient? A) Full-thickness skin grafting B) Local nasal skin flap C) Nasolabial flap D) Radial forearm free flap E) Split-thickness skin grafting

The correct response is Option D. The sartorius muscle classically has a type IV Mathes-Nahai vascular anatomy, which may limit its arc of rotation. In a recent study, even though the sartorius muscle has multiple segmental pedicles, there tend to be codominant superior and inferior pedicles that could possibly allow for the majority of the muscle to be raised on either the superior or inferior dominant pedicle. Mathes-Nahai vascular anatomy classification for muscle flaps: I - Single dominant vascular pedicle II - Single dominant vascular pedicle with secondary minor vascular pedicles III - Codominant major vascular pedicles IV - Multiple segmental vascular pedicles V - Dominant vascular pedicle with segmental secondary pedicles that can supply muscle if dominant is divided Type II vascular anatomy is seen with the rectus femoris, vastus medialis, and gracilis muscles. The rectus abdominis has a type III vascular anatomy. Type V would be a latissimus dorsi muscle flap.

2017 A 75-year-old man who had femoral-popliteal bypass surgery 2 weeks ago has an infection in the proximal groin. A muscle flap to fill the dead space is planned. Which of the following muscle flap options has a type IV Mathes-Nahai (multiple segmental vascular pedicles) vascular anatomy? A) Gracilis B) Rectus abdominis C) Rectus femoris D) Sartorius E) Vastus medialis

The correct response is Option B. The best option to achieve wound closure in this patient is an ipsilateral latissimus dorsi myocutaneous flap. For chronic wounds in an irradiated field, the best option is debridement followed by transfer of healthy, nonirradiated tissue. Negative pressure wound therapy is likely to result in a recurrent chronic wound, albeit a clean one. Split-thickness skin graft would be inappropriate in an irradiated wound bed with exposed bone. Autologous fat grafting can help improve the quality of irradiated tissues in the absence of a wound, and some studies have shown promise in the treatment of superficial radiation ulcers; however, this patient has necrotic rib, and following debridement the wound will be deep and large. Although wound management and fat grafting have been shown to promote healing in isolated cases, this approach is not yet an accepted standard of care. Adjacent tissue transfer will employ irradiated tissue, and is thus prone to necrosis, wound breakdown, and recurrent chronic wound formation. A free flap could be an option, but a forearm flap would not have the volume required for the expected defect. In addition, a free flap is more morbid than a local pedicled flap in this elderly patient.

2017 A 76-year-old woman with a history of left modified radical mastectomy and radiation therapy comes to the office because of a chronic wound of the left axilla associated with limitation of abduction and exposed rib at the wound base. A photograph is shown. Examination of a biopsy specimen excludes malignancy. In addition to appropriate debridement, which of the following is likely to be most effective in achieving wound closure? A) Adjacent tissue transfer B) Left latissimus dorsi myocutaneous flap C) Negative pressure wound therapy D) Radial forearm fasciocutaneous free flap E) Split-thickness skin graft

The correct response is Option D. The most likely subtype of melanoma in this patient is nodular. Nodular melanomas are the second most common subtype (10 to 20%) and are commonly seen in the trunk, head, and neck with a slightly increased incidence in men. Upon invading the dermis, these lesions have a rapid vertical growth phase compared to the radial growth phase of the superficial spreading subtypes. These lesions are often dark and dome-shaped and have the appearance of a blood blister. They have increased metastatic potential. Histologic examination shows a sharply circumscribed epidermal component and extensive dermal proliferation with atypical melanocytes. The superficial spreading subtype is characterized by lateral spreading of malignant melanocytes in the epidermis. This is the most common subtype and exhibits a prolonged radial growth phase before developing a vertical component. These lesions occur usually on sun-exposed skin and often arise in preexisting nevi. They appear flat and become irregular or raised with growth. Lentigo maligna is a rare form of melanoma with low malignant potential. They often arise from lentigo maligna lesions and grow slowly in a radial fashion before a vertical phase. They are more common in older women and have a strong correlation to sun exposure. They are often present in the face, head, and neck as large, tan lesions with convoluted patterns. Acral lentiginous melanoma is more common in dark-skinned patients and is often found on the palms, nail bed, and soles of the feet. These lesions are aggressive and frequently metastasize. Desmoplastic melanoma is also a rare subtype with aggressive local growth; however, these lesions rarely metastasize. They are similar in appearance histologically to spindle cell tumors and are confused with common nevi, Spitz nevi, or hemangiomas.

2017 A fair-skinned 55-year-old man is evaluated because of an irregular dark lesion on his back. Patient history includes frequent sunburn. On examination, the lesion is 8 mm in size, dome-shaped, dark, and has the appearance of a blood blister. The border of the lesion does not appear irregular. A biopsy of the lesion shows a well circumscribed proliferation of atypical melanocytes extending vertically throughout the dermis. There is no lateral extension of the intra-epidermal component. Which of the following is the most likely melanoma subtype? A) Acral lentiginous B) Desmoplastic C) Lentigo maligna D) Nodular E) Superficial spreading

The correct response is Option A. While cyanoacrylate glue closures such as Dermabond and Indermil offer the advantages of speed, ease-of-use, and comfort in the closure process, some studies show that the outcomes are unpredictable, especially for longer lacerations. One pediatric groin hernia incision closure showed a 24% dehiscence rate, while a porcine study of 10-cm lacerations showed a 15% dehiscence rate. Glue closures do have a role in smaller, tension-free lacerations, particularly in children or others who may not easily tolerate traditional closure. This simplicity of closure does come with the cost of a higher dehiscence rate, so glue closures may be inappropriate for longer, more complex wounds. When used as an adjunct to a comprehensive subdermal interrupted closure, it appears that the dehiscence rate normalizes. Data are less convincing on long-term scar results, but it does not appear likely that glue closures improve or worsen scarring to any appreciable extent for wounds that achieve primary healing without disruption.

2017 A patient presents with an 8-cm linear laceration to the forearm from a bicycle accident. Compared with typical suturing techniques, which of the following outcomes is more likely with a cyanoacrylate glue-only closure? A) Dehiscence B) Hyperpigmentation C) Infection D) Keloid scarring E) Pain

The correct response is Option E. The repair of peripheral nerve injuries can be affected by several factors. Younger patients tend to have improved outcomes compared with older patients. Although there is no consensus on the optimal timing for nerve repair, earlier repairs have been shown to have better outcomes than those attempted at later time points. The level at which the injury has occurred can also affect the outcome. The more proximal the injury, the worse the prognosis in terms of motor and sensory return. Moreover, more complete and rapid regain of function occurs in more proximally innervated muscles. Finally, technical aspects of the nerve repair can also affect outcomes. Minimal tension and an increasing number of suture strands crossing the repair site are both associated with improved function.

2017 An 8-year-old boy is brought to the emergency department after sustaining injury to the right upper extremity, 3-cm proximal to the antecubital fossa. Which of the following factors is associated with improved functional outcomes following peripheral nerve repair? A) Fewer suture strands used in the nerve repair B) Higher-tension nerve repair C) Increasing time between nerve injury and repair D) More proximal nerve injury E) Younger patient age

The correct response is Option D. The third, fourth, and fifth intercostal nerves are responsible for innervation of the majority of the breast. The anterior branch of the fourth intercostal nerve provides most erogenous sensation to the nipple. Sensation to the lower abdomen arises from segmental cutaneous branches of the intercostal nerve, which travel through the rectus abdominis muscle. T10 provides sensation to the dermatome, including the periumbilical region, and is most commonly used. The iliohypogastric nerve provides sensation to the lateral gluteal region. The ilioinguinal nerve provides sensation to the upper medial thigh. The genitofemoral nerve provides sensation to the upper anterior thigh and mons pubis. The lateral femoral cutaneous nerve provides innervation to the lateral thigh and is not used for this purpose.

2017 Following a skin-sparing mastectomy, a 39-year-old woman undergoes deep inferior epigastric perforator (DIEP) flap breast reconstruction. To augment flap sensation, the anterior sensory branch of the fourth intercostal nerve is coapted to which of the following nerves within the DIEP flap? A) Genitofemoral B) Iliohypogastric C) Ilioinguinal D) Intercostal E) Lateral femoral cutaneous

The correct response is Option D. The major indication to repair a myelomeningocele defect in the early postnatal period is to prevent infection and bacterial meningitis. Great care is taken to keep the sac sterile and hydrated. Further cardiac, orthopedic, and urologic workup is often necessary, as well as evaluation for hydrocephalus. Although improvement in the return of motor function with early repair has not been shown, neurogenic bladder prognosis is improved. Leakage of cerebrospinal fluid is commonly observed regardless of timing of closure. Although folic acid supplementation has been shown to decrease the neural tube defects and myelomeningocele formation, postnatal supplementation has not been shown to be effective. Approaches to surgical therapy for the treatment of these defects have seen a shift toward prenatal, fetal reconstructive surgery. A recent study by Adzick et al. in the New England Journal of Medicine examined the use of prenatal surgery versus postnatal surgery. In this randomized trial, the authors found a decreased need for cerebrospinal fluid shunt placement and improved motor function outcomes.

2017 The plastic surgeon is called to the neonatal unit to evaluate a newborn with a myelomeningocele. There is an intact 4 × 4-cm sac at the lumbosacral area with minimal fluid oozing from the area. Early operative repair is indicated for which of the following reasons? A) To decrease the need for cerebrospinal fluid shunt placement B) To decrease the need for folic acid supplementation C) To improve motor function return D) To prevent bacterial meningitis E) To prevent cerebrospinal fluid leakage

The correct response is Option E. Serum imbibition describes the earliest stage of skin graft healing. Immediately after placement onto the wound, the graft becomes edematous and may increase its mass up to 30 to 40%. Plasma leaks from recipient bed capillaries and venules and fills the space between the wound bed and the graft. Fibrinogen within the plasma settles out and forms a fibrin bond, which helps anchor the graft to the wound. The graft passively absorbs nutrients from the underlying serum by diffusion during the first 48 hours. Metabolism within the graft becomes anaerobic and the pH level falls to 6.8. The metabolic demands of the graft also fall, with ATP levels falling 70% and glucose levels falling 80%. Revascularization and inosculation describe the second stage of skin graft healing. These processes began shortly after graft placement, but it takes approximately 4 to 5 days for the graft to become vascularized, with maximal flow developing by day 29. Revascularization refers to direct ingrowth of new blood vessels into the graft from the underlying wound bed. Inosculation describes a process by which blood vessels from the underlying wound bed connect with existing vessels in the skin graft. More recent evidence suggests that both of these processes play a role in the development of vascularization within the skin graft: existing vasculature within the skin graft undergoes some level of degeneration. However, the acellular basal lamina persists and provides a conduit for the ingrowth of a new vascular tree from the host wound bed. Primary contraction of a skin graft occurs immediately after harvest and is due to the recoil of elastic fibers within the dermis. Grafts with a larger amount of included dermis (e.g., full-thickness grafts) have greater primary contraction. Secondary contraction refers to contraction after the wound heals. This process is mediated by myofibroblasts and occurs more frequently in grafts with a thinner dermal component (e.g., split-thickness grafts). A larger dermal component appears to suppress proliferation of myofibroblasts within the wound. Encapsulation refers to the development of a fibrous scar capsule around a foreign device such as a breast implant.

2017 Which of the following best describes the immediate mechanism of skin graft survival following placement onto the recipient wound bed? A) Dermal contraction B) Encapsulation C) Inosculation D) Revascularization E) Serum imbibition

The correct response is Option B. In this circumstance, the surgeon should wash the graft with sterile triple antibiotic solution (0.1% gentamicin, 0.1% clindamycin, 0.05% polymyxin) and consider reusing it. This graft is not easily re-harvestable, and therefore reusing it is a reasonable alternative. Triple antibiotic solution is readily available and nontoxic to the bone graft, unlike iodine and chlorhexidine. Studies have shown that iodine does not have the antimicrobial effectiveness of other antimicrobials. Studies also show that while chlorhexidine is most effective in eradicating the microorganisms from the graft, its detrimental effect on corticocancellous bone prevents its use in this clinical scenario. If the dropped graft were skin, cartilage, or composite tissue, chlorhexidine would be the ideal antimicrobial. Discarding the graft is not the ideal alternative given the extremely low likelihood of infection if reused after appropriate decontamination. In this clinical scenario, alloplastic materials are contraindicated due to the patient's age. One should change the wound classification from 1 to 3, not 4. Steam sterilization is not used for such grafts in such an acute setting and this technique would potentially harm the graft.

2017 While performing a cranioplasty on a 6-year-old girl, the plastic surgeon accidentally drops the bone graft on the floor. Which of the following is the most appropriate next step? A) Change the wound classification from 1 to 4 B) Decontaminate with triple antibiotic solution lavage and reuse the graft C) Discard the graft and use an alloplastic material D) Steam sterilize and reuse the graft E) Wash with chlorhexidine and reuse the graft

The correct response is Option B. Layers of the scalp can be easily remembered using the mnemonic SCALP: S (skin), C (subcutaneous), A (galea aponeurotica), L (loose areolar), and P (pericranium). The skin is quite thick and densely adherent to the subcutaneous layer, which contains the vessels and nerves that travel just above the galea. The galea aponeurotica is continuous with extensions of the superficial muscular aponeurotic system: the frontalis anteriorly, the occipitalis posteriorly, and the temporoparietal fascia (superficial temporal fascia) laterally. The loose areolar plane is quite mobile and dissects very easily. As a result, most avulsions occur within this plane. The pericranium is densely attached to the skull and is contiguous laterally with the deep temporal fascia. The temporalis muscle lies below this layer and attaches directly to the temporal bone. Tissue expanders should be placed under the galea/superficial muscular aponeurotic system layer to provide maximal tissue coverage and to protect and optimize the blood supply to the expanded skin. Placement in the subcutaneous plane would require a difficult, tedious dissection and would result in a thin, poorly perfused flap. Conversely, placement of the expander under the pericranium is unnecessary and would result in a painful, difficult, and unpredictable expansion. There is no clinical indication to place a tissue expander under the temporalis muscle.

2018 A 15-year-old girl is evaluated 2 months after a failed scalp replantation because of an avulsion injury. Skin grafting is planned, followed by staged tissue expansion. Photographs are shown. Between which of the following layers of the scalp is the most appropriate anatomic plane for tissue expander placement? A) Skin and galea B) Galea and temporalis muscle C) Temporalis muscle and pericranium D) Pericranium and cranium

The correct response is Option B. Romberg disease, also known as progressive hemifacial atrophy (PHA), is an acquired disease causing severe deformity of the facial contour. The therapeutic goals of surgery for patients with PHA are to minimize psychosocial effects and to correct the appearance and function of involved facial structures. PHA usually presents in the first 20 years of life, after which it is slowly progressive but self-limited. The atrophy affects subcutaneous tissue, fat, muscle, and osteocartilaginous structures, creating a sunken hemiface appearance. Epidermal cutaneous involvement is minimal, but the tongue, gingiva, teeth, and palate may also be involved. For cases with minimal-to-moderate soft-tissue defects, autologous fat grafting can provide correction of the contour deformities. For more severe soft-tissue deficits, additional or other modalities such as adipofascial free flaps can be used. In more severe cases with skeletal and/or dental abnormalities, interventions such as distraction osteogenesis and bone grafting may be necessary. The injection of free silicone is not recommended and has been replaced by the use of autologous fat and other injectable substances. Hyaluronic acid may provide a temporary solution, but it is costly.

2018 A 20-year-old woman is evaluated for facial asymmetry due to loss of volume on one side of the face. She has no history of trauma or infection to that side of her face. Physical examination shows normal dental occlusion with normal bony anatomy and no cranial nerve deficits. She has a mild-to-moderate deficiency of soft tissue along the cheek between the zygomatic and mandibular areas of the face. Which of the following is the most appropriate intervention for correction of this patient's hemifacial atrophy? A) Adipofascial free flap B) Autologous fat grafting C) Distraction osteogenesis D) Hyaluronic acid filler E) Silicone injection

The correct response is Option D. Silicone gel has demonstrated efficacy in improving hypertrophic scars in a number of studies and is supported by level I evidence. Vitamin E, fat injection, allium cepa extract and microneedling are supported by lesser quality studies in a recent comprehensive review of the literature.

2018 A 23-year-old woman comes to the office because of a hypertrophic scar after undergoing abdominoplasty 3 months ago. A multimodal approach to improving the appearance of the scar is planned. Which of the following therapies is supported by the highest quality evidence in this patient? A) Allium cepa extract B) Fat injection C) Microneedling D) Silicone gel E) Vitamin E

The correct response is Option B. Capsular contracture is more common in smooth round silicone implants than in textured implants. It is believed that the texturing of the implant is protective against significant capsule formation. On the other hand, there are several increased risks associated with textured anatomic implants. These include increased risks of late seroma and breast implant-associated anaplastic large cell lymphoma (BIA-ALCL), although this is very rare. Double capsule is a complication more recently noted with the introduction of textured anatomic implants. Malrotation can only be seen in an anatomic textured implant, because smooth round implants are symmetric in shape. In addition, it can be difficult to differentiate between anatomic shaped and smooth round implants, with several studies showing their similar cosmetic outcomes.

2018 A 24-year-old woman comes to the office to discuss augmentation mammaplasty. She is interested in subglandular implant placement and would like to discuss the risks of augmentation. Which of the following risks is more likely with smooth round silicone implants compared with textured anatomic silicone implants? A) Anaplastic large cell lymphoma B) Capsular contracture C) Double capsule D) Late seroma E) Malrotation

The correct response is Option E. Polymethylmethacrylate (PMMA) fillers are indicated for treatment of moderate-to-severe, atrophic, distensible facial scars on the cheeks in patients over the age of 21. Polymethylmethacrylate microspheres (30-50 μm) are suspended in a water-based gel containing 3.5% bovine collagen gel and 0.3% lidocaine. The collagen carrier is resorbed over 1 to 3 months post injection, first replaced by granulation tissue and then later by fibrous tissue. The PMMA microspheres act as a stimulus for tissue regeneration and deposition of autologous connective tissue. Because of the permanent nature of this filler material, care must be taken to place the filler in the appropriate plane (very deep dermis or subcutis). Also, the presence of bovine collagen requires skin testing one month prior to treatment to rule out hypersensitivity. Prior first- and second-generation PMMA fillers were composed of smaller microspheres ( Hyaluronic acid fillers are an off-label option for treatment of scars, but in this case, the patient requested a permanent filler. Depending on the formulation, hyaluronic acid fillers resorb over 8 to 24 months. Intense pulsed-light therapy may help with skin blemishes, improve redness, and smooth fine wrinkles, but it is not effective for the treatment of acne scars. Poly-L-lactic acid filler is very useful for providing volume in the cheeks but is considered "semi-permanent." It is recommended for placement in a deep plane to avoid skin irregularities, so correction of scars may be difficult. Additionally, use for scars would also be considered off-label. Glycolic acid peels are useful for fine rhytids, but because their effect is confined to the epidermis, these peels will not improve the appearance of deep scars.

2018 A 25-year-old woman seeks permanent, nonsurgical treatment of acne scars. Which of the following is the most appropriate treatment option? A) Glycolic acid peel B) Hyaluronic acid filler C) Intense pulsed-light therapy D) Poly-L-lactic acid filler E) Polymethylmethacrylate filler

The correct response is Option D. For bridging long nerve gaps not amenable to primary repair (greater than 3 cm), peripheral nerve autografts are the most reliable choice. Their use is limited by supply and donor site morbidity from additional incisions, loss of sensation, and possible neuromas. Common donor sites include sural nerve, medial antebrachial cutaneous nerve, lateral antebrachial cutaneous nerve, dorsal cutaneous branch of the ulnar nerve, superficial peroneal nerve, and posterior and lateral cutaneous nerves of the thigh. Nerve conduits, such as silicone tubes, synthetic biodegradable tubes (such as polyglycolic acid conduit or collagen conduit), and biologic tubes (such as autologous vein grafts) are limited to use in short gaps (less than 3 cm). Beyond 3 cm, there is no clinically meaningful regeneration. Acellular autografts have recently been used with good success in short nerve gaps (less than 3 cm). The advantage of these products is that they provide the extracellular matrix molecules, which may enhance nerve regeneration.

2018 A 30-year-old man is brought to the emergency department after a rollover motor vehicle collision. Physical examination shows significant soft-tissue loss and a median nerve injury. At the time of surgical repair, there is a 5-cm gap in the nerve. Which of the following treatment options is most likely to provide the best long-term functional outcome for this patient? A) Acellular autograft B) Autologous vein graft C) Collagen conduit D) Peripheral nerve autograft E) Polyglycolic acid conduit

The correct response is Option B. This patient has a desmoid tumor, a rare, locally infiltrative mesenchymal neoplasm that is most commonly found in adolescents and young adults. It does not metastasize but can have a very unpredictable and aggressive natural history. Treatment can range from close observation (as some tumors will self-involute) to radical excision with wide margins. Traditionally, surgical excision can be difficult because of the high risk of local recurrence. Desmoid tumors are not associated with chemical or radiation exposures, or lipomas. Metastatic disease is not a characteristic of desmoid tumors. Although treatment modalities for desmoids are evolving, they are not treated with chemotherapy/radiation therapy as a primary modality. Primary treatment is frequently observation for small or slowly changing tumors. For rapidly growing tumors, radical excision is frequently performed. There does not appear to be a genetically inherited component for desmoid tumors, although they are associated with random mutations of the β-catenin (CTNNB1) and adenomatous polyposis coli (APC) genes.

2018 A 30-year-old woman is evaluated for a mass on the anterior abdominal wall that has been growing over the past several months. Imaging and examination of the specimen obtained on biopsy confirm a diagnosis of desmoid-type fibromatosis arising from the anterior abdominal musculature. Which of the following is the most accurate statement regarding this condition? A) It is associated with previous radiation exposure B) It is locally aggressive with remote risk of metastatic disease C) It usually arises from an untreated lipoma D) Primary treatment is combination chemotherapy/radiation E) There is an autosomal recessive genetic inheritance pattern

The correct response is Option C. Mafenide acetate effectively penetrates burn eschar as well as cartilage and decreases the risk of suppurative chondritis in the setting of burns of the auricle. Twice-daily application is recommended. Care must be taken to monitor for metabolic acidosis, as mafenide acetate is metabolized to sulfamoylbenzoic acid, a carbonic anhydrase inhibitor. Silver sulfadiazine, nanocrystalline silver dressings, honey, and acetic acid have not demonstrated similar efficacy in preventing burn-associated chondritis.

2018 A 32-year-old man is brought to the emergency department with a full-thickness thermal burn injury to the left ear. Which of the following therapeutic agents allows for the most effective preservation of involved cartilage? A) Acetic acid B) Honey C) Mafenide acetate D) Nanocrystalline silver dressings E) Silver sulfadiazine

The correct response is Option A. The preoperative management of pressure sores is vital to successful surgical outcomes and healing. When a patient presents, one of the first assessments must be whether that patient has evidence of osteomyelitis. For patients presenting with chronic exposed wounds at Stage III or Stage IV, the risk of pelvic osteomyelitis is very high. This will require intravenous antibiotic treatment and bony debridement prior to coverage. To establish this diagnosis, especially in a patient with an increased ESR, a bone biopsy is needed. This will help guide initiation and maintenance of treatment. While a positive bone scan may indicate the presence of osteomyelitis, this test carries a high rate of false positives in patients with open wounds. Thus, the next line of workup for osteomyelitis would be a bone biopsy, with the tissue being sent for cultures and sensitivities. This should be done in such a way that some of the deeper pelvic bone is sampled, and not just the superficially exposed bone. This will allow for both diagnosis and a guide for directed antibiotic therapy. In cases where bone biopsy is not available, a gadolinium-enhanced MRI or tagged white blood cell scan can be used to assess for the characteristic inflammation associated with osteomyelitis, and make a diagnosis indicating the need for antibiotic treatment. A CT scan with IV contrast is a last-line test used for this purpose, as it is less sensitive at showing localized inflammation, and is more helpful in assessing depth of invasion and defining exposed structures. Thus, in this case, given the chronic nature of this open wound, and the fact that it is ready for closure based on clinical examination, a bone biopsy would be most helpful as the first step in ensuring there is no osteomyelitis prior to closure.

2018 A 34-year-old T12 paraplegic man presents with a Stage IV sacral pressure sore. The ulcer has been persistent and chronic, with exposed structures down to bone. Serial bedside debridements have maintained a clean and healthy-appearing wound. On admission, the patient has an increased erythrocyte sedimentation rate (ESR). Prior to definitive coverage, which of the following is the most appropriate next step in management? A) Bone biopsy and culture B) CT scan with IV contrast C) Gadolinium-enhanced MRI D) Tagged white blood cell scan E) Triple phase bone scan

The correct response is Option E. The principles of burn wound skin grafting dictate that first, a clean wound must be obtained. This is achieved through operative debridement and washout, to remove all necrotic eschar, and eliminate any possible source of bacteria or infection. In most cases of second-degree burns, a complete fascial excision is not necessary, as this proceeds much deeper than the affected tissue. Instead, a tangential excision is used to remove burn eschar, in layers, until the necrotic tissue has been excised, and viable tissue remains at the base of the wound. Following this, a split-thickness skin graft allows for greatest potential of graft survival. The use of a full-thickness graft will result in lower graft survival rates. Meshing the skin graft prior to placement will improve survival, as it prevents accumulation of fluid or blood under the skin graft. Any collection of seroma or hematoma under the graft will prevent successful outcomes. Placement of either a bolster dressing or a negative pressure (vacuum-assisted closure) dressing on the skin graft, will prevent mechanical shear forces, and improve graft survival rates. Thus, for the first 5 days after graft placement, any early motion or lack of dressing will result in greater risk of shear forces, and lower graft survival rates.

2018 A 34-year-old man sustains deep second-degree burns to the forearm. He is brought to a local burn center. A skin graft is chosen as the coverage method. Which of the following management techniques offers the highest likelihood of skin graft survival in this patient? A) Complete fascial excision B) Early range of motion C) Keeping the graft uncovered postoperatively D) Placing a full-thickness graft E) Placing a meshed graft

The correct response is Option C. The blood supply to the penis arises from the internal pudendal arteries, which continue into the penis as the bulbourethral artery (supplying the bulb of the penis and penile urethra), the deep corporal/cavernosal artery (responsible for erection and supply to the penile shaft), and the dorsal penile artery (supplying the penile skin). A meta-analysis has shown that repair of the dorsal penile artery predicted the best combined sexual function, urinary function, and sensation outcomes. Repair of the deep orporal/cavernosal artery did not correlate with improved outcomes. The helicine artery is one of the terminal branches of the dorsal penile artery, and is distal to the site of amputation. The internal pudendal artery supplies the penis, but more proximal than the site of amputation.

2018 A 34-year-old man with schizophrenia is examined 1 hour after amputating his penis at the base of the shaft with a cleaver. The penis has been retrieved and a decision is made to attempt microvascular replantation. Repair of which of the following arteries is paramount to a successful outcome? A) Bulbourethral B) Deep cavernosal C) Dorsal penile D) Helicine E) Internal pudendal

The correct response is Option E. The anterolateral thigh flap is a versatile coverage tool because of its wide skin island (up to 8 x 25 cm) and long, accessible pedicle (up to 7 cm). The blood supply originates from the lateral femoral circumflex artery descending branch, and sends perforating branches through the vastus lateralis and rectus femoris muscles, and occasionally through the intermuscular septum. The inferior gluteal muscle, while a common muscle flap, is further posterior and proximal. The tensor fascia lata is more lateral to the anterolateral thigh flap zone, though it also has a blood supply from the lateral femoral circumflex system, as the vessel terminates in the tensor fascia lata. The sartorius is more medial and proximal and has a segmental circulation based on the femoral artery branches. The gracilis is more medially based, and is supplied by the medial femoral circumflex.

2018 A 35-year-old man is brought to the emergency department with a 15-cm open wound on the left hip and thigh after he was involved in an accident while using machinery at a construction site. Much of the skin of the lateral thigh is injured, and exposed bone is noted over the trochanter of the hip. Wound coverage using an anterolateral thigh flap from the right side is planned. When the flap is harvested, which of the following muscles must be identified in order to preserve perforators to the flap? A) Gracilis B) Inferior gluteal C) Sartorius D) Tensor fascia lata E) Vastus lateralis

The correct response is Option C. The traditional standard Z-plasty consists of at least three incisions of equal length (two limbs and one central incision) and two angles of equal degree. Ideally, the central incision runs parallel to the long axis of the scar, or the scar itself may be completely excised with the fusiform defect acting as the central incision. The resultant subcutaneous triangular skin flaps are transposed with each other such that the new, central incision lies perpendicular to the original central incision. After closure, the scar is reoriented along the limb incisions, and the new central incision lies within relaxed skin tension lines. The length of the original scar also increases after a Zplasty, which is a useful characteristic when a surgeon desires release of a scar contracture, as in this specific example. In general, as the central incision lengthens (given a constant angle), so does the resultant scar. Additionally, as the angles between the limbs increase (given a constant limb length), so does the resultant scar.

2018 A 40-year-old man sustains burns to 35% of his total body surface area, including the neck, chest, axillae, and upper extremities. After subsequent skin grafting, a right anterior axillary dome scar contracture develops. The patient is scheduled to undergo revision of the scar using Z-plasty. Which of the following lateral limb angles will result in a theoretical 75% gain in central limb length? A) 30 degree B) 45 degree C) 60 degree D) 75 degree E) 90 degree

The correct response is Option E. In this clinical scenario, the patient has a one-vessel runoff in the lower extremity. Preservation of the distal blood supply is critical and that is the main advantage of an end-to-side anastomosis. In general, an end-to-side anastomosis is more technically difficult with longer ischemia time. Kinking of the vessels is still possible with end-to-side anastomosis and therefore is not a major advantage to this technique. There is no added benefit in visualizing the vessel and it is generally more difficult to see the entire vessel compared to an end-to-end anastomosis. This technique does not preferentially allow for an anastomosis in the zone of injury. Finally, it is controversial whether or not the flap survival rate is different between end-to-side and end-to-end anastomosis. An early paper by Godina shows an advantage for end-to-side; however, many subsequent papers have contradicted these results.

2018 A 42-year-old woman presents with a Gustilo Type IIIB open tibial fracture with a large area of soft-tissue loss. Rectus abdominis free flap reconstruction is planned. CT angiography shows a patent posterior tibial artery; however, the peroneal and anterior tibial vessels are not suitable for use because they do not traverse past the level of the fracture. Compared with end-to-end anastomosis, which of the following is the main advantage to using end-to-side arterial anastomosis in this scenario? A) Allows anastomosis in zone of injury B) Decreases ischemia time C) Facilitates visualization D) Minimizes kinking of vessels E) Preserves distal blood supply

The correct response is Option D. Hair follicles contain multipotent stem cells that are activated upon the start of a new hair cycle and upon wounding to provide cells for hair follicle and epidermal regeneration. In the hair follicle, stem cells reside in the bulge area. Bulge cells are relatively quiescent compared with other cells within the follicle but can be recruited during wound healing to support reepithelialization. Sebaceous, apocrine, and eccrine glands secrete fluids that are involved in lubricating, coating, or cooling the skin. Arrector pili are responsible for motility of cutaneous hair in response to tactile stimulation or low temperatures.

2018 A 43-year-old man undergoes wound closure with a split-thickness skin graft harvested from the left thigh. Which of the following skin appendages are the primary source of multipotent stem cells responsible for reepithelialization of the donor site? A) Apocrine glands B) Arrector pili C) Eccrine glands D) Hair follicles E) Sebaceous glands

The correct response is Option C. In the patient described, the most effective way to fixate autografts is negative pressure wound therapy (NPWT). Several studies in burn patients have demonstrated the superiority of NPWT over the other conventional dressings listed. Whether staples or sutures are used has little effect on graft survival, although sutures may offer more precise graft placement. Most dressings for graft fixation use petrolatum gauze of some form, followed by some type of dressing that offers compression to fixate grafts. Dry gauze and a compressive wrap is the simplest option. Cotton balls with a tie-over bolster dressing are also effective, especially for small grafts. Larger surface areas are more difficult to secure with this method. A simple occlusive dressing, while acceptable for a donor site, will not effectively fixate autografts. NPWT offers good graft fixation, exudate removal, and promotion of local perfusion, which may explain the improved graft survival observed.

2018 A 43-year-old woman comes to the emergency department 2 hours after sustaining deep second-degree burns to the right elbow from a campfire. The wound measures 8 × 20 cm. After surgical excision, placement of autografts, and coverage with petrolatum gauze, which of the following dressings is most likely to promote graft survival? A) Cotton balls and tie-over bolster dressing B) Dry gauze and compressive wrap C) Negative pressure wound therapy D) Occlusive dressing E) No additional dressing

The correct response is Option B. Autologous fat grafting is a widely accepted technique in breast reconstruction. A large systematic review recently confirmed the oncologic safety of this technique but did report a significant incidence of benign lesions including cysts and calcifications. Fat grafting is not associated with an increased risk of recurrent breast cancer, infection or hypopigmentation.

2018 A 43-year-old woman undergoes the second stage of tissue expander-based breast reconstruction. Exchange of the tissue expander for a smooth round silicone implant is planned along with a superior capsulotomy and fat grafting to the upper pole for contour improvement. Which of the following is an increased risk associated with fat grafting to the breast in this patient? A) Anaplastic large cell lymphoma B) Benign lesions C) Hypopigmentation D) Infection E) Recurrent breast cancer

The correct response is Option D. Several comparative studies have reported a higher incidence of anastomotic thrombosis and failure with the superficial inferior epigastric artery (SIEA) flap than with flaps based on the deep inferior epigastric artery (DIEA). These failure rates range from 7.35 to 14%. Most of these failures were arterial in nature. Since SIEA flaps do not require an incision into the anterior rectus sheath or rectus muscle, bulges do not occur. Reported fat necrosis rates are similar between SIEA and DIEA flaps. There is no evidence for a difference in donor site dehiscence or umbilical necrosis rates.

2018 A 48-year-old woman is evaluated for bilateral microsurgical breast reconstruction. Compared with the deep inferior epigastric perforator (DIEP) flap, the superficial inferior epigastric artery (SIEA) flap places the patient at a greater risk for which of the following complications? A) Abdominal bulge B) Donor site dehiscence C) Fat necrosis D) Flap failure E) Umbilical necrosis

The correct response is Option C. Preservation of the superficial inferior epigastric veins (SIEV) during flap harvest is a useful preventive measure in microsurgical free tissue transfer operations. These veins can serve as important lifeboats to augment venous outflow in the setting of venous congestion. Typically, if a free flap demonstrates venous congestion, the inset should be taken down and the pedicle, recipient vessels, and anastomoses should be interrogated. Simple issues, such as mechanical compression or twisting of the vein, should be ruled out. Next, the SIEV should be inspected. If engorged, the flap is likely reliant on superficial outflow, and this vein should be connected to a recipient vessel to augment the venous outflow of the flap. Options for recipient veins include an anterograde branch on the pedicle vena comitans, or in a retrograde fashion to the vena comitans that was not used in the initial set of anastomoses. In this case scenario, the flap continued to demonstrate venous congestion intraoperatively. This makes it unlikely that tension or pressure from the inset of the flap was causing the venous outflow obstruction. Furthermore, leech therapy is not indicated for a free flap with global venous congestion. The venous coupling device is safe and effective for the anastomosis of veins in DIEP flap surgery. It has not been associated with patency rates that are different from hand-sewn anastomoses. The coupling device, however, has been shown to reduce the microsurgery time. The use of one or two veins in microsurgical free tissue transfer is a topic that has been debated for several years. While some studies indicate that the use of two venous connections may reduce the velocity of blood flow across the anastomosis, there is not sufficient data to support differences in flap outcomes or thrombotic events. Therefore, the routine use of a second vein is largely up to surgeon preference.

2018 A 48-year-old woman undergoes immediate unilateral breast reconstruction with a free deep inferior epigastric artery perforator (DIEP) flap. At the conclusion of the procedure, the flap skin paddle is noted to have venous congestion. Upon reexploration, the venous anastomosis appears patent with venous outflow detected by handheld pencil Doppler evaluation, but the flap continues to have venous congestion with brisk capillary refill. Which of the following is the most appropriate next step in management? A) Apply leeches postoperatively B) Loosely re-inset the flap and monitor closely C) Perform a second venous anastomosis using the superficial inferior epigastric vein D) Perform a second venous anastomosis using the vena comitans E) Revise the venous anastomosis using a hand-sewn technique

The correct response is Option C. The most appropriate method to reconstruct the wound is a local fascial flap and skin advancement flap. The major principle of tethered cord and myelomeningocele repair is to obtain a well-vascularized layer of soft-tissue coverage between the dural and skin closures. The fascia overlying the paraspinous muscles can be turned over as flaps to cover the underlying dural repair. This vascularized soft-tissue layer will minimize the risk of cerebrospinal fluid leak by reinforcing the dural repair. In addition, the fascial flaps will prevent contact with cutaneous bacteria and subsequent meningitis if either the dural repair or skin repair breaks down. A split-thickness skin graft over the dura would not adequately protect the spinal cord. Closing the skin directly over the dural repair using skin advancement flaps would place the child at risk for meningitis in the event of a cerebrospinal fluid leak or if wound breakdown occurred along the incision line of the widely undermined skin flaps. The use of a regional gluteal or latissimus muscle flap to cover the dural repair is unnecessary because local tissue (paraspinous muscle fascia) is available. Harvesting the gluteal or latissimus muscles also may cause significant donor site morbidity in a child already at risk for ambulatory problems from a neurological deficit.

2018 A 5-year-old boy is evaluated for a cerebrospinal fluid leak and a 3 × 3-cm area of wound dehiscence involving the posterior trunk following tethered cord repair. Which of the following is the most appropriate method for reconstructing the wound? A) Gluteal muscle flap and skin advancement flap B) Latissimus muscle flap and skin advancement flap C) Local fascial flap and skin advancement flap D) Skin advancement flap only E) Split-thickness skin graft

The correct response is Option E. In patients who sustain digital amputation as a result of an avulsion mechanism, there is often an extensive zone of injury that precludes primary vascular anastomosis. Vein grafts permit vascular anastomosis outside of the zone of injury. Bone shortening can sometimes allow excision of the injured vasculature and primary anastomosis. However, in this case, bone shortening is unlikely to make up for a 2-cm vascular gap. Longer vascular gaps can be addressed with vein grafts. Despite the fact that vein grafts involve an additional anastomosis per vessel compared to primary anastomosis, they have been found to exhibit similar rates of thrombosis and replantation survival. Medicinal leech therapy can help address venous congestion, but is typically considered when venous congestion occurs after attempt at surgical replantation, or if no suitable veins can be found for anastomosis. While postoperative warming and systemic heparin are often used adjunctively in patients undergoing replantation, they have not been demonstrated to increase the likelihood of survival of the replanted part, and would most likely not have as significant an effect as restoring perfusion to the amputated part using vein grafts.

2018 A 50-year-old man comes to the emergency department after sustaining amputation of the right long finger involving an avulsion mechanism. The patient is taken to surgery for replantation. During surgery, extensive vascular injury is seen, and an approximately 2-cm vascular gap of the digital arteries and veins results following excision of injured vessels. Which of the following interventions is most likely to increase the probability of functional digit replantation? A) Bone shortening B) Medicinal leech therapy C) Postoperative warming D) Systemic heparin E) Vein grafts

The correct response is Option E. Cylindromas are benign adnexal tumors showing an eccrine and an apocrine differentiation. They are found most commonly on the scalp and face, and are more common in women. Solitary cylindromas are generally sporadic in nature. Multiple cylindromas can be seen in patients with Brooke-Spiegler syndrome as an autosomal dominant trait with variable penetrance. Cylindromas may undergo malignant transformation, and therefore surgical excision is typically recommended, with close postoperative follow-up given high recurrence rates. Cryotherapy, electrodessication and curettage, and imiquimod are not treatments for cylindromas. Radiotherapy has been used to treat malignant cylindromas (also known as cylindrocarcinoma or adenoid cystic carcinoma), but not benign cylindromas.

2018 A 50-year-old woman is evaluated for multiple firm, nodular, pink-colored lesions of the scalp, ranging in size from 2 to 4 mm. Examination of a specimen obtained on biopsy shows benign cylindroma. Which of the following is the most appropriate management of these lesions? A) Cryotherapy B) Electrodessication and curettage C) Imiquimod therapy D) Radiotherapy E) Surgical excision

The correct response is Option B. The need for multiple reexplorations has been found to be a predictor for unsuccessful free flap salvage. Free flap reconstruction of the breast has been associated with higher flap survival rates than other anatomic areas, such as the head and neck and extremities. Higher free flap survival rates have been observed when vascular compromise occurs earlier in the postoperative period (postoperative days 0 to 2) compared with later. Anticoagulants, such as heparin, are sometimes used during free flap salvage attempts. However, their use has not been found to impact flap survivability. The same has been found with regard to thrombolytic agents, such as tissue plasminogen activator.

2018 A 50-year-old woman undergoes reconstruction of a postburn neck contracture with a free anterolateral thigh flap. On postoperative day 2, the flap is explored for vascular compromise and is successfully salvaged. On postoperative day 3, the flap suffers vascular compromise and is explored again. During the operation, the anastomoses are revised using vein grafts. Which of the following factors is most strongly associated with unsuccessful flap salvage in this patient? A) Anatomic site B) Multiple reexplorations C) No use of anticoagulants D) No use of thrombolytic agents E) Postoperative day of initial reexploration

The correct response is Option B. There is evidence that the longer fat is exposed at room temperature, the lower the adipocyte viability. There may be complete loss of stem cell viability by 4 hours at room temperature and 24 hours at 4°C (39°F). There is no high-level evidence suggesting that centrifuging or rinsing fat increases viability. There is also no evidence for enhanced fat survival based on donor site, such as the abdomen, thigh, or arm. Use of local anesthesia does not appear to hinder graft survival. It does appear that less mechanical trauma with low-shear harvesting instruments is helpful. Ultrasonic liposuction is designed to rupture fat cells and would likely hinder graft survival. Fat grafting is an increasingly common tool, although results appear to be operator-dependent based on wide ranges of success in published reports. There is no absolute agreed-upon method of measuring fat survival. Most studies use volumetric analysis with imaging. More future long-term studies are needed. At this time it appears that the commonality of successful results is delicate handling of the fat.

2018 A 52-year-old woman undergoes fat grafting of the upper right breast area to correct a contour indentation after implant reconstruction. Which of the following factors will most likely increase the success of fat grafting? A) Abdominal donor site B) Grafting soon after harvest C) Rinsing the fat with Ringer's lactate D) Ultrasonic liposuction aspiration E) Use of centrifuged fat

The correct response is Option E. After Staphylococcus aureus and S. epidermidis, Pseudomonas aeruginosa is among the the next most common sources of breast infections. It is common in hospitalized or immunocompromised patients, as well as patients with foreign body devices such as catheters or implants. It is a gram negative rod, and common antibiotic treatments include advanced β-lactams (piperacillin, ceftazidime), carbapenems, quinolones, and aminoglycosides. Dual coverage is often recommended in severe infections. In the case of breast implant infections, if there is not rapid improvement on antibiotic therapy, or if significant systemic symptoms develop (vital sign instability, high white blood cell count, fever, renal impairment), then surgical washout and device removal is mandatory. In patients without systemic symptoms, wash out and new implant placement can be an option in carefully selected and counseled patients. Other breast pathogens include Escherichia coli, Propionibacterium, and Corynebacterium. More than 300,000 breast implant procedures are performed each year in the United States. In reconstructive cases, the infection rate averages 6% and the explantation rate 3% (range, 1.5 to 8%). Preventative measures include proper patient selection, preoperative MRSA management when carriers are suspected, routine presurgery chlorhexidine washes, proper antibiotic timing presurgery and continuation of antibiotics in implant reconstruction cases for at least 24 hours (though the optimal treatment duration has not yet been determined). None of the other bacteria listed are common in breast infections, though all are common pathogens. Bacteroides are anaerobic gram-negative rods that are common in gut flora and feces. Enterococcus faecalis is a frequent cause of nosocomial infection, with a high prevalence of multi-drug resistance. It is a gram-positive coccus, and is not commonly seen in breast surgery patients as it primarily colonizes the digestive tract. Mycobaterium marinum is a rare pathogenic cause of hand infections from injuries that occur in aquatic environments. Pasteurella multocida is a frequent cause of animal bite infections, particularly from cats and dogs.

2018 A 54-year-old woman undergoes bilateral immediate tissue expander-based breast reconstruction. BMI is 36 kg/m2. On postoperative day 10, examination shows bilateral breast erythema; empiric antibiotics are started. In addition to Staphylococcus species, which of the following bacteria should be treated as the next most likely pathogen? A) Bacteroides fragilis B) Enterococcus faecalis C) Mycobacterium marinum D) Pasteurella multocida E) Pseudomonas aeruginosa

The correct response is Option B. Administration of tissue plasminogen activator (tPA) during revision of a microanastomosis has a decreased rate of subsequent fat necrosis. The suspected mechanism of action is thrombolysis of distant "shower" emboli in the microvasculature. The administration of tPA as an adjunct to microanastomotic revision has no effect on flap salvage rates. In addition, there is no change in hematoma risk since the dose is low (2 mg) and is usually injected directly into the flap artery, which is maintained locally in the flap. Only if larger doses of tPA were given systemically would there be a risk of operative hematoma.

2018 A 54-year-old woman undergoes breast reconstruction using a deep inferior epigastric artery perforator (DIEP) flap. Arterial thrombosis is noted after performing the microanastomosis. Which of the following is more likely to occur with local administration of tissue plasminogen activator (tPA) as an adjunct to revision microanastomosis as compared with revision microanastomosis alone (without tPA)? A) Decreased flap salvage rate B) Decreased incidence of fat necrosis C) Increased flap salvage rate D) Increased incidence of fat necrosis E) Increased incidence of operative hematoma

The correct response is Option D. Cordeiro et al. updated the largest series of women undergoing two-stage implant breast reconstruction who require postmastectomy radiation. The authors found that radiating the tissue expander, as opposed to the permanent implant, increased the rate of reconstructive failure by 46%. However, aesthetic results were better and capsular contracture was less frequent. There were no differences in patient-reported outcomes. This study confirmed the earlier findings of Nava et al. regarding reconstruction failure (explantation). There are no known differences in cancer recurrence between the two approaches discussed.

2018 A 54-year-old woman with breast cancer undergoes a skin-sparing mastectomy with tissue expander reconstruction. Adjuvant chemotherapy and subsequent radiation therapy have been recommended. Chemotherapy proceeds during tissue expansion. Radiation may be performed either before or after the implant exchange procedure. When compared with radiating the permanent implant, radiating the tissue expander is most likely to increase the risk of which of the following? A) Cancer recurrence B) Capsular contracture C) Device rupture D) Explantation E) Radiation dermatitis

The correct response is Option E. Tretinoin (all-trans-retinoic acid) is one of the best long-term topical therapies available for chronically photoaged skin. Long-term use of tretinoin is associated with improved skin texture, decreased sallowness, a reduction in fine rhytides and actinic keratoses, fading of pigmented macules, and an overall improvement in skin appearance. Histologic effects of tretinoin include the following: increased epidermal and dermal layer thickness, elimination of dysplasia, atypia, and microscopic actinic keratoses, uniform dispersion of melanin granules, increased collagen and glycosaminoglycan deposition in the papillary dermis, diminished dermal elastosis, angiogenesis, and compaction/thinning of the stratum corneum. The mechanism of action of retinoids is regulated through specific nuclear receptors. Ultraviolet radiation activates a series of phosphokinases that stimulate c-Fos and c-Jun proto-oncogenes, and thereby activate activator protein 1 (AP-1) transcription factor. AP-1 causes activation of metalloproteases, such as collagenase, gelatinase, and stromelysin which then break down collagen. Tretinoin results in a 70% inhibition of AP-1 transcription factor binding to DNA and a significant reduction in protease activity. Tretinoin side effects include erythema, photosensitivity, and desquamation. Patients are initially started on a low dose with nightly application until tolerance is achieved. Because tretinoin is a photosensitizer, sunscreen use is absolutely imperative. Topical retinoids should be used for a minimum of 3 to 4 months, with the greatest improvement after 1 year of use. Patients who use alpha-hydroxy acids concomitantly with topical retinoids will see a synergistic effect and this combination is tolerated well in most patients.

2018 A 55-year-old woman comes to the office requesting treatment of fine rhytides due to smoking and photoaging. Topical retinoid therapy is planned. Which of the following effects is most likely to occur with topical retinoid treatment? A) Decreased collagen production B) Decreased hyaluronic acid production C) Thinning of the dermis D) Thinning of the epidermis E) Thinning of the stratum corneum

The correct response is Option C. Because of demonstration of the safety, efficacy, and improvements in the harvest and preparation of fat, autologous fat grafting or liposculpting is gaining popularity to adjust aesthetic breast contour irregularities after all forms of breast reconstruction. The most common complication of fat injection remains the resorption of the grafted fat. Fat graft volume retention has been studied previously. Employing radiologic volumetric data analysis in fat grafting for cosmetic and reconstructive breast surgery demonstrates volume retention between 50 to 80% and this retention may be time and volume dependent. Resorption of grafted fat may be attributable to apoptosis, a reduction in adipocyte volume after transplantation and survival, or a reduction in the fluid content of the grafted mixture.

2018 A 55-year-old woman is scheduled to undergo autologous fat grafting to improve the upper inner quadrant aesthetic contour of the breasts. Six months ago, she underwent bilateral mastectomy and breast reconstruction with deep inferior epigastric perforator flap coverage. The patient does not have or desire breast implants. Which of the following percentage ranges most accurately describes the likelihood of fat graft survival in this patient? A) 10% B) 35% C) 60% D) 85%

The correct response is Option D. Osteogenesis is the formation of new bone by cells in a flap/graft that survive the transfer. This is the primary mechanism by which a vascularized bone graft heals. The pedicle keeps the bone alive so that primary bone healing can occur between the graft and recipient site. Osteoblasts from both locations participate in the formation of new bone at the interface between graft and native bone. Advantages of a vascularized bone graft include the ability to place the graft into a hostile environment such as an irradiated wound bed and immediate structural support with shortened time to bony union compared with nonvascularized cortical grafts. Osteoinduction refers to the direct stimulation of mesenchymal cells at the recipient site by bone morphogenetic protein to differentiate into osteoprogenitor cells. This mechanism of action is associated with the healing of cancellous bone grafts and demineralized bone matrix. Endochondral ossification is the process by which the cartilaginous soft callus covering a fracture is transformed into bone. Osteochondrosis refers to a family of ossification disorders in children. Osteoconduction (creeping substitution) is the primary method by which cortical bone grafts heal. During osteoconduction, cells and blood vessels from the recipient bed grow into the graft. The bone graft becomes a template for the deposition of new bone and the graft resorbs. Neovascularization is complete by 6 to 8 weeks, but ultimate strength of cortical grafts is not seen until 6 to 12 months, at which time the graft is comparable to a vascularized bone graft.

2018 A 55-year-old woman undergoes composite resection of the right mandibular body and floor of the mouth. Closure with a fibular free flap is performed. A photograph is shown. Which of the following mechanisms best describes the healing process associated with vascularized bone transfers? A) Endochondral ossification B) Osteochondrosis C) Osteoconduction D) Osteogenesis E) Osteoinduction

The correct response is Option A. The clinical feature in this particular patient that fulfills the criteria to get Mohs micrographic surgery is that the basal cell carcinoma is arising in a traumatic scar. There are many clear indications for Mohs micrographic surgery for basal cell carcinoma: certain size, histology, and anatomic location, all recurrent basal cell carcinomas, and the occurrence of basal cell carcinoma in irradiated skin, traumatic scars, areas with osteomyelitis/chronic ulceration/inflammation, and/or patients with genetic syndromes. Almost all primary basal cell carcinomas in the H and M zones, regardless of pathology (i.e. superficial, nodular, or aggressive), size, or health status of the patient, are candidates for Mohs micrographic surgery. In the L zone, most basal cell carcinomas are also candidates for Mohs micrographic surgery (except all superficial subtypes [irrespective of health of patient], or those less than 1 cm size in immunocompromised patients or nodular subtypes). Area H: 'Mask areas' of face (central face, eyelids [including inner/outer canthi], eyebrows, nose, lips [cutaneous/mucosal/vermilion], chin, ear and periauricular skin /sulci, temple), genitalia (including perineal and perianal), hands, feet, nail units, ankles, and nipples/areola. Area M: Cheeks, forehead, scalp, neck, jawline, and pretibial surface. Area L: Trunk and extremities (excluding pretibial surface, hands, feet, nail units and ankles). In this clinical case, the patient has a basal cell carcinoma that has a favorable pathology (i.e. superficial subtype) in the L zone, not an indication for Mohs micrographic surgery, irrespective of tumor size. Additionally, with a 1-cm tumor of this pathology subtype, her immunocompromised state is not an indication for Mohs micrographic surgery, either. The radiation was remote from the area she developed her basal cell carcinoma, so it is not an indication, either.

2018 A 56-year-old woman presents with a 1-cm primary superficial basal cell carcinoma on the left volar mid-forearm. Medical history includes renal transplantation, carcinoma of the right breast managed with lumpectomy and radiation, and treatment for a gunshot wound to the left forearm. The basal cell carcinoma is located within the previous traumatic scar. Which of the following clinical features is an indication for Mohs micrographic surgery in this patient? A) Basal cell carcinoma arising in traumatic scar B) History of radiation C) Immunocompromised status of patient D) Size of basal cell carcinoma E) Superficial basal cell carcinoma

The correct response is Option C. For patients who have patent arterial inflow on imaging, spasm is likely to be responsible for their ischemia. Spasm is most common in those with autoimmune disease. Digital sympathectomy involves stripping the adventitia from the radial, ulnar, and digital arteries in an effort to decrease sympathetic input that is the presumed cause of pathologic vasoconstriction. Vascular occlusion with a satisfactory distal target may require an interposition bypass. Occlusion without a distal target for bypass may require venous arterialization. In the absence of evidence of occlusion, there is no indication for thrombolytic therapy.

2018 A 56-year-old woman with a history of systemic sclerosis (scleroderma) is evaluated for intractable pain and progressive ulceration to the right index and middle fingers despite medical management. Duplex ultrasonography shows no identifiable vascular occlusion in the affected digits. Which of the following is the most appropriate surgical management? A) Interposition bypass grafting B) Intra-arterial TPA C) Sympathectomy D) Thrombectomy E) Venous arterialization

The correct response is Option D. A number of risk factors for recurrence have been identified for squamous cell carcinomas. Histologic subtype is one such factor. Adenoid, adenosquamous, and desmoplastic subtypes are considered high risk for recurrence. Anatomic location is another such factor and is typically considered in combination with the size of the lesion. High-risk areas include the "mask" areas of the face (eyelids, eyebrows, periorbital, nose, lips, chin, temple, ear), as well as genitalia, hands, and feet. Lesions greater than 6 mm in these areas indicate a high risk for recurrence. Mid-risk areas include the cheeks, forehead, scalp, and neck, with lesions greater than 10 mm indicating a high risk for recurrence. Low-risk areas include the trunk and extremities, with lesions greater than 20 mm indicating a high risk for recurrence. Based on this, this patient with a 6-mm forehead lesion would not be considered at high risk for recurrence. Rapid growth is a risk factor for recurrence. However, this patient's lesion grew gradually over a period of 2 years and not rapidly. Depth of involvement greater than 2 mm indicates a high risk for recurrence. Other risk factors for recurrence include poorly defined borders, immunosuppression, prior irradiation, site of inflammatory process, neurologic symptoms, moderate/poor differentiation, and perineural/vascular involvement.

2018 A 60-year-old man presents with a 6-mm lesion of the forehead. The patient states that it has enlarged over a period of 2 years. Examination of a biopsy specimen shows squamous cell carcinoma (adenoid subtype) with a 1.5-mm depth of involvement. Which of the following is the most likely risk factor for recurrence of this patient's lesion after surgical excision? A) Anatomic location B) Depth of involvement C) Growth rate D) Histologic subtype E) Size

The correct response is Option C. The effects of ionizing radiation are permanent and may present either acutely or in delayed fashion, even years after the original radiation insult. The mechanism of injury from this radiation is through free radical production which, in turn, directly damages the DNA. In the acute period, the effects of radiation may manifest themselves as erythema and edema of the skin, vasodilation with endothelial edema, and lymphatic obliteration. This eventually leads to capillary thrombosis and subsequent inadequate tissue oxygenation. Over time, nonhealing ulcers can spontaneously develop, sometimes years later. Although recurrent cancer is always a concern in patients with a personal history of cancer, proper, regular, and thorough surveillance can often detect recurrences early, especially in compliant patients. Most recurrences occur within the first 5 years. Abscesses usually present initially with pain, erythema, and localized fluctuance, and often with associated fever and/or malaise. Spontaneous necessitation to the skin would also result in purulent drainage. Lymphedema can be a chronic condition after mastectomy and axillary node dissection, and is usually manifested as generalized edema of the ipsilateral upper extremity. Sinus tract formation is rare. Intertriginous shearing would most often present as superficial epidermal loss with possible superinfection with yeast due to moisture.

2018 A 62-year-old woman is evaluated because of a new 2 × 2-cm open area near her left axillary fold. Medical history is significant for left breast cancer previously treated with bilateral mastectomies, left axillary node dissection, and adjuvant chemoradiation 10 years ago. A photograph is shown. She has been compliant with postoperative oncologic surveillance and has had no recent trauma. Which of the following underlying conditions is most likely responsible for her current presentation? A) Empyema with spontaneous drainage B) Lymphedema drainage tract C) Osteoradionecrosis of the underlying rib(s) D) Recurrent breast cancer E) Skin ulceration from intertriginous shearing forces

The correct response is Option B. Exposed bone does not provide an adequately vascularized bed for skin graft take. Pericranium, in contrast, can support a skin graft. The presence of granulation tissue is a good sign that the wound bed is adequately vascularized for a skin graft to take. Surrounding alopecia decreases aesthetic concerns associated with reconstructive options that do not support hair growth. Defect location does not significantly affect the need for vascularized coverage.

2018 A 63-year-old man has a full-thickness scalp defect following resection of a melanoma. Final pathology has confirmed clear margins. Which of the following criteria is an indication for coverage of the defect with a flap instead of a skin graft? A) Alopecia of the surrounding skin B) Exposed calvarium C) Granulation tissue in the base of the wound D) Intact pericranium E) Posterior location

The correct response is Option E. Papaverine is a phosphodiesterase inhibitor and is a commonly used vasodilating agent in microsurgery. It is administered as a liquid, directly to the adventitia of blood arteries, leading to vasodilation. The proposed mechanism of action of papaverine is by induced increase in cyclic adenosine monophosphate (AMP) levels, causing smooth muscle relaxation in the vessels. It is this mechanism of papaverine that has also led to its use for treatment of cardiac and neurovascular vasospasm. Nifedipine is another common topical vasodilator, which is a calcium channel blocker. The remaining choices are all used to prevent clotting. Glycoprotein IIb/IIIa inhibitors are antiplatelet agents along with aspirin. Heparin inactivates thrombin and factor Xa through an antithrombin dependent mechanism.

2018 An 18-year-old man is brought to the emergency department for thumb replantation. After an uneventful microsurgical anastomosis of the digital arteries and veins, papaverine is applied to the vessels. This medication works as a vasodilator through which of the following mechanisms? A) Blocking calcium channels B) Decreasing platelet aggregation C) Inactivating thrombin and factor Xa D) Inhibiting glycoprotein IIb/IIIa E) Inhibiting phosphodiesterase

The correct response is Option C. The patient above has Poland syndrome. Poland syndrome represents a spectrum of congenital chest wall anomalies ranging from the simple form (depicted in the preoperative photograph) to complex. The pathognomonic feature is the absence of the sternocostal head of the pectoralis major muscle. A variety of other ipsilateral chest wall and upper extremity malformations may be present, including absence of ribs two through five, foreshortening of the limb, brachysyndactyly, hypoplasia/aplasia of the breast and nipple-areola complex, and absence/hypotrophy of various trunk muscles including the latissimus dorsi, serratus anterior and external oblique. Although a few familial cases have been reported, Poland syndrome is believed to be sporadic in nature with an incidence of 1:100,000. There is a male-to-female predilection of 3:1, with a right-sided predominance in boys of 2:1. The patient is shown two months after reconstruction with a right latissimus flap and bilateral silicone gel-filled implants. The trunk develops from the fusion of the ectoderm/neural crest layer with the somatic mesoderm during the fourth week of gestation. Mesoderm gives rise to somites, which differentiate into the dorsolateral dermomyotome and the ventromedial sclerotome. The dermomyotome gives rise to the musculature of the trunk and extremities. The sclerotome surrounds the developing spinal cord and notochord and forms the vertebral bodies and spinal skeletal structure. The notochord acts as a pathway for the development of the spinal cord and vertebral bodies. The notochord degenerates as the vertebral bodies develop and becomes the nucleus pulposus of the intervertebral disks. The exact pathophysiology of Poland syndrome has not been elucidated. One theory suggests a mechanical disruption of the embryonic blood supply in the subclavian/vertebral systems during the sixth to seventh week of gestation. Another theory suggests the etiology to be an injury or developmental failure of the mesodermal plate during the third to fourth week of gestation. Endoderm is the primitive germ layer that gives rise to the epithelium of the respiratory and gastrointestinal tracts. Ectoderm is divided into surface ectoderm and neuroectoderm. Surface ectoderm gives rise to the epidermis, adnexal structures of the skin, and the mammary glands. Neuroectoderm becomes the central nervous system, various neural ganglia, and the branchial arches.

2018 An 18-year-old woman comes to the office to discuss her congenital breast asymmetry. Physical examination shows asymmetry in the size and shape of the right breast and nipple-areola complex compared with the left breast. Additionally, there is an absence of the right anterior axillary fold. A photograph is shown. Which of the following is the most likely origin of the absent muscle in this patient? A) Ectoderm B) Endoderm C) Mesoderm D) Neuroectoderm E) Notochord

The correct response is Option B. When considering how best to treat pectus excavatum in the female patient, it is important to recognize that the majority of the contour deformity is due to the thoracic concavity with only a small portion of the deformity due to actual breast tissue hypoplasia. However, reinsertion of a correction bar is not always successful in correcting the thoracic deformity in post-adolescent patients whose bones have ossified, and is only done in extreme cases. Augmentation mammaplasty is also unsuccessful, regardless of implant volume, since, as stated above, only a small portion of the defect is actually caused by breast tissue asymmetry. Although some might consider autologous fat grafting a viable option for correcting both the chest wall defect and the hypoplastic breast, there have been reports that indicate that injecting between the tight adhesions of presternal skin and bone is technically challenging and rarely successful. Injection of cultured autologous chondrocytes, which is still in the early stages of research and development, would prove to be equally technically challenging, and at this time, de novo generation of cartilage or fat for injection into soft tissueue defects is still unproven. As a result, customized silicone elastomer implants are commonly used in conjunction with augmentation mammaplasty to provide consistent and reliable correction of pectus excavatum and breast asymmetry in the female patient. Studies have shown that placing a custom sternal prosthesis and bilateral breast implants during one surgery is both safe and effective in producing an improved aesthetic result.

2018 An 18-year-old woman is evaluated for chest wall and breast asymmetry. She has a history of pectus excavatum and underwent surgery as a child for placement of a correction bar, which was subsequently removed 3 years after insertion. Since the removal of the bar, she has noticed progressive recurrence of her chest wall deformity with associated breast asymmetry. Physical examination shows a thin patient with a 4-cm deep concavity involving the lower end of the sternum and medial distortion of the right breast. The patient denies shortness of breath or chest pain. Which of the following treatment options is the most appropriate recommendation for contour improvement? A) Autologous transfer of fat tissue to the sternal defect and right breast B) Implantation of a customized silicone elastomer device with concurrent augmentation mammaplasty C) Injection transplantation of cultured autologous chondrocytes D) Placement of bilateral silicone breast implants greater than 550 cc E) Reinsertion of the correction bar with sternal wiring

The correct response is Option C. This patient has hidradenitis suppurative (HS). There are various surgical treatments available to these patients with varying risk of recurrence. Surgical options include incision and drainage, surgical deroofing, local excision, and radical resection of all involved tissue. Reconstructive and closure techniques include secondary healing, primary closure, skin grafting and locoregional pedicled flaps. Numerous studies have looked at the outcomes with various closure techniques and they show that risk of recurrence is likely influenced more by the extent of disease rather than the method of definitive closure. Because of the recurrent nature of this disease, surgery has been considered as the only effective curative therapy for HS. Inadvertent compromise in the margin of resection may diminish the probability of successful healing. Radical resection of all hair-bearing skin with a 1- to 2-cm clear margin of normal tissue is the gold standard and the most important factor in reducing risk of recurrence. Recurrence rates tend to be higher after excision in regions where functional and aesthetic outcomes take priority, and often limit the extent of resection and compromise the ability to obtain clear margins (ie, axilla, perineum, breast). Historically, low-voltage radiation was used as a treatment modality for HS and it is thought to cause complete follicular destruction. Current studies on radiotherapy as treatment for HS are very limited because of concern regarding malignancies arising in radiation fields. Radiation should be reserved for individuals with severely recalcitrant disease and used with extreme caution in younger individuals. Bacterial burden may also diminish the probability of successful wound healing. Topical and systemic antibiotics are still the mainstay treatment for mild HS. Despite their widespread use, few studies have shown their efficacy. In addition, antibiotics do offer relief by reducing the burden of abscesses and pustules in some individuals, but recurrence in these people is frequent. Standard practice of managing acute flares with intra-lesional steroid injections lacks clinical evidence. It has been shown in a series of patients to reduce erythema, edema, size and pain, but no effect on recurrence of disease has been shown.

2018 An obese 47-year-old man comes to the office for pain and drainage in the perineal region. A photograph is shown. Medical history includes hypertension, hypercholesterolemia, and diabetes. Which of the following is the most important factor in reducing the risk of recurrence after surgery? A) Adjuvant radiation B) Antibiotic therapy C) Extent of resection D) Intralesional corticosteroid injection E) Method of closure

The correct response is Option E. The primary indication for excision in this case is the 10-cm measurement in greatest dimension. This makes this congenital nevus a giant nevus. The exact definition of a giant nevus has been debated, but one of the most constant definitions is a nevus that is 20 cm in greatest dimension in an adult or a nevus that will become 20 cm in greatest dimension once the child is fully grown. This latter measurement usually translates to approximately a 9-cm nevus on the head and neck, or approximately a 6-cm nevus on the body of an infant. Other definitions of giant nevi include a nevus that is about 1% of the body surface area in the head and neck region or 2% of the body surface area in the trunk region, a nevus that is greater than 100 cm2 in area, or a nevus that cannot be excised in one stage. The main indication for excision in these giant congenital nevi is the malignant potential that they harbor. The rates of malignant degeneration are also a matter of great debate, with rates quoted anywhere from 0 to 40%. More recently, however, rates of less than 5% are quoted. The other clinical features are not risk factors that indicate excision of the nevus. If a patient has more than three nevi, that potentially does increase the risk for malignant melanoma in the patient; however, this patient has only two.

2018 An otherwise healthy 2-year-old male infant is evaluated because of a congenital melanocytic nevus on his flank. Physical examination shows a hairy, 10 × 5-cm nevus with areas of dark brown pigmentation that is growing proportionate to the infant's growth rate. He has another nevus, which measures less than 2 cm, on his arm. Which of the following is the primary indication for excision? A) Dark color B) Growth rate C) Presence of hair D) Presence of two lesions E) Size of lesion

Please note: Upon further review, this item was not scored as part of the examination. The correct response is Option A. Bilaminate neodermal reconstruction of large scalp defects has been described as a successful, low-morbidity, durable reconstructive technique. A recent systematic review of the literature regarding this technique confirmed substantial evidence that this technique can be used to achieve predictable, durable results in patients with large scalp defects. The bilaminate neodermal matrix consists of an outer silicone sheet and an inner layer of bovine collagen and glycosaminoglycan (derived from shark cartilage), which acts as a scaffold for regeneration. Specifically, there is migration of host fibroblasts into the matrix, which subsequently proliferate and form collagen. Additionally, there is migration of endothelial cells that form a vascular network within the neodermis that can subsequently act as a foundation to support staged split-thickness skin grafting, at which time the silicone outer layer is removed and a thin (usually 6 to 8/1000") split-thickness autograft is applied. The success, or "take," of the skin graft depends on whether the neodermis is vascularized sufficiently to maintain the graft viability. A 2001 study by Moiemen et al, demonstrated clinically and histologically that this process takes 4 weeks on average, without the use of topical negative pressure. Split- and full-thickness skin grafting on exposed bone without periosteum would not result in a successful reconstruction. Scalp rotation flaps alone would not be sufficient to reconstruct a 65% defect, as there is not enough residual tissue. Furthermore, tissue expansion of scalp flaps is possible, but only for defects that are 50% or less of remaining scalp. A free anterolateral thigh flap is an option that would likely result in a successful reconstruction if there is sufficient donor tissue of the thigh to reconstruct this large defect, and would abide by the patient's request to avoid muscle harvest. However, given the size of the defect, it is likely that there would be a donor site defect that requires skin grafting, a longer index operation, and the risks of microvascular free tissue transfer that would outweigh the risks of a bilaminate neodermal matrix reconstruction.

2018 An otherwise healthy 23-year-old man is brought to the emergency department following traumatic avulsion of 65% of his scalp. The avulsed tissue is not replantable due to multilevel injury. A photograph is shown. After complete debridement, there is healthy remaining soft tissue and no contamination, but exposed bone without periosteum. The patient is adamant that he does not want sacrifice of muscle. Which of the following provides durable soft tissue coverage while minimizing donor site morbidity? A) Bilaminate neodermal reconstruction B) Free anterolateral thigh flap C) Full-thickness skin grafting D) Scalp rotation flap after tissue expansion E) Split-thickness skin grafting

The correct response is Option D. Available recent Level III evidence using prospectively collected standardized photographic measurements in patients who have undergone liposuction and/or abdominoplasty versus retrospective controls demonstrated that removal of excess fat through these methods provided long-term reduction in treated areas without fat reaccumulation in either treated or untreated areas of the body. This evidence contradicts the commonly held notion (mostly by the lay public) that removal of fat in one location leads to "return" of fat in another. Short-term reductions of fat with reaccumulation in treated and/or untreated areas can occur if the patient does not remain calorically neutral after surgery. However, in this clinical scenario, it is mentioned that this patient has had no postoperative increase in caloric intake.

2018 An otherwise healthy 32-year-old woman is considering liposuction. Assuming that the patient maintains her current weight, which of the following most accurately describes the anticipated changes in her body fat distribution after surgery? A) Long-term reduction in the abdomen but eventual reaccumulation in the hips B) Long-term reduction in the hips but eventual reaccumulation in the abdomen C) Long-term reduction in treated areas but compensatory increase in fat in untreated areas D) Long-term reduction in treated areas without reaccumulation in treated or untreated areas of the body E) Short-term reduction in treated areas but eventual long-term fat reaccumulation in these areas

Please note: Upon further review, this item was not scored as part of the examination. The correct response is Option B. The contralateral periumbilical perforators are the vessels that need to be ligated during the delay procedure for a pedicled transverse rectus abdominis muscle (TRAM) flap. This intermediary procedure is effective in increasing the reliability of the TRAM flap in high-risk patients by augmenting blood flow to the areas rendered ischemic by the ligation through the opening of choke vessels. Traditionally in the TRAM delay procedure, the deep inferior and the superficial inferior epigastric vessels are ligated. There is some debate whether the contralateral deep and superficial systems need to be sacrificed during the delay procedure. Some authors feel that these vessels should be left uninjured so that they can be used in salvage situations for congested flaps. More recent studies show that ligating the ipsilateral deep epigastric vessels and the contralateral periumbilical perforators enhances the reliability of Zone III in TRAM flaps more effectively. This staged procedure is indicated in patients who require large tissue flaps, have been irradiated, are smokers, are obese, or have low midline incisions on the abdomen. This procedure should be done about 2 to 3 weeks prior to the scheduled TRAM flap. The other options listed are incorrect because they would either interfere with the blood flow to the TRAM flap being used or have no effect on the enhancement of blood flow to the desired TRAM flap. Ligation of the contralateral deep inferior epigastric vessels is not as effective as ligation of the contralateral periumbilical perforators due to maintenance of perfusion via the superior epigastric vessels.

2018 An otherwise healthy 55-year-old woman is evaluated for reconstruction of the left breast using a pedicled transverse rectus abdominis musculocutaneous (TRAM) flap. In addition to ligating the ipsilateral deep inferior epigastric pedicle, ligation of which of the following will maximize perfusion across the midline? A) Contralateral deep inferior epigastric vessels B) Contralateral periumbilical perforators C) Contralateral superior epigastric vessels D) Ipsilateral periumbilical perforators E) Ipsilateral superior epigastric vessels

Please note: Upon further review, this item was not scored as part of the examination. The correct response is Option B. A recent randomized controlled trial has shown that oral propranolol is better tolerated with fewer adverse effects than prednisolone. Based on this and other studies that have been published, oral propranolol has assumed the status of effective first-line treatment of infantile hemangiomas if there is no contraindication to its use. Intralesional corticosteroids can arrest growth but generally do not cause regression. In large dosages, they can also have systematic adverse effects. Surgical excision is typically reserved for urgent/emergent circumstances where a rapidly proliferating hemangioma threatens important structures or function, such as visual, nasopharyngeal, or auditory canal obstruction. Otherwise, on an elective basis, it is usually deferred until later in life (i.e. school-age) when psychosocial concerns present more prominently. Pulsed-dye laser can be used, but requires multiple treatments or anesthesia, and carries generally more risk than oral propranolol, which is the correct choice in this situation.

2018 An otherwise healthy, 2-month-old female infant is brought to the outpatient clinic for evaluation of an enlarging hemangioma of the face. The parents are concerned about its rapid growth and beginning ulceration. Which of the following interventions would provide the most efficacious treatment with the lowest risk of adverse events? A) Embolization B) Oral propranolol therapy C) Pulsed-dye laser treatment D) Surgical excision E) Triamcinolone injection

The correct response is Option D. The most efficient sequence to perform the replantation is structure by structure: bone, tendons, arteries, nerves, and then veins. It has been shown that the time to complete the procedure is significantly shorter if the same anatomic structure on each severed digit is fixed before repairing the next structures, as opposed to completing all aspects of the replantation one digit at a time. With respect to the sequence of repair of the severed structures, the general thought is to have a stable construct prior to starting the delicate microscopic repairs. However, the technical sequence used by microsurgeons varies greatly. The only consistent agreement is starting with bony shortening and fixation. The traditional sequence that follows is extensor and flexor tendon repair, and then vessel/nerve repair. However, individual surgeon preference and patient circumstances dictate the usual sequence thereafter. Some surgeons like to start dorsally and complete the extensor tendon, venous, and skin repair first, and then complete the volar structures next. On the volar side, some surgeons repair the tendon first, followed by the artery and nerve, while others fix the artery and nerve first, followed by the tendon. There are those who believe that the nerve is better repaired in a bloodless field, so that should be done first. Others feel that repairing the vein first reduces blood loss and keeps a bloodless field more reliably for better vision. In patients who present with long ischemia time, it may be beneficial to anastomose the artery first, because this provides the advantages of earlier revascularization and allows easier detection of the most functional veins by their spurting backflow. In short, any of these sequences is fine, as long as it follows the bony fixation. The other options are incorrect sequences for the above reasons.

2018 An otherwise healthy, nonsmoking 30-year-old mechanic has the long, ring, and little fingers amputated sharply through Zone II of the right hand. The amputated digits are stored appropriately, and he is rushed to surgery within 2 hours of the accident. Which of the following sequences is the best method of replantation? A) Digit by digit: bone, tendons, arteries, nerves, veins B) Digit by digit: bone, tendons, arteries, veins, nerves C) Structure by structure: bone, nerves, tendons, arteries, veins D) Structure by structure: bone, tendons, arteries, nerves, veins E) Structure by structure: tendons, bones, veins, arteries, nerves

The correct response is Option E. Preservation of segmental intercostal nerves is a critical aspect of component separation. Component separation is a powerful technique used during abdominal wall reconstruction to advance the abdominal fascia towards the midline and allow for primary closure with reduced tension. A component separation is performed by making an incision longitudinally in the external oblique aponeurosis, just lateral to the linea semilunaris. Only the fascia is cut, not the external oblique muscle. This allows for advancement of a myofascial complex consisting of the rectus abdominus muscle, internal oblique muscle and fascia, and transversalis muscle and fascia. The segmental intercostal nerves that supply the anterior abdominal wall run between the internal oblique and the transversalis muscle. They are deeper than the incision and dissection plane with component separation, and are thus protected from injury. This maintains abdominal wall tone in patients undergoing this procedure. External oblique aponeurotomy, not internal oblique aponeurotomy, is involved in component separation. Incision of the anterior rectus sheath is not part of component separation, although some modifications include incision of the posterior rectus sheath to release more fascia available for primary closure. Plication of the linea semilunaris is not a part of component separation. Ligation of periumbilical perforators is commonly performed in the standard, open approach to component separation, but it is not considered a critical component of the procedure, as it is possible to perform a perforator-sparing component separation.

2018 During a ventral hernia repair and abdominal wall reconstruction, a surgeon wishes to achieve full primary fascial closure over an intraperitoneal mesh. However, the fascia is under tension and requires a component separation to achieve full closure with minimal tension. Component separation technique involves which of the following? A) Incision of the anterior rectus sheath B) Internal oblique aponeurotomy C) Ligation of periumbilical perforators D) Plication of the linea semilunaris E) Preservation of intercostal nerves

The correct response is Option D. Systemic antibiotic prophylaxis is recommended in clean breast surgery. Studies have shown that the use of antibiotic prophylaxis in patients undergoing breast surgery (with or without implant) significantly reduces the risk of surgical site infections. The benefit from routine antibiotic prophylaxis is greater in individuals receiving tissue expanders or breast implants for reconstruction, but patients undergoing breast augmentation or reduction mammaplasty also benefit from antibiotic prophylaxis. With the exception of cosmetic breast surgery, clean operations have not been shown to benefit from routine antibiotic prophylaxis. Therefore, the use of routine antibiotic prophylaxis is not indicated in clean surgical cases of the hand (carpal tunnel release), skin (squamous cell carcinoma of the skin), head and neck, or abdominoplasty. It is indicated in contaminated surgery of the hand or face.

2018 The use of routine systemic antibiotic prophylaxis is indicated in which of the following procedures? A) Abdominoplasty B) Carpal tunnel release C) Excision of squamous cell carcinoma of the skin D) Reduction mammaplasty E) Rhytidectomy

The correct response is Option B. Negative pressure wound therapy (NPWT) has dramatically improved our ability to manage complicated and complex wounds. Advantages of NPWT include decreased healing time, simplified wound care with less frequent dressing changes, and promotion of healthy granulation tissue. However, there are several contraindications to the use of NPWT. The presence of exposed vessels is an absolute contraindication to its use; arterial rupture has been reported with multiple fatalities. In addition to placement over exposed vessels, significant bleeding has also been noted with use of NPWT in groin and sternal wounds, in patients on blood-thinning medications, and during removal of well-integrated foam pieces. Other complications include infections due to retained pieces of foam. In addition to exposed vessels, NPWT should not be used in the setting of active, uncontrolled infection, malignant wounds, wounds with unexplored and/or nonenteric fistulas, and poorly debrided wounds. Although NPWT may not be the definitive wound management in the other scenarios listed, there is no contraindication for its use.

2018 Use of negative pressure wound therapy with a sponge dressing is likely to result in an adverse outcome in which of the following clinical scenarios? A) A hand wound with exposed tendons B) A lower extremity wound with exposed femoral vessels C) A Stage IV sacral pressure sore D) As a bolster over a split thickness skin graft on the arm E) Over a closed surgical incision after hernia repair

The correct response is Option C. Historically, there has been a 10 to 15% reported malignant degeneration in nevus sebaceous in children and the recommendation had been for all lesions to be removed before puberty. More recent studies have shown malignant transformation of these lesions to be less than 1%. Therefore, the decrease in malignant change prompted a more conservative approach to nevus sebaceous based on serial monitoring and excision of lesions that are a source of irritation, difficult to monitor clinically, or aesthetically displeasing. Tissue expansion is an option in patients with giant nevus sebaceous and usually requires one to two rounds of expansion and serial excisions. However, this is not a first line treatment for a small 4-cm lesion. Although serial excision is an alternative reconstructive option, it is recommended only for larger lesions that can be excised in three stages or less. If that is not possible, then tissue expansion is the preferred procedure. Reconstruction with rotational flaps should be planned in consideration of natural facial aesthetic units and an effort should be made to maintain natural brow and hairline position and symmetry and avoid creating tension in the perioral, periorbital, and periauricular regions.

2018 Which of the following is the most appropriate surgical treatment recommendation for a 4-cm round sebaceous nevus of the scalp in a child? A) Excision, skin grafting, and delayed tissue expansion B) Serial excision without tissue expansion C) Serial monitoring and selective excision D) Tissue expansion and excision E) Tissue expansion, excision, and rotational flaps

The correct response is Option E. The two technical factors that increase the likelihood of a "good" scar are placement of sutures that will not leave permanent suture marks and wound-edge eversion. In wounds where the skin is brought precisely together, there is a tendency for the scar to widen. In wounds where the edges are everted or hypereverted in an exaggerated fashion, this tendency is minimized possibly by reducing the tension on the closure. While the most common method of closing a wound is with sutures, there is nothing necessarily superior about sutures or a specific type of suture. Staples, skin tapes, or wound adhesives are also useful in certain situations. Regardless of the method of closure or type of suture used, precise approximation of skin edges without tension is essential to ensure healing with minimal scarring. Simple interrupted suture is the gold standard for suturing wounds closed and everting the skin edges. Retention sutures tend to leave the most obvious and unsightly cross-hatching if they are not removed early. Wounds deeper than the skin are closed in layers. The key is to eliminate the dead space and provide a strong closure to prevent dehiscence and reduce tension. However, not all layers necessarily require separate closure.

2018 Which of the following technical factors has the greatest favorable impact on the final appearance of a surgical scar? A) Closing the wound in a single layer B) Use of an absorbable suture C) Use of topical cyanoacrylate D) Retention suture E) Wound-edge eversion


Set pelajaran terkait

Archer Maternal & Newborn Health

View Set

American History Chapter 4 Worksheets

View Set

1G) Calculating Healthcare Statistics

View Set

Module 37: Public Goods and Services

View Set